Download as pdf or txt
Download as pdf or txt
You are on page 1of 371

See discussions, stats, and author profiles for this publication at: https://www.researchgate.

net/publication/267866066

Mathematical Methods

Book · May 2005

CITATIONS READS
0 34,330

2 authors, including:

Zafar Ahsan
Aligarh Muslim University
115 PUBLICATIONS   509 CITATIONS   

SEE PROFILE

Some of the authors of this publication are also working on these related projects:

Ricci solitons and Symmetries of spacetime manifold View project

Potential of Gravitational Fields in General Relativity View project

All content following this page was uploaded by Zafar Ahsan on 24 April 2021.

The user has requested enhancement of the downloaded file.


Mathematical Methods
Mathematical
Methods

Zafar Ahsan
Nikhat Ahsan
Zafar Ahsan
Department of Mathematics
Aligarh Muslim University
Aligarh-202002, India

Nikhat Ahsan
Formerly at AMU Girl’s High School
Aligarh-202002, India

Copyright © 2017 Real World Education Publishers


REAL WORLD EDUCATION PUBLISHERS
A-216, First Floor, Shaheen Bagh, Abul Fazal Enclave, Jamia Nagar,
New Delhi-110025, India.
E-mail: danishkhan@rwepublishers.com
info@rwepublishers.com

All rights reserved. No part of this publication may be reproduced, stored


in a retrieval system, or transmitted in any form or by any means,
electronic, mechanical, photocopying, recording or otherwise, in part
or as a whole, without the prior written permission of the publishers.

ISBN 978-81-927151-9-3

Price : ` 000.00

Published by Mohd Danish Khan for Real World Education Publishers


A-216, First Floor, Shaheen Bagh, Abul Fazal Enclave, Jamia Nagar,
New Delhi-110025
Printed in India.
Preface
Mathematics, in a broader sense, may be defined as the scientific study
of quantities, including their relationships, operations and measurements
expressed by numbers and symbols. From the wonders of Taj Mahal in
India, Pyramids of Egypt and the Leaning Tower of Pisa to the making
of atomic bomb and space crafts - mathematics is applicable everywhere.
Some of the disciplines where mathematics plays a key role, apart from
mathematical and physical sciences and engineering, are acturial science,
coding theory, commercial surveys, cryptology, economics, financial and
market research analysis, industrial chemistry and robotics, etc.
Mathematics has two branches, viz. pure and applied. Pure
mathematics deals with the concepts and ideas which do not necessarily
have any immediate applications. But these ideas and concepts have
their own intrinsic beauty. Applied mathematics is concerned with the
use of mathematical theories and principles as tools to solve a real life
problem.
The present book is primarilary intended for the undergraduate
students of industrial chemistry, business managment, biosciences, social
sciences as well as the post graduate students of hydrogeology and
biotechnology. It can also be used as a reference book for the post
graduate students of economics and biochemistry.
This book consists of eight chapters and three appendices. In each
chapter, as also in the appendices, an ample amount of theory is given
which is supported by solved examples. The ability to solve the problems
is essential to a proper understanding of the subject. Hence, each chapter
ends up with a carefully selected set of problems along with their
answers. Chapter 1 deals with the theory of quadratic equations and
vi Mathematical Methods

the methods for solving such equations. Complex numbers, their


algebraic and geometric properties along with the related results have
also been discussed in this chapter. The concept of a real function, its
algebra and different types of functions along with the limit of a function
and the related results on how to find the limit of a given function form
the contents of Chapter 2. The continuity of a function and related results
are also given here. The concepts dealt here are then applied to measure
the intensities of loudness and earth quake, pH value of a solution and
depreciation of an equipment, etc. Chapter 3 gives the derivative of a
function and its geometrical and physical meanings; the methods of
finding the derivatives of the elementary functions and a number of
applications of the derivatives in the fields of business management,
social science and biotechnology, etc. Indefinite and definite integrals
of a function and the methods for finding them along with the
application of definite integral in determining the areas of the bounded
figures are given in Chapter 4. The function of several variables and
its derivatives, known as partial derivatives, have been discussed in
Chapter 5 along with the related results. Chapter 6 deals with ordinary
differential equations and the methods for finding the solutions of such
equations have been discussed including applications of differential
equations in the fields of archeology, population studies and business
management, etc. Chapter 7 presents the notions of a matrix and a
determinant along with a number of properties of matrices and
determinants with their applications. The main attraction of this chapter
is the Leontief input-output model which plays a key role in business
management and economics. Chapter 8 introduces the concept of vector
and also deals with the algebraic and geometric properties of vectors
along with their differentiation and integration. Finally, the topics like,
progressions, permutation, combination and binomial theorems have
been discussed in Appendices along with applications of these concepts.
The book finally ends up with a bibliography.
While preparing the book, we have consulted many standard works
and are indebted to the authors of those works. We are grateful to Prof.
Khalil Ahmad, Dean, School of Physical and Molecular Sciences, Al-
Falah University, Faridabad for his critical reading and suggestions in
the manuscript. We also express our gratitude to Real World Education
Publishers for their meticulous processing of the manuscript and
bringing out the book in such a nice form.
Zafar Ahsan
Nikhat Ahsan
April 2015
Contents
Preface v

1. Quadratic Equations and Complex Numbers 1


1.1 Introduction 1
1.2 Quadratic Equations 4
1.3 Methods for Solving a Quadratic Equation 5
1.4 Complex Numbers 12
1.5 Algebra of Complex Numbers 13
1.6 Cube Roots of Unity 16
1.7 Representation of a Complex Number 16
1.8 De Moivre’s Theorem 19
Exercises 20

2. Functions, Limit and Continuity 23


2.1 Introduction 23
2.2 Functions 23
2.3 Operations on Functions 43
2.4 Limit of a Function 46
2.5 Infinite Limits 54
2.6 Right and Left Hand Limits 56
2.7 Continuity of a Function 59
Exercises 68

3. Differentiation 72
3.1 Introduction 72
viii Mathematical Methods

3.2 Derivative of a Function 72


3.3 Differentiation from the First Principle 78
3.4 Formulas for Differentiation 80
3.5 Derivative of a Function of Function 81
3.6 Trigonometric Transformation and Differentiation
by Substitution 83
3.7 Differentiation of Implicit Function 87
3.8 Logarithmic Differentiation 88
3.9 Parametric Differentiation 90
3.10 Higher Order Derivatives 92
3.11 Applications of Derivatives 95
Exercises 121

4. Integration 129
4.1 Introduction 129
4.2 Indefinite Integral 129
4.3 Methods of Integration 136
4.4 Definite Integral 164
4.5 Fundamental Theorems of Integral Calculus 168
4.6 Properties of Definite Integrals 172
4.7 Areas of Bounded Regions 178
Exercises 180

5. Partial Differentiation 186


5.1 Introduction 186
5.2 Partial Derivative 187
5.3 Higher Order Partial Derivatives 188
5.4 Total Differential 190
5.5 Change of Variables 193
5.6 Euler’s Theorem on Homogeneous Functions 195
Exercises 197

6. Differential Equations 199


6.1 Introduction 199
6.2 Formation of a Differential Equation 201
Quadratic Equations and Complex Numbers ix

6.3 Differential Equations of First Order and


First Degree 203
6.4 Higher Order Linear Differential Equations 212
6.5 Solution of Homogeneous Linear Differential
Equations of Order n with Constant Coefficients 212
6.6 Solution of Nonhomogeneous Linear Differential
Equations of Order n with Constant Coefficients 215
6.7 Special Methods for Finding the Paticular Integral 217
Exercises 230

7. Matrices and Determinants 235


7.1 Introduction and Definition 235
7.2 Algebra of Matrices 239
7.3 Determinants 248
7.4 Inverse of a Matrix 257
7.5 Solution of a Matrix Equation 261
7.6 Solution of Linear Equations 262
7.7 Leontief Input-output Analysis 264
Exercises 267

8. Vector Analysis 272


8.1 Introduction 272
8.2 Algebra of Vectors 273
8.3 Vector Calculus 286
8.4 Line, Surface and Volume Integrals and Integral
Theorems 296
Exercises 305
Appendix A : Arithmetic, Geometric and Harmonic
Progressions 309
Appendix B : Permutation, Combination and Binomial
Theorem 324
Appendix C : Some Useful Formulas 348
References 355
Index 357
CHAPTER 1

Quadratic Equations and Complex


Numbers

1.1 Introduction
Solving problems by means of equations is an old tradition of mathematics
and the earliest description about the methods of solving equations has
been described in a book written by Abu Abdallah Mohammad bin Musa
Al-Khwarizmi around 830 A.D. Al-Khwarizmi is considered to be the
grand father of computer science and the father of Algebra. He was a
Persian/Arab mathematician, astronomer, astrologer, geographer and a
scholar in the House of Wisdom (Bait ul Hikma) in Baghdad.
Al-Khwarizmi was the popularizer of Arabic numerals, adopter of zero
(the symbol) and the decimal system. He was born in Persia of that time
around 780 A.D. and died around 850 A.D. He was one of the first
directors of the House of Wisdom and one of the few learned men who
worked in this House in the early 9th Century. Al-Khwarizmi flourished
while working as a member of the House of Wisdom in Baghdad under
the leadership of Khalif al-Mamun, the son of the Khalif Harun al-Rashid.
The House of Wisdom was a scientific research and teachning centre.
Ibn al-Nadim’s Kitab al-Fihrist includes a short biography on
Al-Khwarizmi, together with a list of the books he wrote. Al-Khwarizmi
accomplished most of his work in the period between 813 and 833. After
the Islamic conquest of Persia, Baghdad became the centre of scientific
studies and trade, and many merchants and scientists from as far as
China and India traveled to this city, as did Al-Khwarizmi. He worked
in Baghdad as a scholar at the House of Wisdom, where he studied the
sciences and mathematics, which included the translation of Greek and
Sanskrit scientific manuscripts.
To Al-Khwarizmi we owe the word “algebra” which is derived from
the Arabic word “Al-jabr” (meaning completion, or subtracting a number
from both sides of the equation) appearing in the title of his greatest
2 Mathematical Methods

mathematical work “Al-Kitab al-mukhtasar finisbb al-jabr wal-


muqabala” (Compendious Book on Calculation by Completion and
Balancing). The title of Al-Khwarizmi’s above book literally means
restoration (transposition of negative terms to the other side of the
equation) and reduction (uniting the similar terms).
Following Arab verse was in use at that time:
Cancel minus terms and then
Restore to make your Algebra
Combine your homogeneous terms
And this is Muqabalah
European mathematician came to know the work of Al-Khwarizmi
when his book was translated into Latin as “Liber algebrae et almucabala”
by Robert of Chester in 12th century. The Latin translation began “Dixit
algorithmi...” (meaning “Al-Khwarizmi said...”). Thus the name of Abu
Abdallah Mohammad bin Musa Al-Khwarizmi has been shortened and
the word “algorithm” is derived from the Latinized form of Al-
Khwarizmi’s name “Algorithmi”. Any systematic method to solve a
problem is now known as ALGORITHM and hence the name of Al-
Khwarizmi is considered, by some people, to be as the grand father of
computer science.
A unique Arabic copy of Al-Khwarizmi book al-jabr wal-muqabala
is kept at Oxford and was translated in 1831 by F. Rosen. A Latin
translation is kept in Cambridge. This book is considered to be the
foundational text of modern algebra and corner stone of science. In this
book al-jabr wal-muqabala, Al-Khwarizami presented the first systematic
solution of linear and quadratic equations in Arabic. Several hundred
simple quadratic equations have been worked out, in this book, by analysis
and as well as geometric example. It is the first book to be written on
algebra. In Al Khwarizmi’s own words, the purpose of the book was to
teach what was easiest and most useful in artithmetic, such as what was
constantly required in cases of inheritance, legacies, partition, law-suits,
and trade, and in all their dealings with one another, or where the
measuring of lands, the digging of canals, geometrical computations,
and other objects of various sorts and kinds were concerned.
Al-Khwarizmi is usually credited with the development of lattice
(or sieve) multiplication method of multiplying large numbers, a method
algorithmically equivalent to long multiplication. His lattice method was
later introduced into Europe by Fibonacci.
Al-Khwarizmi’s second major work was on the subject of arithmetic,
which survived in a Latin translation but was lost in the original Arabic.
Quadratic Equations and Complex Numbers 3

The original Arabic title was possibly Kitab al-Jama wal-tafriq bil hisab
al-Hind (“The Book of Addition and Subtraction According to the Hindu
Calculation”). Al-Khwarizmi’s work on arithmetic was responsible for
introducing the Arabic numerals, based on the Hindu-Arabic numeral
system developed in Indian mathematics, to the western world. Al-
Khwarizmi’s book on arithmetic was translated into Latin and published
in Rome in 1857 by Prince Baldassare Boncompagni. Many of his books
were translated into Latin and used as a principle mathematical text
book in European universities until the 16th century. Among them these
two books had important place: “Kitab al-Jama wal-Tafreeq bil Hisab
al-Hind ” and “Kitab al-Jabr wal-muqabala.”
Al-Khwarizmi developed detailed trigonometric tables containing
the sine functions which later included tangent functions. Al-Khwarizmi
also wrote an important work on astronomy, covering calendars,
calculating true positions of the sun, moon and planets, tables of sine
and tangents, spherical astronomy, astrological tables, parallax and eclipse
calculations, and visibility of the moon. Although his astronomical work
is based on that of the Indians, and most of the values from which he
constructed his tables came from Hindu astronomers, Al-Khwarizmi must
have been influenced by Ptolemy’s work too. Al-Khwarizmi performed
detailed calculations of the positions of the sun, moon, and planets, and
did a number of eclipse calculations. In addition to an important treatise
on astronomy, Al-Khwarizmi wrote a book on astronomical tables, which
were also translated into European languages and, later, into Chinese.
Al-Khwarizmi’s another major contribution is his 833’s book “Kitab
Suart al-ard ” (Book on the appearance of the Earth). His works differed
from Ptolemy’s and he corrected Ptolemy’s views in detail. It is a
description of a world (known world at that time) map and contains a
list of the coordinates of the important places on it. He corrected the
distortion that Ptolemy’s map had with regard to the length of the
Mediterranean. It was much more accurate. However, it failed to replace
the Ptolemaic geography used in Europe. Al-Khwarizmi’s work on
geography which gives latitudes and longitudes for 2,402 cities and
landmarks, forming the basis for a world map. The book lists with
latitudes and longitudes, cities, mountains, seas, islands, geographical
regions, and rivers.
A number of minor works were written by Al-Khwarizmi on topics
such as clocks, sundials and astrolabe. He made several important
improvements to the theory and construction of the sundials. He made
tables for these instruments which considerably shortened the time needed
4 Mathematical Methods

to make specific calculations. His sundials were universal and could be


observed from anywhere on the earth. From then onward, sundials were
frequently placed on mosques to determine the time of prayer.
In conclusion, algebra and algorithms are enabling the building of
computers, and the creation of encryption. The modern technology
industry would not exist without the contributions of Muslim
mathematicians like Al-Khwarizami. Abu Abdallah Mohammad bin
Musa Al-Khwarizmi died in 1850 A.D. being remembered as one of the
most seminal scientific minds of early Islamic culture.
This chapter deals with the quadratic equations and the methods for
solving such equations. These methods have been illustrated with the
help of examples. Complex numbers and some of their basic properties
have also been given.

1.2 Quadratic Equations


To obtain a mathematical model of a real life problem, quadratic equations
are often used. We begin with few definitions.
Definition 1.1 An expression of the form
f (x) = a0 x n + a1 x n–1 + a2 x n–2 + ... + an (1)
where a0, a1, a2, ..., an are constants, n is a positive integer, is called a
polynomial of degree n (in x).
Definition 1.2 A polynomial in x of degree one is called a linear
polynomial.
For example, f (x) = ax + b, f (x) = 4x – 6, etc. are polynomials of
degree one.
Definition 1.3 If a polynomial of degree 2 is equated to zero, then such
an equation is called a quadratic equation.
For example, if f (x) = 0 and n = 2 in Eq. (1) then a0 x 2 + a1 x +
a2 = 0 is a quadratic equation.
The most general quadratic equation is of the form
ax2 + bx + c = 0 (2)
where a, b and c are constants and a ≠ 0.
A root of the equation f (x) = 0 is the value of x which makes f (x)
zero. For example, x = –1, x = –5 are the roots of the equation x2 + 6x +
5 = 0 (for, put x = –1 in x 2 + 6x + 5 = 0, we get LHS = (–1)2 + 6(–1) + 5
= 1 – 6 + 5 = 0 = RHS and similarly for x = –5, the equation x2 + 6x + 5
= 0 is satisfied). But x = 1
2 is not a root of the equation 2x2 – 6x + 3 = 0.
The set of all roots of an equation, in a given domain, is called the
solution set of the equation.
Quadratic Equations and Complex Numbers 5

1.3 Methods for Solving a Quadratic Equation


Solving a quadratic equation means finding the roots of the quadratic
equation. In what follows, we shall list some of the commonly known
methods for solving a quadratic equation and illustrate them with the
help of examples.

1.3.1 Factorization Method


Let the given quadratic equation be
ax2 + bx + c = 0, a ≠ 0
or
(dx + e) ( f x + g) = 0
or
d x + e = 0 and f x + g = 0
or
d x = – e and f x = – g
or
x = – e/d and x = – g/ f
Thus, the possible roots (solutions) of the given quadratic equation
ax2 + bx + c = 0 are x = – e/d and x = – g/f .
Example 1.1 Solve the following quadratic equations by factorization
method:
(i) x 2 + 3x – 18 = 0
1 1 1
(ii) + =
3 − 2x 5 + 2x 2
(iii) (x – 1) (x – 2) = 4x 2 – 5x – 3
Solution (i) The given equation can be written as
x2 + 6x – 3x – 18 = 0
or
x(x + 6) – 3(x + 6) = 0
or
(x – 3) (x + 6) = 0
which leads to x = 3, – 6.
(ii) The given equation can be written as
(5 + 2 x) + (3 − 2 x) 1
=
(3 − 2 x )(5 + 2 x ) 2
6 Mathematical Methods

or
2(5 + 2x + 3 – 2x) = (3 – 2x) (5 + 2x)
or
2(8) = 15 + 6x – 10x – 4x2
or
4x2 + 4x + 1 = 0
or
(2x + 1) (2x + 1) = 0
−1 −1
and thus x = , .
2 2
(iii) The given equation can be written as
x 2 – 2x – x + 2 = 4x 2 – 5x – 3
or
x 2 – 4x 2 + 5x – 3x + 2 + 3 = 0
or
3x 2 – 2x – 5 = 0
which on solving leads to x = –1, 5/3.
Example 1.2 The length of a room is 3 ft less than twice its breadth.
What will be the dimension of the room if 252 sq ft carpet is laid down
in the room?
Solution Let b be the breadth of the room. Then, the length is 2b – 3.
The area of the room is (2b – 3)b. Thus, the carpet required to cover the
room is 2b2 – 3b sq ft, i.e.
2b 2 – 3b = 252
which leads to

−21
b = , 12
2
Neglecting the first value, the breadth is 12 ft and the length is 2b – 3 =
21 ft.
Example 1.3 In a chemical factory, there is a chemical reagent vat with
two input pipes A and B. If both pipes are opened, the vat will be filled in
4 h. How much time is required to fill the vat if each of the pipe is
working alone such that pipe A takes 3 h more than pipe B?
Quadratic Equations and Complex Numbers 7

Solution Let t be the time in hours required by pipe B to fill the vat
alone. Then, t + 3 is the time taken by the pipe A to fill the vat alone.
Thus in 1 hour, pipe B fills 1/t of the vat, while pipe A fills 1/(t + 3) of the
vat. Both pipes A and B, working together, fill 1/4 of the vat. Therefore,
we have

1 1 1
+ =
t t +3 4
which on solving leads to the quadratic equation
t 2 – 5t – 12 = 0
The roots of this equation are –1.959 and 8.041. Thus t = 8.041 is the
only possible solution for the given problem. Therefore, if both the pipes
A and B are working alone, then it takes nearly 8 h and 11 h, respectively,
to fill the vat.

1.3.2 Nature of the Roots


If ax 2 + bx + c = 0, a ≠ 0, then the roots are

− b + b 2 − 4ac − b − b 2 − 4ac
x= , x=
2a 2a
The quantity b 2 – 4ac = D is called the discriminant.
Consider the quadratic equation
ax 2 + bx + c = 0, a ≠ 0 (3)
Multiplying this equation by 4a, we get
4a 2x 2 + 4abx + 4ac = 0
or
(2ax)2 + 2(2ax)b + 4ac = 0
or
(2ax)2 + 2(2ax)b + b 2 = b 2 – 4ac
so that
(2ax + b) 2 = b 2 – 4ac (4)
We thus have the following cases:

Case I : If b 2 – 4ac ≥ 0, then b2 − 4ac is a real number and


Eq. (4) leads to

2ax + b = ± b2 − 4ac
8 Mathematical Methods

so that

− b ± b 2 − 4ac
x = (5)
2a
and thus Eq. (3) has two roots. Moreover, the roots given by Eq. (5)
are real and distinct, if b2 − 4ac > 0, and are real and equal if

b2 − 4ac = 0.

Case II : If b 2 − 4ac < 0, then Eq. (4) leads to


(2ax + b) 2 = b2 – 4ac < 0
But for no real value of x, the quantity (2ax + b)2 < 0 and thus the given
quadratic equation (3) has no real roots.
Example 1.4 Find the roots of the following quadratic equations:
(i) x 2 – 4x + 1 = 0
(ii) 2x 2 + x – 1 = 0
(iii) 2x 2 + 5x + 5 = 0
4 2 3
(iv) x – 2x + =0
3 4
Solution (i) Here a = 1, b = – 4, c = 1 and thus b2 – 4ac > 0. The given
4 ± 12
equation has thus real and distrinct roots which are given by 2
.
(ii) Here a = 2, b = 1, c = –1 and thus b2 – 4ac > 0. The given equation
has thus real and distrinct roots which are given by −1 ± 9
4
.
(iii) Here a = 2, b = 5, c = 5 and thus b 2 – 4ac < 0. The given equation
has thus no real roots.
(iv) Here a = 4/3, b = –2, c = 3/4 and thus b 2 – 4ac = 0. The given
equation has thus real and equal roots which are given by 3 3
, .
4 4

1.3.3 Relation Between the Roots and the Coefficients and


Formation of a Quadratic Equation
If ax 2 + bx + c = 0, a ≠ 0 is a quadratic equation with D = b 2 – 4ac ≥ 0
and the roots are

− b + b2 − 4ac − b − b2 − 4ac
α = ,β= (6)
2a 2a
Quadratic Equations and Complex Numbers 9

Then, if the roots of the quadratic equation are given, we can form the
quadratic equation as follows:
From Eq. (6), we have
sum of the roots = α + β = – b/a, product of the roots = αβ = c/a
Also, the Eq. (3) can be written as
b c −b c
x2 + x + = 0 ⇒ x2 – x+ =0
a a a a
Therefore, the required quadratic equation is given by
x 2 – (sum of the roots) x + (product of the roots) = 0 (7)
Example 1.5 Obtain the quadratic equation if the roots of the quadratic
equation are 4, –5 and –1, 3.
Solution Here α = 4, β = –5 which gives α + β = –1, and αβ = –20 and
thus the required quadratic equation, using Eq. (7), is x 2 + x – 20 = 0.
On the other hand, for the roots –1, 3, the required quadratic equation,
again from Eq. (7), is x 2 – 2x – 3 = 0.
Example 1.6 Form the quadratic equation whose one root is 1 + 2 and
sum of its roots is 2.
Solution Given that one root is 1 + 2 and let the other root be α, then
(1 + 2 ) + α = sum of the roots = 2
which gives α = 1 – 2, and thus the roots are 1 + 2 and 1 – 2. The
product of the roots is now –1 and hence, from Eq. (7), the required
quadratic equation is x 2 – 2x – 1 = 0.

1.3.4 Equations Reducible to Quadratic Equations


Here, we shall mention those equations that can be reduced to a quadratic
equation.
1. Equations of the type ax 4 + bx 2 + c = 0
This equation can be reduced to a quadratic equation if we use the
substitution x 2 = y.
Example 1.7 Solve the equation x 4 – 10x 2 + 9 = 0.
Solution Let x 2 = y so that the given equation takes the form
y 2 – 10y + 9 = 0
which on solving yields y = 1, 9. Therefore, the roots of the given equation
are ± 1 and ± 3.
q
2. Equation of the type px + =r
x
10 Mathematical Methods

If we multiply both sides of the above equation by x, we get


px 2 – rx + q = 0
which is a quadratic equation and can be solved by the methods mentioned
above.
4
Example 1.8 Solve the equation 2x + = 9.
x
Solution Multiplying the given equation by x, we get
2x 2 – 9x + 4 = 0
which is a quadratic equation having the roots as 1/2 and 4.
3. Equations of the type ax ± b = cx + d
Squaring both sides of the equation, we get
(ax ± b) = (cx + d )2
which on simplification leads to a quadratic equation.
Example 1.9 Solve the equation 217 − x = x – 7.
Solution Squaring both sides of the given equation, after simplification,
we get
xx 2 – 13x – 168 = 0
which is a quadratic equation having the roots as x = – 8, 21.
4. Equations of the type ax + b + cx + d = e
or ax + b − cx + d = f

Consider the equation ax + b + cx + d = e which can be written as


ax + b = e − cx + d . This equation on squaring leads to a quadratic
equation.
Example 1.10 Solve the equation 4 x − 3 + 2 x + 3 = 6.
Solution The given equation can be written as 4 x − 3 = 6 − 2 x + 3.
Now, squaring the both sides of this equation, we get, after simplification,
x 2 – 114x + 333 = 0 which is a quadratic equation whose roots are
3 and 111.
⎛ 1⎞
5. Equations of the type a ⎛⎜ x 2 + ⎞⎟ + b⎜ x +
1
⎟ + c = 0, x ≠ 0
⎝ 2
x ⎠ ⎝ x⎠
2 2
Since ⎛⎜ x + 1 ⎞⎟ = x 2 + 1 + 2 ⇒ x 2 + 1 = ⎛⎜ x + 1 ⎞⎟ – 2. Substituting
⎝ x⎠ x2 x2 ⎝ x⎠
Quadratic Equations and Complex Numbers 11

⎪⎧⎛ ⎪⎫
2
1⎞ ⎛ 1⎞
it in the given equation, we get a ⎨⎜ x + ⎟ − 2⎬ + b ⎜ x + ⎟ + c = 0.
⎩⎪⎝ x⎠ ⎭⎪ ⎝ x⎠
1
Take y = x + in this equation, we get a( y 2 – 2) + by + c = 0 which is
x
a quadratic equation in y. Let α and β be the roots of this equation, then
1 1
y=x+ = α and y = x + =β
x x
or
x 2 + 1 = α x and x 2 + 1 = βx
or
x 2 – α x + 1 = 0 and x 2 – βx + 1 = 0
which are quadratic equations and can be solved for x.
⎛ 1⎞ ⎛ 1⎞
Example 1.11 Solve the equation 2 ⎜ x 2 + 2 ⎟ − 3⎜ x + ⎟ – 1 = 0, x ≠ 0.
⎝ x ⎠ ⎝ x⎠
2
= y so that ⎛⎜ x + ⎞⎟ = y 2 ⇒ x 2 + 2 + 2 = y 2
1 1 1
Solution Put x +
x ⎝ x⎠ x
1
⇒ x 2 + 2 = y 2 – 2 and the given equation now reduces to 2y 2 –
x
3y – 5 = 0 which is a quadratic equation having the roots y = –1 and
1 1
y = 5/2. Thus x + = – 1 and x + = 5/2 which lead to the quadratic
x x
equations x 2 + x + 1 = 0 and 2x 2 – 5x + 2 = 0. It can be shown that the
former quadratic equation has no real roots, while the later has the
roots as 2 and 1/2. Therefore, the real roots of the given equation are 2
and 1/2.

6. Equations of the type a ⎛⎜ x 2 + ⎞⎟ + b ⎛⎜ x − ⎞⎟ + c = 0, x ≠ 0


1 1
⎝ 2
x ⎠ ⎝ x⎠
1
Take y = x – in the given equation to get a ( y 2 + 2) + by + c = 0 which
x
is a quadratic equation in y. Solve it for y and substitute these values of
y in equation y = x – 1
x
to get a quadratic equation in x.

⎛ 1⎞ ⎛
Example 1.12 Solve the equation 6⎜ x 2 + ⎟ − 25⎜ x −
1⎞
⎟ + 12 = 0,
⎝ 2
x ⎠ ⎝ x⎠
x ≠ 0.
12 Mathematical Methods

1 1
Solution Put x – = y so that x 2 + 2 = y 2 + 2 and the given equation
x x
now reduces to 6( y 2 + 2) – 25y + 12 = 0 which is a quadratic
1
equation having the roots y = 3/2 and y = 8/3. Thus x – = 3/2 and
x
1
x– = 8/3 which lead to the quadratic equations 2x 2 – 3x – 2 = 0 and
x
3x 2 – 8x – 3 = 0 having the roots as –1/2, 2 and –1/3, 3.

1.4 Complex Numbers


Since the begining of the human civilization, the mankind is familiar
with the counting process and the numbers used for such purpose are
known as Natural numbers. We denote these numbers by ¥. Thus the set
of natural numbers is given by ¥ = {1, 2, 3, ...}. The solution of the
equation x + a = b for b = a and b < a, where a and b are natural numbers
leads to an extension in the set of natural numbers ¥. This new set of
numbers is known as the set of Integers. We denote this set by ¢, thus
¢ = {0, ±1, ±2, ±3, ...}. If we have an equation of the form ax = b, where
a ≠ 0, a, b are integers, then the solution of this equation does not belong
to the set of integers and we thus have another number system, known as
the Rational numbers. This system is denoted by ¤ and we have
¤ = {x | x = b/a, a ≠ 0, a and b are integers}. Rational numbers can be
expressed in decimals such that they either terminate or may repeat
themselves after some digit (or digits). For example, 3/4 = .75, 1/6 =
0.1666..., 24591/99000 = 0.2483939..., etc. Numbers which are not
rationals are known as Irrational numbers. We denote this set by I. For
example, 2, 3, π, e are irrational numbers. If we take the union of the
set of rational and irrational numbers, we get a new set of numbers,
known as the set of Real numbers. This set is denoted by ¡. If we represent
the real numbers on a line, then such line is known as number line or
real line.
Moreover, if we have an equation of the form x 2 + 1 = 0, then the
solution of this equation leads to x = −1 which does not belong to the
set of real numbers. Thus for the existence of the solutions of the equations
of the form x 2 + 1 = 0, we need another number system. The set of these
numbers is known as the set of Complex numbers. We denote this set by
£ and we have
Definition 1.4 A number of the form z = x + iy, where x, y are real
numbers and i = −1, is calld a complex number.
Quadratic Equations and Complex Numbers 13

For example, z = 3 + 4i, z = 45 + 27i, z = 10 – 5i, z = 0 – 3i, z = 0


+ 56i, z = 24 + 0, z = 37 – 0, etc. are complex numbers.
It may be noted that i 2 = –1, i 3 = –i, i 4 = 1, i 5 = i, ... etc. If z = x + iy
is a complex number, then x is the real part of z and y is the imaginary
part of z and is written as x = Re(z) and y = Im(z), respectively. Thus, if
z = 3 + 10i is a complex number, then Re(z) = 3 and Im(z) = 10. Since a
real number x can be written as x + 0i, every real number is a complex
number and thus ¡ ⊂ £ where £ = {z = x + iy | x, y are real numbers}.

1.5 Algebra of Complex Numbers


This section deals with the addition, subtraction, multiplication and
division of complex numbers and related results. We have
Definition 1.5 Two complex numbers are equal if and only if their real
and imaginary parts are equal.
Thus, if z1 = x1 + iy1 and z2 = x2 + iy2 are two complex numbers, then
z1 = z2 if x1 = x2 and y1 = y2 and conversely. Therefore, z1 = z2 if and only
if Re(z1) = Re(z2) and Im(z1) = Im(z2).
Definition 1.6 If z1 = x1 + iy1 and z2 = x2 + iy2 are two complex numbers,
then the addition of z1 and z2 is denoted by z1 + z2 and is defined as
z1 + z2 = (x1 + iy1) + (x2 + iy2) = (x1 + x2) + i( y1 + y2)
Properties of addition of complex numbers
(1) Addition is commutative. For any two complex numbers z1 and z2,
z1 + z2 = z2 + z1
(2) Addition is associative. For any three complex numbers z1, z2 and
z3,
(z1 + z2) + z3 = z1 + (z2 + z3)
(3) Additive identity. The complex number 0 = 0 + i0 is the identity
element for addition, i.e., z + 0 = 0 + z = z.
(4) Additive inverse. For any complex number z = x + iy there exists a
complex number –z = – (x + iy) such that z + (–z) = 0 = –z + z.
Then –z is called the inverse for addition.
Definition 1.7 If z1 = x1 + iy1 and z2 = x2 + iy2 are two complex numbers,
then the subtraction of z2 from z1 is denoted by z1 – z2 and is defined as
z1 – z2 = z1 + (–z2) = (x1 + iy1) + (–x2 – iy2) = (x1 – x2) + i( y1 – y2)
Definition 1.8 If z1 = x1 + iy1 and z2 = x2 + iy2 are two complex numbers,
then the multiplication of z1 and z2 is denoted by z1z2 and is defined as
z1z2 = (x1 + iy1) (x2 + iy2)
14 Mathematical Methods

= x1 x2 + ix1 y2 + ix2 y1 + i 2y1 y2


= (x1 x2 – y1 y2) + i(x1 y2 + x2 y1)
Properties of multiplication of complex numbers
(1) Multiplication is commutative. For any two complex numbers z1
and z2,
z1z2 = z2z1
(2) Multiplication is associative. For any three complex numbers z1,
z2 and z3,
(z1z2) z3 = z1(z2z3)
(3) Multiplicative identity. The complex number 1 = 1 + i0 is the
identity element for multiplication, i.e., z.1 = 1.z = z.
(4) Multiplicative inverse. For any complex number z = x + iy there
exists a complex number z1 = (x1 + iy1) such that zz1 = 1 = z1z.
Then, z1 is the inverse for multiplication
(5) Multiplication is distributive. For any three complex numbers
z1, z2 and z3
z1(z2 + z3) = z1z2 + z1z3 and (z1 + z2) z3 = z1z3 + z2 z3
Definition 1.9 If z1 = x1 + iy1 and z2 = x2 + iy2 are two complex numbers,
then the division of z1 by z2 is denoted by z1/z2 and is defined as
z1 x1 + iy1
=
z2 x2 + iy2

x1 + iy1 x2 − iy2
= ⋅
x2 + iy2 x2 − iy2

⎛ x x + y1 y2 ⎞ ⎛ x2 y1 − x1 y2 ⎞
= ⎜ 1 2 +i
⎜ x 2 + y 2 ⎟⎟ ⎜⎜ x 2 + y 2 ⎟⎟
⎝ 2 2 ⎠ ⎝ 2 2 ⎠

Definition 1.10 If z = x + iy is a complex number, then z = x – iy is called


the conjugate of z. We shall denote it by z .
For example, if z = 5 + 6i is a complex number, then z = 5 – 6i is
the conjugate of z.
Properties
If z is a complex number and z is the conjugate of z, then
(1) z = z
(2) z + z = 2Re(z).
Quadratic Equations and Complex Numbers 15

(3) z – z = 2Im(z).
(4) z = z if and only if z is purely real.
(5) z = – z if and only if z is purely imaginary.
(6) z z = [Re(z)]2 + [Im(z)]2.

(7) z1 ± z2 = z1 ± z2

(8) z1 z2 = z1 z2

⎛ z1 ⎞ z1
(9)
⎜⎝ z ⎟⎠ = z , z2 ≠ 0
2 2

Definition 1.11 The modulus of a complex number z = x + iy is defined


as

|z | = x 2 + y 2 , | z | ≥ 0, for every complex number z


It may be noted that in a set of complex numbers, the order relation
is not defined, i.e., z1 > z2 or z1 < z2 are not defined but | z1 | > | z2 |
or | z1 | < | z2 | are defined as | z1 | and | z2 | are real numbers.
Properties
If z, z1 and z2 are complex numbers and z is the conjugate of z, then
(1) | z | = 0 if and only if z = 0, i.e., Re (z) = Im(z) = 0.
(2) | z | = | z | = | – z |
(3) – | z | ≤ Re(z) ≤ | z |, – | z | ≤ Im(z) ≤ | z |
(4) z z = | z |2
(5) | z1z2 | = | z1 | | z2 |

z1 |z |
(6) = 1 , z2 ≠ 0
z2 | z2 |

(7) | z1 ± z2 |2 = | z1 |2 + | z2 |2 ± 2Re( z1 z2 )
(8) | z1 + z2 |2 + | z1 – z2 |2 = 2(| z1 |2 + | z2 |2)

Re(z ) ⎛ Im( z )⎞ z
(9) + i⎜ − 2 ⎟
=
|z|2
⎝ |z| ⎠ |z|
2
16 Mathematical Methods

1.6 Cube Roots of Unity


Let z = 11/3. Then z3 = 1 ⇒ z3 – 1 = 0 ⇒ (z – 1) (z2 + z + 1) = 0 which
gives z = 1 and z2 + z + 1 = 0. This latter equation is a quadratic equation
−1 ± −3 −1 ± i 3
and the roots are = . Thus the roots of the (complex)
2 2
−1 + i 3 −1 − i 3
equation z 3 – 1 = 0 are 1, and . These are known as
2 2
the cube roots of unity. Here one root is real while the other two are
complex.
Remarks

(1) One of the cube roots of unity is 1 and the other two −1 + i 3 and
2
−1 − i 3
are complex conjugate of each other.
2
(2) Each complex cube root of the unity is the square of the other.
−1 + i 3
(3) The cube roots of unity are denoted as 1, ω = and
2
−1 − i 3
ω2 = .
2
(4) The sum of the three cube roots of unity is zero, i.e., 1 + ω +
ω2 = 0.
(5) The product of the three cube roots of unity is one, i.e.,
1⋅ ω ⋅ ω2 = 1.
(6) Each complex cube root of the unity is the reciprocal of the other.
(7) The equation x 2 + x + 1 = 0 has the roots ω and ω2; and the
equation x 2 – x + 1 = 0 has the roots – ω and – ω2.
(8) We can also find the cube roots of any real number. Thus, if a is
a positive real number, then the cube roots of a have the values
a1/3, a1/3 ω, a1/3 ω2 and, if a is a negative real number, then the cube
roots of a have the values – | a |1/3, – | a |1/3ω, – | a |1/3ω2.

1.7 Representation of a Complex Number


A complex number can be represented in the following forms:
(a) Geometrical form : For every complex number z = x + iy there exists
a unique point (x, y) in the plane and conversely. If z = x + iy is purely
real then it is represented on the x-axis and if z = x + iy is purely imaginary
Quadratic Equations and Complex Numbers 17

then it is represented by the points on the y-axis. That is why x-axis is


known as real axis and y-axis as the imaginary axis. The plane in which
a complex number is represented is called the complex plane, or Argand
plane. The point P(x, y), plotted in the Argand plane, is called Argand
diagram (see Fig. 1.1).
The length of the line segment OP is the modulus of z and from
∆OPQ, we have OP 2 = PQ 2 + OQ 2 which gives

| z | = OP = PQ 2 + OQ 2 = x 2 + y 2 = [Re(z )]2 + [Im(z )]2


The angle θ which OP makes with the positive direction of x-axis in
anticlockwise direction is called the argument or amplitude of z and is
denoted by arg (z) or amp (z). From Fig. 1.1, we have
y
P(x, y)

θ Q
x
O
Fig. 1.1 Argand diagram
PQ y Im( z )
tan θ = = =
OQ x Re(z)
and thus
⎡ Im( z ) ⎤
θ = tan–1 ⎢ ⎥
⎣ Re( z ) ⎦
This equation shows that there are infinitely many values (differing
by multiples of 2π) of the angle θ. The unique value of θ such that
–π < θ < π is called the principal argument.
Method for finding argument of z = x + iy
To know the argument of a complex number z = x + iy, we proceed as
follows:
y
(1) Find the value of tan −1 lying between 0 and 2π. Let it be α.
x
18 Mathematical Methods

(2) Find the position of the point P(x, y), i.e. determine in which
quadrant P lies and
(i) If P(x, y) lies in the first quadrant, then arg(z) = α.
(ii) If P(x, y) lies in the second quadrant, then arg (z) = π – α.
(iii) If P(x, y) lies in the third quadrant, then arg(z) = α – π.
(iv) If P(x, y) lies in the fourth quadrant, then arg(z) = – α.
Example 1.13 Find the modulus and argument of the complex numbers
1 + i 3, − 2 + i 2 3, − 3 − i, 2 3 − 2i.

Solution Consider z = 1 + i 3. Here x = 1, y = 3 and thus

|z|= –1 y = tan–1
x 2 + y 2 = 2, α = tan 3 = π/3. But x > 0, y > 0.
x
Therefore arg(z) = π/3. Similarly, for z = –2 + i 2 3, arg(z) = 2π/3 and
| z | = 4, while for z = – 3 – i, arg(z) = –5π/6 and | z | = 2, and for
z = 2 3 – 2i, arg(z) = –π/6 and | z | = 4.
(b) Vector representation of a complex number: A complex number
z = x + iy can be represented by the position vector OP of the point
P (see Fig. 1.1) in a two dimensional plane as the complex number
depends upon the modulus and argument.
(c) Polar form : Let z = x + iy be a complex number and θ be the angle
which OP makes with the x-axis (Fig. 1.1), i.e. ∠POQ = θ. Then from
∆OPQ, we have OP = | z | = x 2 + y 2 , ∠POQ = θ = arg(z) and
PQ y y
sin θ = = =
OP x +y
2 2 |z|
so that
y = | z | sin θ
and
OQ x x
cos θ = = =
OP x +y
2 2 |z|
so that
x = | z | cos θ
Thus
z = x + iy = | z | cos θ + i| z | sin θ
Quadratic Equations and Complex Numbers 19

Hence
z = | z |(cos θ + i sin θ) = r(cos θ + i sin θ)
which is the polar representation of the complex number z where
|z|=r= x2 + y 2 is the modulus of z and θ = arg(z).
The identity
eiθ = cos θ + i sin θ
is known as Euler’s formula and thus
z = r (cos θ + i sin θ) = reiθ
For the general value of the argument of θ, the polar form of the
complex number z is
z = r[cos(2nπ + θ) + i sin(2nπ + θ)]
where r = | z | and θ = arg(z), n ∈ ¢.
Example 1.14 Find the modulii and arguments of the cube roots of unity
and express the roots of the unity in polar form.
−1 + i 3
Solution We know that the cube roots of unity are 1, ω =
2
−1 − i 3
and ω2 = . Consider z = 1 = 1 + 0i ⇒ | z | = r = 1 and
2
tan θ = | y/x | = 0 ⇒ θ = 0 and as x > 0, y = 0, the complex number
z = 1 + 0i lies on the x-axis and arg(z) = 0. Thus the polar form of z = 1 is
z = r(cos θ + i sin θ) ⇒ z = 1(cos 0 + i sin 0) = 1
−1 + i 3
Similarly, when z = ω = , | z | = 1 and tan θ = – 3 ⇒ θ
2
= 2π/3, and thus arg(z) = 2π/3. Therefore, the polar form of z =
−1 + i 3
ω= is
2
z = r (cos θ + i sin θ) ⇒ z = 1[cos (2π/3) + i sin (2π/3)]
−1 − i 3
While for z = ω2 = , the polar form is
2
z = r(cos θ + i sin θ) ⇒ z = [cos(4π/3) + i sin(4π/3)]

1.8 De Moivre’s Theorem


(i) If n ∈ ¢ (set of integers), then
(cos θ + i sin θ)n = (cos n θ + i sin n θ)
(ii) If n ∈ ¤ (set of rational numbers), then (cos n θ + i sin n θ) is one
of the values of (cos θ + i sin θ)n.
20 Mathematical Methods

Remarks
(1) (cos θ + i sin θ)n = (cos n θ + i sin n θ)

1
(2) = (cos θ + i sin θ)–1 = cos θ – i sin θ
cos θ + i sin θ
(3) (sin θ ± i cos θ)n ≠ (sin n θ ± i cos n θ)
n
(4) (sin θ + i cos θ)n = ⎡cos ⎜⎛ π − θ ⎟⎞ + i sin ⎜⎛ π − θ ⎟⎞⎤
⎢ ⎥
⎣ ⎝2 ⎠ ⎝2 ⎠⎦

⎛ π π ⎞
= ⎜ cos n ⎜⎛ − θ ⎟⎞ + i sin n ⎜⎛ − θ ⎟⎞ ⎟
⎝ ⎝ 2 ⎠ ⎝ 2 ⎠⎠
(5) (cos θ + i sin φ)n ≠ (cos n θ + i sin n φ)

EXERCISES
Solve the following quadratic equations using the general expressions
for the roots of a quadratic equation:
1. x 2 – 8x – 48 = 0 [Ans. 12, – 4]
2
2. 5x – 19x + 12 = 0 [Ans. 3, 4/5]
2
3. 25x – 30x + 9 = 0 [Ans. 3/5, 3/5]
4. 16x 2 = 24x + 1 [Ans. (3 ± 10 )/4]
Solve the following equations by reducing them into quadratic equations:

5. 4x 4 – 33x 2 + 8 = 0 [Ans. ± 2 2, ± 1/2]


6. (x 2
+ 3x + 2)2
– 8(x 2
+ 3x) – 4 = 0 [Ans. – 4, – 3, 0, 1]
2 2 2
7. (x – 5x) – 30(x – 5x) – 216 = 0 [Ans. 9, – 4, 2, 3]

x 1 − x 13
8. + = [Ans. 9/13, 4/13]
1− x x 6

9. x + 5 + x + 21 = 6 x + 40 [Ans. 4]
10. 5x+1 + 52–x = 53 + 1 [Ans. 2, – 1]
2 2
−3 −3
11. (5 + 2 6) x + (5 − 2 6) x = 10 [Ans. ±2, ± 2]

a+x+ a−x a
12. = [Ans. ±a]
a+x− a−x x
Quadratic Equations and Complex Numbers 21

13. Evaluate 6 + 6 + 6 + ...∞ [Ans. 3]


14. For what value of m will the equation x 2 – 2(1 + 3m) x + 7(3 + 2m)
= 0 have equal roots? [Ans. 2 or –10/9]
2
15. If the roots of the equation (b – c)x + (c – a)x + (a – b) = 0 are
equal, then prove that 2b = a + c.
16. Form a quadratic equation whose one root is 3 + 2.
[Ans. x2 – 6x + 7 = 0]
17. If α, β are the roots of the equation 2x2
– 3x – 6 = 0, find the
equation whose roots are α2 + 2 and β2 + 2.
[Ans. 4x 2 – 49x + 118 = 0]
18. Perform each of the indicated operations.
(a) (3 + 2i) + (–7 – i) and (–7 – i) + (3 + 2i) [Ans. – 4 + i]
(b) (5 + 3i) + {(–1 + 2i) + (7 – 5i)} and {(5 + 3i) + (–1 + 2i)} +
(7 – 5i) [Ans. 11]
(c) (2 – 3i) (4 + 2i) and (4 + 2i) (2 – 3i) [Ans. 14 – 8i]
(d) (2 – i) {(–3 + 2i) (5 – 4i)} and {(2 – i) (–3 + 2i)} (5 – 4i)
[Ans. 8 + 51i]
(e) (–1 + 2i){(7 – 5i) + (–3 + 4i)} [Ans. –2 + 9i]

19. If z1 = 2 + i, z2 = 3 – 2i and z3 = 1
2
+ 2
3
i, then evaluate each of the
following:
(a) | 3z1 – 4z2 | [Ans. 157 ]
(b) z13 − 3z12 + 4z1 – 8 [Ans. –7 + 3i]

⎡ 1 3 ⎤
(c) ( z3 ) 4 ⎢Ans. − − i⎥
⎣ 2 2 ⎦
2
2 z2 + z1 − 5 − i
(d) [Ans. 1]
2 z1 − z2 + 3 − i

20. If z1 = 1 – i and z2 = –2 + 4i, find Im ( )


z1 z2
z1
[Ans. 2]

21. Find real values of x and y such that 3x + 2iy – ix + 5y = 7 + 5i.


[Ans. x = –1, y = 2]
22 Mathematical Methods

22. Find real values of x and y for which the complex numbers
–3 + ix2y and x2 + y + 4i are conjugate of each other.
23. Find the square root of –15 – 8i and i.

[Ans. ±(1 – 4i), ± 1


2
(1 + i)]
24. If 1, ω, ω2 are three cubes roots of unity, show that
(a) (1 – ω + ω2) (1 + ω – ω2) = 4
(b) (1 – ω + ω2)3 = (1 + ω – ω2)3 = 8
(c) (1 + ω) (1 + ω2) (1 + ω4) (1 + ω8) = 1
25. Solve the equation (x – 1)3 + 8 = 0. [Ans. –1, 1 – 2ω, 1 – 2ω2]
26. Write the polar form of the complex numbers −3 2 + 3 2i,1 + i,
1+ i
– 1 – i, 1 – i, 1− i
, 1, ω and ω2.

[Ans. 6(cos(3π/4) + i sin (3π/4)), 2 (cos(π/4) + i sin(π/4)),


2 (cos (3π/4) – i sin (3π/4)), 2 (cos(π/4) – i sin(π/4)),
(cos(π/2) + i sin(π/2)), (cos 0 + i sin 0), (cos(2π/3)
+ i sin(2π/3)), (cos(4π/3) + i sin(4π/3))]
27. Show that

(cos θ + i sin θ)5


(a) = cos 11θ + i sin 11θ
(cos θ − i sin θ)6
(b) (cos θ + i sin θ)4 (cos θ – i sin θ)2 = cos 2θ + i sin 2θ

(cos3θ + i sin 3θ)4 (cos4θ − i sin4θ)5


(c) =1
(cos4θ + i sin 4θ)3 (cos5θ + i sin5θ)− 4
CHAPTER 2

Functions, Limit and


Continuity

2.1 Introduction
In daily life we often come across with objects which are paired together,
for example, a person has a name, a cricket player has a number on his
uniform, a student has a grade, etc. When we observe a group of people
in sunlight, we observe that each person has exactly one shadow. With
each biological species there is associated a typical number of
chromosomes. Each person has ten finger prints and each finger print is
associated with exactly one person. In science, engineering and business
also, a number is associated with another, e.g. the amount of a radioactive
substance present determines the decay rate; the interest paid on an
investment depends upon the duration (time) of investment. In all these
situations it may be noted that there exist a unique association between
the quantities involved. Such pairing of one object with the other leads
to the concept of a function.
This chapter deals with the concept of real functions, their limit and
continuity and related results. We first begin with real functions.

2.2 Functions
Functions are useful in describing many situations involving two variables
because each value of one variable corresponds to only one value of the
other variable. Figure 2.1 shows the speed of the car over elapsed time in
seconds as the car accelerates from rest, e.g., Fig. 2.1 shows that 5 s after
starting, the car is going 45 miles/h. Fig. 2.2 shows the variation of
blood pressure of a person (systolic and diastolic pressures are the upper
and lower limits is the periodic changes in pressure that produce the
pulse. The length between the peaks is called the period of the pulse).
After 0.8 s, the blood pressure is same as its starting value, 80 mm.
Figures 2.1 and 2.2 illustrate functions, which are rules or proceedures
24 Mathematical Methods

that yield just one value of one variable from any given value of another
variable. We can find the exact speed of the car after 10 s because the car
can not have two speeds at the same time. Similarly, a person can have
only one blood pressure at a given instant.

100
90 3rd–4th
80
Speed in miles per hour

70 2nd–4th
60
1st–2nd
50
40
30
20
10

0 5 10 15 20 25 30 35
Elapsed time in seconds

Fig. 2.1 Speed of a car

Systolic Period = 0.8 sec


pressure
Pressure in mm mercury

120

80
Diastolic
pressure
40

0 0.8 1.6
Time in seconds

Fig. 2.2 Variation of blood pressure


The key to the mathematical analysis of a geometric or scientific
situation is the recognition of relationship between the variables that
describe the situation. Such a relationship may be a formula that expresses
Functions, Limit and Continuity 25

one variable as the function of the other. For example, the area A of a
circle of radius r is A = πr 2. The volume V and surface area S of a sphere
of radius r are given by V = 4
3 πr 3 and S = 4πr 2. The relationship between
the variables can also be described with the help of the following examples.
Examples
1. The cost of leasing an office space in a building depends upon the
number of square feet of space in the office. An association between
the area of an office in sq ft and monthly rent in rupees are given
in Fig. 2.3.
2. Another association may be set up between investment in a mutual
fund and the corresponding earning, assuming an annual return
of 12% (Fig. 2.4). In this example we can use a formula to show
how the numbers in the set X are used to obtain the numbers in the
set Y. If x is a rupee amount from the set X, then the corresponding
annual return y in rupee from the set Y can be found with the
formula y = (.12)x. Here x, the investment in the mutual fund, is
called the independent variable. As the annual return depends on
the amount of investment, y is called the dependent variable. When
a specific number, say 2000 is substituted for x then y takes one
specific value and here y = 240. The variable y is said to be a
function of x.

Square feet Rent (Rs) Investment Annual return

1000 1800 50000 6000

1400 2520 7200 864

350 630 14600 1752

680 1240 89250 10722

Fig. 2.3 Office space and rent Fig. 2.4 Investment and earning
The pair of numbers, one for x and one for y, can be written as an ordered
pair (x, y), where the order of the numbers is important. Using ordered
pairs, the information shown in Fig. 2.4 can be expressed as a set {(50000,
6000), (7200, 864), (14600, 1752), (89350, 10722)}. The set of points in
a plane that corresponds to the ordered pairs of a function is called the
graph of the function. Fig. 2.5 illustrates the graph of y = 0.12 x.
26 Mathematical Methods

Fig. 2.5 Graph of y = 0.12x


We thus have
Definition 2.1 A function f is a rule that assigns to each element x of a
set X exactly one element y from a set Y. We denote it as f : X → Y and
read as f is a function from X to Y and we write as y = f (x).
Examples
1. In Fig. 2.6 an x - value of 11 corresponds to two y - values 8 and
10. In a function, each x must correspond to exactly one y and so
this correspondance is not a function.

16 6

18 12
13 8
9
11 10

Fig. 2.6 Correspondence between two sets


2. The optical reader at the check out counter in many big
departmental stores converts codes to prices. For each code, the
reader produces exactly one price and thus this is a function.
3. The x 2 key on a calculator: The calculator produces one x 2 value
for each value of x entered and this correspondance between the
input and output is a function.
4. The set of ordered pairs with first element mothers and second
element their children: Here the mother is an independent variable
Functions, Limit and Continuity 27

and the child is the dependent variable. For a given mother there
may be several children, and thus this correspondance is not a
function.
5. The set of ordered pairs with first element children and second
element their birth mother: Here the child is an independent
variable and the mother is the dependent variable. Since each child
has only one birth mother, so this is a function.
As we have seen, a function is a correspondance between the elements
of two sets. These sets are given special names and we have
Definition 2.2 The set of all possible values of the independent variable
in a function is called the domain of the function, and the set of all
possible values for the dependent variable is called the range.
It may be noted that the domain and range of a real function are real
numbers.
A function is often defined by an equation. We shall now find which
of the following equations represent a function.
(i) y = – 4x + 11. Here for each value of x there is exactly one value of
y, e.g., for x = 1, y = 7, etc. and thus y is a function of x.
(ii) y 2 = f (x) = x. If x = 36 then y = ± 6. Here for one value of the
independent variable x there are two values of the dependent
variable. Thus y 2 = x does not represent a function.
(iii) (a) If x + y = 1 then y = 1 – x. Here for each x there is one y and
thus this is a function.
(b) If x 2 + y 2 = 1 then y = ± 1 − x 2 . Here for each x there are two
values of y and thus this is not a function.
(c) If x 2 + y = 1 then y = 1 – x 2. Here for each x there is one y and
thus this is a function.
(d) If x + y 2 = 1 then y = ± 1 − x . Here for each x there are two y
and thus this is not a function.
The graphs of these equations are illustrated in Fig. 2.7.
If a graph is to represent a function, each value of x from the domain
must lead to exactly one value of y. In the graph of Fig. 2.7(b) and (d),
since the domain value x leads to two values of y, hence these are not the
graph of the function. These examples suggest the vertical line test for
the graph of a function.
28 Mathematical Methods

y y
2 2

1 x+y=1 x2 + y2 = 1

x x
–2 –1 1 2 –2 2
–1
–2 –2

(a ) (b )

y y
2 2
1 x2 + y = 1 1 x + y2 = 1
x x
–2 –1 1 2 –2 –1 1 2
–1 –1
–2 –2

(c) (d)

Fig. 2.7 Graph of functions


Vertical Line Test. If a vertical line intersects a graph in more than one
point, the graph is not the graph of the function.
To understand how a function works, assume a function as a machine
that takes an input x from the domain and uses it to produce an output
f (x) (see Fig. 2.8).

Domain
x x f
f (x )

Fig. 2.8 Fuction as a machine


One such machine is the familiar pocket calculator with a square
root key on it. When a non-negative number x is entered and the key is
pressed, the calculator display the number x .
Not every function has a rule expressible as a simple one-part formula
such as f (x) = x , e.g.,
Functions, Limit and Continuity 29


f (x) = ⎨ x − 1 if x ≥ 1
⎩ 1 − x, if x < 1
The graph of this function is illustrated in Fig. 2.9.

2
Range: y ≥ 0

1 2 3 4

Domain: all real x

Fig. 2.9 Graph of the function f (x)


We now define some of the frequently occuring functions as follows:
Definition 2.3 The function defined by
f (x) = c
where c is a constant is called a constant function.
Here, domain = (– ∞, ∞) and range = c, where (a, b) = {x | a < x
< b}, [a, b] = {x | a ≤ x ≤ b}, [a, b) = {x | a ≤ x < b} and (a, b] = {x | a <
x ≤ b}, are respectively, known as open, closed, semi-open and semi-
closed intervals.
Definition 2.4 A function is linear if it can be expressed as
f (x) = ax + b
where a and b are real numbers.
The example of linear function is Charle’s law (1787) which states
that when a gas is heated at a constant pressure P, the relationship between
the tempeature T and volume V is linear, i.e. V = kT.
Definition 2.5 The quadratic function is defined by
f (x) = ax 2 + bx + c
where a, b, c are real numbers and a ≠ 0.
The graph of a quadratic function represents a parabola. Lenses in
telescopes, reflectors in car headlights are examples of a parabola.
Definition 2.6 A polynomial function (of degree n), where n is non-
negative, is defined by
f (x) = an x n + an–1 xn–1 + ... + a1x + a0
30 Mathematical Methods

where an, an–1, ..., a1, a0 are real numbers and an ≠ 0.


Definition 2.7 A power function is defined by
f (x) = x n
where n is a real number.
Some special cases of power function are as follows:
For n = 0, f (x) = 1 (a constant function).
For n = 1, f (x) = x (an identity function).
For n = –1, f (x) = 1/x (a reciprocal function, or rectangular hyperbola).
For n = 1/2, f (x) = x (a square root function).
For n = 2, f (x) = x 2 (an upward parabola).
These special cases are depicted in Fig. 2.10.
y y y

f ( x) = 1

1 f ( x) = x

x x x
1
f ( x) =
x

y y

f ( x) = x 2
f ( x) = x

x x

Fig. 2.10 Special cases of power function


Definition 2.8 A function defined by
p( x)
f (x) =
q( x)
where p(x) and q(x) are the polynomial functions such that q(x) ≠ 0, is
called a rational function.
Since any values of x such that q(x) = 0 are excluded from the domain,
a rational function usually has a graph with one or more breaks. For
example, the graph of the rational function y = 2/(1 + x) is shown in
the Fig. 2.11
Functions, Limit and Continuity 31

y
x y
–4 –2/3
–2 –2
0 0
x = –1 2 2
2

x
–2 0 2

2
y=
1+x

Fig. 2.11 Graph of a rational function


Definition 2.9 The function defined by a two-step formula

{
x, if x ≥ 0
f (x) = − x, if x < 0

is called an absolute value function (Fig. 2.12).


y

x
0

Fig. 2.12 Graph of an absolute value function


Definition 2.10 The function defined by a three-step formula

⎧⎪ 1, if x > 0
f (x) = ⎨ 0, if x = 0
⎪⎩−1, if x < 0
is called signum function (Fig. 2.13).
Definition 2.11 The function
f (x) = ka x
32 Mathematical Methods

where k, a are constants, ≠ 0, a > 0 is called an exponential function.


The well known exponential function is f (x) = e x (Fig. 2.14).
y

Fig. 2.13 Signum function

Fig. 2.14 Exponential function


Definition 2.12 The function defined by
f (x) = loga x
is called the logarithmic function (Fig. 2.15).
y

Fig. 2.15 Logarithmic function


Definition 2.13 The function defined by
f (x) = [x]
where [x] denotes the greatest integer less than or equal to x, is called the
greatest integer function. The graph of this function is given in
Fig. 2.16.
Functions, Limit and Continuity 33

Fig. 2.16 Greatest integer function


The logarithmic and exponential functions are inverse of each
other. Thus, if the graph of y = 2x is known we can draw the graph of
y = log2 x. The logarithms were largely developed by the mathematicians
John Napier and Henry Briggs. Henry Briggs also constructed extensive
logarithmic tables.
The graph of exponential function is either increasing or decreasing.
When a real life problem is modelled by an increasing exponential
function, we then have exponential growth; and if the problem is modelled
by a decreasing exponential function then we have exponential decay.
Growth of population, either humans or bacteria is the example of
exponential growth; while the decay of a radioactive substance is the
example of an exponential decay (for more details, see [3]).
Applications of functions
Here we shall mention some of the applications of the functions, studied
so far, to real life problems and we have
(i) Brain Weight: The linear function occurs when we consider the
relationship between the brain weight and body weight of human beings.
If B denotes the weight of human brain and W, the weight of human
being then
B ∝ W ⇒ B = kW
Example: (a) It is known that a person who weighs 200 pounds has a
brain which weighs 5 pounds. Obtain an equation in B and W. Find the
constant and interpret the resulting equation.
(b) What is the weight of the person’s brain whose weight is 120 pounds?
Solution: (a) Here B = kW, B = 5, W = 200 so that k = B/W = 0.025 and
B = 0.025 W. Also B = 2.5% W. That is the weight of the brain is 2.5% of
the weight of the body.
(b) Here B = 0.025 W so that B = 0.025(120) = 3 pounds approx.
34 Mathematical Methods

(ii) Estimation of height of a male/female: Linear functions can be used


to estimate the height of a male/female if the length of a certain bone is
known.
If x is the length of humerus (a bone from elbow to shoulder) in cm,
then the height M(x) of a male is
M(x) = 2.89x + 70.64
and the height F(x) of female is
F(x) = 2.75x + 71.48
Thus, for example a 45 cm humerus was found in ruins
(a) if it was from a male, how tall was he? [using x = 45 cm in the above
equation, we get M(x) = 200.69 cm].
(b) if it was from a female, how tall was she? (using x = 45 cm in the
above equation, we get F(x) = 195.23 cm].
(iii) Track and field events: Linear functions can be used to predict/
achieve the world record in any race. In particular, for the mile run
R = – 0.00582x + 15.3476
where R is the world record in minutes, x, the year in which the record is
achieved.
Athelete Roger Bannister surprised the world by running a mile in
four minutes in 1954. The actual record was 3:59:40. Take x = 1954 in
the above equation to get R = 3 : 58 : 50 (3.97532 approximately). This
model has some limitations, for example, when one minute-mile record
be set (take R = 1 in the above equation, we get x = 2465). Most track
coaches assume that one minute-mile run is beyond human capabilities.
(iv) Daily protein intake: This phenomenon can be explained with the
help of the following example.
Example: It is known that 100 gram of soybean contains 35 gram of
protein and 100 gram of dried lentils contain 26 gram of protein. Men of
average size living in moderate climate need 70 gram of protein in their
daily food. Assuming a man wants to provide these 70 gram of protein
by eating soybean and/or lentils, find the relationship between the amounts
of soybean and lentils daily.
Solution: Let x and y denote the amount of soybean and lentils daily,
respectively (x and y are measured in the units of 100 gram). The protein
taken with soybean is 35x and with lentils is 26y a day. The total daily
amount of protein is 70 gm. Thus we have
35x + 26y = 70
which leads to the following linear relationship
Functions, Limit and Continuity 35

35 70
y=− x+
26 26
(v) Sensitivity to drugs: The sensitivity S to a drug is related to the dosage
x through the quadratic function
S = 1000x – x 2
[One can draw the graph, which is an inverted parabola; and the
maximum sensitivity can be obtained as (x, S) = (500, 250, 000).]
(vi) Photosynthesis: Photosynthesis is how plants use light and water to
make sugar. Sugar is created in the green parts of a plant and every
animal on earth depends on it. Without plants we would have no food to
eat or oxygen to breath. Plants absorb a common gas called carbon dioxide,
pull water up through their roots and use light to make sugar. Plants use
the sugar to grow. Plants give off oxygen as a by-product. The green
parts of the plant makes the sugar and oxygen. That is
Carbon dioxide + water + sunlight = sugar + oxygen
The chemical reaction is
6CO2 + 12H2O + Light ⎯→ C6H12O6 + 6O2 + 6H2O
where C6H12O6 represents glucose. Thus photosynthesis changes light
(sun) energy into food (chemical) energy.
The quadratic function can be used to study the phenomenon of
photosynthesis and it is seen that the rate of photosynthesis R of a certain
plant depends upon the intensity of light x according to the equation
R = 270x – 90x2
One can find the intensity that gives the maximum rate.
(From the above equation, R′ = 270 – 180x so that R′ = 0 leads to x =
1.5. Also R″ = – 180 which is negative, thus for x = 1.5, the maximum
rate, from above equation, is R = 270x – 90x2 = 202.5, for details about
maxima/minima, see Article 3.11.6).
(vii) Carton industry: Polynomial function of degree three can be used
in carton industry which can be seen from the following example.
Example: A square piece of cardboard with a side 18 feet is to be made
into a box without top by cutting a square from each corner and folding
up the flops to form the box. What should be the side of the square to be
cut off so that the volume of the box is maximum possible?
Solution: Let each side of the square to be cut off from each corner be of
length x feet, then length = breadth of the box = 18 – 2x and thus volume
V is
V = (18 – 2x) (18 – 2x) x = 324x – 72x 2 + 4x 3
36 Mathematical Methods

Here V ′ = 12(27 – 12x + x 2) so that V ′ = 0 leads to x = 3, 9. Also,


V ″ = – 144 + 24x < 0 for x = 3. Thus V = 432 is maximum for x = 3 feet
(for details about maxima/minima, see Article 3.11.6).
(vii) Compound Interest: The greatest integer function can be used for
calculating the earning on the investments through compound interest.
If ` 1000/- are invested at 12% interest compounded quarterly, the amount
in the account at the end of x-months for one year period is given by

⎧1000, if 0≤ x<3
⎪1030, if 3≤ x <6

f (x) = ⎨1061, if 6≤ x<9
⎪1093, if 9 ≤ x < 12
⎪⎩1126, if 12 = x

It is easy to notice that the graph of this function f (x) represents the
greatest integer function.
(ix) Automatic ventilation system: The most efficient building ventilation
is an issue that investor and developers are increasingly focusing on
today. Companies are now offering an efficient system solution for
decentralized ventilation-automatic ventilation with windows closed.
Such ventilation controls air exchange without opening the windows.
This optimizes energy consumption, the room climate and air quality.
The exponential function can well be used to deal with such problems.
In a building, the automatic ventilation system operates when the
concentration of CO2 reaches a certain level. Suppose that when the
ventilation system operates, the x cubic feet of CO2 in an 8000 cubic feet
room depends upon time t minutes according to the equation
x = 4.8 + 11.2e– t/4
Example: (a) Find the initial amount of CO2 in the room and concentration
of CO2 (as %) at this time.
(b) How long does it take to have the concentration of 0.07% CO2?
Solution: (a) Put t = 0 in the above equation, we get x = 16 cubic feet of
16 × 100
CO2 and thus percentage of CO2 = 8000
= 0.2.
(b) 0.07% concentration means 0.0007 (8000) = 5.6 cubic feet CO2 and
the above equation leads to 5.6 = 4.8 + 11.2e–t/4 so that t = 10.56 minutes.
(x) Marine life: In lakes and sea, plant life can only exist in the top layer
which is roughly 10 meter deep since daylight is gradually absorbed by
water. How does light intensity decreases with increasing thickness of
the layer? The answer is given by Bouguer-Lambert’s law [Pierre Bouguer
Functions, Limit and Continuity 37

(1698-1758), French scientist and explorer, studied the absorption of


light in atmosphere. Johann Heinrich Lambert (1728-1777), Alsatian
mathematician, astronomer and physicist, studied the law in general].
Consider a vertical beam entering the water with original intensity
I0. Let I be the reduced intensity in a depth of x meters, then the law
states that
I = I0e– µx
The parameter µ > 0 is called the absorption coefficient. It depends upon
the purity of water and the wave length of the beam. Strictly speaking,
the intensity I will never be exactly zero. However, for sufficiently large
x the remaining light can no longer be perceived. Table below
shows how the light intensity decreases in fairly clean sea water
(assuming µ = 1.4 m–1).

Depth(m) 0 1 2 3 4 5 6
Intensity of light (%) 100 25 6 1.5 0.4 0.1 0.02
The Bouguer-Lambert’s law is applicable to any homogeneous,
transparent substance such as glass, liquids and thin layer viewed under
microscope. In addition to light waves, other electromagnetic waves such
as X-rays, and γ-rays, behave the same way. The Bouguer-Lambert’s law
is basic in photometry.
(xi) Pollution in Lakes: Exponential function appears in the study of
such problems and the pollution level in a certain lake is given by the
equation
x = 0.05 + 0.18e– 0.38t
where x is the volume of the pollutants (cc km) and t, the time (years).
Example: (a) Find the initial pollution level. (b) How long before x is
30% of the initial level?
Solution: (a) Take t = 0 in the above equation, we get x = 0.23 (cc km).
(b) Now 30% of 0.23 = 0.069 = x. With this value of x, the above equation
becomes 0.069 = 0.05 + 0.18e– 0.38t and that t = 5.9 years.
Further applications of exponential function are discussed in
Chapter VI.
(xii) pH value: The pH (Hydrogen potential) value is a scale for measuring
acidic or basic nature of a solution. The concept of pH was first introduced
by Danish chemist Sren Lauritz Srensen at the Carlsberg Laboratory in
1909. pH value of a solution is defined as the negative logarithm of its
hydronium ion concentration [H3O+]. Thus
38 Mathematical Methods

pH ≡ –log10 [H3O+]
It is measured in mole/litre. The hydronium ion concentration is an
important factor in living tissues as well as in the soil where the plant
grows. pH measurements are important in medicine, biology, chemistry,
food science, environmental science, oceanography and many other
applications. Since [H3O+] = 1 × 10–7 mole/litre, the pH for pure water at
25°C (77°F) is close to 7.0. Pure water is said to be neutral, that is,
neither acidic nor basic. Solutions with a pH less than 7 are said to be
acidic and solutions with a pH greater than 7 are said to be basic or
alkaline. The solutions having pH values between 0 and 2 are strongly
acidic, between 2 and 4 are moderately acidic and between 4 and 7 are
weakly acidic; whereas solutions having pH values between 7 and 10 are
weakly basic, between 10 and 12 are moderately basic and more than 12
are strongly basic.
Most of the food we eat are acidic in nature, for example, the pH
value of vinegar and apple juice is 3.0 and 3.5, respectively; the pH
value of oranges, banana and spinach, respectively lies between 3.0-4.0,
4.5 - 4.7 and 5.1-5.7. On the other hand, the pH value of sea water, egg
and blood are 8.15, 7.8 and 7.4, respectively. Moreover, for human brain
it is found that cerebrospinal fluid has hydronium ion concentration as
4.8 × 10– 8 mole/litre. Thus, for cerebrospinal fluid, pH = – log[H3O+] =
–log(4.8 × 10– 8) = – (log 4.8 + log 10– 8) = – (0.68 – 8 log 10) = – (0.68
– 8) = 7.32.
(xiii) Brightness of a star: The experienced brightness of a star is by no
means proprotional to the light energy received by the eye. A linear
relationship between brightness and the logarithm of light intensity is
given by
m = c – 2.5 log I
where I is the light intensity, c is a constant determined by the unit in
which I is measured and m is the apparent magnitude of a star.
It may be noted that the stars of magnitude + 6 are visible to the
naked eye. The magnitude of brightness of some of the prominant stars,
such as Sirius, Betelgeuse and Vega are – 1.6, – 0.9 and 0.1, respectively;
while the Sun has the magnitude of brightness as –26.
(xiv) Noise Pollution: Noise health effects are both health and behavioural
in nature. The unwanted sound is called noise. This unwanted sound can
damage physiological and psychological health. The loudness of sound
(level of sound) is defined as
Functions, Limit and Continuity 39

I I
L = log (in bel) = 10 log (in decibel)
I0 I0
where I0 is the minimum intensity detectable by human ear (the tick of a
clock at a distance of 20 feet under very quiet conditions). At a frequency
of 1000 Hz (Hz = cycles/sec) the threshold of audiobility or lowest intensity
that can be heard is I0 = 10–12 watt/m2. Also, for I = I0, L = 0dB. When
one sound is 10 times as intensive as another, its loudness is 1 bel louder.
If one sound is 100 times as intensive as another, then it is louder by 2
bel and so on. This unit of sound is called bel (after Graham Bel-who
invented telephone), but it is too large; so we use a sub unit (1/10)th as
large, called decibel (dB). Subject to 45 dB of noise, the average person
cannot sleep. At 120 dB the ear registers pain, but hearing damage begins
at a much lower level, about 85 dB. Apart from hearing loss, such noise
can cause lack of sleep irritability, heartburn, indigestion, ulcers, high
blood pressure, and possible heart disease. One burst of noise, as from a
passing track, is known to alter endocrine, neurological and
cardiovascular functions in many individuals; prolonged or frequent
exposure to such noise tends to make the physiological disturbances
chronic. In addition, noise-induced stress creates severe tension in daily
living and contributes to mental illness.
Example: Find the loudness, in dB, of conversational speech, having an
intensity I = 106 I0 [using the given value of I in the above formula, we
get L = 60 dB].
(xv) Earthquake: An earthquake (also known as a tremor or temblor) is
the result of a sudden release of energy in the Earth’s crust that creates
seismic waves. Earthquakes are recorded with a seismometer and the
recorded message is called a seismograph. Minor earthquakes occur nearly
constantly around the world in places like California and Alaska in the
U.S., as well as in Guatemala, Chile, Peru, Indonesia, Iran, Pakistan,
the Azores in Portugal, Turkey, New Zealand, Greece, Italy, and Japan,
but earthquakes can occur almost anywhere, including New York City,
London, and Australia. Most of the world’s earthquakes (90%, and 81%
of the largest) take place in the 40,000-km-long, horseshoe-shaped zone
called the circum-Pacific seismic belt, also known as the Pacific Ring of
Fire, which for the most part bounds the Pacific Plate. Massive
earthquakes tend to occur along other plate boundaries, too, such as along
the Himalayan Mountains.
40 Mathematical Methods

The destructive power of earthquake is measured by a scale, known


as Richter Scale (named after American Geologist Charles Richter) and
is defined as
I
R = log
I0
where R is the Richter number, I0, the minimum intensity used for
comparison and I, the intensity of the earthquake.
When an earthquake is 10 times as intense as another, its magnitude
on Richter scale is 1 higher; when an earthquake is 100 times as intense
as another, its magnitude on Richter scale is 2 higher; and so on. Thus,
an earthquake of magnitude 7 on Richter scale is 10 times as intense as
an earthquake of magnitude 6.
Example: (a) Find R if I = 3,160,000I0. (b) For Oct. 08, 2005 (Jammu
and Kashmir) R = 7.8, find intensity.
I
Solution: (a) From equation R = log , we have
I0
R = log 3,160,000 = 6.5.
(b) From above equation, we have I = 107.8 I0 = 63,100,000I0.
Moreover, If E is the energy released, measured in joules, during an
earthquake then the magnitude of the earthquake is given by
2 E
M = log
3 E0
where E0 = 104.4 joules.
Problem: If 8 × 1014 joules of energy is released during the earth quake,
what was the magnitude of the earth quake? [Answer: Put the values in
the above formula, we get magnitude = 7].
Problem: How much energy will be released in an earthquake with a
magnitude of 5.9? [Answer: Using the values of M and E0, we get
E = 1013.25 joules].
(xvi) Depreciation of an Equipment: In general, the value of an equipment
(except for antiques) goes down during the passage of time. Depreciation
is term used in accounting, economics and finance to spread the cost of
an asset over the span of several years. In simple words we can say that
depreciation is the reduction in the value of an asset due to usage, passage
of time, wear and tear, technological outdating or obsolescence, depletion,
inadequacy, rot, rust, decay or other such factors.
If C is the original cost of an equipment, V(t) is its value after t years
and n is the life expectancy (in years) then
Functions, Limit and Continuity 41

t
⎛ 2⎞
V(t) = C ⎜1 − ⎟
⎝ n⎠
This equation is known as double-declining balance formula.
Definition 2.14 Trigonometric functions. The function that associates
the number sin x to each number x is called the sine function. Here x is
the radian measure of an angle. The value of sin x always lies between 1
and –1. Also, sin(x + 2π) = sin x and sin(x + π) = – sin x for all x in ¡.
Similarly, we have other trigonometric functions cos x, tan x, cot x,
sec x and csc x (Fig. 2.17). It may be noted that the domain of each of the
trigonometric function is not the same. For sine and cosine functions,
the domain is the set of real numbers, while for the function tan x, the
π
domain is ¡ – ⎧⎨(2n + 1) ⎫⎬, where n is an integer.
⎩ 2⎭
y y

1
1
π/2
x x
–2π –π 0 π 2π 2π
–1 –1 π
y = sin x one period y = cos x
one period

y y

– 3π/2 –π/2 π/2 3 π/2


–π 0 π x –2π –π 0 π/2 π 3 π/2 2π x

y = tan x y = cot x

y = cos x y = sin x
1
–π π –π 1
– 2π
– 3 π/2 π/2 2π x – 2π π/2 2π x
–π/2 –1 –1

y = sec x y = cosec x

Fig. 2.17 Graphs of trigonometric functions


42 Mathematical Methods

Definition 2.15 The inverse trigonometric functions are sin–1 x, cos–1 x,


tan–1 x, cot–1 x, sec–1 x, cosec–1 x. The domains and ranges of these
−π π
functions, respectively, are [–1, 1], 2
≤y≤ 2
; [–1, 1], 0 ≤ y ≤ π
;
2
−π π π
(– ∞, ∞), 2
≤y≤ 2
; (– ∞, ∞), (0, π); ¡ – (–1, 1), [0, π] – 2
and
¡ – (–1, 1), ⎡⎣ −π , π ⎤ – {0}. The graphs of these functions are depicted in
2 2⎦
the Fig. 2.18.
y y y

2π 2π
π

π π
π/2
x x x
–1 1 0
– π/2
–π –π

–1 –1 –1
y = sin x y = cos x y = tan x
(or, x = sin y) (or, x = cos y) (or, x = tan y)
y y y

3π/2 2π 2π
π π π
π/2 x x
x 0 0
x –π –π
0
– π/2 –2π – 2π

–1 –1 –1
y = cot x y = sec x y = cosec x
(or, x = cot y) (or, x = sec y) (or, x = cosec y)

Fig. 2.18 Graphs of inverse trigonometric functions


Definition 2.16 The functions defined by the equations f (–x) = f (x)
and f (–x) = – f (x) are, respectively, known as even and odd functions.
The graph of even function is symmetric about y-axis, i.e. the point
(x, y) lies on the graph if and only if (–x, y) lies on the graph. While the
graph of an odd function is symmetric about the origin, i.e. the point
(x, y) lies on the graph if and only if the point (–x, –y) lies on the graph.
Definition 2.17 The function defined by the equation f (x + T ) = f (x) is
called the periodic function, where T is the period. The graph of the
periodic function repeats its shape after a certain gap. This gap is the
period of the function.
Functions, Limit and Continuity 43

The domain and range of a function can be obtained as follows:


“Given y = f (x), find all values of x so that the function y = f (x) is
defined. The set of all these values of x is the domain of the given
function.”
“Express y = f(x) as x = f ( y) and find all values of y so that the function
x = f ( y) is defined. The set of all these values of y is the range of the
given function.”
Example 2.1 Find the domain and range of y = x/(1 – x).
Solution This function is defined for all x except x = 1. Here, domain is
¡ – {1}. Also, x = y/( y + 1) which is defined for all y except y = –1 and
thus the range is ¡ – {–1}.
Example 2.2 Find the domain and range of y = 1/(x 2 – 1).
Solution The given function is defined for all x except x = ±1. Here,
domain is ¡ – {1, –1}. Also, x = y /( y + 1) which is defined for all y
except y = 0 and –1 < y < 0 (because x then takes imaginary values) and
thus the range is ¡ – (–1, 0], i.e. the set of all real numbers except y = 0
and the real numbers lying between 0 and –1.
Example 2.3 Find the domain and range of y = (x + 1)/(x 2 – 1).
Solution This function is defined for all x except x = ±1. Here, domain is
¡ – {1, –1}. Also, x = (1 + y)/y which is defined for all y except y = 0 and
thus the range is ¡ – {0}.
Example 2.4 Find the domain and range of y = (x 2 – 1)/(x – 1).
Solution The given function is defined for all x except x = 1. Here, domain
is ¡ – {1}. Also, x = y – 1 which is defined for all y except y = 2 (as we
get again x = 1) and thus the range is ¡ – {2}.
Example 2.5 Find the domain of y = sin–1 2x.
Solution Here, domain is the set of all real numbers x such that – 1 ≤ 2x
≤ 1 and thus the domain is the set [–1/2, 1/2].
Example 2.6 Find the domain of y = cosec x.
Solution The given function is not defined for those values of x for which
sin x = 0, i.e., x = nπ, n ∈ ¢. Thus domain is the set ¡ – {nπ | n ∈ ¢}.

2.3 Operations on Functions


Two functions can be combined in various ways to create new functions.
If f and g are real functions and c is a fixed real number, then
44 Mathematical Methods

(a) (c f ) (x) = c f (x)


(b) ( f + g) (x) = f (x) + g(x) (sum of two functions)
(c) ( f – g) (x) = f (x) – g(x) (difference of two functions)
(d) ( f g) (x) = f (x) g (x) (product of two functions)

⎛ f⎞ f ( x)
(e) ⎜ ⎟ ( x) = , g(x) ≠ 0 (quotient of two functions)
⎝ g⎠ g ( x)
We can combine two functions in yet another way known as composition
of functions.
Definition 2.18 Let f and g be two functions such that the range of g is in
the domain of f. The function whose values are given by f (g(x)) is called
the composite of f with g.
Alternatively, if f is a function from a set A to a set B and g is a
function from the set B to the set C, then g composition f is a function
from the set A to the set C, and we write it as (g o f ) (x) = g( f (x)). In
general, (g o f ) (x) ≠ ( f o g) (x). This can easily be verified by considering
f (x) = 2x – 3, g(x) = x 2 + 1; f (x) = sin x, g(x) = 2x. It may be noted that
for the functions f (x) = [x], g(x) = x, ( f o g) (x) ≠ (g o f ) (x)
when x = – 1.6; while for the same function when x = 1.6, ( f o g) (x) =
(g o f ) (x).
An important instance in which these two composite functions are
equal occurs when f (g(x)) = g( f (x)) = x, we call such functions the
inverse of each other and we have

Definition 2.19 Two functions f and g are inverse of each other if


f (g(x)) = x, for each x in the domain of g and g ( f (x)) = x, for each x in
the domain of f and we denote g by f –1.
For inverse functions, f and g, the range of g must be equal to the
domain of f and vice versa. The inverse of a function can also be defined
as: “If f is a one-one and onto function from the set A to the set B, then g
is a function from B to A and is known as the inverse of f ”. For example,
f (x) = 2x 3 – 1 and g(x) = ( x + 1)/2 are inverse of each other. The
graph of the functions f and f –1 are mirror images of each other (with
respect to the line y = x) (Fig. 2.19). The reflective property of inverse
function can be stated as follows:
The graph of f contains the point (a, b) if and only if the graph of f –1
contains the point (b, a).
Functions, Limit and Continuity 45

y
f
y=x

(a, b )
–1
f

(b, a)

Fig. 2.19 Graph of f –1 is a reflection of the graph of f in the line y = x


y y

x2 + 3 (–2, 3) (2, 3)
f (x ) =
–1

2 3
y=3
4 y=x
2

3 f ( x) = 2 x – 3 x
0, –3 –2 –1 1 2 3
2
(2, 1)
–1
x f (x) = x – 1
2

3 ,0 2 4 6 –2
2

Fig. 2.20 Graph of f and f –1 Fig. 2.21 Graph of f (x) = x 2 – 1


This geometric property suggests an algebraic proceedure for finding
the inverse of a function. Since (a, b) lies on the graph of f if and only if
(b, a) lies on the graph of f –1, we can find the inverse of a function by
interchanging the roles of x and y. For example, the inverse of the function
y = f (x) = 2 x − 3 is g(x) = ( y 2 + 3)/2 (Fig. 2.20).
Not all the functions have an inverse - for the inverse to exist a
function must be one-one and onto. This can be verified easily if we
consider the function defined by f (x) = x 2 – 1. This function is not one-
one. The same conclusion can be made if we solve this function for x so
that x = ± y + 1 (Fig. 2.21).
46 Mathematical Methods

2.4 Limit of a Function


One of the fundamental ideas that distinguishes calculus from algebra
and trigonometry is the concept of limit of a function. This concept can
be introduced through an old problem of elementary geometry;
If regular polygons are inscribed in a circle, then using the perimeter
of these polygons, find the circumference of the circle.
Consider three-, six-, and twelve-sided regular polygons inscribed
in a circle. From Fig. 2.22, it is clear that if we double the number of
sides of the polygons then the perimeter comes closer to the circumference
of the circle. Continuing this process of doubling the sides, we can make
the perimeter as close to the circumference as we wish. In such a case,
we say that the circumference of a circle is the “limit” of the perimeter of
the inscribed regular polygon when the number of sides of the regular
polygon increases.

Fig. 2.22 Inscribed polygons


It is known that the circumference C and the diameter D of a circle
are related through the equation C = πD, when D = 1, the value of π was
approximated by Archimedes (287-212 B.C.) as the “limit” of the
perimeter of the regular polygons.*
To understand the limit of a function, consider the function y = f (x)
= 3x – 1. By assigning different values to x the values of y can be obtained.
For example, when x = 0, 1, 1/3, the function y = f (x) takes the values
–1, 2, 0. These values can then be used to draw the graph of the function
(see Fig. 2.23).

* Muhyi al-Din-al-Maghribi, a Spanish Muslim mathematician and astronomer


of 13th century has found the perimeters of inscribed and circumscribed regular
polygons having ninety six sides. He calculated the value of π using interpolation
formulas. The value of π has also been calculated by Al-Khwarizmi.
Functions, Limit and Continuity 47

2 (1, 2)

x
–1 1
1
,0
–1 3

Fig. 2.23 Graph of f (x) = 3x – 1


It may be noted that when x = 1, then f (1) = (value of the function at
x = 1) = 2. We shall now investigate the behaviour of f (x) = 3x – 1 when
x ≠ 1 but is approaching towards 1 from left and from right. The values
that f (x) may take when x is approaching towards 1 either from left or
right are given in Table 2.1.
Table 2.1 Values of f (x) as x approaches 1

x f (x) f (x) – 2 | f (x) – 2|

0.75 1.25 – 0.75 0.75


0.916 1.75 – 0.25 0.25
0.95 1.86 – 0.13 0.13
0.999 1.997 – 0.003 0.003
..... ..... ..... .....
1.25 2.75 0.75 0.75
1.08 2.25 0.25 0.25
1.04 2.13 0.13 0.13
1.001 2.003 0.003 0.003
Table 2.1 shows that as x comes closer and closer to 1, either from left or
right, then f (x) comes closer to 2 and | f (x) – 2 | can be made as small as
we wish by taking x sufficiently close to 1. In this case, we say that f (x)
has limit 2 as x is approaching to 1 and we have
Definition 2.20 The number L is said to be the limit a function f (x) as x
is approaching to a (we write it as x → a) if for every ε > 0 there exists a
δ > 0 such that | f (x) – L | < ε whenever 0 < | x – a | < δ. We write it as
48 Mathematical Methods

lim f (x) = L and read it as limit of f (x) is equal to L as x approaches to


x →a
a.
Definition 2.21 If x approaches to a from left, then we have left hand
limit while if x approaches towards a from right, we then have right
hand limit and we write them as
lim f ( x) = L1 = left hand limit
x → a−

and lim f ( x) = L2 = right hand limit


x → a+

If L1 = L2 = L, then lim f ( x) = L.
x →a

The following fundamental results will be used while finding out


the limits of a given function. The proofs of these theorems can be found
in any elementary book on calculus (cf., [5]).
Theorem 2.1 If f (x) and g(x) are two real functions and k is any constant,
then
(i) lim k = k .
x →a

(ii) lim [ f ( x) ± g ( x)] = lim f ( x) ± lim g ( x).


x→a x→ a x→a

(iii) lim [ f ( x) g ( x)] = lim f ( x) lim g ( x).


x→a x→ a x→ a

⎡ f ( x) ⎤ lim x → a f ( x)
(iv) lim ⎢ ⎥= .
x → a ⎣ g ( x) ⎦ lim x → a g ( x)

(v) lim k f ( x) = k lim f ( x).


x→a x→a

(vi) lim f ( x) = lim f ( x).


x→a x→ a

(vii) lim [ f ( x)]m / n = [ lim f ( x)]m / n .


x→a x→ a

(viii) lim { f [ g ( x)]} = lim f ( x ), where b = lim g ( x).


x→a x→b x→ a

(ix) If f (x) ≤ g(x) then lim f ( x ) ≤ lim g ( x). But, if f (x) > g(x) then
x→a x→ a
lim f ( x) will not be less than lim g ( x).
x→ a x→a
Functions, Limit and Continuity 49

(x) Let f (x), g(x) and h(x) be functions defined on the same domain D
and f (x) ≤ g(x) ≤ h(x), for every x ∈ D. If lim f ( x) and lim h( x)
x→ a x→a
exist and are equal, then xlim g ( x) exist and equals to lim f ( x).
→a x→a
This result is known as Sandwich theorem.
Theorem 2.2 If n is an integer or fraction, then
xn − an
lim = na n–1 provided that a > 0.
x→a x − a

Theorem 2.3 If m, n are rational numbers, then


xm − am m m − n provided that a > 0.
lim n n
= a
x→a x −a n
Theorem 2.4 lim sin θ = 0, lim cos θ = 1
θ→ 0 θ→ 0

sin θ
Theorem 2.5 lim = 1, when θ is measured in radians.
θ→ 0 θ

Theorem 2.6 lim e x = 1.


x→0

ex − 1
Theorem 2.7 lim = 1.
x→0 x
log (1 + x)
Theorem 2.8 lim = 1.
x→0 x
log x
Theorem 2.9 lim = 1.
x →1 x −1
ax − 1
Theorem 2.10 lim = log a.
x→0 x
Theorem 2.11 lim log x = 1.
x→e

Theorem 2.12 lim e x = e.


x →1

We shall now make use of the above results in finding the limits of
certain functions.

⎡ 1 1 ⎤
Example 2.7 Evaluate lim ( x − 9) ⎢ +
2
.
x→3 ⎣ x + 3 x − 3⎥⎦
50 Mathematical Methods

Solution We have
⎡ 1 1 ⎤ ⎡ ( x − 3) + ( x + 3) ⎤
lim ( x 2 − 9) ⎢ + ⎥ = xlim ( x 2 − 9) ⎢ ⎥
x→3 ⎣ x + 3 x − 3 ⎦ → 3 ⎣ ( x + 3) ( x − 3) ⎦
= lim [ x − 3 + x + 3] = 6
x→3

(1 + x)6 − 1
Example 2.8 Evaluate lim .
x→0 (1 + x) 2 − 1
Solution We have

(1 + x)6 − 1 (1 + 6 x + 15 x 2 + ...) − 1
lim = lim
x→0 (1 + x ) − 1
2
x→0 (1 + 2 x + x 2 ) − 1

6 x + 15 x 2 + ...
= lim
x→0 2 x + x2
6 + 15 x + ...
= lim =3
x→ 0 2+ x

x2 − 4
Example 2.9 Evaluate lim .
x→ 2 3x − 2 − x + 2
Solution We have
x2 − 4 x2 − 4
lim = lim
x→2 3x − 2 − x + 2 x→2 3x − 2 − x + 2
3x − 2 + x+2
×
3x − 2 + x+2
( x + 2) ( 3x − 2 + x + 2)
= lim =8
x→2 2

Example 2.10 If lim x − 1 = lim x − k , find the value of k.


4 3 3

x →1 x − 1 x → k x2 − k 2

Solution Using Theorem 2.2, we have


x4 − 1
lim = 4(1)4–1 = 4,
x →1 x −1
and
Functions, Limit and Continuity 51

x3 − k 3
x −k 3(k )3−1 3
3 3
lim = lim 2x − k 2 = = k
x→ k x2 − k 2 x→ k x − k 2(k ) 2 −1 2
x−k
Thus
x4 − 1 x3 − k 3 3 8
lim = lim ⇒4= k⇒k =
x →1 x − 1 x → k x2 − k 2 2 3
x n − 2n
Example 2.11 If lim = 80, find the value of n.
x→ 2 x − 2

Solution Using Theorem 2.2, we have

x n − 2n
lim = n(2)n–1 = 80 = 5(2)4 = 5(2)5–1 ⇒ n = 5
x→2 x − 2

sin x cos x
Example 2.12 Evaluate lim .
x→0 3x
Solution We have
sin x cos x 1 ⎡ sin x ⎤ 1 1
lim = lim ⎢ ⎥ lim cos x = (1) (1) =
x→0 3x 3 x → 0 ⎣ x ⎦ x → 0 3 3

1 − cos6 x
Example 2.13 Evaluate lim .
x→0 x
Solution We have

1 − cos6 x 2sin 2 3x
lim = lim
x→0 x x→0 x

⎛ sin3 x ⎞
= lim ⎜ 6 × × sin3 x ⎟
x → 0⎝ 3x ⎠

⎛ sin3 x ⎞
= 6 lim ⎜ ⎟ lim (sin 3x) = 6 × 1 × 0 = 0
x → 0 ⎝ 3x ⎠ x → 0

tan x − sin x
Example 2.14 Evaluate lim .
x→0 x3
Solution We have
52 Mathematical Methods

tan x − sin x sin x(1 − cos x )


lim 3
= lim
x→0 x x→0 x3 cos x

sin x(2sin 2 x / 2)
= lim
x→0 x3 cos x

⎡⎛ sin x ⎞ ⎛ sin x /2 ⎞2 ⎛ 1 ⎞⎤
= 2 lim ⎢⎜ ⎟⎜ ⎟ ⎜ ⎟⎥
x → 0 ⎢⎝ x ⎠ ⎝ x /2 ⎠ ⎝ 4cos x ⎠⎥
⎣ ⎦
2
2 ⎛ sin x ⎞ ⎛ sin x /2 ⎞ ⎛ 1 ⎞
= lim ⎜ ⎟ lim ⎜ ⎟ lim ⎜ ⎟
4 x → 0 ⎝ x ⎠ x → 0 ⎝ x / 2 ⎠ x → 0 ⎝ cos x ⎠

1 1
= (1)(1)2 (1) =
2 2
1 + cos2 x
Example 2.15 Evaluate lim .
π x→ 2 (π − 2 x) 2

Solution Put π – x = t so that when x → π/2, t → 0, and we have


2

1 + cos2 x 1 + cos( π − 2t )
limπ = lim
x→ 2 (π − 2 x) 2 t→0 4t 2
1 − cos2t
= lim
t →0 4t 2
2sin 2 t
= lim
t →0 4t 2
2
1 ⎛ sin t ⎞
= lim ⎜ ⎟
2t →0 ⎝ t ⎠
1 1
= ×1 =
2 2
1 − tan x
Example 2.16 Evaluate lim .
π x→4 x − π
4

π
Solution Put x = + t so that when x → π/4, t → 0, and we have
4
Functions, Limit and Continuity 53

⎛π ⎞
1 − tan ⎜ + t ⎟
1 − tan x ⎝4 ⎠
lim = lim
x → π4 x − π /4 t→0 t
1 ⎡ 1 + tan t ⎤
= lim ⎢1 −
t →0t ⎣ 1 − tan t ⎥⎦
−2tan t
= lim
t → 0 t (1 −
tan t )

= lim ⎜
⎛ −2 tan t ⎞
× ⎟
t → 0 ⎝ 1 − tan t t ⎠
2
= − ×1=–2
1
ax − 1
Example 2.17 Evaluate lim .
x → 0 bx − 1

Solution We have
ax − 1 ⎡ax − 1 x ⎤
lim x
= lim ⎢ × x ⎥
x→0 b −1 x→0
⎣ x b − 1⎦
ax − 1 bx − 1
= lim ÷ lim
x→0 x x→ 0 x
log a
= = logb a
log b
e x − e− x
Example 2.18 Evaluate lim .
x→0 x
Solution We have
e x − e− x e2 x − 1
lim = lim
x→0 x x→0 xe x
e2 x − 1 2
= lim × x
x → 0 2x e
e2 x − 1 2
= lim × lim x
x→0 2x x→0 e

2
= 1× = 2
1
54 Mathematical Methods

Example 2.19 Evaluate lim log x .


x→1 x − 1
Solution Put x = 1 + y so that when x → 1, y → 0 and thus
log x log(1 + y )
lim = lim
x→1 x −1 x→ 0 y
1
= lim log(1 + y )
y→ 0 y
= lim log(1 + y )1/ y
y→ 0
= log e = 1
esin x − 1
Example 2.20 Evaluate lim .
x→ 0 x
Solution We have

esin x − 1 ⎛ esin x − 1 sin x ⎞


lim = lim ⎜ × ⎟ = 1×1 = 1
x→ 0 x x → 0 ⎜ sin x x ⎟⎠

2.5 Infinite Limits


1
Consider the function f (x) = . When x becomes very large then
x
1
f (x) = comes very close to 0 and we say that the limit of the function
x
f (x) = 1/x is 0 when x is approaching towards infinity. We write it as
lim (1/x) = 0. This result is frequently used in evaluating the limits of
x→∞

functions when x approaches to infinity. Another way to calculate the


infinite limit of a function is that by using the substitution y = 1/x and
taking the limit at zero. These methods are illustrated with the help of
the following examples.
(3x − 1) (4 x − 2)
Example 2.21 Evaluate lim .
x→∞ ( x + 8)( x − 1)
Solution We have
(3x − 1) (4 x − 2) [3 − (1/ x)][4 − (2/ x)]
lim = lim
x→∞ ( x + 8)( x − 1) x→∞ [1 + (8/ x)][1 − (1/ x)]
Functions, Limit and Continuity 55

[3 − lim x → ∞ (1/ x)][4 − lim x → ∞ (2/ x)]


=
[1 + lim x → ∞ (8/ x)][1 − lim x → ∞ (1/ x)]
= 12
ax + b
Example 2.22 If f (x) = , lim f ( x) = 2 and lim f ( x) = 1, then
x +1 x→0 x→∞
show that f (–2) = 0.
Solution Given that lim f ( x) = 2 and lim f ( x) = 1, then
x→0 x→∞

ax + b
lim f ( x) = lim =b (1)
x→0 x→0 x +1
while
ax + b
lim f ( x) = lim =a (2)
x→∞ x→∞x +1
Thus from the given condition, Eqs. (1) and (2) yield a = 1 and b = 2 and
x+2
the given function f (x) takes the form f (x) = which clearly shows
x +1
that f (–2) = 0.
Theorem 2.13 For all x
n
⎛ x⎞ x
lim ⎜1 +
n → ∞⎝
⎟ =e
n⎠
so that for x = 1, we have
n
⎛1⎞
lim ⎜1 +
⎟ =e
n → ∞⎝n⎠
which may be treated as the definition of e.

Example 2.23 Evaluate lim sin x .


x→∞ x
Solution. Put x = 1/y so that when x → ∞, y → 0, and thus
sin x sin1/ y 1
lim = lim = lim y sin
x→∞ x y → 0 1/ y y→0 y
But sin (1/y) oscillates between –1 and 1. Thus
1
lim y sin =0
y→0 y
56 Mathematical Methods

Hence
sin x
lim =0
x→∞ x

2.6 Right and Left Hand Limits


As defined earlier, a function has a limiting value only if its right hand
limit is same as its left hand limit. We shall now illustrate this concept
by considering the behaviour of the function f (x) = | x |/x, x ≠ 0 when x
approaches to 0 from right as well as from left. The values which this
function may take when x approaches towards 0 either from right or left
are given in Table 2.2 as follows:
Table 2.2 Behaviour of f (x) = | x |/x

x f ( x)
0.1 1
0.01 1
0.001 1
0.0001 1
______ ______
− 0.1 −1
− 0.01 −1
− 0.001 −1
− 0.0001 −1

Thus function f (x) = | x |/x, x ≠ 0 is not defined at x = 0 but its values


can be determined at any point which is as close to 0 as we wish. First
consider x → 0+, then at all points on the right of x = 0, the value of
| f (x) – 1 | is zero. Thus the value of | f (x) – 1 | can be made smaller than
any positive number (however small). Thus
|x|
lim f ( x) = lim =1
x → 0+ x → 0+ x

Now, if x → 0– , then at all points on the left of x = 0, the value of


| f (x) – (–1) | is zero. Thus the value of | f (x) – (–1) | can be made smaller
than any positive number (however small). Thus

lim f ( x ) = | x|
lim = –1
x → 0− x → 0− x
Functions, Limit and Continuity 57

and we have

|x| ≠ | x|
lim lim
x → 0+ x x → 0− x
|x|
Therefore lim does not exist. The same has been illustrated in
x→0 x
|x|
Fig. 2.24. The graph of the function has two branches. One branch
x
is in the first quadrant and the other is in the third quadrant and also,
both of these branches are parallel to the x-axis and are at a distance of
one unit from x-axis. From the Fig. 2.24, it may be noted that when x is
approaching towards 0 from the right, then f (x) approaches towards 1
while for x approaching towards 0 from left, f (x) moves towards –1. It
may be noted that the graph of this function has a break at the point
x = 0 and the function has no limiting value at that point. At all the other
remaining points, the graph of the function has no breaks and one can
find the limit of the function at these points.
We shall now give the working rule for finding the right and left
hand limits of a given function as follows:
For obtaining the left hand limit, we substitute x = a – h in f (x) and then
take limit of f(a – h) as h → 0, i.e.
lim f ( x) = lim f (a − h)
x → a− h→0

while for finding the right hand limit, we put x = a + h in f (x) and then
take limit of f (a + h) as h → 0, i.e.
lim f ( x) = lim f (a + h)
x → a+ h→0
y

x
0

–1

_ x_
Fig. 2.24 Graph of f (x) =
x
58 Mathematical Methods

Example 2.24 Prove that lim[ x] does not exist.


x →1
Solution Here
left hand limit = lim f ( x)
x → 1−

= lim f (1 − h)
h→0

= lim [1 − h] = lim 0 = 0
h→0 h→0
and
right hand limit = lim f ( x)
x → 1+

= lim f (1 + h)
h→0

= lim [1 + h] = lim 1 = 1
h→0 h→0

Since left hand limit ≠ right hand limit, lim [ x] does not exist.
x →1

Example 2.25 Establish the existence of the limit of the function f (x)
defined as follows:
⎧ 1
⎪ x, if 0 ≤ x <
2

⎪ 1
f (x) = ⎨0, if x =
⎪ 2
⎪ 1
⎪ x − 1, if < x ≤1
⎩ 2
Solution Here
right hand limit = lim f ( x)
x → 12 +

= lim ( x − 1)
x → 12 +

= lim ⎡⎢ + h − 1⎤⎥
1
h → 0 ⎣2 ⎦
⎡ 1⎤ 1
= lim ⎢h − ⎥ = −
h→0⎣ 2⎦ 2
and
left hand limit = lim f ( x)
x → 12 −
Functions, Limit and Continuity 59

= lim ( x)
x → 12 −

⎡1 ⎤ 1
= lim ⎢ − h⎥ =
h → 0 ⎣2 ⎦ 2
Since left hand limit ≠ right hand limit, lim f ( x ) does not exist.
1 x→ 2

Example 2.26 Find the limit as x → 0 of the function f (x) defined as


follows:
⎧1, if x ≠ 0
f (x) = ⎨
⎩2, if x = 0
Solution Here
right hand limit = lim f ( x) = 1
x → 0+

and
left hand limit = lim f ( x) = 1
x → 0−

Since left hand limit = right hand limit, lim f ( x) exists and its value
x→0
is 1.

2.7 Continuity of a Function


While plotting the input and output values of a data, we connect the
plotted points with a curve that has no breaks in it, i.e. the drawn curve
is continuous. This section discusses the continuity of a real function
and the related results.
Definition 2.22 A function y = f (x) is said to be continuous at a point
x = a if the following conditions are satisfied:
(i) f (x) is defined at x = a, i.e. f (a) exists.
(ii) lim f ( x) exists.
x→a

(iii) lim f ( x) = f (a).


x→a

If any one of the above conditions is not satisfied, then the function is
said to be discontinuous.
Definition 2.23 A function y = f (x) is said to be continuous in an open
interval (a, b) if it is continuous at every point of the interval (a, b).
60 Mathematical Methods

Definition 2.24 A function y = f (x) is said to be continuous in the closed


interval [a, b] if it is continuous at every point of the open interval (a, b)
and also continuous at the point a from the right and continuous at the
point b from the left, i.e., lim f ( x) = f (a) and lim f ( x) = f (b).
x → a+ x → b−

It may be noted that when we are talking about the continuity of a


real function, without mentioning the domain of the function, it means
that the function is continuous at every point of the domain. Roughly
speaking, a function is continuous if there is no break or jump in the
graph of this function. We shall now mention some of the important
results concerning the continuity of a given function.
Theorem 2.14 If f and g are continuous functions, then
(i) f + g is continuous.
(ii) f – g is continuous.
(iii) f g is continuous.
(iv) A f is continuous, where A is aconstant.
(v) f /g is continuous at those points where g does not take the value
zero.
Following are some of the frequently occuring functions which are
also continuous.
1. Constant function. If f (x) is a constant function, i.e., if f (x) = a,
where a is a constant, then its graph is a line parallel to the x-axis
at a distance a from the x-axis and thus there is no break in the
graph, which means that the constant function is a continuous
function.
2. Identity function. The identity function f (x) = x is a continuous
function because the graph of this function is a line passing through
the origin.
3. Polynomial function. We have seen that the identity function
f (x) = x is a continuous function and from Theorem 2.14, the
product of two continuous functions is also a continuous function,
thus f (x) = x 2, x 3, ..., x n are continuous functions. Moreover, the
constant function f (x) = a is also continuous which leads to the
functions of the types f (x) = a x n to be the continuous one as the
product of the continuous functions is also a continuous one.
Therefore, the polynomial function f (x) = a0 + a1x + a2 x 2 + a3 x 3
+ ... + an x n is also a continuous function.
Functions, Limit and Continuity 61

4. Rational function. Since every polynomial is a continuous function,


and from Theorem 2.14, the quotient of two continuous functions
is also a continuous function except when the denominator is zero.
Therefore, a rational function is a continuous function except at
the points where the denominator is zero.
5. The functions sin x, cos x, e x, a x are continuous functions
everywhere.
6. The functions sin–1 x and cos–1 x are continuous functions in
[–1, 1].
7. The functions tan–1 x and cot–1 x are continuous everywhere.
8. The functions sec–1 x and cosec–1 x are continuous in (– ∞, –1] U
[1, ∞).
Properties of the continuous functions
The continuous functions defined on some domain do share the following
properties:
(a) If f (x) is a continuous function at x = a and g (x) is another
continuous function at x = f (a) then the composite function
( g o f ) (x) or g ( f (x)) is also a continuous function at x = a. For
example, if f (x) = x 2 + 1 and g(x) = x + 1 are two continuous
functions in some domain, then ( g o f ) (x) = x 2 + 2 is also a
continuous function.
(b) If f (x) is a continuous function and f (x) ≠ 0, then 1/ f (x) is also a
continuous function. For example, if f (x) = x 4 is continuous, then
1/x 4 is also continuous except at x = 0.
(c) The graph of a continuous function is a continuous curve without
having any break(s) in it. Therefore, if the function is passing
from any value to any other value within the range for which the
function is continuous, then the given function must pass atleast
once through every intermediate value.
(d) If the graph of a continuous function is passing from one side of
an axis to another side of the same axis, then it must cut the axis at
some intermediate points (cf., the graphs of trigonometric functions,
Fig. 2.17).
(e) A continuous function on an interval has a least and greatest value.
(f ) If f (x) is a continuous function in [a, b] then for each number c
between f (a) and f (b) there exists a number d in [a, b] such that
f (d ) = c. This is known as Intermediate value property. For
62 Mathematical Methods

example, consider the function f (x) = 16 − x 2 which is


continuous in [– 4, 0] and the given intermediate value is c = 7.
Here f (– 4) = 0, f (0) = 4 and 0 < 7 < 4. We then have to find a
number d such that f (d ) = c = 7. Now, 16 − d 2 = c = 7
which gives d = ±3, and thus the desired value of d is –3.
We shall now work out a number of examples about the continuity
of some functions as follows:
Example 2.27 Test the continuity of the function at x = 0
⎧2 x − 1, if x < 0
f (x) = ⎨
⎩2 x + 1, if x ≥ 0
Solution Here
right hand limit = lim f ( x) = 1
x → 0+

and
left hand limit = lim f ( x) = –1
x → 0−

Since the left hand and right hand limits are not same, this means that
the limit of the given function does not exist at x = 0, and therefore, the
given function is not continuous at x = 0.
Example 2.28 Examine the continuity of the following function at x = 2
⎧2 x − 1, if x < 2
f (x) = ⎨
⎩3 x /2, if x ≥ 2
Solution Here
right hand limit = lim f ( x ) = 3
x → 2+

and
left hand limit = lim f ( x ) = 3
x → 2−

Since the left hand and right hand limits are same, this means that the
limit of the given function exists at x = 2, and equals to 3, moreover
f (2) = 3; thus the value of the function is same as the limit of the function.
Therefore, the given function is continuous at x = 2.
Example 2.29 Test whether the following function is continuous or not
at x = 2
Functions, Limit and Continuity 63

⎧ x 4 − 16
⎪ , if x ≠ 2
f (x) = ⎨ x − 2
⎪16, if x = 2

Solution Here, f (2) = 16 = value of the function at x = 2 and lim f ( x)
x→2

= 32 which shows that the limit of the given function, at x = 2, is different


from the value of the function at x = 2. Therefore, the given function is
not continuous at x = 2.
It may be noted that the above function can be made continuous if
the value of the function at x = 2 is taken as 32, and thus the discontinuity
of this function can be removed. Such type of discontinuity is known as
removable discontinuity.
Example 2.30 Examine the continuity of the following function at x = 2
⎧| x |, if x ≤ 2
f (x) = ⎨
⎩ x, if x > 2
Solution Here
right hand limit = lim f ( x ) = 2
x → 2+

and
left hand limit = lim f ( x ) = 2
x → 2−

Since the left hand and right hand limits are same, this means that the
limit of the given function exists at x = 2, and equals to 2, moreover
f (2) = 2; thus the value of the function is same as the limit of the function.
Therefore, the given function is continuous at x = 2.
Example 2.31 Show that the function defined as
⎧ x
⎪ , if x ≠ 0
f (x) = ⎨| x |
⎪0, if x = 0

is not continuous at x = 0.
Solution Here
x x
right hand limit = lim f ( x) = lim = lim = lim 1 = 1
x → 0+ x → 0+ | x | x → 0+ x x → 0 +
and
64 Mathematical Methods

x x
left hand limit = lim f ( x) = lim = lim = lim (−1) = −1
x → 0− x → 0− | x | x → 0− − x x → 0 –
Since the left hand and right hand limits are not same, this means that
the limit of the given function does not exist at x = 0. Therefore, the
given function is not continuous at x = 0.
Example 2.32 Show that the function
⎧ sin x
⎪ , if x < 0
f (x) = ⎨ x
⎪⎩ x + 1, if x ≥ 0
is continuous at x = 0.
Solution Here
right hand limit = lim f ( x) = lim ( x + 1) = 1
x → 0+ x → 0+

and
sin x
left hand limit = lim f ( x ) = lim =1
x → 0− x → 0− x
Since the left hand and right hand limits are same, this means that the
limit of the given function exists at x = 0, and moreover, the value of the
function at x = 0 is 1. Thus the limit of the function and the value of the
function are same at x = 0. Therefore, the given function is continuous at
x = 0.
Example 2.33 Show that the function
⎧ e1/ x − 1
⎪ , if x ≠ 0
f (x) = ⎨ e1/ x + 1
⎪0, if x = 0

is discontinuous at x = 0.
Solution Here
1
1−
right hand limit = lim f ( x) = lim e1/ x − 1 e1/ x
= lim =1
x → 0+ x → 0+ e1/ x + 1 x → 0+ 1
1 + 1/ x
e
as x → 0+, 1/x → + ∞ which gives e1/x → ∞ and 1/e1/x → 0. Also
e1/ x − 1 0 −1
left hand limit = lim f ( x) = lim = lim = −1
1/ x x → 0− 0 + 1
x → 0− x → 0− e +1
Functions, Limit and Continuity 65

as x → 0–, 1/x → – ∞ which gives e1/x → e– ∞ = 0. Since the left hand and
right hand limits are different, this means that the limit of the given
function does not exist at x = 0, and therefore the given function is
discontinuous at x = 0.
Example 2.34 If

⎧1, if x ≤ 3

f (x) = ⎨a x + b, if 3 < x < 5
⎪7, if 5 ≤ x

Find the values of the constants a and b such that f (x) is continuous.
Solution. Here
right hand limit = lim f ( x) = lim (a x + b) = 3a + b
x → 3+ x → 3+

and
left hand limit = lim f ( x) = lim (1) = 1
x → 3− x → 3−

The limit of the function at x = 3 will exist only when the right hand and
left hand limits at x = 3 are equal. Also, f (3) = 1. Thus
1 = 3a + b (3)
(i.e., f (x) is continuous at x = 3 if 3a + b = 1). Further
right hand limit = lim f ( x) = lim (7) = 7
x → 5+ x → 5+

and
left hand limit = lim f ( x) = lim (a x + b) = 5a + b
x → 5− x→5 −

The limit of the function at x = 5 will exist only when the right hand and
left hand limits at x = 5 are equal. Also, f (5) = 7 and thus
7 = 5a + b (4)
(i.e., f (x) is continuous at x = 5 if 5a + b = 7 ).
Solving Eqs. (3) and (4), we get a = 3 and b = – 8, and for these
values the given function is continuous.
Example 2.35 Examine the continuity of the function

⎧ x− | x |
⎪ , if x ≠ 0
f (x) = ⎨ x
⎪⎩2, if x = 0
66 Mathematical Methods

Solution. Here
x− | x | x−x
right hand limit = lim f ( x) = lim = lim =0
x → 0+ x → 0+ x x → 0+ x
and
x− | x | x − (− x)
left hand limit = lim f ( x) = lim = lim
x → 0− x → 0− x x → 0− x

2x
= lim =2
x → 0– x
(using the definition of the absolute value function). Since the left hand
and right hand limits are not same, this means that the limit of the given
function does not exist at x = 0. Therefore, the given function is
discontinuous at x = 0.
Example 2.36 Examine the continuity of the function

⎧sin 2 x
⎪ , if x ≠ 0
f (x) = ⎨ x
⎪⎩1, if x = 0
Solution Here
sin 2 x
right hand limit = lim f ( x) = lim
x → 0+ x → 0+ x

sin 2(0 + h) ⎤ ⎡ ⎛ sin 2h ⎞⎤


= lim ⎡⎢ ⎥ = lim ⎢2⎜ ⎟⎥ = 2
h → 0⎣ 0+h ⎦ h → 0 ⎣ ⎝ 2 h ⎠⎦
and
sin 2 x
left hand limit = lim f ( x) = lim
h → 0− h → 0− x

⎡ sin 2(0 − h) ⎤ ⎡ sin 2h ⎤ ⎡ ⎛ sin 2h ⎞⎤


= lim ⎢ = lim ⎢ = lim ⎢2⎜ ⎟ =2
h → 0⎣ 0 − h ⎦ h → 0 ⎣ h ⎦ h → 0 ⎣ ⎝ 2h ⎠⎥⎦
⎥ ⎥

Since the left hand and right hand limits are same, this means that the
limit of the given function exists at x = 0, and equals to 2. Also f (0) = 1,
thus the limit and value of the given function, although they exist, are
not same. Therefore, the given function is discontinuous at x = 0.
Functions, Limit and Continuity 67

Example 2.37 Examine the continuity of the function


⎧ 1
⎪ x sin , if x ≠ 0
f (x) = ⎨ x
⎪⎩0, if x = 0
at x = 0.
Solution Here
1 ⎡ 1 ⎤
right hand limit = lim f ( x) = lim x sin = lim ⎢(0 + h)sin
x → 0+ x → 0+ x h→0 ⎣ 0 + h ⎥⎦
⎡ 1⎤
= lim ⎢h sin ⎥ = 0 × a finite quantity = 0
h→0⎣ h⎦
(as h → 0, 1/h → ∞ and –1 ≤ sin (1/h) ≤ 1). Also
1 ⎡ 1 ⎤
left hand limit = lim f ( x) = lim x sin = lim (0 − h)sin
x → 0− x → 0− x h → 0 ⎢⎣ 0 − h ⎥⎦

= lim ⎡⎢−h sin ⎤⎥ = lim ⎡⎢h sin ⎤⎥ = 0 × a finite quantity = 0


1 1
h→0⎣ − h ⎦ h→0 ⎣ h⎦
Since the left hand and right hand limits are same, this means that the
limit of the given function exists at x = 0, and equals to 0. Also f (0) = 0,
thus the limit and value of the given function, are same. Therefore, the
given function is continuous at x = 0.
Example 2.38 Show that the function
⎧ e1/ x
⎪ , if x ≠ 0
f (x) = ⎨1 + e1/ x
⎪0, if x = 0

is discontinuous at x = 0.
Solution.  Here
e1/ x
right hand limit = lim f ( x) = lim
x → 0+ x → 0+ 1 + e1/ x

e1/ h 1
= lim = lim =1
h → 0 1 + e1/ h h →0 e−1/ h + 1
as limh → 0 e–1/h = 0. Also

e1/ x e−1/ h
left hand limit = lim f ( x) = lim = lim =0
x → 0− x → 0− 1 + 1/ x
e h → 01 + e −1/ h
68 Mathematical Methods

Since the left hand and right hand limits are different, this means that
the limit of the given function does not exist at x = 0. Therefore, the
given function is discontinuous at x = 0.

EXERCISES
1. If f is the exponential function and g is the logarithmic function,
then find
(a) ( f + g) (1) (b) ( f g) (1)
(c) ( f o g) (1) (d) (2 f ) (1)
[Ans. e, 0,1, 2e]
2. If f (x) = e2x and g(x) = log x , x > 0, then find
(a) ( f g) (x) (b) ( f o g) (x)

(c) ( g o f ) (x) [Ans. e2x log x , x, x]


x2
3. Show that f (x) = 5 – is an even function.
4. If f (x) = x2 + kx + 1 for all x and f is an even function, find k.
[Ans. 0]
5. If y = f (x) is an identity function, then find ( f o f ) (x) and ( f f ) (x).
[Ans. x, x 2]
6. Find the domain of the following functions

x
(a) f (x) = (b) ( x − 2) (4 − x)
x − 3x + 2
2

(c) x2 − 5x + 6 [Ans. ¡ – {1, 2}, [2, 4], (– ∞, 2] ∪ [3, ∞)]


7. Evaluate the following limits

x2 − 1 x −1
(a) lim (b) lim
x →1 x − 1 x → 1 2 x3 + x−3

x 4 − 3x3 + 2 (2 x − 3)( x − 1)
(c) lim (d) lim
x → 1 x3 + 5 x 2 + 3x + 1 x →1 2x2 + x − 3

x3/5 − a3/5 x5 − 32
(e) lim (f ) lim
x→a x1/3 − a1/3 x→2 x2 − 4
Functions, Limit and Continuity 69

( x + 2)5/3 − (a + 2)5/3
(g) lim
x→a x−a

⎡ x + h − x⎤ ⎡ 1 + x − 1 + x2 ⎤
(h) lim ⎢ ⎥ (i) lim ⎢ ⎥
h→0
⎣ h ⎦
x→0 ⎢ x ⎥⎦

1
[Ans. 2, 1/7, 5/4, –1/10, (9/5) a4/15, 20, (5/3) (a + 2)2/3, , 1/2]
2 x

f ( x) − f (1) 1
8. If f (x) = − 25 − x 2 , then show that lim = .
x →1 x −1 24
9. Evaluate the following limits

⎛ sin 3x ⎞ ⎛ sin x cos x ⎞


(a) lim ⎜ ⎟ (b) lim ⎜ ⎟
x → 0 ⎝ 5x ⎠ x → 0⎝ 3x ⎠

⎛ 3 ⎞ ⎛ sin 5 x − sin 3 x ⎞
(c) lim ⎜ x ⎟ (d) lim ⎜ ⎟
x → 0 ⎜ sin x 2 ⎟ x → 0⎝ sin x ⎠
⎝ ⎠

3sin x − sin3x ⎞ 1 − cos x ⎞


(e) lim ⎛⎜ ⎟ (f ) lim ⎛⎜ ⎟
x → 0⎝ x3 ⎠ x → 0⎝ x ⎠

⎛ cos 2 x ⎞ ⎛ cosec x − cot x ⎞


(g) lim ⎜ ⎟ (h) lim ⎜ ⎟
x → π2 ⎜ 1 − sin x ⎟ x → 0⎝ x ⎠
⎝ ⎠

⎛ sin θ − cos θ ⎞
(i) limπ (sec θ – tan θ) ( j) lim ⎜ ⎟⎟
θ→ 4 ⎜ π
⎝ θ− 4
θ→ 2 π

⎛ 1 − tan θ ⎞ ⎛ x cos x − sin x ⎞


(k) lim ⎜ ⎟ (l) lim ⎜⎜ ⎟⎟
θ→ 4 ⎜ θ − π ⎟
2
x→0
⎝ x sin x ⎠
π
⎝ 4 ⎠

⎛ 1 − sin 3 x ⎞
(m) limπ ⎜⎜ 2 ⎟⎟
x→ 2
⎝ cos x ⎠
[Ans. 3/5, 1/3, 0, 2, 4, 0, 2, 1/2, 0, 2, –2, –1/3, 3/2]
70 Mathematical Methods

10. Prove that lim (1 + a x)1/x = e.


x→0

⎛ ax − bx ⎞
11. Show that lim ⎜ ⎟⎟ = log (a/b)
x → 0⎜
⎝ x ⎠

⎡ ⎛ x ⎞⎤
log 1 − ⎟⎥
12. Show that lim ⎢ ⎜⎝ 2 ⎠⎥ = –1/2.
x→0⎢
⎢ x ⎥
⎣ ⎦

⎛ e x + e− x − 2 ⎞
13. Show that lim ⎜ ⎟⎟ = 0.
x → 0⎜ x
⎝ ⎠
⎛ a x − 1⎞
14. If lim ⎜ ⎟⎟ = log a, then prove that
x → 0⎜
⎝ x ⎠
⎛ 2x − 1 ⎞
lim ⎜ ⎟ = 2 log 2
x → 0 ⎜ (1 + x )1/2 − 1 ⎟
⎝ ⎠
15. Evaluate the following limits
⎡ ⎤
(a) lim ⎢ ( x + 1)(2 x + 3) ⎥ ( b) lim ⎡ x − x 2 + x ⎤
x → ∞ ⎣ ( x + 2) (3 x + 4) ⎦ ⎢
x→∞ ⎣ ⎥⎦

⎡ 2 3 2 ⎤
(c) lim ⎢ x + 1 − x − 1 ⎥ [Ans. 2/3, –1/2, 1]
x→∞ ⎢4 4
⎣ x + 1 − x + 1 ⎥⎦
5 4

16. If the function


⎧ x 2 − 10
⎪ , if x ≠ 4
f (x) = ⎨ x − 4
⎪c, if x = 4

is continuous, then find the value of c. [Ans. 8]
17. For what value of c is the following function

⎪⎧cx 2 , if x ≤ 2
f (x) = ⎨
⎪⎩3, if x > 2
continuous? [Ans. 3/4]
Functions, Limit and Continuity 71

18. For what value of c is the following function

⎧ 7x + 2 − 6x + 4
⎪ , if x ≥ −2/7 and x ≠ 2
f (x) = ⎨ x−2
⎪c, if x = 2

continuous? [Ans. 1/8]
19. Examine the continuity of the function

⎧| x − a |
⎪ , if x ≠ a
f (x) = ⎨ x − a
⎪⎩1, if x = a
[Ans. Discontinuous]
20. Examine the continuity of the function

⎧1 + x, if x ≤ 2
f (x) = ⎨
⎩5 − x, if x > 2
[Ans. Continuous at x = 2]
21. Show that the function

⎧sin x
⎪ + cos x, when x ≠ 0
f (x) = ⎨ x
⎪⎩2, when x = 0
is continuous at x = 0.
22. Show that the function

⎧ x2 − x − 6
⎪ 2 , when x ≠ 3
f (x) = ⎨ x − 2 x − 3
⎪5/3 when x = 3

is discontinuous at x = 3.
23. Prove that the function

⎧ 1
⎪( x − a ) sin , when x ≠ a
f (x) = ⎨ x−a
⎪⎩0, when x = a
is continuous at x = a.
72 Mathematical Methods

CHAPTER 3

Differentiation

3.1 Introduction
Differential calculus deals with the problems which involve rate of change.
Though the subject was developed by Newton, Leibnitz, Pascal, Fermat
and others during 17th century, it were Newton and Leibnitz who not
only introduced the concepts of derivatives and differentials, but also the
symbols which greatly simpified the calculations. They applied differential
calculus in solving a number of problems of geometry and mechanics.
Differential calculus now finds a wide range of applications in basic
sciences, computer sciences, engineering, economics and medicine. It
can be used to find the slopes of curves, velocities and accelerations of
moving bodies, the motion of a projectile which in turn leads to the
firing angle of a cannon, the motion of a planet such as when a planet is
nearest or farthest from the sun, etc. Differential calculus also helps us
to know and predict the behaviour of a machinery, the attitude of a person
towards learning a particular subject, the sensitivity of a particular
medicine to human beings, the nature of demand and supply of a certain
commodity, etc.
This chapter deals with the concept of the derivative of a real function
and the related results along with applications.

3.2 Derivative of a Function


Definition 3.1 Let y = f (x) be a single valued real and finite function of
x. Let ∆x be a small increment (change) in x and ∆y be the corresponding
change in y. Then y + ∆y = f (x + ∆x) so that ∆y = f (x + ∆x) – y = f (x +
∆x) – f (x). Divide this equation by ∆x and take the limit as ∆x approaches
to zero, i.e.
∆y f ( x + ∆x) − f ( x)
lim = lim
∆x → 0 ∆x ∆x → 0 ∆x
Differentiation 73

This limit, if exists, is known as the differential coefficient of y with


respect to x, or the derivative of y with respect to x, and is denoted by the
symbol dy/dx (or f ′ (x) or d f (x)/dx or y ′ or y1 or Dy). Thus
dy f ( x + ∆x ) − f ( x )
= lim (1)
dx ∆x → 0 ∆x
Equation (1) is also known as the derivative of a function by first principle.
Definition 3.2 The derivative of a function y = f (x) with respect to x at
the point x = a is defined as
⎡ dy ⎤ f ( a + h) − f ( a )
⎢ dx ⎥ = lim (2)
⎣ ⎦ x =a h→0 h
provided that the limit exists and is denoted by f ′ (a).
A function is said to be derivable or differentiable at x = a if it
possesses a derivative at x = a, while a function is said to be differentiable
on an interval if it is differentiable at every point of the interval.
The right hand and left hand derivatives of the function y = f (x) at
the point x = a are defined as
f ( a + h) − f ( a )
R f ′(a) = lim , h>0 (3)
h→0 h
and
f ( a − h) − f (a )
L f ′(a) = lim , h>0 (4)
h→0 −h
For the existence of the derivative, the limits on the right hand sides
of Eqs. (3) and (4) must be the same, i.e., R f ′(a) = L f ′(a). If these two
limits are different, then the derivative will not exist and the function
will not be differentiable.
Example 3.1 Find the derivative of the function
⎧ x −1
⎪⎪ 2 , if x ≠ 1
2x − 7x + 5
f (x) = ⎨
⎪− 1 , if x = 1
⎪⎩ 3
at x = 1.
Solution From the definition
f (1 + h) − f (1)
f ′ (1) = lim (5)
h→0 h
(1 + h) − 1 h
When x ≠ 1, f (1 + h) = =
2(1 + h) 2 − 7(1 + h) + 5 2h 2 − 3h
74 Mathematical Methods

1
When x = 1, f (1) = – .
3
Thus, from Eq. (5), we have
⎛ h ⎞ ⎛ 1⎞
⎜ 2 ⎟ − ⎜− ⎟
⎝ 2h − 3h ⎠ ⎝ 3 ⎠ = lim 3h + (2h − 3h)
2
f ′ (1) = lim
h→0 h h → 0 3h(2h − 3h)
2

2 2
= lim = = –2/9
h → 0 3(2h− 3) 3(0 − 3)

Q
y
y = f (x )

R
P

ψ
x
M N

Fig. 3.1 Geometrical meaning of the derivative


Geometrical and physical interpretation of the derivative of a function
Let y = f (x) be a continuous function whose graph is shown in Fig. 3.1,
and also let P and Q be two points on the curve having the coordinates as
(x, y) and (x + ∆x, y + ∆y).
From P and Q draw PM and QN perpendiculars on the x-axis and
PR perpendicular on NQ, then from the Fig. 3.1,
∆y
tan ∠ QPR = (6)
∆x
It can be easily seen that when ∆x → 0, Q → P. In the limiting case,
the secant PQ becomes the tangent at P and ∠ QPR will tend to angle ψ
which the tangent at P makes with the positive direction of x-axis. Thus
in the limiting position as Q → P, ∆ x → 0 and Eq. (6) becomes
∆y
lim = lim tan ∠ QPR
∆x → 0 ∆x Q→P
Differentiation 75

dy
or, = tan ψ = slope of the tangent at P(x, y)
dx
Therefore, the derivative at any point P is the tangent of the angle which
the tangent to the curve at P makes with the positive direction of
x-axis.
Remark. If the tangent is parallel to x-axis, then ψ = 0 and dy/dx = tan ψ
= tan 0 = 0, while, if the tangent is perpendicular to x-axis, then ψ = 90°
and cot ψ = 0, and thus dx/dy = 0.
Let a particle P be moving in a straight line and we are measuring
its displacement x from a point O on this line. As the displacement
depends upon time t, we can write t = f (x). Also, let Q be any other point
on this line. Then the displacement OP = x at time t = f (t) and the
displacement PQ at time (t + h) = f (t + h), and therefore, the displacement
at time (t + h) – t = h = f (t + h) – f (t). Hence
f (t + h) − f (t )
average velocity of the particle in the interval h =
h
Now, when we make h smaller and smaller, we can obtain the average
velocity in smaller and smaller intervals near a. The limit of the average
velocity as h → 0 represents the instantaneous velocity of the particle at
time t. Therefore
dx f (t + h) − f (t )
velocity at time t = v = = lim
dt h → 0 h
Thus, the derivative is related to a function, in the same way as
the velocity is related to the distance travelled by a moving particle
(Fig. 3.2).
O P Q

Fig. 3.2 Distance travelled by a particle


We shall state the following theorem on the differentiability of a
given function and illustrate it with the help of examples.
Theorem 3.1 If a function is differentiable at a point x then it is continuous
at that point, but the converse need not be true.
Example 3.2 Show that function
⎧ x, if x > 0
f (x) = | x | = ⎪⎨− x, if x < 0
⎪ 0, if x = 0

is continuous at x = 0 but not differentiable at x = 0.
76 Mathematical Methods

Solution The function given in this example is the absolute value function
and from the graph of this function (cf., Fig. 2.12), it is clear that this is
a continuous function. The continuity of this function can also be verified
otherwise as follows:
We have
right hand limit = lim f ( x) = lim | x | = lim x = 0
x → 0+ x → 0+ x → 0+

and
left hand limit = lim f ( x) = lim | x | = lim (− x) = 0
x → 0− x → 0− x → 0−

which shows that the left and right hand limits exist and are equal, thus
lim f ( x) = 0. Moreover, f (0) = 0, which is same as the limit of the
x→0
function at x = 0. Therefore, the given function is continuous at x = 0.
We shall now check the differentiability of this function as follows:
We have
f (0 + h) − f (0) |0+h|−|0| h
R f ′(0) = lim = lim = lim = 1
h→0 h h → 0 h h → 0 h
and
f (0 − h) − f (0) |0−h|−|0| h
L f ′ (0) = lim = lim = lim = –1
h→0 −h h→0 −h h → 0 −h

which shows that the right and left hand derivatives are not same.
Therefore, the given function is not differentiable at x = 0.
Example 3.3 Show that the function
⎧⎪ x 2 sin(1/ x), if x ≠ 0
f (x) = ⎨
⎪⎩0, if x = 0
is continuous and differentiable at x = 0.
Solution We have
f (0 + h) − f (0) (0 + h) 2 sin (1/(0 + h)) − 0
R f ′(0) = lim = lim
h→0 h h→0 h
= lim h sin (1/ h) = 0
h→0

and similarly L f ′(0) = 0, which shows that the function f (x) is


differentiable at x = 0. Therefore, from Theorem 3.1, the given fuction is
continuous at x = 0.
Differentiation 77

Example 3.4 Test whether the function


⎧1 + x, if x ≤ 2
f (x) = ⎨
⎩5 − x, if x > 2
is continuous and differentiable at x = 2.
Solution Here f (2) = 3, and
right hand limit = lim f ( x) = lim f (2 + h) = lim {5 − (2 + h)} = 3
x → 2+ h →0 h→0

and
left hand limit = lim f ( x) = lim f (2 − h) = lim {1 + (2 − h)} = 3
x → 2− h →0 h→0

which shows that the left and right hand limits exist and are equal, thus
lim f ( x) = 3, which is same as the value of the function at x = 2.
x→2
Therefore, the given function is continuous at x = 2.
The differentiability of this function can be checked as follows:
We have
f (2 + h) − f (2) [5 − (2 + h)] − 3 −h
R f ′(2) = lim = lim = lim = −1
h→0 h h→0 h h→0 h
and
f (2 − h) − f (2) [1 + (2 − h)] − 3 −h
L f ′(2) = lim = lim = lim =1
h→0 −h h → 0 −h h → 0 −h
which shows that the right and left hand derivatives are not same.
Therefore, the given function is not differentiable at x = 2.
Example 3.5 Examine the differentiability of the function
⎧⎪1 − x, if x < 1
f (x) = ⎨ 2
⎪⎩ x − 1, if x ≥ 1
at x = 1.
Solution The differentiability of this function can be checked as follows:
We have
f (1 + h) − f (1) [(1 + h) 2 − 1] − 0
R f ′(1) = lim = lim = lim (2 + h) = 2
h→0 h h→0 h h→0
and
f (1 − h) − f (1) [1 − (1 − h)] − 0 h
L f ′(1) = lim = lim = lim = −1
h→0 −h h→0 −h h → 0 −h
78 Mathematical Methods

which shows that the right and left hand derivatives are not same.
Therefore, the given function is not differentiable at x = 1, although it is
continuous at x = 1.

3.3 Differentiation from the First Principle


When the derivative of a function is obtained directly from the definition
of the derivative of a function, i.e. from the Eq. (1), it is known as the
differentiation from the first principle. We shall illustrate this method
with the help of following example.
Example 3.6 Differentiate the following functions from the first principle:
(a) x (b) x2 (c) ax + b (d) (ax + b)2 (e) (x + 2)/(3x + 5) (f) sin x
(g) ex (h) log x.
Solution (a) Let y = f (x) = x. Then, from Eq. (1), we have
dy f ( x + ∆x ) − f ( x ) x + ∆x − x ∆x
= lim = lim = lim =1
dx ∆x → 0 ∆x ∆x → 0 ∆x ∆x → 0 ∆x
(b) Let y = f (x) = x2. Then, from Eq. (1), we have
dy f ( x + ∆x ) − f ( x ) ( x + ∆x ) 2 − x 2
= lim = lim
dx ∆x → 0 ∆x ∆x → 0 ∆x

x 2 + 2 x∆x + (∆x) 2 − x 2
= lim = lim (2x + ∆x) = 2x
∆x → 0 ∆x ∆x → 0

(c) Here y = f (x) = a x + b. Using Eq. (1), we have


dy f ( x + ∆x) − f ( x) a( x + ∆x) + b − (a x + b)
= lim = lim
dx ∆x → 0 ∆x ∆x → 0 ∆x

a x + a ∆x + b − a x − b a ∆x
= lim = lim =a
∆x → 0 ∆x ∆x → 0 ∆x

(d) Here y = f (x) = (a x + b)2. Using Eq. (1), we have

dy a 2 ( x + ∆x)2 + 2ab( x + ∆x) + b 2 − (a x + b) 2


= lim = 2a(a x + b)
dx ∆x → 0 ∆x
(e) Let y = f (x) = (x + 2)/(3x + 5), then Eq. (1) leads to
( x +∆x ) + 2 x+2
dy f ( x + ∆x ) − f ( x ) 3( x +∆x ) + 5
− 3x +5 −1
= lim = lim =
dx ∆x → 0 ∆x ∆x → 0 ∆x (3x + 5) 2
(f) Here y = f (x) = sin x, Eq. (1) now leads to
Differentiation 79

dy f ( x + ∆x ) − f ( x )
= lim
dx ∆x → 0 ∆x
sin( x + ∆x) − sin x
= lim
∆x → 0 ∆x
2cos( x +∆2x + x )sin ( x +∆2x − x )
= lim
∆x → 0 ∆x
cos (x + ∆x
2 )sin (∆2x )
= lim ∆x
∆x → 0
2

⎛ ∆x ⎞ sin ( ∆2x )
= lim cos ⎜ x + ⎟ lim = cos x
∆x → 0 ⎝ 2 ⎠ ∆x → 0 ∆2x
(g) Here y = f (x) = e x. Using Eq. (1), we have
dy f ( x + ∆x ) − f ( x )
= lim
dx ∆x → 0 ∆x
e x +∆x − e x
= lim
∆x → 0 ∆x
e x e ∆x − e x
= lim
∆x → 0 ∆x

x e ∆x − 1
= e lim = ex
∆x → 0 ∆x
(h) Here y = f (x) = log x. Using Eq. (1), we have
dy f ( x + ∆x ) − f ( x )
= lim
dx ∆x → 0 ∆x
log( x + ∆x) − log x
= lim
∆x → 0 ∆x

dy log ( x +∆
x )
x
= lim
dx ∆x → 0 ∆x
log (1 + ∆x
x )
= lim
∆x → 0 ∆x

1 log (1 + )
∆x
x
= lim
∆x → 0 x ∆x
x
80 Mathematical Methods

log (1 + A) 1
= lim
A→0 A x
1 log (1 + A) 1
=lim =
x A→0 A x
where A = ∆x/x and as ∆x → 0, A → 0.
In a similar way we can find the derivatives of the well known
functions. These functions and their derivatives are given in Table 3.1.

3.4 Formulas for Differentiation


Although the derivative of every well defined function can be obtained
with the help of first principle, but the process of finding the derivative
can be made more simpler, if we have certain rules for differentiation.
Some of the rules are given by the following theorem.
Theorem 3.2 Let f, g and h be real functions and c is a constant. Then
d df dg dc
(i) ( f ± g) = ± (ii) =0
dx dx dx dx
d df d df dg
(iii) (cf ) = c (iv) ( fg ) = g + f
dx dx dx dx dx
df dg
d ⎛ f ⎞ g dx − f d
( f ± g ± h) =
df dg dh
± ±
(v) ⎜ ⎟=
dx (vi)
dx ⎝ g ⎠ g2 dx dx dx dx

d df dg dh
(vii) ( f g h) = gh + fh + fg
dx dx dx dx
Table 3.1 Functions and their derivatives

y d y/d x y d y/d x

c, a constant 0 xn nx n – 1
(a x + b) n na (a x + b)n – 1 ex ex
ax ax log a log x 1/x
sin x cos x cos x – sin x
tan x sec2 x cot x – csc2 x
sec x sec x tan x csc x – cot x csc x

sin–1 x 1/ 1 − x 2 cos–1 x −1/ 1 − x 2


tan–1 x 1/(1 + x 2) cot–1 x –1/(1+ x 2)
Differentiation 81

sec–1 x 1/ x x 2 − 1 csc–1 x −1/ x x 2 − 1


sinh x cosh x coshx sinh x
tanh x sech 2x coth x – csch2 x
sech x – sech x tanh x csch x – csch x coth x
sinh–1 x 1/ 1 + x 2 cosh–1 x 1/ x 2 − 1

1
tanh–1 x 1/(1 – x 2) coth–1 x
1 − x2
1
sech–1 x csch–1 x −
−1/ x 1 − x 2 x 1 + x2

We shall now illustrate the use of Theorem 3.2 and Table 3.1 to find
the derivative of a given function with the help of the following examples.
Example 3.7 Find the derivatives of the functions given below:
(i) y = x 2 + 3x + 1 + 3x – 1.
(ii) y = x sin x.
(iii) y = x cos x log x.
(iv) y = (1 – 2 tan x) (5 + 4sin x).
(v) y = (x + sin x)/(x + cos x).
(vi) y = (log x)/x.
(vii) y = e x/x.
Solution Using Theorem 3.2 and Table 3.1, we have
(i) y ′ = 2x + 3 – 3x – 2
(ii) y ′ = 1 ⋅ sin x + x cos x
(iii) y ′ = cos x log x + x (– sin x) log x + x cos x (1/x)
(iv) y ′ = (–2sec2 x) (5 + 4sin x) + (1 – 2tan x) (4cos x)
(v) y ′ = [(1 + cos x) (x + cos x) – (x + sin x) (1 – sin x)]/(x + cos x)2
(vi) y ′ = [(1/x)x – log x]/x 2 = (1 – log x)/x 2
(vii) y ′ = (e x x – e x)/x 2 = e x (x – 1)/x 2

3.5 Derivative of a Function of Function


Uptill now we have obtained the derivatives of those types of functions
in which the dependent and independent variables are directly expressible
in terms of the other. Here, we shall discuss those types of functions in
which the two variables are connected through some other functions. All
82 Mathematical Methods

such functions are connected in a chain. Let y = f (u) and u = g(x), i.e. y
is a function of u and u is a function of x, then
dy dy du
= × (7)
dx du dx
Similarly, if y = f (u), u = g(v), v = h(w) and w = i(x), then
dy dy du dv dw
= × × × (8)
dx du dv dw dx
Equations (7) and (8) are often known as derivative of a function by
chain rule.
Example 3.8 Find dy/dx of the following functions:
(i) y = cos x 2 (ii) y = log (log x)
(iii) y = ecos x (iv) y = sin log (1 + x 2)

⎡ b + a cos x ⎤
(v) y = log {x + x 2 + a 2 } (vi) y = cos–1 ⎢ ⎥
⎣ a + b cos x ⎦
Solution (i) Put u = x 2 so that y = cos u and d y/du = – sin u and d u/dx =
2x. Thus

dy dy du
= = – 2x sin x 2
dx du dx
(ii) Put u = log x so that y = log u. Thus
dy dy du 1
= =
dx du dx x log x
(iii) Put u = cos x so that y = e u. Thus
dy dy du
= = e u (– sin x) = – sin xecos x
dx du dx
(iv) Put z = log(1 + x 2) and t = 1 + x 2 so that y = sin z and z = log t which
on differentiation leads to dy/dz = cos z, dz/dt = 1/t and dt/dx = 2x. Thus
dy dy dz dt ⎛ 1⎞ 2x
= = (cos z ) ⎜ ⎟ (2 x) = coslog(1 + x 2 )
dx dz dt dx ⎝t ⎠ 1 + x2
(v) Here y = log {x + x 2 + a 2 }. Put z = {x + x 2 + a 2 } and
t= x 2 + a 2 so that y = log z and z = x + t. Thus
Differentiation 83

dy 1 dz dt x x + x2 + a2
= , =1+ =1+ =
dz z dx dx x2 + a2 x2 + a2
Therefore
dy dy dz 1
= =
dx dz dx x2 + a2
⎡ b + a cos x ⎤ b + a cos x
(vi) Here y = cos–1 ⎢ ⎥. Put z = a + b cos x so that y = cos–1 z
⎣ a + b cos x ⎦
and we have
dy 1 dz ( a + b cos x)( − a sin x) + (b + a cos x) (b sin x)
=− and =
dz 1− z 2 dx ( a + b cos x) 2
Therefore

dy dy dz a 2 − b2
= =
dx dz dx a + b cos x

3.6 Trigonometric Transformation and Differentiation by


Substitution
To differentiate a given function, sometimes it is convenient to simplify
the given function by making use of a trigonometric or algebraic
transformation. The following transformations may prove helpful in the
simplification of a given function (see also Appendix C).
Trigonometric functions
(i) sin 2 A = 2sin A cos A
(ii) cos 2A = cos2 A – sin 2 A = 2 cos2 A – 1 = 1 – 2 sin2 A
2tan A
(iii) sin 2A =
1 + tan 2 A
1 − tan 2 A
(iv) cos 2A =
1 + tan 2 A
2tan A
(v) tan 2A =
1 − tan 2 A
(vi) sin 3A = 3sin A – 4sin3 A
(vii) cos 3A = 4 cos3 A – 3 cos A
84 Mathematical Methods

3tan A − tan 3 A
(viii) tan 3A =
1 − 3tan 2 A
(ix) sin(A ± B) = sin A cos B ± cos A sin B
(x) cos(A ± B) = cos A cos B B sin A sin B
tan A ± tan B
(xi) tan (A ± B) =
1 B tan A tan B
Inverse trigonometric functions
(i) sin –1 (sin x) = sin (sin – 1 x) = x
(ii) cos – 1 (cos x) = cos (cos – 1 x) = x
(iii) tan – 1 (tan x) = tan (tan – 1 x) = x
⎛x± y⎞
(iv) tan – 1 x ± tan – 1 y = tan – 1 ⎜ ⎟
⎝ 1 B xy ⎠
Hyperbolic functions
e x − e− x e x + e− x
(i) sinh x = , cosh x =
2 2
(ii) cosh 2 x – sinh 2 x = 1, sech 2 x = 1 – tanh 2 x,
2 sinh x cosh x = sinh 2x, cosh2 x + sinh2 x = cosh 2x.
Inverse hyperbolic functions
e y − e− y
Let y = sinh –1 x so that x = sinh y = which leads to e 2y – 2xe y
2
– 1 = 0. This is quadratic equation in e y and we have

x 2 + 1 ⇒ y = log[ x + x + 1]
2
ey = x ±
It may be noted here that x + x 2 + 1 is positive and x – x 2 + 1 is
negative for every real number x, whether positive or negative. Since the
logarithm of a negative number has no meaning in the set of real numbers
and thus x – x 2 + 1 need not to be considered. Therefore

sinh–1 x = log[ x + x2 + 1], cosh–1 x = log[ x + x 2 − 1],


1+ x
tanh–1 x = log , | x | < 1.
1− x
The following example illustrates the use of some of the trigonometric
transformations.
Differentiation 85

Example 3.9 Find dy/dx of the following functions:


⎛ 1 − x2 ⎞
(i) y = tan −1 ⎜⎜ 2⎟⎟
⎝1 + x ⎠

(
(ii) y = sin −1 2 x 1 − x 2 )
(iii) y = cos –1 (4x 3 – 3x)
−1 ⎛ 2 x ⎞
(iv) y = sin ⎜ ⎟
⎝1 + x2 ⎠
⎛ 1 + x2 − 1 ⎞
−1
(v) y = tan ⎜ ⎟
⎜ x ⎟
⎝ ⎠
⎧ 2⎫
−1 ⎪ 1 + x + 1 − x ⎪
2
(vi) y = tan ⎨ ⎬
⎩⎪ 1 + x − 1 − x ⎭⎪
2 2

Solution (i) Let x2 = tan θ. Then


⎛ 1 − tan θ ⎞ −1 ⎡ ⎛π ⎞⎤ π π
y = tan −1 ⎜ −1 2
⎟ = tan ⎢tan ⎜ − θ ⎟⎥ = − θ = − tan x
⎝ 1 + tan θ ⎠ ⎣ ⎝4 ⎠⎦ 4 4
Therefore
dy 2x
=−
dx 1 + x4
–1
(ii) Let x = sin θ so that θ = sin x. Then

y = sin ⎡⎢2sin θ 1 − sin θ ⎤⎥ = sin–1 (2 sin θ cos θ)


−1 2
⎣ ⎦
= sin–1 (sin 2θ) = 2θ = 2sin–1 x
Therefore
dy 2
=
dx 1 − x2
–1
(iii) Put x = cos θ so that θ = cos x and
y = cos –1 (4 cos3 θ – 3 cos θ) = cos –1 (cos 3θ) = 3θ = 3cos –1 x
Therefore
dy 3
= −
dx 1 − x2
–1
(iv) Put x = tan θ so that θ = tan x and
86 Mathematical Methods

−1 ⎛ 2tan θ ⎞ –1 –1
y = sin ⎜ ⎟ = sin (sin 2θ) = 2θ = 2tan x
⎝ 1 + tan 2 θ ⎠
Therefore
dy 2
=
dx 1 + x 2
(v) Put x = tan θ so that θ = tan –1 x and
⎡ 1 + tan 2 θ − 1⎤
−1
y = tan ⎢ ⎥
⎢⎣ tan θ ⎥⎦

−1 ⎛ sec θ − 1 ⎞
= tan ⎜ ⎟
⎝ tan θ ⎠
⎛ 1 − cos θ ⎞
= tan −1 ⎜ ⎟
⎝ sin θ ⎠
−1 ⎛ 2sin 2 θ /2 ⎞
= tan ⎜⎜ ⎟⎟
⎝ 2sin θ /2 cos θ / 2 ⎠
1
= tan–1 (tan θ/2) = θ/2 = tan–1 x
2
Therefore
dy 1
=
dx 2(1 + x 2 )
(vi) Put x2 = cos 2θ so that

−1
⎡ 1 + cos2θ + 1 − cos2θ ⎤
y = tan ⎢ ⎥
⎣⎢ 1 + cos2θ − 1 − cos2θ ⎦⎥
⎡ cos θ + sin θ ⎤
= tan −1 ⎢ ⎥
⎣ cos θ − sin θ ⎦
−1 ⎡1 + tan θ ⎤
= tan ⎢ ⎥
⎣1 − tan θ ⎦
−1 ⎡ ⎛π ⎞⎤
= tan ⎢tan ⎜ + θ ⎟⎥
⎣ ⎝ 4 ⎠⎦
π π 1
= + θ = + cos −1 x 2
4 4 2
Differentiation 87

Therefore
dy x
=−
dx 1 − x4

3.7 Differentiation of Implicit Function


So far we have calculated the derivative of a function y = f (x), i.e. when
y is explicitly written as a function of x. However, there are situations
when an equation involving x and y is expressed in such a manner that y
depends on x implicitly. If we differentiate this equation, then such type
of differentiation is known as implicit differentiation. In this method, we
differentiate each term of the given equation; and if the term contains x
alone, it is differentiated by the methods we have learnt so far while the
terms containing y are first differentiated with respect to y (using the
above methods as in the case when y is a function of x) and then multiplied
by dy/dx. Finally, the coefficients of dy/dx are collected to get the desired
value. The following examples illustrate the method.
Example 3.10 Find dy/dx of the following functions:
(i) y 2 = 4ax
(ii) ax 2 + 2hxy + by 2 + 2gx + 2 f y + c = 0
(iii) x 2/3 + y 2/3 = 64
Solution (i) Differentiating both sides of y 2 = 4ax, we get
dy dy 2a
2y = 4a ⇒ =
dx dx y
(ii) Differentiating both sides of ax + 2hxy + by 2 + 2gx + 2 f y + c = 0,
2

we get
⎛ dy ⎞ dy dy
2ax + 2h ⎜ x + y ⋅ 1⎟ + 2by + 2 g + 2 f =0
⎝ dx ⎠ dx dx
which leads to
dy ax + hy + g
=−
dx hx + by + f
(iii) Differentiating both the sides of the given equation, we have
1/3
2 23 −1 2 23 −1 dy dy ⎛ y⎞
x + y =0 ⇒ = −⎜ ⎟
3 3 dx dx ⎝x⎠
Example 3.11 If y = x+ x+ x + ...∞ , then find the derivative of y
with respect to x.
88 Mathematical Methods

Solution Here

y = x + x + x + ...∞ ⇒ y = x + y ⇒ y2 = x + y
which on differentiation leads to
dy dy dy 1
2y =1+ ⇒ =
dx dx dx 2 y − 1

3.8 Logarithmic Differentiation


Sometimes it becomes easier if we take the logarithm of the given function
before finding out the derivative of this function. This type of
differentiation is known as logarithmic differentiation. Such situations
often occur in the following two cases:
(a) When the given function is a product (or quotient) of a number of
simpler functions. In this case, the logarithm converts the product (or
quotient) into the sums (or differences) which in turn makes the
differentiation process much simpler.
(b) When the variable x occurs in the exponent and here the logarithm
converts the given function into a manageable form.
We shall now illustrate this process with the help of the following
example.
Example 3.12 Find dy/dx when

x4 x + 6
(i) y = (x + 1)2 (x + 2)3 (x + 3)4 (ii) y =
(3x + 5) 2
x
x
(iii) y = x x (iv) y = x
x + e x ... ∞
x+e
(v) y = e (vi) x y = e x–y
yx
(vii) y = x
Solution (i) Here y = (x + 1)2 (x + 2)3 (x + 3)4 which on taking the
logarithm of both sides leads to log y = 2 log (x + 1) + 3 log (x + 2) +
4log(x + 3). Differentiation of both sides of this equation now yields
1 dy 2 3 4
= + +
y dx x + 1 x + 2 x + 3
so that
dy ⎡ 2 3 4 ⎤
= (x + 1)2 (x + 2)3 (x + 3)4 ⎢ + + ⎥
dx ⎣ x + 1 x + 2 x + 3⎦
Differentiation 89

(ii) Taking logarithm of both sides of the given function, we get


⎡ x4 x + 6 ⎤
log y = log ⎢
2⎥
= log x 4 + log x + 6 – log(3x + 5)2
⎣ (3x + 5) ⎦
1
= 4log x + log (x + 6) – 2log(3x + 5)
2
Now, differentiating with respect to x, we get
1 dy 1 1 1
=4 + (1) − (2) (3)
y dx x 2( x + 6) 3x + 5
Thus
dy ⎡4 1 6 ⎤
= y⎢ + − ⎥
dx ⎣ x 2( x + 6) 3 x + 5 ⎦
(iii) Taking the logarithm of both sides of y = x x, we get log y = log x x
= x log x. Now, differentiate both sides of the equation so that
1 dy 1
= x + log x (1)
y dx x
Thus
dy
= y(1 + log x) = x x (1 + log x)
dx
(iv) Taking the logarithm of both sides of the given function, we have
log y = x x log x. Taking the logarithm again so that
log log y = log(x x log x) = log x x + log log x = x log x + log log x
Now, differentiating with respect to x, we get
1 1 dy 1 1 1
= x + log x(1) +
log y y dx x log x x
so that
dy xx x ⎛ 1 ⎞
= x x log x ⎜1 + log x + ⎟
dx ⎝ x log x⎠
(v) Here y = e x + y. Taking the logarithm of both sides, we get
log y = (x + y) log e = x + y which on differentiation leads to
1 dy dy
= 1+
y dx dx
90 Mathematical Methods

Thus
dy y
=
dx 1− y
(vi) Taking the logarithm of both sides of the given function, we get
x
y log x = (x – y) log e = x – y so that y = which on differentiation
1 + log x
gives
dy log x
=
dx (1 + log x) 2
(vii) Taking the logarithm of both sides of the given function, we get
log y = y x log x which on taking the logarithm again becomes log log y
= x log y + log log x. Now, differentiating both sides of this equation, we
get

1 1 dy 1 dy 1 1
=x + log y (1) +
log y y dx y dx log x x
so that, after simplifying, we get

dy y log y (1 + x log x log y )


=
dx x log x(1 − x log y )

3.9 Parametric Differentiation


If x and y are functions of another variable, say, t, i.e. x = f (t) and
y = g(t), then these equations are known as parametric equations and t is
the parameter. In such situations, the derivative of y with respect to x is
given by
dy
dy dt
= dx
dx dt
Such type of differentiation is known as the parametric differentiation
and is illustrated with the help of the following example.
Example 3.13 Find the differential coefficient of y with respect to x
when
a(1 − t 2 ) 2bt
(i) x = and y =
1 + t2 1 + t2
(ii) x = a(θ – sin θ) and y = a (1 – cos θ)
Differentiation 91

Solution (i) Here


a(1 − t 2 ) dx (1 + t 2 ) (−2at ) − a(1 − t 2 ) (2t ) − 4at
x= ⇒ = =
1 + t2 dt (1 + t 2 )2 (1 + t 2 )2
and
2bt dy (1 + t 2 ) (2b) − (2bt ) (2t ) 2b(1 − t 2 )
y= ⇒ = =
1 + t2 dt (1 + t 2 ) 2 (1 + t 2 ) 2
Therefore
dy
dy dt −b(1 − t 2 )
= dx
=
dx dt
2at
(ii) Here x = a(θ – sin θ) and y = a(1 – cos θ) and thus
dx dy
= a (1 − cos θ), = a sin θ
dθ dθ
which leads to
dy
dy dθ sin θ
= dx
=
dx dθ
1 − cos θ
x2
Example 3.14 Differentiate with respect to x 2.
1 + x2
x2 du
Solution Let u = and v = x 2. Then, we have to find . Now
1 + x2 dv
du 2x dv du du dv 1
= , = 2x, and therefore = / = .
2 2 dv dx dx (1 + x 2 ) 2
dx (1 + x ) dx
Example 3.15 Differentiate cos –1 x with respect to 1 − x .
Solution Ley y = cos –1 x and z = 1 − x . Then
dy 1 dz −1
= − and = .
dx 2 1− x x dx 2 1 − x
Therefore
dy dy dx 1
= × =
dz dx dz x
⎛ 2x ⎞
Example 3.16 Differentiate tan −1 ⎜ ⎟ with respect to
⎝1 − x2 ⎠
⎛ 2x ⎞
sin −1 ⎜ ⎟.
⎝ 1 + x2 ⎠
92 Mathematical Methods

2x ⎞ ⎛ 2x ⎞
Solution Let y = tan −1 ⎛⎜ and z = sin −1 ⎜ ⎟ , then we have to
2⎟
⎝1 − x ⎠ ⎝1 + x2 ⎠
find dy/dz. Substituting x = tan θ in each of the equation for y and z, we
get
−1 ⎛ 2tan θ ⎞
y = tan ⎜ ⎟ = tan–1 (tan 2θ) = 2θ
⎝ 1 − tan 2 θ ⎠
and
−1 ⎛ 2tan θ ⎞ –1
z = sin ⎜ ⎟ = sin (sin 2θ) = 2θ
⎝ 1 + tan 2 θ ⎠
These two equations lead to y = z and thus dy/dz = 1.

3.10 Higher Order Derivatives


In some geometrical and physical problems, we need the notion of higher
derivatives which we shall define as follows:
Let y = f (x) be a function. If it is differentiable, then y ′ = f ′(x) is
another function. We call this function f ′ or y′ as the first dervative of y.
If y′ or f ′ is differentiable, we then call it the second derivative of f and
denote it by f ″ or y″. Also, if f ″ is differentiable, we get another function
f ′″, known as the third derivative of f. Continuing this process of
differentiation n times, we get the n th derivative of y = f (x). This
derivative is denoted by y(n), or f (n) (x), or d (n) y/dx (n).
Example 3.17 Find the second derivative of
(i) y = log log x (ii) (a + bx) e y/x = x
Solution (i) Differentiate y = log log x with respect to x, we get
dy 1 d 1
= (log x) =
dx log x dx x log x
Differentiating again, using the quotient formula for differentiation, we
get
d2y 1 + log x
2
=−
dx ( x log x) 2
(ii) Taking logarithm of both sides of the given function so that
y
log (a + bx) + log e = log x ⇒ y = x[log x – log(a + bx)]
x
which on differentiation leads to
Differentiation 93

dy a
= log x – log(a + bx) +
dx a + bx
Differentiating again, we get
d2y a2
=
dx 2 x(a + bx) 2
Example 3.18 If x = a(θ – sin θ) and y = a(1 – cos θ), then find the
second derivative of y with respect to x at the point θ = π.
Solution: Differentiate x = a(θ – sin θ) and y = a(1 – cos θ) with respect
to θ, we get
dx dy dy dy dx sin θ
= a(1 – cos θ), = a sin θ ⇒ = / =
dθ dθ dx d θ d θ 1 − cos θ
Differentiating again, we get
d2y d ⎛ sin θ ⎞ d θ 1
= ⎜ ⎟ =−
dx 2 d θ ⎝ 1 − cos θ ⎠ dx a(1 − cos θ)2
Therefore at θ = π, d 2y/dx 2 = – 1/4a.
Example 3.19 Find the nth derivative of the following functions:
(i) y = (ax + b)n
(ii) y = a0 + a1x + a2 x 2 + ... + an x n
(iii) y = log (a x + b)
(iv) y = e ax
(v) y = sin (a x + b)
Solution (i) Differentiate y = (a x + b) n successively with respect to x, we
get
dy
= na(a x + b)n–1
dx
d2y
= n(n – 1) a2(ax + b)n – 2
dx 2

d3y
= n(n – 1) (n – 2)a3 (a x + b)n – 3
dx3
.......................................................
.......................................................
dny
= n(n – 1) (n – 2) ... {n – (n – 1)} a n (ax + b)n–n = n! a n
dx n
94 Mathematical Methods

(ii) Let y = a0 + a1 x + a2 x 2 + ... + an x n. Differentiating y with respect to


x, n times, we get
dy
= a1 + 2a2 x + ... + n an x n – 1
dx
d2y
= 2a2 x + ... + n(n – 1) an x n – 2
dx 2
d3y
= 3a3 x + ... + n(n – 1) (n – 2) an x n – 3
dx3
.......................................................
.......................................................
dny
= n(n – 1) (n – 2) ... {n – (n – 1)} a nx n – n
dx n
= n(n – 1) (n – 2) ... 1an x0 = n! an
(iii) Differentiating n times the given function y = log (ax + b) with
respect to x, we get
dy 1
= a = (a x + b)–1 a
dx ax + b
d2y
= (– 1) (a x + b)– 2 a 2
dx 2
d3y
= (–1) (–2) (ax + b)–3a 3 = (–1)2 2! (ax + b)–3a3
dx3
d4y
= (–1) (–2) (–3) (ax + b)– 4 a4 = (–1)3 3! (ax + b)– 4 a4
dx 4
...........................................................................................
...........................................................................................
dny an
= (–1)n – 1 (n – 1)! (ax + b)– n a n = (–1)n – 1 (n – 1)!
dx n (ax + b) n
(iv) Differentiate y = e ax successively, we get
y′ = ae a x, y ″ = a 2e ax, y ″′ = a 3e ax, ..., y(n) = a ne a x
(v) Differentiating y = sin (ax + b) with respect to x, n times, we get
dy π
= a cos (a x + b) = a sin{(a x + b) + }
dx 2
Differentiation 95

d2y ⎛ π⎞ ⎧⎛ π ⎞ π⎫
= a 2 cos ⎜ ax + b + ⎟ = a 2 sin ⎨⎜ ax + b + ⎟ + ⎬
dx 2 ⎝ 2 ⎠ ⎩⎝ 2 ⎠ 2⎭

⎧ π⎫
= a sin ⎨(ax + b) + 2 ⎬
2
⎩ 2⎭

d3y ⎛ π⎞ ⎧⎛ π ⎞ π⎫
= a cos ⎜ ax + b + 2 ⎟ = a sin ⎨⎜ ax + b + 2 ⎟ + ⎬
3 3
dx 3
⎝ 2⎠ ⎩⎝ 2 ⎠ 2⎭

⎧ π⎫
= a sin ⎨(ax + b) + 3 ⎬
3
⎩ 2⎭
...................................................................................
...................................................................................
dny ⎧ nπ ⎫
= a sin ⎨(ax + b) + ⎬
n
n
dx ⎩ 2⎭

3.11 Applications of Derivatives


As discussed in Sec. 3.1, differential calculus has a wide range of
applications in science and engineering as well as in social sciences and
business and it is therefore this section is devoted to some of the
applications of differential calculus.
3.11.1 Motion in a Straight Line
When the distance s travelled by a particle, from a given fixed point, is
expressed as a function of time, say s = f (t), then f ′(t) = ds/dt represents
the velocity of the particle (denoted by v) and f ″(t) = d 2s/dt 2 represents
the acceleration of the particle at time t, and is denoted by a.
Example 3.20 A particle moves along a straight line according to the
law s = 12 t 3 – 2t. Find its velocity and acceleration at time t = 2.
1 3
Solution Differentiate s = t – 2t twice go get
2
ds 3 d 2s
= v = t 2 − 2 and = a = 3t
dt 2 dt 2
so that at t = 2,
v = 4, and a = 6
96 Mathematical Methods

Example 3.21 A car is running on a straight road. The distance travelled


and the time taken are connected by the formula s = t 2 – 2t, where t is
measured in hours and s in kilometers. What is the reading of the
speedometer when the odometer reading is 15?
Solution It may be noted that odometer measures the distance s and the
speedometer measures the velocity v. Here, we can take t ≥ 0. The time
taken to cover the distance of 15 kilometer (odometer reading) can be
obtained by putting s = 15 in the equation s = t2 – 2t, which on solving
for t yields t = –3, t = 5. The value t = –3 can be neglected. Now
differentiate s = t 2 – 2t with respect to t so that v = ds/dt = 2t – 2. Thus
the velocity of the car at t = 5 is 8. Therefore, the speedometer reading is
8 km/h when the odometer reading is 15.
3.11.2 Motion Under Gravity
The motion of a body falling under gravity is a particular instance when
the body is moving in a straight line. The acceleration of the falling
body due to gravity, towards the centre of the earth, is known to be as
g = 32 ft/s2 or 9.8 meter/s2. Here, we shall use differentiation to solve
certain problems involving such type of motion.

Example 3.22 A stone thrown vertically upwards falls back on the ground
after 8 s. Assuming that the equation of motion is given by s = ut – 4.9t 2,
when s is in meters and t is in seconds, find the velocity at t = 0 and
t = 1.

Solution Here, the stone takes 8 s to fall back on the ground,


which means that the distance covered in 8 s is zero and thus equation
s = ut – 4.9t 2, leads to u = 39.2. Now, differentiating s = ut – 4.9t 2,
we get
ds ds
= u – (4.9) (2)t ⇒ = 39.2 – 9.8t (as u = 39.2)
dt dt
Therefore
⎡ ds ⎤
velocity at t = 0 is ⎢ ⎥ = 39.2 m/s
⎣ dt ⎦ t =0
and
ds
velocity at t = 1 is ⎡⎢ ⎤⎥ = 39.2 – (9.8) (1) = 29.4 m/s
⎣ dt ⎦ t =1
Differentiation 97

Example 3.23 A person standing on a pole of height 12 m throws a stone


vertically upward. It moves in a vertical line slightly away from the
line of the pole, and falls on the ground. If the equation of motion is
s = 16.9 t – 4.9 t 2, how much time does it take for the upward and
downward motions?

Solution Differentiate s = 16.9 t – 4.9 t 2, to get ds/dt = 16.9 – 9.8 t. This


velocity is zero when the height is maximum, i.e., 16.9 t – 4.9 t 2 = 0,
which gives t = 169/98 s. Thus the time taken for the upward motion is
169/98 s. Also, the time taken for the downward motion is equal to the
time taken for the upward motion plus time taken for covering 12 m. To
find the time for downward motion of 12 m, we put s = 12 in the equation
s = 16.9 t – 4.9 t 2 to get t = 1 (neglecting the negative value). Hence, the
time taken for the downward motion is (169/98) + 1 = 267/98 s.

Example 3.24 Two balls are thrown up simultaneously. Their equations


of motions are, respectively, s = 19.6t – 4.9 t 2 for the first ball and
s = 9.8 t – 4.9 t 2 for the second ball. What is the height of the second ball
when the height of the first ball is maximum?

Solution The equation of motion for the first ball is s = 19.6 t – 4.9 t 2
which on differentiation leads to ds/dt = v = 19.6 – 9.8 t. When the
height is maximum, this velocity is zero and this leads to t = 2. Therefore
the first ball reaches the maximum height after 2 seconds.

Now, the equation of motion for the second ball is s = 9.8 t – 4.9 t 2
which at t = 2 gives the height attained by the second ball as s = (9.8) (2)
– (4.9) (22) = 0. Therefore, the second ball will have fallen back on the
ground when the first ball reaches the maximum height.
3.11.3 Rate of Change of Quantities
From the above discussion, it may be noted that ds/dt represents velocity
which is the rate of change of distance s with respect to time t. In a
similar way, whenever one quantity y varies with another quantity x,
according to the rule y = f (x), then f ′ (x0) represents the rate of change
of y with respect to x at x = x0. In this section, we shall consider some
examples of this nature. Moreover, if both x and y are changing with
time t (i.e., x and y are functions of time t), then
dy dy dx dx
= = f ′( x)
dt dx dt dt
98 Mathematical Methods

Thus, the rate of change of one variable can be calculated if the rate of
change of other variable is known.

Example 3.25 The weekly profit P, in rupees, of a company is determined


by the number x of television sets produced per week according to the
formula P = 200 x – 0.03 x 2 – 15000. Find the rate at which the profit is
changing when the production level x is 1000 television sets per week.

Solution The profit is governed by the equation P = 200 x – 0.03 x 2 –


15000 which gives the rate of change of profit P with respect to the
production level x as dP/dx = 200 – 0.06 x so that when x = 1000,
dP/dx = 140 per television set.

Example 3.26 An edge of a variable cube is increasing at the rate of 3


cm. per second. How fast is the volume of the cube increasing when the
edge is 10 cm. long?

Solution Let x be the side of the cube and V the volume of the
cube at time t. Then V = x 3 ⇒ dV/dt = (3x 2) dx/dt. But it is given
that dx/dt = 3 cm/sec. Therefore when x = 10, dV/dt = 3 × (10)2 × 3 = 900
cm3 s–1.

y
60 m

Fig. 3.3
Example 3.27 A man is walking at the rate of 8 km per hour towards the
foot of a tower 60 m high. At what rate is he approaching the top when
he is 80 m from the foot of the tower?

Solution At any instant, let x be the distance of the man from the foot
and y be his distance from the top of the tower, then from the Pythagoras
theorem (Fig. 3.3) y 2 = x 2 + (60)2, which on differentiation leads to

dy dx dy x dx
2y = 2x ⇒ = (9)
dt dt dt y dt
Differentiation 99

When x = 80 m, y = (80)2 + (60) 2 = 100 m. But it is given that


dx/dt = – 8 km/h. Therefore, from Eq. (9)
dy/dt = (80/100) × (– 8) = – 6.4 km/h

Example 3.28 A conical vessel whose height is 10 m and the radius of


whose base is 5 m is being filled with a liquid at a uniform rate of 3/2
cubic meter per minute. Find the rate at which the level of water in the
vessel is rising when the depth is 4 m.

Solution From the Fig. 3.4, ∆OCD ~ ∆OAB. Therefore, r/5 = h/10, which
gives r = h/2. Let V be the volume of the liquid. Then

1 2 π h2 π 3
V= πr h = h= h
3 3 4 12
so that
dV π 2 dh
= 3h
dt 12 dt

A
B

D
C

Fig. 3.4
But it is given that dV/dt = 3/2 when h = 4. Thus from the above equation,
we have dh/dt = (3/8 π) m/min.
Example 3.29 Liquid is running out of a conical funnel at the rate of
5 cm3 s– 1. If the radius of the base of the funnel is 10 cm and the altitude
is 20 cm, find the rate at which the level of the liquid is decreasing when
it is 5 cm from the top.
Solution Let r be the radius and h the height of the surface of the liquid
at time t, and V, the volume of the liquid in the conical funnel. From the
similar triangles (Fig. 3.5), we have
100 Mathematical Methods

r h 1
= ⇒ r= h
10 20 2
Moreover,
1 2 1 dV 1 2 dh
V= π r h = π h3 and = πh
3 12 dt 4 dt
dV
so that when = –5 and h = 20 – 5 = 15
dt
dh 4
=– cm3 s–1
dt 45π

Fig. 3.5

3.11.4 Tangents and Normals


Let y = f (x) be a continuous function, then we have seen earlier that
dy/dx at any point P(x, y) of the curve y = f (x) represents the slope of the
tangent at the point P. If the angle which a tangent makes with the positive
direction of x-axis (Fig. 3.1) and is denoted by ψ, then
dy
= tan ψ = slope of the tangent at (x, y)
dx
If the tangent is parallel to x-axis, then ψ = 0, and we have
dy
= tan ψ = tan 0 = 0
dx
If the tangent is perpendicular to x-axis, then ψ = 90°, and we have
dx 1 1
= = =0
dy dy / dx tan ψ
Differentiation 101

P(x1, y1)

ψ
x
O

Fig. 3.6 Tangent to a curve


Consider Fig. 3.6 and let P(x1, y1) be any point on the curve y = f (x),
then the equation of the tangent at P is a line whose equation is
y – y1 = m(x – x1)
⎡ dy ⎤
where m = slope of the tagent at P = ⎢ ⎥ .
⎣ dx ⎦ ( x1 , y1 )
It is known that the normal to the curve y = f (x) is a line perpendicular
to the tangent at the point P, and the two straight lines are perpendicular
to each other if the product of their slopes is –1. Thus the slope of the
normal is –1/m = −1/(dy / dx)( x1 , y1 ) . Therefore, the equation of the normal
is
1
y – y1 = − (x – x1)
m
⎡ dy ⎤
where m = slope of the tangent at P = ⎢ ⎥ .
⎣ dx ⎦ ( x1 , y1 )
Example 3.30 Find the equation of the tangent to the ellipse
x2 y2
+ = 1 at the point (x1, y1).
a2 b2

dy b2 x
Solution Differentiate the given equation of the ellipse to get =− 2 .
dx a y
Thus
⎡ dy ⎤ b 2 x1
slope of the tangent at (x1, y1) = ⎢ ⎥ = −
⎣ dx ⎦ ( x1 , y1 ) a 2 y1
102 Mathematical Methods

Therefore, the equation of the tangent at (x1, y1) is


⎡ b2 x ⎤
y – y1 = ⎢− 2 1 ⎥ (x – x1)
⎢⎣ a y1 ⎥⎦
which can be written as

xx1 yy1 x12 y12


+ = +
a2 b2 a2 b2
Now, since the point (x1, y1) lies on the ellipse, we have

x12 y12
+ =1
a2 b2
Hence, the equation of the tangent at (x1, y1) is
xx1 yy1
2
+ =1
a b2
Example 3.31 Find the equation of the normal at the point (am 2, am3)
for the curve ay 2 = x 3.
Solution Differentiate the equation of the curve ay 2 = x 3 with respect to
x, we get
dy dy 3 x 2
2ay = 3x 2 ⇒ =
dx dx 2 ay

⎛ dy ⎞
Therefore, the slope of the tangent at (am2, am3) is ⎜ ⎟ =
⎝ dx ⎠( am2 , am3 )
3 a 2 m 4 3m
= ; and the slope of the normal at (am2, am3) is given
2 a 2 m3 2
2
by – . Therefore, the equation of the normal at (am2, am3) is
3m
2
y – am3 = − (x – am 2) ⇒ 2x + 3my – am 2 (2 + 3m 2) = 0
3m

3.11.5 Increasing and Decreasing Functions


Definition 3.3 If y = f (x) is a function of x and y increases as x increases
in a certain interval, then y is called an increasing function in that interval;
while if y = f (x) is a function of x and y decreases as x increases in a
certain interval, then y is called a decreasing function in that interval.
Differentiation 103

Thus, y = f (x) is an increasing function of x over a subset of its


domain if x1 < x2 ⇔ f (x1) < f (x2), and y = f (x) is a decreasing function
of x over a subset of its domain if x1 < x2 ⇔ f (x1) > f (x2) (Fig. 3.7).
y y
y = f ( x)

y = f (x)

f ( x2) f (x1)
f (x2)
x x
0 x1 x2 0 x1 x2

Fig. 3.7 Increasing and decreasing function


From the graph of the function f (x) = sin x (cf. Fig. 2.17) it may be
π
noted that it is an increasing function of x in the interval ⎛⎜ 0, ⎞⎟ and a
⎝ 2⎠
⎛π ⎞
decreasing function of x in the interval ⎜ , π ⎟ . At x = π/2, the function
⎝2 ⎠
sin x attains its maximum value 1, and at this point the function is neither
increasing nor decreasing. At this point the tangent to the curve is parallel
to x-axis and so f ′(x) = 0. The values of x for which f ′(x) = 0 {i.e., the
function f (x) is stationary} are known as critical points (turning point).
These are the points at which the curve changes its nature from increasing
to decreasing or from decreasing to increasing (Fig. 3.8).
y

y = f ( x)

a b
x

Fig. 3.8 Critical points of a curve


However, the increasing/decreasing nature of a function can easily
be known with the help of the following theorem.
104 Mathematical Methods

Theorem 3.3 Test for Increasing/Decreasing Function. A function


y = f (x) is increasing at a point if its derivative f ′(x) at that point is
positive, and y = f (x) is decreasing at a point if its derivative f ′(x) is
negative at that point.

We shall now work out few examples to illustrate the application of


this theorem.

Example 3.32 Examine whether the logarithmic function is increasing


or decreasing.

Solution Let y = log x. Then dy/dx = 1/x. Since log x is defined only for
positive values of x, therefore, 1/x > 0, which means that dy/dx > 0 and
hence the function y = log x is increasing wherever it is defined.

Example 3.33 Prove that the exponential function e x is an increasing


function.

Solution Let y = e x. Then dy/dx = e x. We have

x 2 x3 x 4
ex = 1 + x + + + + ... ≥ 1
2! 3! 4!

When x is positive, then e x is positive; and when x is negative, e x =


1/e– x = 1/(a positive quantity) is a positive quantity. Moreover, when
x = 0, then e x = 1 which is also a positive quantity. Thus dy/dx is positive
for all values of x. Therefore, the function e x is an increasing function.

It may be noted that when the interval on which the function is


defined is not mentioned, then the nature of the function (whether it is
increasing or decreasing) has to be examined on its domain, which in
our case is the set of real numbers, i.e. ¡.

Example 3.34 Show that the function x 3 – 3x + 3x – 100 is increasing


on ¡.

Solution Let f (x) = x 3 – 3x 2 + 3x – 100. Then f ′(x) = 3x 2 – 6x + 3 =


3(x – 1)2. The right hand side of this equation is a square quantity which
is always positive for all values of x, i.e., f ′ (x) = 3(x – 1)2 is positive for
all values of x. Therefore, the given function f (x) is an increasing function
on ¡.
Differentiation 105

Example 3.35 Show that the function f (x) = x 2 – x + 1 is neither


increasing nor decreasing on the interval (0, 1).

Solution Here f ′(x) = 2x – 1 which when equated to zero leads to


x = 1/2, that is, the critical value is 1/2. For x < 1/2, f ′(x) is negative and
hence f (x) is decreasing while, for x > 1/2, f ′(x) is positive and hence
f (x) is increasing. As the point x = 1/2 lies between 0 and 1, it means
that in the interval (0, 1) there is a point on the left of which f (x) is
decreasing and on the right of which, f (x) is increasing, i.e. at some
parts of the interval (0, 1), the function is increasing; and at the other
part, the function is decreasing. Hence, f (x) is neither increasing nor
decreasing on (0, 1).

3.11.6 Maxima and Minima


We shall now apply differentiation to find the maximum and minimum
values of a function and we have

Definition 3.4 A function f (x) is said to have a maximum value in the


interval I at a point x = x0, if x0 is in I and if f (x0) ≥ f (x) for all x in I. The
number f (x0) is called the maximum value of f (x) in I and x0 is called a
point of maximum of f (x) in I.

Definition 3.5 A function f (x) is said to have a minimum value in the


interval I at a point x = x0, if x0 is in I and if f (x0) ≤ f (x) for all x in I. The
number f (x0) is called the minimum value of f (x) in I and x0 is called a
point of minimum of f (x) in I.

Definition 3.6 Let f (x) be a real function and let x0 be an interior point
in the domain of f (x). We say that x0 is a local maximum of f (x) (or a
point of local maximum of f (x), or a maximum of f (x)), if there is
an open interval containing x0 such that f (x0) > f (x) for every x in
that interval.

Definition 3.7 Let f (x) be a real function and let x0 be an interior point
in the domain of f (x). We say that x0 is a local minimum of f (x) (or a
point of local minimum of f (x), or a minimum of f (x)), if there is an
open interval containing x 0 such that f (x 0) < f (x) for every x in
that interval.

The following theorem is helpful in finding the local maximum and


local minimum of functions.
106 Mathematical Methods

Theorem 3.4 Let y = f (x) be a differentiable function. Then, f ′ (x)


vanishes at every local maximum and at every local minimum.
It may be noted that the converse of this theorem is not true, i.e.
every point at which f ′(x) = 0 need not be a local maximum or
local minimum. Also, since f ′(x) is the slope of the tangent to the curve
y = f (x) at the point x0, therefore when x0 is a point of local maximum or
local minimum, the tangent at x0 is parallel to the x-axis. The values of
the function corresponding to points of local maximum or points of local
minimum are called the extreme values of the function.
The derivative f ′(x) gives us the points of local maximum or
points of local minimum. To distinguish whether the point x0 satisfying
f ′(x) = 0 is a point of local maximum or a point of local minimum, we
have the following theorem.

Theorem 3.5 First Derivative Test. Let f (x) be a differentiable function


on I and let x0 ∈ I. Then
(a) x0 is a point of local maximum of f (x) if
(i) f ′(x0) = 0,
(ii) f ′(x0) > 0 at every point close to and to the left of x0; and
f ′(x0) < 0 at every point close to and to the right of x0.
(b) x0 is a point of local minimum of f (x) if
(i) f ′(x0) = 0,
(ii) f ′(x0) < 0 at every point close to and to the left of x0; and
f ′(x0) > 0 at every point close to and to the right of x0.
(c) If f ′(x0) = 0, but f ′(x) does not change sign as x increases through
x0, then x0 is neither a point of local maximum nor a point of local
minimum.
If f ′(x0) = 0, and x0 is neither a point of local maximum nor a point
of local minimum, then x0 is called a point of inflexion.

Example 3.36 Find the local maximum/local minimum, if any, for the
following functions using the first derivative test
(i) f (x) = x 3 – 3x (ii) f (x) = sin x + cos x, 0 < x < π/2
(iii) f (x) = x 2 – 4, x > 0
3

Solution (i) Differentiating the given function, we get f ′(x) = 3x 2 –


3 = 3(x + 1) (x – 1) which when equated to zero leads to x = –1, 1.
For x = –1, let the two nearby points on the left and right of this point be
Differentiation 107

x1 = –2, x2 = 0. Then at x1 = –2, f ′(x) > 0, while at x2 = 0, f ′(x) < 0, which


shows that the given function has a maxima at x = –1. In a similar
manner, for x = 1, let the two nearby points on the left and right of this
point be x1 = 0, x2 = 2. Then at x1 = 0, f ′(x) < 0, while at x2 = 2,
f ′(x) > 0, which shows that the given function has a minima at x = 1.
Therefore the maximum value of the given function at x = –1 is f (x) = 2
while the minimum value of the given function at x = 1 is f (x) = –2.
(ii) The given function f (x) = sin x + cos x is defined in the interval
0 < x < π/2, which on differentiation yields f ′(x) = cos x – sin x. Equate
it to zero so that sin x = cos x ⇒ tan x = 1 ⇒ x = tan– 1 (1) = π/4. Consider
x1 = π/6, x2 = π/3, the two nearby points on the left and right of the
point x = π/4, so that at x1 = π/6, f ′(x) = cos(π/6) – sin(π/6) > 0, and at
x2 = π/3, f ′(x) = cos(π/3) – sin(π/3) < 0. Thus at x = π/4, the given
function has a maxima and the maximum value as 2.
(iii) The given function can be written as f (x) = x2/3 – 4, which on
differentiation leads to f ′(x) = 23 x– 1/3. Equating it to zero, we get 2 = 0
which is not possible. Hence the given function has neither maxima nor
minima.

Example 3.37 Prove that the functions


(i) y = e x (ii) y = log x (iii) y = x 3 + x 2 + x + 1
do not have maxima or minima.

Solution (i) Here y = e x ⇒ y ′ = e x. Equating it to zero, we get e x = 0,


which is not possible as e x > 0 ⇒ y = e x has neither maxima nor minima.
1
(ii) Here y = log x ⇒ y′ = . Equating it to zero, we get 1/x = 0 which
x
is not possible as x > 0 ⇒ y = log x has neither maxima nor minima.
(iii) Here y = x 3 + x 2 + x + 1 which on differentiation gives y′ = 3x 2 + 2x
−2 ± −8
+ 1. Equate it to zero and solve to get x = 6 which is a complex
number. Thus y′ ≠ 0 for any real x. Therefore, the given function has no
maxima/minima for real x.

Maximum and Minimum Values in a Closed Interval. Let f (x) be a function


defined in a closed interval [a, b] whose graph is given in Fig. 3.9. What
are the maximum and minimum values of f (x) in [a, b]? From the
Fig. 3.9, it is clear that the maximum value is f (a) and minimum value
is f (b), and let x = x1 be a local point of minimum, then the local minimum
108 Mathematical Methods

value of f (x) is f (x1). Similarly, if x = x2 is a point of local maximum,


then the local maximum value of f (x) is f (x2). But neither f (x2) is the
maximum value of f (x) in [a, b] nor f (x1) is the minimum value of f (x)
in [a, b]. In the interval [a, b], f (a) is called the absolute maximum
value of f (x), and f (b) is called the absolute minimum value of f (x). It is
possible that in a given interval, the absolute maximum value of the
function and the local maximum value may be the same.

y
A Q

P
B

x
0 a x1 x2 b

Fig. 3.9
The following theorems are helpful in finding the absolute maximum
and absolute minimum values of a function on an interval I.

Theorem 3.6 Let f (x) be a continuous function on a closed interval


I = [a, b] and differentiable throughout the open interval (a, b), then f (x)
has the maximum value and attains it atleast once in I. Also, f (x) has the
minimum value and attains it atleast once in I.
This theorem tells us that f (x) has the maximum and the minimum
value in a given closed interval.

Theorem 3.7 If a differentiable function f (x) attains its maximum value


at an interior point x0 of its domain, then f ′(x) = 0.

Theorem 3.8 If a differentiable function f (x) attains its minimum value


at an interior point x0 of its domain, then f ′(x) = 0.

These theorems allow us to frame the following rule for finding the
maximum or minimum values of a function in a given interval.

Step 1. Find all the points where f ′(x) takes the value zero.
Step 2. Consider the end points of the interval.
Differentiation 109

Step 3. Find the values of the function at the end points of the interval
and at the points obtained in step 1.
Step 4. Take the maximum and minimum values out of the values
calculated in step 3. These are the absolute maximum and absolute
minimum values of the function in a given interval.
We shall now workout few examples to illustrate the use of this rule
for finding the absolute maximum/minimum of a function.

Example 3.38 Find the absolute maximum and absolute minimum of


the following functions in the given intervals
(i) f (x) = x 3 – 3x + 2, 0 ≤ x ≤ 2
(ii) f (x) = (x – 1)1/3 (x – 2), 1 ≤ x ≤ 9
(iii) f (x) = sin x + cos x in [0, π]

Solution (i) The given function is continuous in [0, 2] and differentiable


in (0, 2) and therefore the above theorems are applicable. Now,
f ′(x) = 3x 2 – 3 = 3(x – 1) (x + 1). Take f ′ (x) = 0 which gives x = –1, 1.
Only x = 1 belongs to the interval [0, 2]. Moreover, the end points are
0 and 2. Therefore, at the point x = 1 and the end points 0 and 2, the
values of the given function, respectively, are f (1) = 0, f (0) = 2 and
f (2) = 4. Hence, the given function has an absolute maximum value
equal to 4 at the point x = 2, and the absolute minimum value equal to 0
at the point x = 1.
(ii) Differentiating the given function f (x) = (x – 1)1/3 (x – 2), we get
4x − 5
f ′(x) = which on equating to zero leads to x = 5/4. Now
3( x − 1) 2/3
finding the value of f (x) = (x – 1)1/3 (x – 2) at the point x = 5/4 and at the
end points, we get f (5/4) = –3/44/3, f (1) = 0 and f (9) = 14. Therefore,
the absolute minimum value is –3/44/3 and the absolute maximum
value is 14.
(iii) Here f (x) = sin x + cos x which on differentiation yields f ′(x) =
cos x – sin x. Put f ′(x) = 0 so that cos x – sin x = 0 ⇒ tan x = 1
⇒ x = π/4. Now, find the value of the function f (x) = sin x + cos x at
x = 0, π/4, π so that f (0) = 1, f (π/4) = 2 and f (π) = –1. Hence, the
absolute maximum value of the function is 2 and the absolute minimum
value is –1.

Example 3.39 If at x = 1, the function f (x) = x 4 – 62x 2 + ax + 9 attains


its maximum value on the interval [0, 2]. Find the value of a.
110 Mathematical Methods

Solution The given function f (x) = x 4 – 62x 2 + ax + 9 when differentiated,


leads to f ′(x) = 4x 3 – 124x + a. Put f ′(x) = 0 to get 4x 3 – 124x + a = 0.
But, it is given that x = 1 is a root of this equation (as the function attains
its maximum value at this point), and thus 4 – 124 + a = 0 ⇒ a = 120.
The given function, with this value of a becomes f (x) = x 4 – 62x 2 + 120x
+ 9, which leads to f (0) = 9, f (2) = 17 and f (1) = 68. This clearly shows
that the given function f (x) = x 4 – 62x 2 + ax + 9 attains its maximum
value at x = 1 only when a = 120.

It may be noted that Theorem 3.5 helps us in finding the local


maximum or local minimum, but it takes time to verify how f ′(x) is
changing sign as x passes through the points given by f ′(x) = 0. We thus
have another test, known as second derivative test, which enables us to
find the points of local maxima or local minima in an easier manner.

Theorem 3.9 Second Derivative Test. Let f (x) be a differentiable function


on an interval I and let x0 ∈ I. Let f ″(x) be continuous at x0. Then
(a) x0 is a local maximum if both f ′(x0) = 0 and f ″(x0) < 0.
(b) x0 is a local minimum if both f ′(x0) = 0 and f ″(x0) > 0.

Remarks
(i) The second derivative test fails if f ″(x0) = 0. In that case, either we
have to go back to the first derivative test to find the local maxima/
minima, or have to calculate the third derivative of f (x) and then look
for the sign of this third derivative at the point x0.
(ii) If f ″(x0) = 0 and x0 is not a point of local maxima/minima, then x0 is
a point of inflexion. We have the following working rule for finding the
maxima/minima of a given function.

Step 1. Find f ′(x) and equate it to zero. Solve this equation for real
values of x. Let these values be a, b, c, ...
Step 2. Find f ″(x) and put x = a, b, c one by one. If f ″(x) is negative for
x = a, then f (x) has a maxima at x = a and the corresponding maximum
value of f (x) is f (a). If f ″(x) is positive for x = a, then f (x) has a minima
at x = a. Similarly, one can verify for the points x = b, c, ...
Step 3. Find f ″(x) = 0 at x = a but f ″′(x) ≠ 0 at x = a, then x = a is a point
of inflexion (i.e., neither a maxima nor a minima). But if f ″′(x) = 0, then
find f iv(x) and see whether for x = a, f iv(x) is positive or negative. If
f iv(x) is negative for x = a, then we have a point of maxima at x = a; on
the other hand if f iv(x) is positive for x = a, then f (x) has a minima at
Differentiation 111

x = a; and the corresponding minimum value of f (x) is f (a). If f iv(a) = 0


then find the next derivative of f (x) and repeat the above process.
This method of finding the maxima/minima of a given function has
been illustrated by the following examples.

Example 3.40 Find the maxima and minima, if any, for the following
functions
(i) f (x) = 2x 3 – 21x 2 + 36x – 20 (ii) f (x) = x 5 – 5x 4 + 5x 3 – 1

Solution (i) Differentiate the given function f (x) = 2x 3 – 21x 2 + 36x –


20, we get f ′(x) = 6x 2 – 42x + 36. For maxima and minima, put f ′(x) =
0 ⇒ 6(x – 1) (x – 6) = 0 ⇒ x = 1, 6. Also, f ″(x) = 12x – 42 which is –30
for x = 1, and 30 for x = 6. Therefore, at the point x = 1, the function has
a maxima and the maximum value is f (1) = –3; while at the point x = 6,
the given function has a minima and the minimum value is f (6) = –228.
(ii) Given that f (x) = x 5 – 5x 4 + 5x 3 – 1, then
f ′(x) = 5x 4 – 20x 3 + 15x 2, f ″(x) = 20x 3 – 60x 2 + 30x
and f ″′(x) = 60x 2 – 120x + 30
For the maxima and minima, put f ′(x) = 0 so that x = 0, 1, 3. At x = 0,
f ″(x) = 0 but f ″′(x) = 30 ≠ 0. Therefore, at x = 0, the function has neither
maxima nor minima. On the other hand, at x = 1, f ″(x) = –10 which
means that the function has a point of maxima at x = 1. Moreover, at
x = 3, f ″(x) = 90 which shows that the function has a point of minima at
x = 3.

Example 3.41 Find two positive numbers whose sum is 16 and the sum
of whose cubes is minimum.

Solution Let the two positive numbers be x and y. Then x + y = 16. Also,
let S denote the sum of the cubes of these two numbers, then S = x 3 + y 3
= x 3 + (16 – x)3 which on differentiation leads to S ′ = 3x 2 – 3(16 – x)2.
Put S ′ = 0 so that x = 8. For x = 8, S″ = 6x + 6(16 – x) = 96 which
is a positive quantity, and thus for x = 8, S is minimum. Therefore,
y = 16 – 8 = 8. Hence, the required numbers are 8 and 8.

Example 3.42 Show that among all rectangles of given area the square
has the least perimeter.

Solution Let x and y be the length and breadth of the rectangle. Then,
⎛ c⎞
the given area is xy = c and the perimeter P = 2(x + y) = 2⎜ x + ⎟ which
⎝ x⎠
112 Mathematical Methods

⎛ c⎞
on differentiation gives P′ = 2⎜1 − 2 ⎟ . For maxima or minima, take
⎝ x ⎠
P′ = 0 which gives x = c and y = c/x = c . Now for x = c , P″ =
4c/x 3 = 4c/ ( c )3 which is a positive quantity. Therefore, P is minimum
when x = c , y = c , i.e. the perimeter is minimum when rectangle is
a square.

Example 3.43 Show that among all the rectangles of given perimeter,
the square has the greatest area.

Solution Let x and y be the length and breadth of the rectangle. The
perimeter P is given and P = 2x + 2y = 2c (say). Also, the area A = xy =
x(c – x) = cx – x2 which on differentiation gives A′ = c – 2x. For maxima
or minima, put A′ = 0 so that x = c/2, y = c/2. Also, A″ = –2 which is a
negative quantity and therefore A is maximum when x = y = c/2. Thus
x = y and hence rectangle must be a square.

Example 3.44 A square piece of tin of side 18 cm is to be made into a


box without top, by cutting a square from each corner and folding up the
flops to form the box. What should be the side of the square to be cut off
so that the volume of the box is maximum possible.

Solution Let each side of the square to be cut off from each corner be
x cm, then the length of the box is 18 – 2x and the breadth of the box is
18 – 2x. The height of the box being x, thus the volume V of the box to be
formed by folding the flops is V = (18 – 2x) (18 – 2x)x = 324x – 72x 2 +
4x3. Differentiate V so that V ′ = 12(27 – 12x + x2) which, for maxima or
minima, is equated to zero so that x = 3, 9. Now, V″ = –144 + 24x which
is a negative quantity for x = 3. Hence, the side of the square to be cut is
3 cm to get the maximum possible volume of the box.

Example 3.45 Of all the closed cylindrical cans (right circular) that
enclose a given volume of 100 cc, find the one which has the minimum
surface area.

Solution Let r and h be the radius and height, respectively, of a right


circular cylinder. Then V = 100 = π r2h ⇒ h = 100/π r2. Also, the
surface area S = 2π r 2 + 2π rh = 2π r 2 + 2π r(100/π r2) = 2π r 2 + (200/r)
⇒ S′ = 4π r – (200/r 2) so that S ′ = 0 ⇒ r = (50/π)1/3. Further, S ″ = 4π
+ (400/r 3) which is 12π, a positive quantity, for r 3 = 200/4π. Thus, for
the surface of the can to be minimum, the radius of the cylindrical can
must be (50/π)1/3 and the height is twice of this radius.
Differentiation 113

Example 3.46 Sensitivity to Medicine. The reaction r of the body to a


dose of medicine is described through the equation
2⎛c m⎞
r= m ⎜ − ⎟
⎝2 2 ⎠
where c is a positive constant and m the amount of medicine in the
blood. If the reaction r is a change in blood pressure, then it is measured
in milimeters of mercury; while if the reaction is a change in temperature,
then r is measured in degrees, and so on. The rate of change of r with
respect to m, dr/dm, is a function of m and is known as the sensitivity of
the body to the medicine. Find the amount of medicine to which the
body is most sensitive by finding the value of m that maximize the
derivative dr/dm.

Solution Given that


2⎛c m⎞
r= m ⎜ − ⎟
⎝2 2 ⎠
so that the sensitivity S of the body to the medicine is
dr 3
S= = mc − m 2
dm 2
Here we have to find the value of m such that S is maximum. Now
dS d 2r
= = c – 3m
dm dm 2
Equating this equation to zero, we get m = c/3. Also S ″ = –3 which is
negative for m = c/3. Thus, the body is most sensitive to medicine only
when m = c/3 and the maximum sensitivity is c 2/6.
3.11.7 Marginal Analysis in Business and Economics
One of the most important application of differential calculus in business
and economics is the study of marginal analysis. For example, a
manufacturer is not only interested in the total cost at certain production
levels but also in the rate of change of costs at different production levels.
The word ‘marginal’ in economics, refers to a rate of change, i.e. to a
derivative.
If C(x) denotes the total cost of manufacturing x units during some
unit of time, then C′(x) is the rate of change in cost per unit change in
manufacturing at an output level of x units and is known as marginal
cost. Also, if R(x) represents the total revenue, then R′(x) is the change
in revenue per unit change in production at a given output level, and is
114 Mathematical Methods

known as marginal revenue. If P(x) = R(x) – C(x) denotes the total profit,
then P′(x) = R′(x) – C′(x) is the change in profit per unit change in
production at the given output level. This P′(x) is called the marginal
profit. Hence, the marginal cost, revenue and profit represent the changes
in cost, revenue and profit, respectively, that are obtained from a unit
increase in production (for a detailed account of marginal analysis and
its applications, see [4]). The relationships between these concepts have
been illustrated through the following examples.

Example 3.47 A radio company manufactures and sells AM/FM radio


sets. If x is the demand at p per radio and the demand equation is
x
x = 6000 – 30p or p = 200 – ; while the cost equation is C(x) = 72000
30
+ 60x. Then find the
(i) marginal cost
(ii) revenue equation in terms of x
(iii) marginal value
(iv) R′(1500) and R′(4500), and interpret
(v) profit equation in terms of x
(vi) marginal profit
(vii) P′(1500) and P′(3000), and interpret.

Solution Given that


x
x = 6000 – 30p or p = 200 – (10)
30
C(x) = 72000 + 60x (11)
Then, we have
(i) Marginal cost = C ′(x) = 60 [from Eq. (11)].
(ii) The revenue equation is the amount of money R(x) received by the
company for manufacturing and selling x units at p per unit. It is given
by
x2
R(x) = (Number of units sold) (Price per unit) = xp = 200x – (12)
30
[using Eq. (10)].
(iii) From Eq. (12), the marginal revenue is 200 – (x/15).
(iv) From (iii), we have R′(1500) = 100; the revenue is increasing at
` 100 per unit increase in production at the 1500 output level.
Differentiation 115

Also, R′(4500) = –100; the revenue is decreasing ` 100 per unit increase
in production at the 4500 output level.
(v) The profit equation in terms of x and p, from Eqs. (11) and (12), is
x2
P(x) = R(x) – C(x) = – + 140x – 72000 (13)
30
(vi) From Eq. (13), the marginal profit is
x
P′(x) = – + 140 (14)
15
(vii) From Eq. (14), we have P′ (1500) = 40; the profit is increasing at
` 40 per unit increase in production at the 1500 output level.
Further, P ′(3000) = – 60; the profit is decreasing at ` 60 per unit
increase in production at 3000 output level.

Example 3.48 A company manufactures and sells x tooth brushes per


week. If the weekly cost and demand equations are
x
C(x) = 5000 + 2x, p = 10 – , 0 ≤ x ≤ 8000 (15)
1000
then find for each week
(i) the maximum revenue
(ii) the maximum profit, the production level that will yield the maximum
profit and the price that the company should charge for each tooth brush.

Solution (i) From Eq. (15), the revenue received for selling x tooth brushes
at ` p per tooth brush is
x2
R(x) = xp = 10x – (16)
1000
Differentiate this equation to get R′(x) = 10 – (x/500) which when equated
to zero leads to x = 5000. Also, R″(x) = –1/500 which is negative for all
x. Thus for x = 5000, the revenue is maximum and from Eq. (16) it is
` 25000.
(ii) We know that the profit is the difference of the revenue and the cost.
Thus, from Eqs. (15) and (16), we have

x2
P(x) = R(x) – C(x) = – + 8x – 5000 (17)
1000
Differentiate Eq. (17) so that P′(x) = 8 – (x/500). Now, put P′(x) = 0 to
get x = 4000. Moreover, P″(x) = –1/500 which is negative for all x. Thus
116 Mathematical Methods

for x = 4000, the profit is maximum and from Eq. (17), this maximum
profit is ` 11000. Using x = 4000 in Eq. (15), we get p = ` 6.
Therefore, the company has a maximum profit of ` 11000 per week
if 4000 tooth brushes are produced weekly and sold for ` 6 each.
3.11.8 Mean Value Theorems
Consider the function f (x) = sin x. This function takes values zero at the
points 0, ± π, ± 2π, .... The derivative of sin x is cos x and it vanishes at
π 3π
the points ± , ± , .... It may be noted that between any two points
2 2
where sin x vanishes, there is a point where its derivative cos x vanishes.
π
For example, between 0 and π, there is ; between π and 2π, there is
2

; and so on. Consider another function f (x) = x2 – 4x + 3. This
2
function vanishes at x = 1 and x = 3. Now, f ′(x) = 2x – 4 which vanishes
at x = 2. Here again 2 is in between 1 and 3.
In these two examples, we have observed that between the two points
where the function vanishes, there is a point where its derivative also
vanishes. From these observations it may be noted that either the observed
result is accidental in the above examples or it is true, in general, that
between any two points where f (x) vanishes, there is atleast one point
where f ′(x) vanishes? The following theorem, known as Rolle’s theorem,
tells us that the above observed result is true, in general.

Theorem 3.10 Rolle’s Theorem. Let f (x) be a real function defined in


the closed interval [a, b] such that
(i) f (a) = f (b) = 0,
(ii) f (x) is continuous in the closed interval [a, b],
(iii) f (x) is differentiable in the open interval (a, b).
Then, there is a point c in the open interval (a, b) such that f ′(x) = 0.

Remark
1. It may be noted that in some of the existing literature, the above theorem
is also stated by relaxing condition (i) as f (a) = f (b), without requiring
them to be necessarily zero. This statement is also correct and, in fact,
these two statements are equivalent. Therefore, for verifying Rolle’s
theorem in specific problems, it is enough to ensure f (a) = f (b), instead
of requiring f (a) = f (b) = 0.
Differentiation 117

2. There may be more than one such point c as f (x) is continuous on


[a, b] and attains its maximum value at some point in [a, b] (as explained
above). If this point is an interior point, then from Theorem 3.7, f ′(x)
vanishes at that point.
y

a b
x x
O a b O a b

Fig. 3.10 Geometrical meaning of Rolle’s theorem


3. Geometric meaning of Rolle’s theorem. Consider Fig. 3.10 which
represents the graph of a function f (x). Under the assumption of Rolle’s
theorem, the graph starts at (a, 0) and ends at (b, 0). From the figure it is
clear that there is point c between a and b such that the tangent to the
graph at (c, f (c)) is parallel to the x-axis. It may be noted that f ′(c) = 0
means that slope of the tangent at (c, f (c)) is zero, and also, if the graph
is fixed, but the segment [a, b] is moved upwards and downwards keeping
it parallel to the x-axis, it must become a tangent to the curve somewhere
as shown in Fig. 3.10.
We shall now verify the truth of Rolle’s theorem with the help of the
following examples.

Example 3.49 Verify Rolle’s theorem for the following functions


(i) f (x) = x 2 – 4x + 3 on [1, 3]
(ii) f (x) = sin x – sin 2x on [0, π]
(iii) f (x) = log (x 2 + 2) – log 3 on [–1, 1]
(iv) f (x) = 3 + (x – 1)2/3 on [0, 1]
(v) f (x) = [x] on [–1, 1]

Solution (i) Here f (x) = x 2 – 4x + 3 is a polynomial function defined on


[1, 3], and therefore, it is a continuous function on [1, 3]. Also f ′(x) = 2x
– 4 exists in (1, 3). Further, f (1) = f (3) = 0. Now, since all the three
conditions of Rolle’s theorem are satisfied, therefore there must exist at
least one value of x, say c in (1, 3) such that f ′(c) = 0. Now, f ′(c) = 2c –
4 = 0 ⇒ c = 2 which lies in the open interval 1 < x < 3. Hence, for the
given function, Rolle’s theorem in applicable.
(ii) As we know that the sine and cosine functions are continuous
functions, therefore the function f (x) = sin x – sin 2x is continuous on
118 Mathematical Methods

[0, π], and f ′(x) = cos x – 2 cos 2x which shows that f ′(x) exists in (0, π).
Moreover, f (0) = f (π) = 0. Thus there is atleast one value of x, say c, in
(0, π) such that f ′(c) = 0. Now, f ′(c) = cos c – 2 cos 2c = 0 ⇒
4 cos2 c – cos c – 2 = 0 ⇒ cos c = 0.8431, – 0.5931. Therefore,
c = cos–1 (0.8431) which belongs to (0, π). Hence Rolle’s theorem is
verified.
(iii) The function f (x) = log(x2 + 2) – log 3, being the difference of
logarithmic function and constant function, is continuous on [–1, 1].
2x
Also, f ′(x) = 2 is defined for all x. Therefore, f (x) is differentiable
x +2
in (–1, 1). Moreover, f (–1) = log(1 + 2) – log 3 = 0, f (1) = log(1 + 2) –
log 3 = 0 which means that f (–1) = f (1). Thus all the three conditions of
Rolle’s theorem are satisfied. Therefore, equating f ′(c) to zero leads to
c = 0 which belongs to (–1, 1). Hence, Rolle’s theorem is valid for the
given function.

(iv) Here f (x) = 3 + (x – 1)2/3 is continuous on [0, 1] and f ′(x) =


2
(x – 1)–1/3 which is not defined at x = 1 ∈ (0, 3). Thus, Rolle’s theorem
3
is not applicable for the given function f (x) = 3 + (x – 1)2/3.
(v) The function f (x) = [x] defined on [–1, 1] is the greatest integer
function which is known to be not continuous and thus Rolle’s theorem
is not applicable for this function.

Example 3.50 Find the point on the curve y = sin 2x defined on [0, π]
where the tangent is parallel to x-axis.

Solution It can be easily verified that the function y = sin 2x is continuous


on [0, π]. Also, y′ = 2cos 2x which means that y = sin 2x is differentiable
in (0, π). Moreover, y(0) = y(π) = 0. It is known that the tangent to the
curve y at a point x = c is parallel to x-axis only when y′(c) = 0 (which in
fact is the conclusion of Rolle’s theorem). Thus y′(c) = 2 cos 2c = 0
π ⎛π ⎞
⇒c= ∈ (0, π). Therefore, ⎜ ,1⎟ is the point where the tangent to
4 ⎝4 ⎠
the curve y = sin 2x is parallel to the x-axis.
In Rolle’s theorem, it was assumed that the ends points of the graph
were on the x-axis, and concluded that at some point of the graph, the
tangent is parallel to the x-axis. Now, we improve this result by asserting
that the x-axis is not at all important, and say that if the end points of the
Differentiation 119

graph are on a line, then there is a point on the graph where the tangent
is parallel to that line. In other words, there is always a point on the
graph, where the tangent is parallel to the line joining the end points of
the graph. The line joining any two points of the curve is known to be
the chord of the curve. Then, our improved theorem must read as: Given
any chord of the graph of a function f (x), there is a point on the graph
where the tangent is parallel to this chord (Fig. 3.11).

y y

x x
a c b O a b
O

Fig. 3.11 Lagrange’s mean value theorem


We thus have

Theorem 3.11 Lagrange’s Mean Value Theorem. Let f (x) be a real


function defined in the closed interval [a, b] such that
(i) f (x) is continuous in the closed interval [a, b],
(ii) f (x) is differentiable in the open interval (a, b).
Then, there is a point c in the open interval (a, b) such that
f (b ) − f ( a )
f ′(c) = . This theorem is also known as first mean value
b−a
theorem.
Remarks
f (b) − f (a )
1. When f (a) = f (b), then becomes zero. Thus f (a) = f (b)
b−a
⇒ f ′(c) = 0 where c ∈ (a, b). Thus Rolle’s theorem becomes a particular
case of Lagrange’s mean value theorem.
2. Physical meaning of Lagrange’s mean value theorem. Let a car be
running on a straight road and let f (a) be its position at a time t = a,
while at a later time t = b, f (b) be its position. Then, f (b) – f (a) is the
f (b) − f (a )
distance travelled in time b – a and thus is the average
b−a
speed of the car. Now, consider the speedometer of the car - the
120 Mathematical Methods

speedometer reading is the value of f ′(x) at that time. Therefore,


Lagrange’s mean value theorem means that at some time between a and
b, the speedometer reading would have been equal to the average speed
of the car. Here, it may be noted that the word “mean” means average,
f (b) − f (a )
and we are considering the average value and we are
b−a
asserting that it is one of the values taken by the derivative f ′(x).
3. Geometrical meaning of Lagrange’s mean value theorem. If f (x) is
continuous on [a, b] and a tangent to the curve can be drawn at every
point, then there exists atleast one point c ∈ (a, b) such that the tangent
at this point is parallel to the chord joining the two end points a and b.

Example 3.51 Find c in Lagrange’s mean value theorem for the function
f (x) = (x – 1) (x – 2) (x – 3) defined on [0, 4].

Solution Here a = 0, b = 4 and f (x) = x 3 – 6x 2 + 11x – 6. Then


f (a) = – 6, f (b) = 6. Differentiating f (x), we get
f ′(x) = 3x 2 – 12x + 11 ⇒ f ′(c) = 3c2 – 12c + 11
Now, using Lagrange’s mean value theorem, we have
f (b) − f (a ) 6 − (− 6)
f ′(c) = ⇒ 3c 2 − 12c + 11 =
b−a 4−0
2 3
⇒ 3c 2 – 12c + 8 = 0 ⇒ c=2±
3

Example 3.52 Find the point on the parabola y = (x – 3)2 where the
tangent is parallel to the chord joining the points (3, 0) and (4, 1).

Solution Here the given function f (x) = x 2 – 6x + 9 is defined on [3, 4],


which on differentiation leads to f ′(x) = 2x – 6. Also, f (3) = 0, f (4) = 1,
and
f (b) − f (a ) 1− 0
f ′(c) = ⇒ 2c – 6 =
b−a 4−3
⇒ 2c – 6 = 1 ⇒ c = 7/2 ∈ (3, 4)
Therefore, the point is (7/2, 1/4).

Theorem 3.12 Cauchy’s Mean Value Theorem. Let f (x) and g(x) be two
real functions defined in the closed interval [a, b] such that both are
(i) continuous in the closed interval [a, b]
Differentiation 121

(ii) differentiable in the open interval (a, b)


(iii) g′(x) ≠ 0, for every x ∈ (a, b).
Then, there is a point c in the open interval (a, b) such that
f ′(c) f (b) − f (a )
=
g ′(c ) g (b) − g (a)
This theorem is also known as second mean value theorem.

Example 3.53 For the functions f (x) = x 2 and g(x) = x 3 defined on the
interval [1, 2] verify Cauchy’s mean value theorem.

Solution Both the given functions f (x) = x 2 and g(x) = x 3 are continuous
in [1, 2] and differentiable in (1, 2) and g ′(x) ≠ 0 for any x ∈ (1, 2). Thus
all the three conditions of Cauchy’s mean value theorem are satisfied
and consequently there exists a point c ∈ (1, 2) such that
f ′(c ) f (2) − f (1)
=
g ′(c) g (2) − g (1)
2c 4 −1 3 14
⇒ = = ⇒ c = 0 or c =
3c 2
8 −1 7 9
14
Now, c = 0 does not belong to (1, 2) but c = ∈ (1, 2) and hence
9
Cauchy theorem is verified.

EXERCISES
3+ x
1. If y = and x ≠ 0, then from the definition find y′(2). [Ans. 6]
3− x
2. If g(x) = 2 x − 1, then find g′(5) from the definition. [Ans. 1/3]
3. Show that the function defined by
⎧x sin (1/ x), if x ≠ 0
f (x) = ⎨
⎩0, if x = 0
is not differentiable at x = 0.
x
4. Show that f (x) = is continuous but not differentiable
1 + e1/ x
at x = 0.
5. Show that the greatest integer function is neither continuous nor
differentiable at x = 2.
122 Mathematical Methods

6. Find the derivative of the following functions using the first


principle:

(i) y = ax + b [ Ans. a /2 ax + b ]
(ii) y = (ax + b)/(cx + d) [Ans. (ad – bc)/(cx + d )2]
(iii) y = x n [Ans. nx n – 1]
(iv) y = (ax + b) n [Ans. n(ax + b)n–1 a]
(v) y = cos x [Ans. – sin x]
(vi) y = tan x [Ans. sec2 x]
7. Find dy/dx for the following functions:
(i) y = x 2 – 3x + 2 [Ans. 2x – 3]
1
2 −3
(ii) y = x 2/3 + 2x 2 [Ans. x + 4x ]
3
2
⎛ 1⎞
(iii) y = ⎜ x + ⎟ [Ans. 2x – 2x–3]
⎝ x⎠

(iv) y = (2x – 1) (5x1/2 + 7) x–1/2 [Ans. 10 + 7x–1/2 + 7


2
x–3/2]

x ⎡ a2 − x2 ⎤
(v) y = ⎢Ans. 2 ⎥
a +x
2 2
⎣ (a + x 2 )2 ⎦

1+ x
(vi) y = [Ans. (1 + x)–1/2 (1 – x)–3/2]
1− x
1
  (vii) y = [Ans. – (2ax + b) (ax2 + bx + c)–2]
ax + bx + c
2

x +1 + x −1
(viii) y = [Ans. 1 + x(x 2 – 1)–1/2]
x +1 − x −1
8. Differentiate the following functions with respect to x using chain
rule:
⎡ 1 ⎤
(i) 1 + x2 ⎢Ans. ⎥
⎣⎢ 2 1 + x 2 ⎦⎥
Differentiation 123


( ) ⎥⎥
n⎤
⎢ n x + x2 − 1

(x + )
n
⎢Ans.
(ii) x −1
2
⎢ x2 − 1 ⎥
⎣⎢ ⎦⎥

a2 − x2
(iii) [Ans. – 2a2x (a2 + x2)–1 a4 − x4 ]
a +x
2 2

(iv) tan–1 (sec x + tan x) [Ans. 1/2]


⎛ x − x −1 ⎞
(v) cos–1 ⎜⎜ −1 ⎟
⎟ [Ans. –2/(x2 + 1)]
⎝x+ x ⎠

(vi) 1 ( x a 2 − x 2 ) + 1 a2 sin– 1 (x/a) [ Ans. a 2 − x 2 ]


2 2
9. Find dy/dx when

1 + cos x
(i) y = cos–1 [Ans. 1/2]
2

1 − cos x
(ii) y = tan–1 [Ans. 1/2]
1 + cos x

1 + sin x
(iii) y = tan– 1 [Ans. 1/2]
1 − sin x

(iv) y = sin– 1 (2 x 1 − x2 ) [ Ans. 2/ 1 − x 2 ]


2x ⎞
(v) y = tan–1 ⎛⎜ ⎟ [Ans. 2/(1 + x 2)]
⎝1 − x2 ⎠
⎛1 + x ⎞
(vi) y = cot– 1 ⎜ ⎟ [Ans. –1/(1 + x2)]
⎝1 − x ⎠
⎛ 3a 2 x − x 3 ⎞
(vii) y = tan–1 ⎜ [Ans. 3a/(a2 + x2)]
⎜ a(a 2 − 3 x 2 ) ⎟⎟
⎝ ⎠

(viii) y = sin–1 [ 1 − x ] + cos–1 ( x ) [ Ans. 1/ x − x2 ]


10. Differentiate
(i) sin–1 x with respect to cos −1 1 − x 2 [Ans. 1]
124 Mathematical Methods

⎛1 − x ⎞
(ii) sin–1 ⎜ ⎟ with respect to x [Ans. –2/(1 + x)]
⎝1 + x ⎠
⎛ 1 + x2 − 1⎞
⎜ ⎟
(iii) tan–1 ⎜ x
–1
⎟ with respect to tan x [Ans. 1/2]
⎝ ⎠
11. Find d y/dx if
(i) 4x 2 + 9y 2 = 36 [Ans. – 4x/9y]
(ii) x3
+ 8x y + = 64 [Ans. –y3 (3x 2
+ 8y)/(8x + 3y 2)]
12. Differentiate the following functions with respect to x
sin x
(i) (sin x)log x [Ans. (sin x)log x (cos x log x + )]
x
⎡ −1 ⎛ sin
−1
x log x ⎞⎤
⎢Ans. xsin x ⎜
−1
(ii) xsin x + ⎟⎥
⎜ ⎟
⎢⎣ ⎝ x 1 − x2 ⎠⎥⎦
(iii) (x log x)log log x

⎡ loglog x ⎛ log x + 2log(log x ) + log(log x )(log x ) ⎞⎤


⎢Ans. ( x log x) ⎜ ⎟⎥
⎣ ⎝ x log x ⎠⎦

⎡ e sin x cos x ⎤
(iv) e sin x ⎢Ans. ⎥
⎢⎣ 2 sin x ⎥⎦
x x
(v) ee [ Ans. e x ee ]
13. Find dy/dx when
a x ...∞
x ax ⎡ y 2 log a ⎤
(i) y = a ⎢Ans. ⎥
⎣ 1 − xy log a ⎦
(ii) x m y n = (x + y)m+n [Ans. y/x]
⎡ cos x ⎤
(iii) y = sin x + sin x + sin x + ...∞ ⎢Ans. ⎥
⎣ 2 y − 1⎦
14. Find dy/dx if x = at 2 and y = 2at [Ans. 1/t]
15. Find dy/dx if x = a (cos t + t sin t) and y = a (sin t – t cos t)
[Ans. tan t]
Differentiation 125

16. Find the second derivative of the following


(i) y = x sin x [Ans. –x sin x + 2 cos x]
(ii) y = e kx [Ans. k 2 e kx]
17. If x = a (θ – sin θ) and y = a(1 – cos θ), then find d y/dx 2 at θ = π
2

[Ans. –1/4a]
18. Find the nth derivative of the following functions:
(i) x n+ 1 [Ans. (n + 1)! x]
(ii) a mx [Ans. m n a mx (log a)n]
19. In biophysics, the equation (L + m) (V + n) = k is called the
fundamental equation of muscle contraction, where m, n, k are
constants and V the velocity of the shortening of muscle fibre for a
muscle subject to a load L. Find dL/dV. [Ans. – (L + m)/(V + n)]
20. A particle moving according to the law s = 10 + 20t – t 2, starts
from a distance of 10 meters from a mark and moves in a line
farther and farther from the mark. How far from the mark does it
go, before it starts moving in the opposite direction?
[Ans. 110 meters]
21. A particle moves according to the law s = (at + bt + c)1/2, prove
2

that at a time t, the acceleration of this particle varies as s–3.


22. On a certain planet, a ball is thrown upward. The motion of the
ball obeys the law s = 10t – 3t2, where s is in meters and t is
measured in seconds. What is the acceleration due to gravity of
this planet and after how many seconds, the ball will fall on the
planet? [Ans. – 6, 10/3]
23. Two balls are thrown up simultaneously according to the laws
s = 19.6t – 4.9t 2 and s = 9.8t – 4.9t 2, for the first and second balls,
respectively. What is the height of the second ball when the height
of the first ball is maximum? [Ans. 0 meter]
24. Gas is escaping from a spherical balloon at the rate of
900 cm3 s–1. How fast is the surface area shrinking when the radius
is 360 cm? [Ans. –5]
25. An airforce plane is ascending vertically at the rate of 10 km/h. If
the radius of the earth is r km, how fast is the area of the earth,
visible from the plane, increasing at 2 minutes after it started
⎡ 200πr 2 ⎤
ascending? ⎢Ans. ⎥
⎣ (r + 5) 2 ⎦
126 Mathematical Methods

26. A company manufactures pocket calculators according to the cost,


revenue and profit equations
x2
C = 5000 + 2x, R = 10x – and P = R – C
1000
respectively, where the production oputput in one week is x pocket
calculators. If the production is increasing at the rate of 500 pocket
calculators per week when production is 2000 pocket calculators,
find the rate of increase in: (i) cost (ii) revenue (iii) profit.
27. Find the equation of the tangent to the curve y = sin x + sin 2x at
x = π/3; and the equation of the normal to the curve y = 2sin2 3x at
x = π/6. [Ans. 2y = 3 3, x = π/6]
28. Determine the points on the curve y = x 2 – 4x + 3 at which the
normal is parallel to a line whose slope is 1/2. [Ans. (1, 0)]
29. Find the least value of a such that the function f (x) = x 2 + ax + 1
is increasing on [1, 2]. [Ans. – 4]
⎛ π⎞
30. Prove that sin x is increasing in the interval ⎜ 0, ⎟ and decreasing
⎝ 2⎠
⎛π ⎞
in the interval ⎜ , π ⎟.
⎝2 ⎠
x
31. Determine whether the function f (x) = – + sin x is increasing or
2
⎛ π π⎞
decreasing in the interval ⎜ − , ⎟ . [Ans. increasing]
⎝ 3 3⎠
32. Using the second derivative test, find the local maxima or minima
of the following functions:
(i) x 3 – 6x 2 + 9x + 15, 0 ≤ x ≤ 6.
[Ans. Local maxima at x = 1, local max. value is 19]
(ii) (x – 1) (x + 2)2.
[Ans. Local minima at x = 3 and local minimum value is 15]
33. Find the absolute minimum and absolute maximum values of the
following functions on the indicated intervals:
2
⎛1 ⎞
(i) f (x) = ⎜ − x ⎟ + x 3, [–2, 2.5].
⎝2 ⎠
[Ans. minimum value is –7/4, maximum value is 157/8]
Differentiation 127

x +1
(ii) f (x) = , [0, 2].
x2 + 1
[Ans. minimum value is –3(4)– 4/3, maximum value is 14]
(ii) f (x) = 2 cos 2x – cos 4x, [0, π].
[Ans. minimum value is –3, maximum value is 3/2]
34. Find two positive numbers x and y such that x + y = 60 and xy 3 is
maximum. [Ans. 15, 45]
35. Prove that the perimeter of a right angled triangle of given
hypotenuse is maximum when the triangle is isosceles.
36. If the sum of the perimeters of a square and a circle are given,
show that the sum of their areas is least when the side of the square
is equal to the diameter of the circle.
37. Prove that the rectangle having maximum area that can be inscribed
in a circle is a square.
38. A wire of length 28 m is cut into two pieces. One of the piece is to
be made into a square and the other into a circle. What should be
the length of the two pieces so that the combined area of the square
and the circle is minimum.
[Ans. The wire is to be cut at a distance of 28π/(π + 4) m
from one end]
39. Prove that a closed right circular cylinder of given surface and
maximum volume is such that its height is equal to the diameter
of its base.
40. Show that a cylindrical vessel of given volume requires the least
surface area when its height is twice its radius.
41. Verify Rolle’s theorem for the following functions:
(i) f (x) = x 2 – 1 defined on [–1, 1]. [Ans. c = 0, applicable]

(ii) f (x) = sin x + cos x – 1 defined on ⎡0, π ⎤ .


⎢ 2⎥
⎣ ⎦
[Ans. c = π/4, applicable]

⎛ 1 ⎞ ⎛ π⎞ .
(iii) f (x) = cos2⎜ x − π ⎟ defined on ⎜ 0, ⎟
⎝ 4 ⎠ ⎝ 2⎠
[Ans. c = π/4, applicable]
128 Mathematical Methods

(iv) f (x) = | x | defined on [– 1, 1]. [Ans. not applicable]


x
(v) f (x) = e sin x defined on [0, π]. [Ans. c = 3π/4, applicable]
(vi) f (x) = x(x + 3) e– x/2 defined on [–3, 0]. [Ans. c = –2, applicable]
42. It is given that Rolle’s theorem holds for the function f (x) = x 3 –
1
6x 2 + ax + b defined on [1, 3] with c = 2 + . Find the values of
3
a and b. [Ans. 11, – 6]
43. Verify Lagrange’s mean value theorem for the following functions:
(i) f (x) = 2x – x 2 defined on [0, 1]. [Ans. c = 1/2, applicable]
(ii) f (x) = sin x – sin 2x defined on [0, π]. [Ans. not applicable]
(iii) f (x) = log x defined on [1, 2]. [Ans. c = 1/log 2, applicable]
(iv) f (x) = ax 3 + bx 2 + cx + d defined on [0, 1]. [Ans. applicable]
44. If in the Cauchy mean value theorem, we have f (x) = e x and
g(x) = e–x defined on [a, b], find the value of c.
1
(a + b), applicable]
[Ans. c =
2
45. Verify Cauchy mean value theorem for the functions f (x) = x2,
g(x) = x4 defined on [0, 1].
CHAPTER 4

Integration

4.1 Introduction
In Chapter 3, we studied the differentiation and some of its applications.
In this chapter we shall study another important concept known as
integration. This has number of applications in science and technology
as well as in social sciences. It can be used to determine the future position
of a body from its present position and also determines the nature of
forces acting on it. The areas of irregular regions, the length of the curves,
volumes and masses of irregular bodies can also be obtained from a
knowledge of integration.
Integration is of two types, viz., (i) indefinite and (ii) definite.

4.2 Indefinite Integral


We know that
d n d 1
(x n + 1) = ( n + 1) x , (log x) =
dx dx x
d x d
(e ) = e x, (sin x) = cos x, etc.
dx dx
Now, the problem is how to obtain x n + 1 from x n, log x from 1/x and
sin x from cos x. More generally, if
d
[ f (x)] = F (x)
dx
then we have to obtain f (x) from F (x). Thus if the differential coefficient
of a function is known, then we have to find the given function itself.
The process of determining such function is called integration and it is
due to this reason that integration is known to be as the inverse process
of differentiation, and we have
Definition 4.1 Let F (x) be a given function of x, and if it is possible to
find a function f (x) such that
130 Mathematical Methods

d
[ f (x)] = F (x)
dx
then f (x) is called the integral of F (x), and we write

∫ F ( x) dx = f (x) (1)
where dx indicates that the integration is performed with respect to the
variable x. Since
d d d
[ f ( x) + c] = [ f ( x)] + (c ) = F ( x ) + 0 = F ( x )
dx dx dx
Therefore

∫ F ( x) dx = f (x) + c (2)
The arbitrary constant c is called the constant of integration and
f (x) + c is called the indefinite integral.
It may be noted that the two integrals of the same function differ
only by a constant, for example
d 4
(x + 6) = 4 x 3 ⇒ ∫ 4 x dx = x4 + 6
3
dx
d
∫ 4 x dx = x 4 + 12
3
and (x 4 + 12) = 4 x 3 ⇒
dx
In Chapter 3, we have calculated the derivatives of a number of functions
(cf., Table 3.1). From these formulas, we can write down the
corresponding formulas for integration which are listed in Table 4.1.
Moreover, we have seen in Chapter 3 that there are certain rules for
finding the derivative of a given function. In case of integration, we do
have rules for the integration of a given function. Some of these rules are
given in the following theorem.
Theorem 4.1 Let f (x), g (x) and h(x) be well defined functions and c is a
constant. Then
d
dx ∫
(i) [ f ( x) dx] = f (x) (i.e., integration and differentiation are
inverse process)
(ii) ∫ cf ( x) dx = c ⋅ ∫ f ( x) dx
(iii) ∫ f ( x) dx ± ∫ g ( x) dx ± ∫ h( x) dx = ∫[ f ( x) ± g ( x) ± h( x)] dx
(iv) ∫ f ′( g ( x)) g ′( x) dx = f ( g ( x))
(v) ∫[ f ( x) g ′( x) + g ( x) f ′( x)] dx = f (x) g (x)
Integration 131

Table 4.1 Formulas for differentiation and integration


Differentiation Integration
d n +1 n x n +1
dx
(x ) = (n + 1) x n ∫x dx =
n +1
+ c, n ≠ – 1
d
dx
(x) = 1 ∫(1) dx =x+c
d 1 1
dx
(log x) =
x ∫ x dx = log | x | + c
d x x
dx
(e ) = e x ∫e dx = e x + c

d x ax
dx
(a x) = a x loge a ∫a dx =
log e a
+c

d
dx
(sin x) = cos x ∫ cos x dx = sin x + c
d
dx
(cos x) = − sin x ∫sin x dx = – cos x + c
d
dx
(log | cos x |) = – tan x ∫ tan x dx = – log | cos x | + c
d
dx
(log | sin x |) = cot x ∫cot x dx = log | sin x | + c
d
dx
(log | sec x + tan x |) = sec x ∫sec x dx = log | sec x + tan x | + c
d
(log | csc x + cot x |) = – csc x
dx ∫ cscx dx = – log | csc x + cot x | + c
d
(tan x) = sec2 x ∫ sec
2
x dx = tan x + c
dx
d
(cot x) = – csc2 x ∫ csc
2
x dx = – cot x + c
dx
d
dx
(sec x) = sec x tan x ∫sec x tan x dx = sec x + c
d
dx
(csc x) = – csc x cot x ∫ cscx cot x dx = – csc x + c

d 1 1
dx
(sin −1 x) = ∫ 1 − x2
dx = sin–1 x + c
1 − x2
d 1 1
dx
(cos−1 x) = − ∫ 1 − x2
dx = – cos–1 x + c
1 − x2
d −1 1 1
dx
(tan x) =
1 + x2
∫1 + x 2 dx = tan–1 x + c
d 1 1
(cot −1 x) = − ∫1 + x 2 dx = – cot– 1 x + c
dx 1 + x2
d 1
(sec−1 x ) = 1
dx x x2 − 1 ∫ x x 2 − 1 dx = sec–1 x + c
d 1 1
(csc−1 x ) = − ∫ x x 2 − 1 dx = – csc–1 x + c
dx x x2 − 1
132 Mathematical Methods

Example 4.1 Evaluate the following integrals:

⎛ x 5 2 ⎞ ax 4 + bx 2 + c
(i) ∫ ⎜⎝ 2e − x + 3sec x⎟⎠ dx (ii) ∫ x4
dx

x4 + 1 (1 − 2 x 2 ) 2
(iii) ∫ x 2 + 1 dx (iv) ∫ x3 x
dx

dx
(v) ∫ 1 + sin 2x dx (vi) ∫ sin 2 x cos 2 x
(vii) ∫ tan
2
x dx

Solution (i) We have


⎛ x 5 ⎞ dx
∫⎜⎝ 2e −
x
+ 3sec 2 x ⎟ dx = 2 ∫ e x dx − 5∫ + 3∫ sec 2 x dx
⎠ x
= 2e x – 5log x + 3tan x + c
(ii) We have

ax 4 + bx 2 + c −2
∫ dx = ∫ (a + bx + cx − 4 ) dx
x4
cx −3
= ∫ adx + ∫ bx − 2 dx + ∫ cx − 4 dx = ax − bx −1 − +d
3
(iii) Here

⎡ x 4 + 1⎤ ⎡ 2 2 ⎤ ⎡ 2 ⎤
∫ ⎢ x 2 + 1⎥ dx = ∫ ⎢x − 1 + 2 ⎥ dx = ∫ ( x − 1) dx + ∫ ⎢ 2
2
⎥ dx
⎣ ⎦ ⎣ x + 1⎦ ⎣ x + 1⎦

1 x3
∫x dx − ∫ dx + 2∫ dx = – x + 2tan–1 x + c
2
=
x2 + 1 3
(iv) Here

(1 − 2 x 2 ) 2
dx = 1 + 4 x − 4 x dx
4 2
∫ 3
x x ∫ x 4/3
12 11/3 12 5/3
= ∫ ( x − 4/3 + 4 x8/3 − 4 x 2/3 ) dx = −3 x −1/3 + x − x +c
11 5
Integration 133

(v) The given integral is

∫ 1 + sin2 x dx =
∫ sin 2 x + cos 2 x + 2sin x cos x dx

= ∫ (sin x + cos x) 2 dx = ∫ (sin x + cos x) dx = – cos x + sin x + c


(vi) We have
dx sin 2 x + cos 2 x
∫ sin 2 x cos 2 x = ∫ sin 2 x cos 2 x
dx

sin 2 x cos 2 x
= ∫ sin 2 x cos2 x dx + ∫ sin 2 x cos2 x dx
∫ sec x dx + ∫ csc 2 x dx = tan x – cot x + c
2
=
(vii) We have

∫ tan ∫ (sec x − 1) dx
2
2
x dx =

∫ sec x dx − ∫ dx
2
= = tan x – x + c

Example 4.2 Evaluate the following integrals:

∫ sin
4
(i) xdx

cos 2 x + 2sin 2 x
(ii) ∫ cos 2 x
dx

dx
(iii) ∫ 1 + sin x
Solution (i) Here

∫ sin ∫ (sin
4
xdx = 2
x ) 2 dx
1
4∫
= (1 − cos2 x)2 dx

1 1 1
4∫
= dx − ∫ cos 2 x dx + ∫ cos 2 2 x dx
2 4
1 1 1
=
4 ∫ dx − ∫ cos 2 x dx + ∫ (1 + cos 4 x) dx
2 8
1 1 1 1
= x − sin2 x + x + sin 4 x + c
4 4 8 32
134 Mathematical Methods

(ii) We have

cos 2 x + 2sin 2 x 1 − 2sin 2 x + 2sin 2 x


∫ dx = ∫ cos 2 x
dx
cos 2 x
∫ sec
2
= x dx = tan x + c
(iii) Here
dx 1 1 − sin x
∫1 + sin x = ∫ 1 + sin x ⋅ 1 − sin x dx
1 − sin x
= ∫ 1 − sin 2 x dx
1 − sin x
= ∫ cos 2 x
dx

⎡ 1 sin x ⎤
= ∫ ⎢⎣ cos 2 x − cos x cos x ⎥⎦ dx
∫ (sec x − sec x tan x ) dx = tan x – sec x + c
2
=
Before proceeding further on integration, we shall now make a
comparison between differentiation and integration. Both of them are
operations on function and the result of their outcome is also a function.
The comparison between these two operations is described as follows:

Comparison between differentiation and integration


1. In case of differentiation, we have
d d d
[ f ( x) ± g ( x)] = f ( x) ± g ( x)
dx dx dx
i.e., the derivative of the sum/difference of two functions is same as the
sum/difference of the derivatives of these functions; while for integration

∫[ f ( x) ± g ( x)] dx = ∫ f ( x) dx ± ∫ g ( x) dx
i.e., in order to integrate the sum/difference of two functions, it is enough
to integrate them separately and then add/subtract the two integrals
obtained.
2. For differentiation, we have
d d
[cf ( x)] = c f ( x)
dx dx
Integration 135

i.e., the constant can be taken outside the differentiation sign. Also

∫ cf ( x) dx = c ∫ f ( x) dx
i.e., the constant can be taken outside the integration sign.
3. Every function can not be differentiated the same is true for integration.
4. If the derivative of a function exists, then it is unique, while the integral
of a function is not unique. It varies upto a constant.
5. When a polynomial function p (x) is differentiated, then the result is a
polynomial function of degree one less than p (x). While, if a polynomial
function is integrated, then the result is a polynomial function whose
degree is one more than the given polynomial function.
6. If a function is obtained by a finite number of steps by taking sum,
product, quotient, from polynomials, logarithm, exponential and
trigonometric functions, etc., then it is possible to find the derivative of
the given function; while for such function one cannot find the integral.
For example, if
f (x) = cos(log sin x) + 16 log tan x2
then we have methods for finding the derivative (cf., Chapter 3). On the
other hand, if
f (x) = cos (log sin x) + 16 log tan x 2
then we do not have methods to integrate such functions.
7. The derivative of a function can be defined at a point, while the integral
of a function can not be defined at a point.
8. The geometric meaning of the derivative of a function is the slopes of
the tangents to a curve. On the other hand, the geometric meaning of the
integral of a function is the area bounded by some regions.
9. The derivative is used to finding the physical quantities such as the
velocity, acceleration, etc. of a moving particle. The integral is used to
calculate some physical quantities like centre of mass, momentum, etc.
10. Differentiation is a process involving limits. Similar is the case for
integration.
11. We have
d ⎡
f ( x ) dx⎤⎦ = f (x)
dx ⎣∫
and
d
∫ dx [ f ( x)] dx = f (x) + c
i.e., the differentiation and integration are inverse processes.
136 Mathematical Methods

4.3 Methods of Integration


If the given function can not be integrated directly by using the standard
formulas mentioned in Table 4.1, then there is a need for developing
some methods for integration. In the remaining part of this chapter, some
such methods are studied and illustrated with the help of a number of
examples. Some of these methods are based on
(a) Substitution (b) Partial fractions (c) Integration by parts and
(d) Successive reduction.
4.3.1 Integration by Substitution
Here, by a suitable substitution, the variable x in ∫ f (x) dx is changed into
another variable t so that the integrand f (x) is changed into F (t) which
is then easily integrable. Usually we make a substitution for a function
whose derivative also occurs in the integral. However, it may be noted
that there are no set rules for making suitable substitution. The following
examples will illustrate this method.
Example 4.3 Evaluate the following integrals

(i) 3x2 (ii) e x (1 + x)


∫ 1 + x6 dx ∫ cos2 ( xe x ) dx
e x + e− x
(iii) ∫ e x − e− x dx (iv) ∫ (4 x + 2) x 2 + x + 1 dx

sin (tan −1 x)
(v) ∫ cot xdx (vi) ∫ 1 + x2
dx

2cos x − 3sin x sin x cos x


(vii) ∫ 6cos x + 4sin x dx (viii) ∫ 2
a cos 2 x + b 2 sin 2 x
dx

Solution (i) Put x 3 = t ⇒ 3x 2dx = dt so that


3x2 dt
∫ 1 + x6 dx = ∫ 1 + t 2 = tan – 1 t = tan – 1 x 2 + c

(ii) Put xe x = t ⇒ e x (1 + x) dx = dt and


e x (1 + x) dt
∫ cos2 ( xe x ) dx = ∫ cos2 t = ∫ sec t dt = tan t + c = tan (xe x) + c
2

(iii) Let e x – e – x = t. Then (e x + e – x) dx = dt and the given integral


reduces to
e x + e− x dt
∫ e x − e− x dx = ∫t = log t + c = log (e x – e – x) + c
Integration 137

(iv) Put t = x 2 + x + 1 so that dt = (2 x + 1) dx and the given integral


becomes
4 3/2 4
∫ (4 x + 2) x 2 + x + 1 dx = 2 ∫ t dt =
3
t + c = ( x 2 + x + 1)3/ 2 + c
3
(v) Put t = sin x so that dt = cos x dx and the given integral becomes
cos x dt
∫ cot x dx = ∫ sin x dx = ∫ t = log t + c = log sin x + c

dx
(vi) Put t = tan – 1 x ⇒ dt = and we have
1 + x2
sin (tan −1 x)
∫1 + x2
dx = ∫ sin t dt = – cos t + c = – cos (tan – 1 x) + c

(vii) Put t = 3cos x + 2sin x so that dt = (– 3 sin x + 2cos x) dx and


we have
2cos x − 3sin x 1 2cos x − 3sin x
∫ 6cos x + 4sin x dx =
2 ∫ 3cos x + 2sin x
dx

1 dt 1 1

2 t
= log t + c = log (3cos x + 2sin x) + c
=
2 2
(viii) Put t = a2 cos2 x + b2 sin2 x which leads to
dt = (– 2a2 cos x sin x + 2b2 sin x cos x) dx
so that
sin x cos x dt 1
∫ a 2 cos 2 x + b2 sin 2 x dx = ∫ 2(b2 − a 2 )t = 2(b2 − a 2 ) log t + c
1
= log (a2 cos2 x + b2 sin2 x) + c
2(b − a 2 )
2

Example 4.4 Evaluate

sec 2 x tan x
(i) ∫ sec2 x + tan 2 x dx (ii) ∫ sin 3x cos 2 x dx
sin 2 x dx
(iii) ∫ cos 2 2 x dx (iv) ∫ 1 + cos x
∫ sin
2
(v) x cos3 x dx
138 Mathematical Methods

Solution (i) The given integral can be written as


sec 2 x tan x sec 2 x tan x sec 2 x tan x
∫ sec2 x + tan 2 x dx = ∫ (1 + tan 2 x) + tan 2 x dx = ∫ 1 + 2tan 2 x dx
Now, put t = tan x so that dt = sec2 x dx and we have
sec 2 x tan x sec2 x tan x 1 dz t dt
∫ sec2 x + tan 2 x dx = ∫ 1 + 2tan 2 x dx =∫ 1 + 2t 2
4∫ z
= log z + c=

where z = 1 + 2t 2 and dz = 4tdt. Therefore, the given integral is

sec 2 x tan x 1 1
∫ sec2 x + tan 2 x dx = 4 log(1 + 2t )+c= log(1 + 2tan 2 x ) + c
2
4
(ii) We have
1
∫ sin 3x cos 2 x dx =
2∫
2sin3 x cos 2 x dx

1
2∫
= [sin(3x + 2 x) + sin(3 x − 2 x)] dx

1
2∫
= (sin5 x + sin x) dx

1 ⎛ cos5 x ⎞
= ⎜− − cos x ⎟ + c
2⎝ 5 ⎠
(iii) Here
sin 2 x sin 2 x
∫ cos 2 2 x dx = ∫ cos 2 x cos 2 x dx
dt
= ∫ tan 2 x sec2 x dx = ∫ tan t sec t 2
1 1
= sec t + c = sec 2 x + c
2 2
where t = 2x and dt = 2dx.
(iv) We have
dx dx
∫ 1 + cos x = ∫ 2cos 2 ( x /2)
1
2∫
= sec2 ( x /2) dx = ∫ sec 2 t dt

= tan t + c = tan (x/2) + c


Integration 139

1 1
where x = t and dx = dt.
2 2
(v) The given integral can be written as

∫ sin x cos3 xdx = ∫ sin 2 x (1 − sin 2 x)cos x dx


2

Now, put t = sin x so that dt = cos x dx and we have

∫ sin ∫ sin x(1 − sin 2 x)cos x dx = ∫ t 2 (1 − t 2 ) dt


2
x cos3 xdx = 2

t3 t5 sin 3 x sin 5 x
= ∫ (t 2 − t 4 ) dt = − +c= − +c
3 5 3 5
4.3.2 Some Special Integrals
We shall now list the following formulas which are used to find the
integrals of a number of functions.
dx 1 x−a
1. ∫ x2 − a2 =
2a
log
x+a
+c

dx 1 a+x
2. ∫ a2 − x2 =
2a
log
a−x
+c

dx 1 −1 x
3. ∫ x2 + a2 =
a
tan
a
+c

dx
4. ∫ = log[ x + x2 − a2 ] + c
x −a2 2

dx x
5. ∫ = sin −1 +c
a2 − x2 a

dx
6. ∫ = log[ x + x2 + a2 ] + c
a +x2 2

1 1 ⎛ x⎞
7. ∫ a 2 − x 2 dx = x a 2 − x 2 + a 2 sin −1 ⎜ ⎟ + c
2 2 ⎝a⎠
1 1
8. ∫ a 2 + x 2 dx =
2
x a 2 + x 2 + a 2 log[ x + a 2 + x 2 ] + c
2
1 1
9. ∫ x 2 − a 2 dx =
2
x x 2 − a 2 − a 2 log[ x + x 2 − a 2 ] + c
2
140 Mathematical Methods

Remarks
The above formulas can be proved easily either by expressing the given
function in a suitable form or by a proper choice of substituion. We shall
give below the hints for the proof of these formulas.
1. Express 1/(x 2 – a2) as
1 1
=
x −a
2 2 ( x + a)( x − a)
1 1 1 1
= + =− +
( x + a ) (−a − a) (a + a) ( x − a) 2a ( x + a ) 2a ( x − a )
and then integrate to get the required formula.
2. Write 1/(a2 – x 2) as
1 1
=
a −x
2 2 (a + x) (a − x)
1 1 1 1
= + = +
(a + x) [a − (− a )] (a + a) (a − x) 2a (a + x) 2a(a − x)
which on integration with respect to x leads to the required formula.
dx
3. Put x = a tan θ and dx = a sec2 θ dθ in ∫ a2 + x2 and integrate to get
the right hand side of formula 3.
dx
4. Put x = a sec θ and dx = a sec θ tan θ dθ in ∫ and integrate.
x − a2
2

dx
5. Put x = a sin θ and dx = a cos θ dθ in ∫ and integrate.
a − x2
2

dx
6. Put x = a tan θ and dx = a sec2 θ dθ in
a + x2
∫ and integrate.
2

7. Put x = a sin θ and dx = a cos θdθ in the left hand side of formula 7
and integrate to get the right hand side.
8. Put x = a tan θ in the left hand side of formula 8 and integrate.
9. Put x = a sec θ.
1
(a) Method to integrate the function of the form
ax + bx + c
2

First we write
Integration 141

⎛ 2 b c⎞
ax 2 + bx + c = a ⎜ x + x + ⎟
⎝ a a⎠

⎛ 2 b c b2 b2 ⎞ ⎡⎛ b ⎞
2
⎛ b 2 − 4ac ⎞⎤
= a ⎜⎜ x + x + + 2 − 2 ⎟⎟ = a ⎢⎜ x + ⎟ − ⎜⎜ ⎟⎟⎥
⎝ a a 4a 4a ⎠ ⎢⎣⎝ 2a ⎠ ⎝ 4a
2
⎠⎥⎦
so that when b 2 – 4ac > 0
dx 1 dx
∫ ax 2 + bx + c a ∫ X 2 − B2
= (3)

b b 2 − 4ac
where X = x + , B2 = ; and when b 2 – 4ac < 0, then
2a 4a 2
dx1 dx
∫ ax 2 + bx + c ∫
a X + B2
2
= (4)

The integrals given by Eqs. (3) and (4) can now be evaluated using the
above formulas 1 to 3.

Example 4.5 Evaluate


dx dx dx
(i) ∫ 2 x2 + x − 1 (ii) ∫1 + x − x2 (iii) ∫ 2 x2 − 2 x + 1
Solution (i) We have
dx dx 1 dx
∫ 2 x2 + x − 1 = ∫ 2 x2 + 1 x − 1 =
2 ∫ (x + 1 )2 − 9
( 2 2 ) 4 16

1 dx
= ∫
2 (x + 1 )2 − ( 3 )2
4 4

1 1 x+ 1 − 3
= log 4 4
+c
2 2(3/4) x+ 1
4
+ 3
4

1 2x − 1
= log +c
3 2x + 2
(ii) The given integral is
dx dx dx
∫1 + x − x2 = −∫ = −∫
x2 − x − 1 (x − 12 )
2
− 5
4
142 Mathematical Methods

dx dx
= ∫5 = ∫
− (x − 1 2
) ( )
2
4 2 2
5
− (x − 2)
1 2

1
5
+x− 1
= log 2 2
+c
2( 5 / 2) 2
5
−x+ 1
2

1 5 − 1 + 2x
= log +c
5 5 + 1 − 2x
(iii) Here
dx 1 dx
∫ 2 x2 − 2 x + 1 =
2 ∫ x2 − x + 1
2

1 dx
= ∫
2 x − x + 14 −
2 1 + 1
4 2

1 dx
= ∫
2 (x − 1 )2 + 1
2 4

1 dx
2 ∫ (x − 1 )2 + (1 )2
=
2 2

1 ⎛ 1 ⎞ −1 x − 1
= ⎜ 1 ⎟ tan
2
+c
2 ⎜⎝ 2 ⎟⎠ 1
2

= tan – 1 (2x – 1) + c
1
(b) Method to integrate the function of the form
ax + bx + c 2

Such types of functions can be treated exactly as in the above method


except for the square root sign so that
dx 1 dx
∫ =
a
∫ (5)
ax 2 + bx + c ± X 2 ± B2
which may then be evaluated with the help of the corresponding formulas
4 to 6.
Integration 143

Example 4.6 Evaluate


dx dx
(i) ∫
3x − x − 2
2
(ii) ∫ 1 − 4 x − 2 x2
Solution (i) We have
dx 1 dx
∫ = ∫
3x − x − 2
2 3 x − 13 x −
2 2
3

1 dx
= ∫
3 (x 2
− 13 x + 1
36 )− 1
36
− 2
3

1 dx
= ∫
(x − 16 ) − (56 )
3 2 2

1 ⎡ 1 ⎤
= log ⎢ x − + 3x 2 − x − 2 ⎥ + c
3 ⎣ 6 ⎦
(ii) The given integral is
dx 1 dx
∫ = ∫
1 − 4x − 2x 2
2 1
2
− 2 x − x2

1 dx
=
2
∫ 1 − ( x + 2 x + 1) + 1
2
2

1 dx
=
2
∫ 2
⎛ 3⎞
⎟⎟ − ( x + 1)
2
⎜⎜
⎝ 2 ⎠
1 x +1
= sin −1 +c
2 (3/2)
px + q
(c) Method to integrate the function of the form
ax + bx + c
2

px + q
and
ax 2 + bx + c
Put
144 Mathematical Methods

⎡d ⎤
px + q = λ ⎢ (ax + bx + c)⎥ + µ = λ(2ax + b) + µ = 2aλx + λb + µ
2
⎣ dx ⎦
Now, equating the coefficients of x and constant, we get
p
2aλ = p ⇒ λ =
2a
and
pb
λb + µ = q ⇒ µ = q – λb = q –
2a
Thus
px + q λ (2ax + b) + µ
∫ ax 2 + bx + c dx = ∫ ax 2 + bx + c
dx

p 2ax + b dx
2a ∫ ax 2 + bx + c
= dx + µ ∫ 2 (6)
ax + bx + c
The first integral on the right hand side can be solved by the substitution
method, while the second integral may be evaluated by one of the methods
px + q
discussed above. In the same manner, we can evaluate ∫ dx
ax 2 + bx + c
and we have
px + q λ (2ax + b) + µ
∫ dx = ∫ dx
ax 2 + bx + c ax 2 + bx + c
p 2ax + b dx
= ∫
2a ax + bx + c
2
dx + µ ∫ (7)
ax + bx + c
2

Example 4.7 Evaluate


5x − 2 6x + 7
(i) ∫ 1 + 2 x + 3x 2 dx (ii) ∫ dx
x − 9 x + 20 2

Solution (i) Here p = 5, q = – 2, a = 3, b = 2 and c = 1. Thus from Eq. (6),


we have
5x − 2 5 6x + 2 11 dx
∫ 1 + 2 x + 3 x2 dx = 6 ∫ 3x 2 + 2 x + 1 dx − 3 ∫ 3x 2 + 2 x + 1
The first integral on the right hand side can be calculated by the
substitution method while the second one can be obtained using
Example 4.5 and we have
Integration 145

5x − 2 5 11 3x + 1
∫1 + 2 x + 3x2 dx = 6 log (3x
2
+ 2 x + 1) − +c tan −1
3 2 2
(ii) Here p = 6, q = 7, a = 1, b = – 9, c = 20, and thus from Eq. (7), we
have
6x + 7
I = ∫ dx
x 2 − 9 x + 20
6 2x − 9 ⎡ ( − 9)(6) ⎤ dx
= ∫
2 x − 9 x + 20
2
dx + ⎢7 −

⎥ ∫
2 ⎦ x − 9 x + 20
2

2x − 9 dx
= 3∫ dx + 34 ∫= I1 + I 2 (8)
x − 9 x + 20
2
x − 9 x + 20 2

Now, put t = x2 – 9x + 20 ⇒ dt = (2x – 9) dx and we have


2x − 9 dt
I1 = 3 ∫ dx = 3∫ = 3(2 t ) = 6 x 2 − 9 x + 20
x − 9 x + 20
2 t
dx dx
and I2 = 34 ∫ = 34∫
x 2 − 9 x + 20 (x − 92 )
2
− 1
4

⎛ 9 ⎞
= 34log ⎜ x − + x − 9 x + 20 ⎟
2
⎝ 2 ⎠
Substituting the values of I1 and I2 in Eq. (8), we get
6x + 7 ⎛ 9 ⎞
∫ dx = 6 x 2 − 9 x + 20 + 34log ⎜ x − + x 2 − 9 x + 20 ⎟ + c
⎝ 2 ⎠
x − 9 x + 20
2

p( x)
(d) Method to integrate the function of the form where
ax + bx + c
2

p(x) is a polynomial of degree 2 or more


In this case, we first carry out the division and write the given function
in the form
p( x) px + q
= polynomial +
ax + bx + c
2
ax + bx + c
2

and then integrate.

x2 + x + 1
Example 4.8 Integrate
x2 − x
146 Mathematical Methods

Solution We have
x2 + x + 1 x2 − x + 2 x + 1 2x + 1 2x − 1 + 2
= =1+ =1+
x −x
2
x −x2
x −x
2
x2 − x
Thus
x2 + x + 1 ⎡ 2x + 1⎤ ⎡ 2x − 1 + 2⎤
∫ x −x
2
dx = ∫ ⎢1 + 2
⎣ x − x⎦⎥ dx = ∫ ⎢1 +
⎣ x 2 − x ⎥⎦
dx

2x − 1 dx ⎛ x − 1⎞
= ∫ dx + ∫ dx + 2 ∫ dx = x + log (x2 – x) + 2log ⎜ ⎟ +c
x −x
2
x −x
2
⎝ x ⎠

(e) Method to integrate the function of the form ax 2 + bx + c and


(px + q) ax 2 + bx + c
In order to evaluate
∫ ax + bx + c dx, substitute y = px + q for suitable
2

p and q and reduce the integral in such a way so that any one of the
formulas 7 to 9 can be used. While for the integral

∫ ( px + q ) ax 2 + bx + c dx, write px + q = A(2ax + b) + B and put


y = ax 2 + bx + c so that the given integral can be reduced to A∫ y dy
+ B ax 2 + bx + c dx.

Example 4.9 Evaluate

(i) ∫ x 2 + 2 x + 5 dx (ii) 3 − 2 x − 2 x 2 dx (iii) ∫ ( x − 5) x + x dx.


2

Solution (i) The given integral

∫ x 2 + 2 x + 5 dx = ∫ ( x + 1) 2 + 4 dx

is of the form a 2 + y 2 dy where y = x + 1 and a = 2. Therefore



∫ (
x 2 + 2 x + 5 dx = 2log x + 1 + x 2 + 2 x + 5 + ) x +1 2
2
x + 2x + 5 + c
(ii) Here we have
3
∫ 3 − 2 x − 2 x 2 dx = 2 ∫
2
− x − x 2 dx

2
3 ⎛ 2 1⎞ 1 ⎛ 7⎞ ⎛ 1⎞
2
= 2∫ − ⎜ x + x + ⎟ + dx = 2 ∫ ⎜⎜ ⎟⎟ − ⎜ x + ⎟ dx
2 ⎝ 4⎠ 4 ⎝ 2 ⎠ ⎝ 2⎠
Integration 147

1
which is of the form ∫ a 2 − y 2 dy, where a = 7 / 2 and y = x + 2 .
Thus from formula 7, we have
2x + 1 7 2 −1 ⎛ 2 x + 1 ⎞
∫ 3 − 2 x − 2 x 2 dx =
4
3 − 2 x − 2 x2 +
8
sin ⎜
⎝ 7 ⎠
⎟+c

(iii) Let y = x 2 + x. Then, dy = (2x + 1) dx. Now, write x – 5 in the form


A(2x + 1) + B which leads to 2A = 1 and A + B = – 5. Thus A = 1/2 and
1 11
B = – 11/2. Therefore x – 5 = (2x + 1) – , and we have
2 2
⎡ 1 11⎤ 2
∫ ( x − 5) x + x dx = ∫ ⎢⎣ 2 (2 x + 1) − 2 ⎥⎦ x + x dx
2

2
1 11 1 11 ⎛ 1⎞ 1
=
2 ∫ y dy − ∫ x 2 + x dx = y 3/2 − ∫ ⎜ x + ⎟ − dx
2 3 2 ⎝ 2⎠ 4

1 2 11 ⎡⎛ x + 1 ⎞ 2 1 ⎛ 1 ⎞⎤
= ( x + x)3/2 − ⎢⎜ 2
⎟⎟ x + x − log ⎜ x + + x + x ⎟⎥ + c
2
3 2 ⎢⎣⎜⎝ 2 ⎠ 8 ⎝ 2 ⎠⎥⎦

Example 4.10 Evaluate


dx dx
(i) ∫ a + b cos x (ii) ∫ a + b sin x
Solution (i) Let t = tan (x/2). Then,
1 1
dt = sec2 (x/2) dx = (1 + t2 ) dx
2 2
Moreover
1 − tan 2 ( x /2) 1 − t2
cos x = =
1 + tan 2 ( x /2) 1 + t2
Therefore
dx 2dt 1 dt
∫ a + b cos x = ∫ 1 + t 2 a + b 1−t 2
= 2∫ (9)
( a − b) t 2 + a + b
1+ t 2

We now have the following three different possibilities:


Case I : When a > b. Here both a – b and a + b are positive and the given
dy
integral is of the form ∫ 2 . Therefore the required integral, from
y + c2
Eq. (9), is
148 Mathematical Methods

dx 2 dt 2 a−b ⎛ a−b⎞
= ∫ = tan −1 ⎜⎜ t ⎟⎟ + c
∫ a + b cos x a − b t + a −b a − b a + b
2 a + b
⎝ a+b⎠
2⎛ a−b x⎞
= tan −1 ⎜ tan ⎟ + c
⎜ a+b 2 ⎟⎠
a −b 2
⎝ 2

Case II : When a < b. From Eq. (9), the required integral is of the form

dy
∫ y2 − c2 , and we have

dx ⎡ b+a + t⎤
2 dt 2 1
∫ a + b cos x = b − a ∫ b+ a − t 2 = b − a b+ a log ⎢⎢ b− a ⎥+c
b −a 2 b− a ⎣
b+ a
b −a
− t ⎥⎦

1 ⎡ b + a + t b − a⎤
= log ⎢ ⎥+c
b −a
2
⎣ b + a −t b − a⎦
2

⎡ b + a + b − a tan ( x / 2) ⎤
1
= log ⎢ ⎥+c
b −a 2
⎣ b + a − b − a tan ( x / 2) ⎦
2

Case III : When a = b. From Eq. (9), we have


dx dt 1 1
∫ a + b cos x = 2∫ = t + c = tan ( x / 2) + c
a+a a a
2 dt
(ii) Let t = tan (x/2). Then, x = 2tan – 1 t and dx = . Also,
1 + t2
2t
sin x = . Therefore, the given integral reduces to
1 + t2
dx 1 2dt dt
∫ a + bsin x = ∫ a + 2bt2 1 + t 2 = 2∫ at 2 + 2bt + a
1+ t

2 dt 2 dt
a ∫ t 2 + 2abt + 1 a ∫ (t +
= = (10)
b 2
a ) + a 2 −b 2
a2
We now have the following possibilities:
Case I : When a 2 > b 2. From Eq. (10), we have

dx 2 dt
∫ a + b sin x = a ∫
( )
2
(t + )
b 2
a
+ a 2 −b2
a2
Integration 149

2 a ⎛ at + b ⎞ 2 ⎛ a tan 2x + b ⎞
= tan −1 ⎜ ⎟+c= tan −1 ⎜ ⎟+c
a ⎜ 2 2 ⎟ ⎜ 2 2 ⎟
a 2 − b2 ⎝ a −b ⎠ a 2 − b2 ⎝ a −b ⎠
Case II: When a 2 < b 2. From Eq. (10), we have
dx 2 dt
∫ a + b sin x =
a∫
( )
2
(t + ba )
2 b2 − a 2

a2

⎡t + b
− b2 − a 2 ⎤
2 a ⎢ a a2 ⎥
= log ⎢ ⎥+C
a 2 b −a
2 2 b b2 − a 2
⎢⎣ t + a
+
a2 ⎥⎦

⎡ a tan x + b − b 2 − a 2 ⎤
1
= log ⎢ 2 ⎥+c
b −a ⎢ x 2 ⎥
⎣ a tan 2 + b + b − a ⎦
2 2 2

2 2
Case III : When a = b .
1. When b = a.
dx 1 dx 1 dx
∫ a + b sin x =
a ∫ 1 + sin x a ∫ 1 − cos ( π2 + x)
=

1 ⎛π x⎞ 1 ⎛π x⎞
2a ∫
= csc2 ⎜ + ⎟ dx = − cot ⎜ + ⎟ + c
⎝ 4 2 ⎠ a ⎝ 4 2⎠
2. When b = – a.
dx 1 dx 1 dx
∫ a + bsin x =
a ∫ 1 − sin x a ∫ 1 + cos ( π2 + x)
=

1 ⎛π x⎞ 1 ⎛π x⎞
=
2a ∫ sec2 ⎜ + ⎟ dx = tan ⎜ + ⎟ + c
⎝ 4 2⎠ a ⎝ 4 2⎠

4.3.3 Integration by Means of Partial Fractions


Now, we give a method for integrating rational functions (for the definition
of rational function, see Chapter 2). For the sake of convenience, we
shall be concerned here with those rational functions whose denominator
has degree 2, 3 or 4. Already, we have developed some methods to integrate
some simple rational functions such as
(i) polynomial functions (here the degree of the denominator is 1).
150 Mathematical Methods

A
(ii) , where n is a positive integer and A is a constant.
( x − a )n
px + q
(iii) , where p, q, a, b are constants.
x 2 + ax + b
When the given integrand is a rational function, which may not be
of the above types, it is then always possible to express the integrand as
the sum of simpler rational functions by a method known as partial
fraction decomposition so that the integration can be carried out easily
by the already known methods. Table 4.2 gives a list of possible partial
fractions that are associated with the rational fractions. It may be noted
that the number of constants to be determined is same as the degree of
the denominator.

Table 4.2 Rational functions and partial fractions

Form of the rational function Form of the partial fraction

px + q A B
1. , a≠b +
( x − a ) ( x − b) x−a x−b
px + q A B
2. +
( x − a)2 x − a ( x − a )2

px + q
3. , where x 2 + ax + b No reduction. Keep it as it is.
x 2 + ax + b
can not be factorized.

px 2 + qx + r A B C
4. , + +
( x − a) ( x − b) ( x − c) x−a x−b x−c
where a, b, c are constants.

px 2 + qx + r A B C
5. , where a ≠ b. + +
x − a ( x − a) 2 x − b
( x − a ) 2 ( x − b)

px 2 + qx + r A B C
6. + +
( x − a )3 x − a ( x − a ) 2 ( x − a )3

px 2 + qx + r A Bx + C
7. , where x 2 + bx + c +
( x − a ) ( x 2 + bx + c) x − a x 2 + bx + c

can not be factorized.

The following examples shall illustrate the integration of functions


through partial fraction decomposition.
Integration 151

Example 4.11 Integrate the following functions


x x2 x2
(i) (ii) (iii)
( x + 1) ( x + 2) x3 + 1 ( x − 1)3 ( x + 1)
Solution (i) Let
x A B
= + (11)
( x + 1) ( x + 2) x + 1 x + 2
which may also be written as
x A B A( x + 2) + B ( x + 1)
= + =
( x + 1) ( x + 2) x + 1 x + 2 ( x + 1) ( x + 2)
which leads to
A(x + 2) + B(x + 1) = x
Now, equating the coefficients of x and constant terms, we get
A + B = 1 and 2A + B = 0 which on solving lead to A = – 1 and B = 2.
Thus Eq. (11) reduces to
x −1 2
= +
( x + 1) ( x + 2) x + 1 x + 2
Therefore
x −1 2
∫ ( x + 1) ( x + 2) dx = ∫ x + 1 dx + ∫ x + 2 dx
= – log (x + 1) + 2log (x + 2) + c
(ii) Let
x2 A Bx + C
+ 2 = (12)
x +1 3x +1 x − x +1
which on simplification yields x 2 = A(x 2 – x + 1) + (Bx + C ) (x + 1) so
that after comparing the corresponding coefficients, we get a system of
three equations
A+B = 1
–A+B +C = 0
A+C = 0
Solving these equations, we have A = 1/3, B = 2/3, C = –1/3 and Eq. (12)
now reduces to

x2 1 2x − 1
= +
x +1
3 3( x + 1) 3( x − x + 1)
2
152 Mathematical Methods

Thus
x2 1 2x − 1
∫ x3 + 1 dx = ∫ 3( x + 1) dx + ∫ 3( x2 − x + 1) dx = I1 + I2 (13)

Now
1 1
I1 = ∫ 3( x + 1) dx = 3 log( x + 1)
and
2x − 1 1
I2 = ∫ 3( x 2 − x + 1) dx = 3 log (x 2 – x + 1) (by taking t = x 2 – x + 1)

Substituting these values of I1 and I2 in Eq. (13), we get

x2 1 1
∫ x3 + 1 dx = 3 log( x + 1) + 3 log(x 2 – x + 1) + c
(iii) Here the denominator can be factorised into linear factors with some
repeated factors and thus we have

x2 A B C D
= + + +
( x − 1) ( x + 1)
3 x − 1 ( x − 1) 2
( x − 1) 3 x +1
which on simplification leads to x 2 = [A(x – 1)2 + B(x – 1) + C] (x + 1)
+ D(x – 1)3 so that when x = 1, x = 0, x = 2 and x = – 1, this equation,
respectively, yields the following set of equations
2C = 1
A–B+C–D = 0
3(A + B + C ) + D = 4
– 8D = 1
Solving these equations, we get A = 1/8, B = 3/4, C = 1/2, D = – 1/8
and thus

x2 A B C D
= + + +
( x − 1) ( x + 1)
3 x − 1 ( x − 1) 2
( x − 1) 3
x +1

1 3 1 1
= + + −
8( x − 1) 4( x − 1) 2
2( x − 1) 3
8( x + 1)
Integration 153

Therefore

x2 1 3
∫ ( x − 1)3 ( x + 1) dx = ∫ 8( x − 1) dx + ∫ 4( x − 1)2 dx
1 1
+∫ dx − ∫ dx
2( x − 1) 3 8( x + 1)

1 3 1 1 1
= log ( x − 1) − + ⋅ − log ( x + 1) + c
8 4( x − 1) 2 (− 2) ( x − 1) 2 8

1 x −1 3 1
= log − − +c
8 x + 1 4( x − 1) 4( x − 1) 2
Remark
p( x)
If in the rational function , the degree of the numerator p(x) is
q( x)
greater than or equal to the degree of the denominator q(x), then we
divide p(x) by q(x) and write
p ( x) P ( x)
= r ( x) +
q ( x) q ( x)
P( x)
where r(x) is a polynomial function and is a rational function, and
q( x)
the degree of P(x) is less than the degree of q(x). The integration is then
P( x)
carried out after splitting into partial fraction. The following
q( x)
example shall illustrate the method.
Example 4.12 Integrate the following functions

x2 + 1 x2
(i) (ii)
x2 − 5x + 6 ( x 2 + 1) ( x 2 + 4)
Solution (i) We have

x2 + 1 5x − 5 5x − 5
=1+ =1+
x2 − 5x + 6 x2 − 5x + 6 ( x − 2) ( x − 3)
Now, let
5x − 5 A B
= +
( x − 2) ( x − 3) x − 2 x − 3
154 Mathematical Methods

which after simplification leads to 5x – 5 = A(x – 3) + B(x – 2) so that


when x = 2, A = – 5 and when x = 3, B = 10 and thus

x2 + 1 5x − 5 A B 5 10
=1+ =1+ + =1− +
x − 5x + 6
2 ( x − 2) ( x − 3) x−2 x−3 x−2 x−3
Therefore

x2 + 1 5 10
∫ x 2 − 5x + 6 dx = 1∫ dx − ∫ x − 2 dx + ∫ x − 3 dx
= x – 5 log (x – 2) + 10 log (x – 3) + c
(ii) Put t = x 2, then

x2 t A B
= = +
( x + 1) ( x + 4)
2 2 (t + 1) (t + 4) t + 1 t + 4
The values of A and B can be obtained in a similar manner as in the
above examples, and we have A = –1/3, B = 4/3. Thus

x2 A B ( −1/3) (4/3)
= + = +
( x + 1)( x + 4)
2 2
t +1 t + 4 t +1 t+4

1 4
= − +
3( x + 1)
2
3( x + 4)
2

Therefore

x2 1 dx 4 dx
∫ ( x 2 + 1) ( x 2 + 4) dx = − ∫ + ∫ 2
3 x +1 3 x + 4
2

1 −1 2 x
= − tan x + tan −1 + c
3 3 2
Note that we have used the substitution in order to apply the partial
fraction decomposition method easily, the substitution was not made for
the integration purpose. The following example illustrates the integration
of a function which involves both the substitution method and the partial
fraction method.

(3sin θ − 2)cos θ
Example 4.13 Evaluate ∫ 5 − cos2 θ − 4sin θ d θ
Integration 155

Solution Put t = sin θ then dt = cos θ d θ, and we have


(3sin θ − 2)cos θ (3t − 2) dt
∫ 5 − cos2 θ − 4sin θ d θ = ∫ 5 − (1 − t 2 ) − 4t
3t − 2 3t − 2
= ∫ t 2 − 4t + 4 dt = ∫ (t − 2)2 dt
Now, let
3t − 2 A B
= +
(t − 2) 2 t − 2 (t − 2)2
then, after simplification we get 3t – 2 = A(t – 2) + B. Comparing the
corresponding coefficients and solving the resulting equations, we get
A = 3 and B = 4. Therefore
(3sin θ − 2)cos θ 3t − 2
∫ 5 − cos 2 θ − 4sin θ d θ = ∫ (t − 2)2 dt
A B 3 4
= ∫ t − 2 dt + ∫ (t − 2)2 dt = ∫ t − 2 dt + ∫ (t − 2)2 dt
⎛ 1 ⎞ 4
= 3log (t – 2) + 4 ⎜ − ⎟ + c = 3 log (sin θ – 2) – sin θ − 2 + c
⎝ t − 2⎠
4.3.4 Integration by Parts
This method is useful in finding the integrals of the product of two
functions. Let f (x) and g (x) be two functions, then
⎡d ⎤
∫ f ( x) g ( x) dx =
f ( x) ∫ g ( x) dx − ∫ ⎢ f ( x) ∫ g ( x) dx⎥ dx
⎣ dx ⎦
(14)

Equation (14) can be stated as:


Integral of the product of two functions = First function (integral of the
second fnction) – Integral of [the derivative of the first function (integral
of the second function)].
Equation (14) is known as the formula of integration by parts. The other
equaivalent forms of this formula are as follows:

∫ fdg = fg– ∫ gdf (15)


or
∫ f ( x) dg ( x) = f (x) g(x) – ∫ g ( x) df ( x) (16)
or

∫ f ( x) g ′( x) dx = f (x) g(x) – ∫ g ( x) f ′( x) dx (17)


156 Mathematical Methods

Remarks
In order to use Eq. (14) for finding the integral of the product of two
functions, we should choose the second function in such a way that its
integral can be obtained easily. However, the following guidelines will
be helpful in making a choice of the first/second function.
1. If both the functions f and g in the product can easily be integrated
separately, and one of them is of the form x m, where m is a positive
integer (i.e., either f = x m or g = x m), then f or g should be taken as the
first function.
2. If one of the functions in the product is a logarithmic or an inverse
trigonometric function, then it should be taken as the first function.
3. Equation (14) can also be used in finding the integral of a single
function f, as f can always be written as f × 1, and in such cases 1 should
be taken as the second function.

Example 4.14 Evaluate

x 2 dx
(i) ∫ x cos xdx (ii) ∫ x 2 e3x dx (iii) ∫ (a + bx)2
Solution (i) Here we take x as the first function and use the formula
(i.e., Eq. (14)) for the integration by parts so that

∫ x cos x dx = x sin x – ∫ (1)sin x dx

= x sin x − ∫ sin xdx


= x sin x + cos x + c
(ii) Take x2 as the first function and use Eq. (14) to get

⎛1 ⎞ ⎛1 ⎞
∫x e dx = x 2 ⎜ e3 x ⎟ − ∫ (2 x)⎜ e3 x ⎟ dx
2 3x
⎝ 3 ⎠ ⎝3 ⎠
1 2 3x 2
= x e − ∫ xe3 x dx
3 3
1 2 3x 2 ⎡ ⎛ 1 3x ⎞ 1 ⎤
= x e − ⎢ x ⎜ e ⎟ − ∫ (1) e3 x dx⎥
3 3 ⎣ ⎝3 ⎠ 3 ⎦
⎛1 2 2⎞
= ⎜ x 2 − x + ⎟ e3 x + c
⎝ 3 9 27 ⎠
Integration 157

(iii) Here by taking x 2 as the first function and integrating by parts, we


get
x 2 dx
∫ (a + bx)2 = ∫x
2
( a + bx ) −2 dx

⎡ (a + bx) −1 ⎤ ⎡ (a + bx) −1 ⎤
= x2 ⎢ ⎥ − ∫2x ⎢ ⎥ dx
⎣ −b ⎦ ⎣ −b ⎦
x2 2 bx
= − + 2∫ dx
b(a + bx) b a + bx
x2 2 (a + bx) − a
b(a + bx ) b2 ∫ a + bx
= − + dx

x2 2 ⎡ a ⎤
= − + 2 ∫ ⎢1 − dx
b(a + bx) b ⎣ a + bx ⎥⎦
x2 2 ⎡ a ⎤
= − + ⎢ x − b log (a + bx)⎥ + c
b(a + bx) b 2 ⎣ ⎦
Example 4.15 Evaluate
log x dx −1
(i) ∫ x log x dx (ii) ∫ ( x + 1)2 (iii) ∫ x tan x dx

Solution (i) Taking log x as the first function and integrating by parts,
we get
⎛1 2⎞ 1⎛1 2⎞
∫ x log xdx = (log x) ⎜ x ⎟ − ∫ ⎜ x ⎟ dx
⎝2 ⎠ x⎝2 ⎠
1 2 1 1
x log x − ∫ xdx = x 2 (2log x − 1) + c
=
2 2 4
(ii) The given integral can be written as
log x dx −2
∫ ( x + 1)2 = ∫ log x( x + 1) dx

Now, choosing log x as the first function and integrating by parts, we


have
−2 1
∫ log x( x + 1) dx = log x [– (x + 1) – 1] –
∫ x [− ( x + 1)
−1
] dx
158 Mathematical Methods

log x 1 log x ⎛1 1 ⎞
x + 1 ∫ x( x + 1) x + 1 ∫⎝ x x + 1⎠
= − + dx = − + ⎜ − ⎟ dx

log x
= − + log x – log (x + 1) + c
x +1
(iii) We have
2
−1
x dx = (tan −1 x) x − 1 x2
∫ x tan 2 ∫ 1 + x2 2
dx

1 2 −1 1 ⎛ 1 ⎞ 1 2 −1 1
= x tan x − ∫ ⎜1 − ⎟ dx = ( x + 1) tan x − x + c
2 2 ⎝ 1 + x2 ⎠ 2 2

Example 4.16 Evaluate


(i) ∫ log x dx (ii) ∫ sin −1 x dx (iii) ∫ tan
−1
x dx

Solution (i) The given integral can be written as

∫ log xdx = ∫ (log x)(1) dx


Taking log x as the first function and 1 as the second function and
integrating by parts, we get
1
∫ log xdx = ∫ (log x)(1) dx = (log x) x − ∫ x xdx = x log x – x + c

(ii) The given integral can be written as


−1
∫ sin xdx = ∫ (sin −1 x )(1) dx
Taking sin–1 x as the first function and 1 as the second function and
integrating by parts, we get
−1 1
∫ sin xdx = ∫ (sin −1 x)(1) dx = (sin −1 x) x − ∫ x dx
1 − x2
−1 1 − 2 x dx
= x sin −1 x + 1 − x 2 + c
2 ∫ 1 − x2
= x sin x +

(iii) Taking tan –1 x as the first function and integrating by parts, we get
−1 −1
∫ tan xdx = ∫ tan x(1) dx

1 1
= (tan – 1 x) x – ∫1 + x2 x dx = x tan – 1 x – log (1 + x 2) + c
2
Integration 159

Example 4.17 Evaluate


−1 ⎡ 2x ⎤
(i) ∫ sin x log cos x dx (ii) ∫sin ⎢1 + x 2 ⎥ dx
⎣ ⎦
1− x x sin −1 x
(iii) ∫ tan −1
1+ x
dx (iv) ∫ 1 − x2
dx

x tan −1 x
(v) ∫ (1 + x2 ) 3/2 dx
Solution (i) Put cos x = t then – sin x dx = dt and the given integral
reduces to

∫ sin x log cos x dx = − ∫ log t dt = cos x [1 – log (cos x)] + c


(ii) Put x = tan θ so that θ = tan –1 x and we have
⎡ 2x ⎤ −1 ⎡ 2tan θ ⎤
sin −1 ⎢ –1 –1
⎥ = sin ⎢1 + tan 2 θ ⎥ = sin (sin 2θ) = 2θ = 2tan x
⎣1 + x 2 ⎦ ⎣ ⎦
The given integral thus becomes
−1 ⎡ 2x ⎤ −1 x dx = 2x tan – 1 x – log (1 + x 2) + c
∫sin ⎢1 + x 2 ⎥ dx = 2∫ tan
⎣ ⎦
1− x 1 − cos θ
(iii) Put x = cosθ, then tan −1 = tan −1
1+ x 1 + cos θ
2sin 2 θ /2 1
= tan
−1
= tan – 1 (tan θ/2) = θ/2 = cos −1 x. Therefore
2cos θ / 2
2 2

1− x 1
−1
dx = ∫ cos −1 x dx
∫ tan
1+ x 2
which on integrating by parts leads to

1− x 1⎡ −1 −1 ⎤
∫ tan
−1
dx = ⎢(cos x) x − ∫ x dx⎥
1+ x 2 ⎢⎣ 1− x 2
⎥⎦
1⎡ 1 ⎤
= ⎢(cos −1 x) x − ∫ (1 − x 2 )−1/2 (−2 x) dx⎥
2⎣ 2 ⎦
1⎡
= (cos −1 x) x − 1 − x 2 ⎤ + c
2 ⎢⎣ ⎥⎦
160 Mathematical Methods

(iv) Put x = sin θ, then dx = cos θ dθ and the given integral reduces to
x sin −1 x θ sin θ

1− x 2
dx = ∫ cos θ d θ = ∫ θ sin θ d θ
1 − sin 2 θ
Now, integrating by parts, we get
x sin −1 x
∫ 1 − x2
dx = – θ cos θ + sin θ + c = – θ 1 − sin 2 θ + sin θ + c

= − 1 − x sin – 1 x + x + c
2

(v) Put x = tan θ, then dx = sec2 θ dθ and θ = tan–1 x. Therefore


x tan −1 x θ tan θ sec 2 θ d θ θ tan θsec2 θ d θ
∫ (1 + x 2 )3/2 dx = ∫ (1 + tan 2 θ)3/2 = ∫ sec3 θ
= ∫ θsin θ d θ = – θ cos θ + sin θ + c = (– θ + tan θ) cos θ + c

tan θ − θ tan θ − θ x − tan −1 x


= +c= +c= +c
sec θ 1 + tan 2 θ 1 + x2
4.3.5 Integration by Successive Reduction
When we are integrating a function by parts, sometimes it happens that
we get another integral that is similar to the one with which we have
started. This can be integrated by parts leading to yet another integral of
the same form. By successive repetition of this, we can get an easily
integrable function, which ultimately leads to the result that we want to
obtain. With the help of examples, we shall illustrate here that how we
can use this method to obtain the integral of some important functions.
n ax
Example 4.18 Evaluate ∫x e dx.
Solution For each positive integer n, let
n ax
In = ∫x e dx (18)
Now, considering x n as
the first function and e ax as
the second function
and integrating the right hand side of Eq. (18) by parts, we get

eax e ax 1 n
n ax
∫ x e dx = x
n
− ∫ nx n −1 dx = x n eax − I n −1 (19)
a a a a
which is a relation between In and In–1, known as reduction formula.
Integration 161

Remark
Put n = n – 1in Eq. (19), we get
1 n −1 ax n − 1
x e −
In–1 = I n− 2
a a
In a similar way, we can have In–2, In–3, ...

m
Example 4.19 Evaluate ∫x (log x ) n dx.
Solution Let
Im, n = ∫ x m (log x)n dx
Then, taking (log x)n as the first function and integrating by parts,
we get
m +1
Im, n = x (log x) n −
n
x m (log x)n −1 dx
m +1 m +1∫
Therefore
x m +1 n
Im, n = (log x) n − I m, n −1
m +1 m +1
n
Example 4.20 Evaluate ∫x sin x dx.
Solution Let for each positive integer n,

∫x
n
In = sin xdx
Then
n
In = ∫x sin xdx

= x n (– cos x) – ∫ nx n −1 ( − cos x ) dx
n−2
= – x n cos x + nxn – 1 sin x – n(n – 1) ∫ x sin xdx
= – x n cos x + nxn – 1 sin x – n(n – 1) In– 2

Example 4.21 Evaluate ∫ sin m x cos n x dx, where m and n are natural
numbers.

Solution Let

Im, n = ∫ sin m x cos n xdx = ∫ sin m x cos n −1 x cos x dx


162 Mathematical Methods

Now, taking cosn –1 x as the first function and sinm x cos x as the second
function and integrating by parts, we get
m +1 ⎞
n −1 ⎛ sin x n−2 sin m +1 x
Im, n = cos x ⎜⎜ ⎟⎟ − ∫ (n − 1) cos x(− sin x) dx
⎝ m +1 ⎠ m +1

1 ⎡ n −1
= cos x sin m +1 x + (n − 1) ∫ cos n − 2 x sin m + 2 xdx⎤
m + 1⎣ ⎦

1
= [cosn– 1 x sinm+1 x
m +1
n− 2
+ (n – 1) ∫ cos x sin m x (1 – cos2 x) dx]

1
= [cosn– 1 x sinm+ 1 x + (n – 1) (Im, n – 2 – Im, n)]
m +1
Therefore
(m + n) Im, n = cosn– 1 x sinm + 1 x + (n – 1) Im, n – 2

Example 4.22 Evaluate ∫ sin n xdx.

Solution Let
n n −1
In = ∫ sin xdx = ∫ sin x sin xdx
Taking sinn–1 x as the first function and sin x as the second function and
integrating by parts, we get

In = sinn – 1 x(– cos x) – ∫ (n − 1) sinn – 2 x cos x (– cos x) dx

= – sinn – 1 x cos x + (n – 1) ∫ sin n − 2 x cos2 x dx

n−2
= – sinn – 1 x cos x + (n – 1) ∫ sin x (1 – sin2 x) dx

n−2
= – sinn– 1 x cos x + (n – 1) ∫ sin xdx − (n − 1) ∫ sin n xdx

= – sinn–1 x cos x + (n – 1) In–2 – (n – 1) In


Therefore

sin n −1 x cos x n − 1
In = − + In – 2
n n
Integration 163

Remark
In a similar way, we can find the reduction formulas for some other well
known functions, for example, cosn x, (log x)n, tann x, etc. The reduction
formulas for such functions, alongwith the formulas evaluated through
Examples 4.18 - 4.22 are :

Reduction Formulas

n ax 1 n ax n
(i) If In = ∫x e dx, then In = x e − I n −1.
a a
m
(ii) If Im, n = ∫x (log x ) n dx, then

x m +1 n
Im, n = (log x) n − I m, n − 1
m +1 m +1
(iii) If In = ∫ (log x)n dx, then In = x (log x)n – nIn – 1.
n
(iv) If In = ∫x sin x dx, then
In = – x n cos x + n xn– 1 sin x – n(n – 1) In – 2
(v) If Im, n = ∫ sin m x cos n xdx, then
(m + n)Im, n = cosn – 1 x sinm +1 x + (n – 1) Im, n – 2
(vi) If Im, n = ∫ sin m x cos n xdx, then
(m + n) Im, n = – sinm – 1 x cosn + 1 x + (m – 1) Im – 2, n
(vii) If In = ∫ sin n xdx, then

sin n −1 x cos x n − 1
In = – + I n−2
n n
(viii) If In = ∫ cos n xdx, then

cos n −1 x sin x n − 1
In = – + I n−2
n n
n
(ix) If In = ∫ tan x dx, then

tan n −1 x
In = – In – 2
n −1
164 Mathematical Methods

4.4 Definite Integral


We have studied the indefinite integral of a function as the inverse of
diffferentiation and have discussed a number of methods for finding the
anti-derivative (or indefinite integral) of a given function. We shall now
be concerned with the definite integral of a function and see that these
two notions namely indefinite integral and definite integrals are closely
related. The concept of definite integral arose from the problems of finding
the areas of bounded regions. We are already familiar with the formulas
for calculating the areas of some of the well known figures, e.g., the
areas of triangle, rectangle, trapezium, circle, etc. These are all bounded
regions. Now, is it possible to evaluate the area of a region whose boundary
is not familiar? The answer is yes, and this is what we intend to do in
this and the forthcoming sections.
y

y = f (x )

1 2 3 n–1

x
O a a + h a + 2h a(n – 1)h b

Fig. 4.1 Area bounded by y = f (x), x = a, x = b and x-axis


Let y = f (x) be a continuous function defined on a closed interval
[a, b], where a, b are finite and b > a. Divide this interval into n equal
parts, each of width h so that nh = b – a. By doing so, we get rectangles.
In Figure 4.1, these rectangles are numbered as 1, 2, 3, ... n – 1.
The areas of these rectangles, respectively, are h f (a), h f (a + h),
h f (a + 2h), ..., h f [a + (n – 1)h]. Now, add all these areas and take the
limit as h approaches to zero (or n approaches to infinity). If this limit
exists, then it is known as the definite integral of the function y = f (x).
We thus have
Definition 4.2 Definite integral as the limit of a sum. Let y = f (x) be a
continuous function defined on [a, b]. Then
lim h[ f (a) + f (a + h) + f (a + 2h) + ... + f {a + (n − 1)h}]
h→ 0
Integration 165

where nh = b – a, is called the definite integral of f (x) with respect to


b
x between the limits a and b and is written as ∫ f ( x) dx. Thus
a

∫ f ( x) dx = hlim
→ 0 h[ f (a) + f (a + h) + ... + f {a + (n – 1) h}]
a

⎪⎧ ⎪⎫
r = n −1

= h → 0 ∑ f (a + rh)⎬
lim ⎨ (20)
⎩⎪ r = 0 ⎭⎪
Remarks

(i) Equation (20) can also be written as


b

∫ f ( x)dx = (b – a) lim
1
n→∞ n
[ f (a) + f (a + h) + ... + f {a + (n – 1)h}] (21)
a

(ii) Equation (20)/(21) represents the area of the region bounded by the
curve y = f (x), x-axis, the ordinates x = a and x = b.
b
(iii) In the expression ∫ f ( x) dx, a and b are called lower and upper
a
limits of integration, respectively, f (x) is known as the integrand and x
is the variable of integration.
(iv) The above definition is true even if f (x) takes negative values and in
such case, we have
(a) Some portion of the region may exist below the x-axis.
b
(b) The area given by ∫ f ( x) dx has a negative sign, i.e. the area of the
a

portion below the x-axis has a negative sign.


We shall now illustrate the use of Definition 4.2 with the help of
following examples.
2
Example 4.23 Find ∫ xdx as the limit of a sum.
1

Solution From the Definition 4.2, we have


166 Mathematical Methods

b
1
∫ f ( x) dx = (b – a) lim
n→∞ n
[ f (a) + f (a + h) + f (a + 2h) + ...
a

+ f {a + (n – 1)h}]
Here a = 1, b = 2, f (x) = x and h = (b – a)/n = 1/n. Therefore
2
1⎡ ⎛ 1⎞ ⎛ 2⎞ ⎛ n − 1 ⎞⎤
∫ xdx = lim
n→∞ n⎣
⎢ f (1) + f ⎜1 + n ⎟ + f ⎜1 + n ⎟ + ... + f ⎜1 + n ⎟⎥
⎝ ⎠ ⎝ ⎠ ⎝ ⎠⎦
1

1 ⎡ ⎛ 1⎞ ⎛ 2⎞ ⎛ n − 1 ⎞⎤
= lim ⎢1 + ⎜1 + n ⎟ + ⎜1 + n ⎟ + ... + ⎜1 + 1 ⎟⎥
n→∞ n ⎣ ⎝ ⎠ ⎝ ⎠ ⎝ ⎠⎦

1 ⎡ ⎛1 2 n − 1 ⎞⎤
= lim ⎢(1 + 1 + ... + 1) n times + ⎜ n + n + ... + n ⎟⎥
n→∞ n ⎣ ⎝ ⎠⎦
1 ⎡ 1 ⎤
= lim
n ⎢n + n {1 + 2 + ... + (n − 1)}⎥
n→∞ ⎣ ⎦
1 ⎡ 1 (n − 1) (n) ⎤
= lim
n ⎢n + n ⎥
n→∞ ⎣ 2 ⎦

= 1 ⎡ 3n − 1⎤
lim
n→∞ n ⎢
⎣ 2 ⎦

= ⎡3 1 ⎤ 3
lim ⎢ − ⎥ =
n → ∞ ⎣2 2n ⎦ 2
2
Example 4.24 Find ∫ ( x 2 + 1) dx as the limit of a sum.
0
Solution From the definition, we have
b
1
∫ f ( x) dx = (b − a) nlim
→∞ n
[ f (a) + f (a + h) + f (a + 2h)
a
+ ... + f {a + (n – 1)h}]
Here a = 0, b = 2, f (x) = x 2 + 1 and h = (b – a)/n = 2/n. Therefore
2
1 ⎡ ⎛ 2⎞ ⎛4⎞ ⎛ 2n − 2 ⎞⎤
∫(x + 1) dx = 2 lim ⎢ f (0) + f⎜ ⎟+ f ⎜ ⎟ + ... +
2
f⎜ ⎟⎥
n→∞ n ⎣ ⎝n⎠ ⎝n⎠ ⎝ n ⎠⎦
0
Integration 167

2
⎡ ⎛ 2 ⎞ ⎛ 2 ⎞ 2 ⎤
∫(x
2
+ 1) dx = 2 lim 1 ⎢1 + ⎜ 2 + 1⎟ + ⎜ 4 + 1⎟ + ... + ⎛⎜ 2n − 2 ⎞⎟ + 1⎥
n→∞n⎢ ⎜ 2 ⎟ ⎜ 2 ⎟ ⎝ n ⎠2
0 ⎣ ⎝n ⎠ ⎝n ⎠ ⎥⎦

1
= 2 lim [(1 + 1 + ... + 1)n times
n→∞ n
1
+ (22 + 42 + ... + (2n – 2)2]
n2
1⎡ 22 2 2 ⎤
= 2 lim ⎢n + 2 (1 + 2 + ... + (n − 1) )⎥
2
n→∞ n n
⎣ ⎦
1⎡ 4 (n − 1) n(2n − 1) ⎤
= 2 lim ⎢n+ 2 ⎥
n→∞ n ⎣ n 6 ⎦
⎡ 2⎛ 1⎞⎛ 1 ⎞⎤ ⎛ 4 ⎞ 14
= 2 lim ⎢1 + ⎜1 − ⎟ ⎜2 − ⎟ = 2 ⎜1 + ⎟=
n→∞ ⎣ 3⎝ n⎠⎝ n ⎠⎥⎦ ⎝ 3⎠ 3

b
x
Example 4.25 Find ∫ e dx as the limit of a sum.
a

Solution From the definition, we have


b

∫ f ( x) dx = lim h[ f (a) + f (a + h) + f (a + 2h) + ... + f {a + (n – 1)h}]


h→0
a
Here f (x) = e x and nh = b – a, and we have
b
x
∫e dx = lim h[e a + e a+ h + e a + 2h + ... + e a + (n – 1)h]
h→0
a

= lim he a[1 + e h + e 2h + ... + e(n – 1)h]


h→0

⎡ enh − 1⎤
= lim he a ⎢ ⎥
h
h→0
⎣ e −1⎦
[using the formula for the sum of n terms of a geometric progression (see
Appendix A) whose first term is 1 and common ratio is e h]. Thus
b
x a ⎡ h ⎤
∫ e dx = hlim e (enh − 1) ⎢ h ⎥
a
→0 ⎣ e − 1⎦
168 Mathematical Methods

b ⎡ 1 ⎤ b a
∫ e x
dx = lim e a b−a
( e − 1) ⎢1 h h2
⎥ =e –e
a
h→0 ⎢⎣1! + 2!
+ 3!
+ ...⎥

4.5 Fundamental Theorems of Integral Calculus


In the above examples we have evaluated the definite integral of some
simpler functions using the definition of the definite integral as the limit
of a sum. However, if the given function is a complicated one then it is
very difficult to use either of the equation (20) or (21). One can now ask:
is there any simpler way to calculate the definite integral of a given
function? The answer is yes.
It may be noted, from the above remark, that the definite integral of
a function f (x) represents the area of the region bounded by the curve,
x-axis and the ordinates. This area depends on the value of x, i.e. this
area is a function of x. If we denote this function of x by A(x) and call this
function A(x) as area function, then
b
A(x) = ∫ f ( x) dx
a
We thus have
Theorem 4.2 First Fundamental Theorem of Integral Calculus. Let the
area function be defined by
b
A(x) = ∫ f ( x)dx for all x ≥ a then A′(x) = f (x)
a
where A′(x) denotes the derivative of A with respect to x.
This theorem can also be stated as:
x
“If f (x) is continuous on [a, b], then F(x) = ∫ f (t ) dt has a derivative at
a
x
dF d
f (t ) dt = f (x), a ≤ x ≤ b.”
dx dx ∫a
every point of [a, b] and =

Remarks
dF
1. The equation = f (x) has a solution for every continuous
dx
function f (x)
Integration 169

2. Every continuous function is the derivative of some other function


x
(namely ∫ f (t ) dt ).
a
3. Every continuous function has an antiderivative.
4. The processes of integration and differentiation are inverse of each
other.
Theorem 4.3 Second Fundamental Theorem of Integral Calculus. Let
F (x) be an anti-derivative of f (x). Then
b

∫ f ( x) dx = [F ( x)]ba = F (b) – F (a) (22)


a

where f (x) is continuous at every point of [a, b] (for proofs of Theorems


4.2 and 4.3, see [5]).
Theorem 4.3 provides a method for evaluating the definite integral
b
of a function, (i.e., ∫ f ( x) dx) without evaluating the limit of a sum as
a
follows:

(i) Find the indefinite integral ∫ f ( x) dx, by the methods given in


Section 4.3. Let it be F (x) (there is no need to put the constant of
integration).
b
(ii) Take [F ( x )]ba. This is F(b) – F(a) and is the value of ∫ f ( x) dx.
a

This proceedure is now illustrated through the following examples:


2
Example 4.26 Evaluate
∫ xdx.
1

Solution We have
2
x2
2
⎡ x2 ⎤ 2 2 12 3
∫ xdx = 2
⇒ ∫ xdx = ⎢ ⎥ = − =
1 ⎣ 2 ⎦1 2 2 2

It may be noted that the definite integral calculated in this example


has also been evaluated using the definition of the definite integral as
170 Mathematical Methods

the limit of a sum (cf., Example 4.23). We can observe that the simplicity
in the calculations is due to the use of Second Fundamental Theorem of
Integral Calculus.

Example 4.27 Evaluate the following integrals


1 2
2x + 3 5x
(i) ∫ 5x2 + 1 dx (ii) ∫ x 2 + 4 x + 3 dx
0 1

π /4
a
(iii) ∫ (1 − x 2 )sin 2 x dx (iv)
∫ a 2 − x 2 dx
0
0
e
dx
(v) ∫ x (log x)1/3
1/ e
Solution (i) We have
1 1 1
2x + 3 2x 3
∫ 5 x2 + 1 dx = ∫ 5x2 + 1 dx + ∫ 2
+1
dx
0 0 0 5x

1 1
1 10 x 3 dx
= ∫
5 0 5x + 1
2
dx + ∫ 2
5 0 x + 1/5
1
⎡1 3 ⎛ 1 ⎞ 1 ⎛ x ⎞⎤
= ⎢ log(5 x 2 + 1) + ⎛⎜ ⎞⎟ ⎜ ⎟ tan ⎜ ⎟⎥
⎣5 ⎝ 5 ⎠ ⎝ 1/ 5 ⎠ ⎝ 1/ 5 ⎠⎦ 0
1
(log 6 + 3 5 tan −1 5)
=
5
(ii) The given integral is
2 2 2
5x 5x ⎡ 15 5 ⎤
∫ x2 + 4 x + 3 dx = ∫ ( x + 1) ( x + 3) dx = ∫ ⎢⎣ 2( x + 3) − 2( x + 1) ⎥⎦ dx
1 1 1
2
⎡15 5 ⎤ 1
= ⎢ log ( x + 3) − log( x + 1)⎥ = (15 log 5 – 25 log 2 – 5 log 3)
⎣2 2 ⎦1 2
(using the method of partial fractions).
(iii) Here
π /4 π /4 π /4
⎡ cos 2 x ⎤ ⎛ cos2 x ⎞
∫ (1 − x 2 )sin 2 x dx = ⎢− (1 − x 2 )
⎣ 2 ⎥⎦ 0
− ∫ (− 2 x) ⎜⎝ −2 ⎠
⎟ dx
0 0
Integration 171

π /4 π /4
1
∫ (1 − x 2 )sin2 x dx =
2
− ∫ x cos2 xdx
0 0

π /4
1 ⎡π ⎛ cos2 x ⎞⎤ 3 π
= − +⎜ ⎟ = −
2 ⎢⎣ 8 ⎝ 4 ⎠⎥⎦ 0 4 8

(using the method of integration by parts).

(iv) Put x = a sin θ so that dx = a cos θ d θ and when x = a, θ = π/2;


x = 0, θ = 0. The given integral can now be expressed as
a π /2

∫ a 2 − x 2 dx = ∫ (a 2 − a 2 sin 2 θ) (a cos θ) d θ
0 0

π /2
= a2 ∫ cos θdθ
2

π /2
a2
=
2 ∫ (1 + cos2θ) dθ
0

π /2
a2 ⎡ sin2θ ⎤ πa 2
= ⎢θ + 2 ⎥ =
2 ⎣ ⎦0 4
(v) Put log x = t so that dx/x = dt. Also when x = e, t = 1 and when
x = 1/e, t = – 1. The given integral thus becomes

e 1
dx
1
dt ⎡ t 2/3 ⎤
∫ 1/3
= ∫ 1/3
=⎢ ⎥ =0
1/ e x (log x ) −1 t ⎣ 2/3 ⎦ −1
Before discussing more about definite integrals, we shall now make
a comparison between definite and indefinite integrals of a function as
follows:
Comparison between indefinite and definite integrals
1. The indefinite integral of a function is another function, while the
definite integral of a function over an interval is a number. This number
varies as the interval varies.
172 Mathematical Methods

2. The indefinite integral ∫ f ( x) dx of a function is not unique. It can


b
vary by a constant. On the other hand, the definite integral ∫ f ( x) dx is
a
a unique number.
3. For indefinite integral, we have

∫ f ( x) dx, F ( y) = ∫ f ( y ) dy
F (x) =

which never allows us to write F ( y) = ∫ f ( x) dx. While for definite


integral, we have
b b b

∫ f ( x) dx = ∫ f ( y ) dy = ∫ f (t ) dt , etc.
a a a

4.6 Properties of Definite Integrals


Following are some important properties of definite integrals (the proofs
can be seen in [5]) which are useful in evaluating the definite integrals
more easily.
b a
1. ∫ f ( x) dx = − ∫ f ( x) dx
a b
b c b
2. ∫ f ( x) dx = ∫ f ( x) dx + ∫ f ( x) dx
a a c
a
3. ∫ f ( x) dx =0
a
a a
4. ∫ f ( x) dx = ∫ f (a − x) dx
0 0
a a
5. ∫ f ( x) dx = 2∫ f ( x) dx if f (x) is an even function.
−a 0
a
6. ∫ f ( x) dx = 0 if f (x) is an odd function.
−a
2a a
7. ∫ f ( x) dx = 2∫ f ( x) dx if f (x) = f (2a – x)
0 0
Integration 173

2a
8. ∫ f ( x) dx = 0 if f (x) = – f (2a – x)
0

2a a a

9. ∫ f ( x) dx = ∫ f ( x) dx + ∫ f (2a − x) dx
0 0 0

na a
10. ∫ f ( x) dx = n∫ f ( x) dx if f (x) = f (a + x)
0 0

We shall now workout a number of examples to illustrate the use of


these properties of definite integrals.

Example 4.28 Evaluate the following integrals


4
(i) ∫ f ( x) dx where
1

⎧7 x + 3, if 1 ≤ x ≤ 3
f (x) = ⎨
⎩8 x, if 3 ≤ x ≤ 4

9
(ii) ∫ f ( x) dx where
0

⎧sin x, if 0 ≤ x ≤ π /2

f (x) = ⎨1, if π / 2 ≤ x ≤ 3
⎪ x −2
⎩e , if 3 ≤ x ≤ 9
Solution
(i) The given integral is
4 3 4

∫ f ( x) dx = ∫ (7 x + 3) dx + ∫ 8 xdx = 62
1 1 3

(ii) We have
9 π /2 3 9
π
∫ f ( x) dx = ∫ sin xdx + ∫ (1) dx + ∫ e x − 2 dx = 4 − + e7 − e
0 0 π /2 3
2
174 Mathematical Methods

Example 4.29 Evaluate the following integrals

π /2 π /2
π /2
(i) ∫ cos x dx (ii) ∫ sin xdx2
(iii) ∫ sin 7 xdx
− π /2 − π /2
− π /2
Solution (i) Since cos x is an even function, therefore we have
π /2 π /2

∫ cos x dx = 2[sin x]0


π /2
∫ cos x dx = 2 =2
−π /2 0

(ii) Since sin2 x is an even function, therefore


π /2 π /2 π /2 π /2
1 − cos2 x ⎡ sin 2 x ⎤ π
∫ sin 2 xdx = 2 ∫ sin 2 xdx = 2 ∫ 2
dx = ⎢x − 2 ⎥
⎣ ⎦0
=
2
−π /2 0 0

(iii) Since sin7 (– x) = – sin7 x, therefore sin7 x is an odd function and we


have
π /2

∫ sin 7 xdx = 0
−π /2

Example 4.30 Evaluate the following integrals


π /2 π /2
sin x
(i) ∫ sin x + cos x
dx (ii) ∫ logtan xdx
0 0

π π /4
x sin xdx
(iii) ∫1 + cos2 x (iv) ∫ log(1 + tan x) dx
0 0

π π /2
(v) ∫ xsin xdx (vi) ∫ logsin xdx
3

0 0

Solution (i) Let


π /2
sin x
I = ∫ sin x + cos x
dx (23)
0
Then
π /2 π /2
sin(π /2 − x) cos x
I= ∫ sin (π /2 − x) + cos(π / 2 − x)
dx = ∫ cos x + sin x
dx (24)
0 0
Integration 175

Adding (23) and (24), we get


π /2 π /2
sin x cos x
2I = ∫ sin x + cos x
dx + ∫ cos x + sin x
dx
0 0

π /2 π /2
sin x + cos x π
= ∫ sin x + cos x
dx = ∫ dx =
2
0 0

Therefore, the given integral is


π /2
sin x π
I= ∫ sin x + cos x
dx =
4
0
(ii) Here
π /2 π /2
⎛ sin x ⎞
∫ log tan xdx = ∫ log ⎜ ⎟ dx
⎝ cos x ⎠
0 0

π /2 π /2
= ∫ log sin x dx − ∫ log cos xdx
0 0

π /2 π /2
= ∫ logsin xdx − ∫ logcos(π /2 − x) dx
0 0

π /2 π /2
=
∫ logsin xdx − ∫ logsin xdx = 0
0 0
(iii) Let
π
x sin xdx
I = ∫ 1 + cos2 x (25)
0
Then
π π
(π − x)sin (π − x) dx (π − x ) sin xdx
I= ∫ 1 + cos (π − x)
2
=∫
1 + cos2 x
(26)
0 0
Adding Eqs. (25) and (26), we get
π
π sin xdx
2I = ∫1 + cos2 x
0
176 Mathematical Methods

Now, put t = cos x and dt = – sin x dx so that the above integral reduces
to
−1 1
π dt dt 1 π2
2I = – ∫ 1 + t2 = π∫ = π ⎡⎣tan −1 t ⎤⎦ =
−11 + t2 −1 2
1

Therefore
π
x sin xdx π2
I= ∫1 + cos2 x =
4
0
(iv) Let
π /4
I= ∫ log(1 + tan x) dx
0
Then
π /4
⎡ ⎛π ⎞⎤
I = ∫ log ⎢⎣1 + tan ⎜⎝ 4 − x ⎟⎠⎥⎦ dx
0

π /4
⎛ 1 − tan x ⎞
= ∫ log ⎜⎝1 + 1 + tan x ⎟⎠ dx
0

π /4
⎛ 2 ⎞
= ∫ log ⎜⎝ 1 + tan x ⎟⎠ dx
0

π /4 π /4
= ∫ log2 dx − ∫ log(1 + tan x) dx
0 0

π
= log 2 – I
4
Therefore
π /4
π
I = ∫ log(1 + tan x) dx = 8 log 2
0
(v) Here
π

∫ x sin
3
I = xdx
0
Integration 177

∫ (π − x)sin (π − x) dx
3
=
0
π π
= ∫ (π − x)sin xdx = π∫sin xdx − I
3 3

0 0
Therefore
π π
3sin x − sin3x π⎡ cos3x ⎤
2I = π ∫ dx = ⎢−3cos x +
0
4 4⎣ 3 ⎥⎦ 0
which leads to
π

∫ xsin xdx =
3
I =
0
3
(vi) Let
π /2
I = ∫ logsin xdx (27)
0
Then
π /2 π /2
I= ∫ logsin (π /2 − x ) dx = ∫ logcos xdx (28)
0 0
Adding Eqs. (27) and (28), we get
π /2 π /2 π /2
2I = ∫ logsin xdx + ∫ logcos xdx = ∫ log (sin x cos x) dx
0 0 0

π /2 π /2 π /2
⎛ sin 2 x ⎞
= ∫ log ⎜
⎝ 2 ⎠
⎟ dx = ∫ logsin 2 xdx − ∫ log 2dx
0 0 0
Substituting 2x = y in the first integral on the right side of the above
equation, we get
π π /2
1
2 ∫0 ∫ log 2 dx
2I = log sin ydy −
0
π /2
⎛1⎞ π
= ⎜ ⎟ (2)
⎝2⎠
∫ logsin y dy − 2 log2
0
π /2
π π
= ∫ logsin xdx − 2 log 2 = I − 2 log 2
0
178 Mathematical Methods

Therefore
π /2
π
I = ∫ logsin x dx = −
2
log 2
0

4.7 Areas of Bounded Regions


Now, we shall use definite integrals to compute the areas bounded by
some curves. First we give the formulas for calculating the areas of certain
regions and then illustrate them with the help of examples.
(i) Area bounded by the curve y = f (x), x-axis and the ordinates x = a,
x = b is given by (Fig. 4.2)
b

A = ∫ f ( x) dx (29)
a
y

y = f (x )

x
O x=a x=b

Fig. 4.2 Area given by Eq. (29)


(ii) Area bounded by the curve x = f ( y), y-axis and the abscissa y = a, y
= b is given by (Fig. 4.3)
b
A = ∫ f ( y) dy (30)
a
y

y=b

x = f ( y)
y=a

x
O

Fig. 4.3 Area given by Eq. (30)


Integration 179

(iii) Area enclosed between the curves y1 = f1(x) and y2 = f2(x), and the
ordinates x = a, x = b is given by (Fig. 4.4)
b
A = ∫[ f1 ( x) − f2 ( x)]dx (31)
a

where y2 > y1.

y1 = f1(x) y2 = f2(x)

x
O x=a x=b

Fig. 4.4 Area enclosed between two curves


Example 4.31 Find the area bounded by the curve x4 = a2(x 2 – y 2), the
x-axis and the ordinates x = 0 and x = a.

x 2
Solution Solve the equation of the curve for y to get y = a − x2 .
a
The required area A is
a a
x 2
A= ∫ ydx = ∫ a
a − x 2 dx
0 0

Taking x = a sin θ, this integral after simplification leads to

a2
A= sq. units
3
Example 4.32 Find the area bounded by the curve y = 4x 2, the y-axis
and the abscissas y = 1 and y = 4.

Solution The required area is given by


4 4
y 7
∫ xdy = ∫ 4
dy =
3
1 1
180 Mathematical Methods

Example 4.33 Find the area enclosed between the curves y 2 = 4ax and
x 2 = 4ay.
Solution Solving the equations of the curves y 2 = 4ax and x 2 = 4ay, we
get x = 0 and x = 4a and thus y = 0 and y = 4a. Hence the required area
A, enclosed between the curves y1 = 4ax and y2 = x 2/4a, is
4a 4a
⎛ x2 ⎞ 16a 2
A= ∫ ( y1 − y2 ) dx = ∫⎜
⎜ 4 ax − ⎟
4a ⎟⎠
dx =
3
0 0⎝

EXERCISES
1. Evaluate the following integrals
⎛ 6 x ⎞
(i) ∫ ⎜⎝ 1 + x 2 + 10 − csc 2 x ⎟ dx

[Ans. 6tan–1 x + 10xlog10 e + cot x + c]

sin 2 x − cos 2 x
(ii) ∫ sin 2 x cos 2 x
dx [Ans. tan x + cot x + c]

dx
(iii) ∫ ax + b − ax + c
2
[Ans. [(ax + b)3/2 + (ax + c)3/2] + d]
3a (b − c)

(iv) ∫ (tan x + cot x) [Ans. tan x – cot x + c]


2
dx

dx x
(v) [Ans. tan + c]
∫ 1 + cos x 2

x x
(vi) ∫ 1 + sin x dx [Ans. 2(sin
2
− cos ) + c]
2
sin x
(vii) ∫ 1 + sin x dx [Ans. sec x – tan x + x + c]

2. Evaluate
2x − 1
(i) ∫ x − x −1
2
dx [Ans. 2 x 2 − x − 1 + c]
Integration 181

sin x
(ii) ∫ 1 + cos 2 x dx [Ans. – tan – 1 (cos x) + c]

dx ⎡ 1 ⎤
(iii) ∫ x(2 + log x)2 ⎢Ans. − + c⎥
⎣ 2 + log x ⎦

e x (1 + x)
(iv) ∫ cos2 ( xe x ) dx [Ans. tan (xe x) + c]

2
(v) ∫ 2 + sin3x cos 3x dx [Ans. (2 + sin 3x)3/2 + c]
9
sin x ⎡ 1 ⎤
(vi) ∫ (a + b cos x)2 dx ⎢Ans.
⎣ b( a + b cos x)
+ c⎥

sec 4 x ⎡ 1 5/2 ⎤
(vii) ∫ dx ⎢Ans. 2[ tan x + 5 (tan x) ]⎥
tan x ⎣ ⎦

2ax + b
(viii) ∫ dx [Ans. 2 ax 2 + bx + c + d]
ax + bx + c
2

sec2 x
(ix) ∫ 1 + tan x dx [Ans. log(1 + tan x) + c]

e2 x
(x) ∫ 1 + e x dx [Ans. e x – log (1 + e x) +c]

3. Evaluate
dx ⎡ 2 2x + 1 ⎤
(i) ∫ 1 + x + x2 ⎢Ans. tan − 1 + c⎥
⎣ 3 3 ⎦
dx ⎡ 1 −1 ⎛ 3 x − 2 ⎞ ⎤
(ii) ∫ 8 − 12 x + 9 x 2 ⎢Ans. 6 tan ⎜ 2 ⎟ + c⎥
⎣ ⎝ ⎠ ⎦
dx ⎡ 1 ⎛3 + x⎞ ⎤
(iii) ∫ 3 − 2 x − x2 ⎢Ans. 4 log ⎜ 1 − x ⎟ + c⎥
⎣ ⎝ ⎠ ⎦
4. Evaluate
dx ⎡ 1 ⎛ 1 ⎞ ⎤
(i) ∫ ⎢Ans. log ⎜ x − + 3 x 2 − x − 2 ⎟ + c⎥
3x − x − 2
2
⎣ 3 ⎝ 6 ⎠ ⎦
182 Mathematical Methods

dx
(ii) ∫ [Ans. sin – 1 (2x – 5) + c]
5x − 6 − x 2

5. Evaluate
2x
(i) ∫ x 2 + 2 x + 2 dx
[Ans. log(x 2 + 2x + 2) – 2tan– 1 (x + 1) + c]
x+3 ⎡ −1 x + 2 ⎤
(ii) ∫ dx ⎢Ans. − 5 − 4 x − x + sin + c⎥
2

5 − 4x − x 2 ⎣ 3 ⎦
6. Evaluate

(i) ⎡ 1
∫ 2 x 2 + 3 x + 4 dx ⎢Ans.
⎣ 4 2
(4 x + 3) 2 x 2 + 3 x + 4

23 ⎡⎛ 3⎞ ⎤ ⎤
+ log ⎢⎜ x + ⎟ + 2 x + 3 x + 4 ⎥ + c⎥
2
32 ⎣⎝ 4⎠ ⎦ ⎦
⎡ 1
⎢Ans. 24 (8 x + 10 x − 1) x − x + 1
2 2
(ii) ∫ ( x + 1) x 2 − x + 1 dx

9 ⎡ ⎛ 1 ⎞⎤ ⎤
+ log ⎢ x 2 − x + 1 + ⎜ x − ⎟⎥ + c⎥
16 ⎣ ⎝ 2 ⎠⎦ ⎦
7. Evaluate
3x + 1 ⎡ 1 1 ⎛ x −1⎞ ⎤
(i) ∫ ( x − 1)2 ( x + 3) dx ⎢Ans. − x − 1 + 2 log ⎜ x + 3 ⎟ + c⎥
⎣ ⎝ ⎠ ⎦
dx
(ii) ∫ ( x 2 + x) ( x 2 − 1)
⎡ 3 1 1 ⎤
⎢Ans. − log x + 4 log( x + 1) − 2 x + 2 + 4 log( x − 1) + c⎥
⎣ ⎦
2sin2θ − cos θ
(iii) ∫ 6 − cos2 θ − 4sin θ d θ
⎡ 19 5 ⎤
⎢Ans. 6 log(sin θ − 5) + 6 log(sin θ + 1) + c⎥
⎣ ⎦
Integration 183

x −1 ⎡ 1 ⎤
(iv) ∫ ( x + 1) ( x 2 + 1) dx ⎢Ans. − log( x + 1) + 2 log ( x + 1) + c⎥
2
⎣ ⎦

x2
(v) ∫ ( x − 1)2 ( x 2 + 1) dx
⎡ 1 1 1 ⎤
⎢Ans. 2 log ( x − 1) − 2 x − 2 − 4 log ( x + 1) + c⎥
2
⎣ ⎦
8. Integrate, with respect to x, the following functions by parts
(i) x sec2 x [Ans. x tan x + log cos x + c]
1 2
(ii) x 3 cos x 2 [Ans. (x sin x 2 + cos x 2) + c]
2
1 2 1 1
(iii) x 2 sin 2x [Ans. – x cos 2x + x sin 2x + cos 2x + c]
2 2 4
1 1
(iv) x log (1 + x) [Ans. [(x 2 – 1) log (1 + x) – x 2 + x] + c )]
2 2
1 2 1 2 1
(v) x 2 tan – 1 x [Ans. x tan –1 x – x + log (x 2 + 1) + c]
3 6 6
(vi) log (1 + x 2) [Ans. x log (1 + x 2) – 2x + 2tan –1 x + c]

(vii) cos –1 (1/x) [Ans. x sec –1 x – log [ x 2 − 1 + x] + c]

(viii) (sin – 1 x)2 [Ans. x(sin – 1 x)2 + 2 1 − x 2 sin – 1 x – 2x + c]

sin −1 x ⎡ x sin −1 x 1 ⎤
(ix) ⎢Ans. + log(1 − x 2 ) + c⎥
(1 − x 2 )3/2 ⎢⎣ 1− x 2 2 ⎥⎦

x 2 tan −1 x 1 (tan – 1 x)2 + c]


(x) [Ans. x tan – 1 x – log 1 + x2 −
1+ x 2
2

⎛ x ⎞ ⎡ −1 ⎛ x ⎞

(xi) sin −1 ⎜ ⎟ ⎢Ans. (a + x) tan ⎜⎜ ⎟⎟ − ax + c⎥
⎜ a+ x ⎟⎠
⎝ ⎢⎣ ⎝ a⎠ ⎥⎦

xe x ⎡ ex ⎤
(xii) ⎢Ans. − + c⎥
( x + 1) 2 ⎣ x +1 ⎦
184 Mathematical Methods

9. Prove the reduction formulas (iii), (vi), (viii) and (ix) given after
Example 4.22.
10. Evaluate the following definite integrals as the limit of a sum
3
(i) ∫ x dx [Ans. 65/4]
3

2
x
(ii) ∫e dx [Ans. e 2 – 1]
0

11. Evaluate the following integrals


π /2
(i) ∫ cos [Ans. π/4]
2
xdx
0

π
(ii) ∫ cos2 x logsin x dx [Ans. – π/2]
0

π /3 ⎡
cos xdx 1 ⎛ 3 + 2 3 ⎞⎤
(iii) ∫ 3 + 4sin x
⎢Ans. log ⎜⎜ ⎟⎥
3 ⎟⎠⎥⎦
0 ⎣⎢ 4 ⎝
π /2
sin θ d θ ⎡Ans. 2 2 − 2⎤
(iv) ∫ 1 + cos θ
⎣ ⎦
0


x(tan −1 x)2 dx
(v) ∫ (1 + x 2 )3/2
[Ans. π – 2]
0

sin −1 xdx
1/2
⎡ π 2 ⎤
(vi) ∫ (1 − x2 )3/2 ⎢Ans.
⎣ 6 3
− log ⎥
3⎦
0

π /2
dx ⎡ 2 −1 1 ⎤
(vii) ∫ 5 + 4sin x ⎢Ans. 3 tan 3⎥
⎣ ⎦
0

π /2
12. Evaluate ∫ sin | x | dx. [Ans. 2]
−π /2

∫x
4
13. Show that sin xdx = 0.
−π
Integration 185

π
x tan xdx π2
14. Show that ∫ sec x + cos x 4 .
=
0

1
log(1 + x) π
15. Show that ∫ 1 + x2
dx =
8
log 2.
0


16. Show that ⎛ 1 ⎞ dx = π log 2 (Hint: Put x = tan θ)
∫ log ⎜⎝ x + x ⎟⎠ 1 + x2
0

π /2
π
17. Prove that ∫ log cos x dx = – 2
log 2.
0
18. Find the area of the parabola y 2 = 4ax bounded by the latus rectum.
[Ans. 8a2/3]
19. Find the area enclosed between the coordinate axes and the curve

y 2 = 4a(x + λ) in the second quadrant.


⎡ 4 ⎤
⎢Ans. 3 aλ λ ⎥
⎣ ⎦
x2 y2
20. Find the area of the region bounded by the ellipse + = 1.
a2 b2
[Ans. π ab]
21. Compute the area of the region bounded by the two parabolas y = x 2
and x = y 2. [Ans. 1/3]
22. Find the area bounded by the parabola x 2 = 4ay + 4a 2 and the line
3x + 4y = 0. [Ans. 125 a2/24]
23. Determine the area bounded by the curve x 2 = 4y and the straight
line x + 2 = 4y. [Ans. 9/8]
24. Calculate the area bounded by the straight lines x + 2y – 2 = 0,
y – x – 1 = 0 and 2x + y – 7 = 0. [Ans. 6 sq. units]
25. Find the area of the region bounded by the curves y = sin x and
y = cos x between x = 0 and x = π/4. [Ans. 2 – 1 sq. units]
26. Calculate the area bounded by the curves y = x and y = x 3.
[Ans. 1/2 sq. units]
27. Find the area of the region {(x, y) : 0 ≤ y ≤ x 2 + 1, 0 ≤ y ≤ x + 1,
0 ≤ x ≤ 2}. [Ans. 23/6]
186 Mathematical Methods

CHAPTER 5

Partial Differentiation

5.1 Introduction
We know that the volume V of a right circular cylinder of radius r and
height h is V = πr 2h. For a given pair of values of r and h, one can find
the specific value of V. Here, the volume V depends upon two quantities,
namely r and h. Similarly, the area A of a triangle PQR is given by
A= 1
2
pq sin R and its value depends upon three variables p, q and R.
Thus, we noticed that volume V is a function of two variables, while the
area A is a function of three variables. In a similar way, the height h
(above sea level) on a point on the surface of the earth depends upon the
latitude φ and longitude ψ, i.e., for a pair of values of φ and ψ, we can
find h. In business, the equations governing the simple and compound
interests S and C, respectively, are S(P, r, t) = P(1 + rt) and C(P, r, t, n) =
P (1 + nr ) nt , where P denotes the principal amount, r the annual rate, t
the time in years and n the compound periods in years. Here, the simple
interest S is a function of three variables; while the compound interest C
is a function of four variables. In social sciences, the intelligence quotient
(IQ) level Q of a person is described through the equation Q(m, c) =
100 m/c, where m and c are mental and chronological ages of a person.
It may be noted here that Q is a function of two variables m and c.
Moreover, in the field of medicine, the resistance R for blood flow in a
vessel is governed by an equation of the form R(L, r) = kL/r4, where L is
the length of the vessel, r being the radius of the vessel and k is a constant.
Here, the resistance R depends upon two quantities L and r and thus is a
function of two variables.
In an analogous way, we may have situations where we have to deal
with a function of more than three variables. The differentiation of a
function of more than one variable leads to the concept of partial
differentiation. In this chapter, we shall deal with partial differentiation
and the related results.
Partial Differentiation 187

Definition 5.1 A variable z is said to be a function of two variables x and


y if for each given pair (x, y) we are able to find one or more values of z.
Such a function is denoted by z = f (x, y), or, z = F (x, y), or, z = z(x, y),
etc. and will mean to denote the value of the function at (x, y). Here, z is
the dependent variable and x and y are the independent variables. The
function z = f (x, y) is called single-valued if any one value of z corresponds
to each pair (x, y) for which the function is defined. If there is more than
one value of z, the function is said to be multiple-valued and can be
considered as a collection of single-valued functions. The set of values
(x, y) for which the function is defined is known as the domain of the
function.
The concept of a function of two variables can easily be extended.
Thus F = F(x1, x2, x3, ..., x n) is a function of n variables x1, x2, x3, ..., xn.

5.2 Partial Derivative


In Chapter 3 we discussed a number of methods for finding the derivatives
of different types functions of one independent variable. This section
shall be concerned with the derivative of a function of more than one
variable and we have

Definition 5.2 The ordinary derivative of a function of several variables


with respect to one of the independent variables, when all other
independent variables are regarded as constant, is called the partial
derivative of the function with respect to the variable.
Thus the partial derivative of f (x, y) with respect to x is the ordinary
derivative of f (x, y) when y is kept as a constant. It is denoted by any one
of the following symbols
∂f
, f , Dx f, fx (x, y)
∂x x
Similarly, the partial derivative of f (x, y) with respect to y is the
ordinary derivative of f (x, y) when x is kept as a constant. It is denoted
by any one of the following symbols
∂f
, f , D f, fy (x, y)
∂y y y
Therefore, from the definition, we have
∂f f ( x + ∆x , y ) − f ( x , y )
= lim (1)
∂x ∆x → 0 ∆x
188 Mathematical Methods

and
∂f f ( x , y + ∆y ) − f ( x , y )
= lim (2)
∂y ∆y →0 ∆y
provided that the limits in Eqs. (1) and (2) exist.
If the partial derivatives in equations (1) and (2) are evaluated at a
point (x1, y1), then they are, respectively, denoted as
⎡ ∂f ⎤ ⎡ ∂f ⎤
⎢ ∂x ⎥ and ⎢ ⎥
⎣ ⎦ ( x1 , y1 ) ⎣ ∂y ⎦ ( x1 , y1 )

5.3 Higher Order Partial Derivatives


As in the case of ordinary derivative we have higher order derivatives,
similar is the case with the partial derivatives and we have

Definition 5.3 If f (x, y) has partial derivatives at each point (x, y), then
∂ f /∂ x and ∂ f /∂ y are themselves functions of x and y which may also
have partial derivatives. These are known as second derivatives and are
denoted as

∂ ⎛ ∂f ⎞ ∂2 f ∂ ⎛ ∂f ⎞ ∂ 2 f
⎜ ⎟ = = f , ⎜ ⎟=
∂y ⎝ ∂y ⎠ ∂y 2 = fyy (3a)
xx
∂x ⎝ ∂x ⎠ ∂x 2

∂ ⎛ ∂f ⎞ ∂ 2 f ∂ ⎛ ∂f ⎞ ∂ 2 f
⎜ ⎟ = = f yx , ⎜ ⎟= = fxy (3b)
∂x ⎝ ∂y ⎠ ∂x ∂y ∂y ⎝ ∂x ⎠ ∂y ∂x
Similarly, we can have other higher order derivatives. For example, fyxx
denotes the partial derivative of f once with respect to y and twice with
respect to x.
It may be noted that if fxy and fyx are continuous, then fxy = fyx, and
the order of partial differentiation is immaterial; otherwise they may not
be equal.

2
Example 5.1 If f (x, y) = x 3 y + e x y , then compute fx , fy , fxx , fyy , fxy ,
and fyx.

Solution Here
∂f ∂ 2 2
fx = = ( x3 y + e xy ) = 3 x 2 y + y 2 e xy
∂x ∂x
Partial Differentiation 189

∂f ∂ 3 2 2
fy = = ( x y + e xy ) = x3 + 2 xye xy
∂y ∂y

∂2 f ∂ ⎛ ∂f ⎞ ∂ 2 2
fx x = = ⎜ ⎟= (3 x 2 y + y 2 e xy ) = 6 xy + y 4 e xy
∂x 2
∂x ⎝ ∂x ⎠ ∂x
∂2 f ∂ ⎛ ∂f ⎞ ∂ 3 2 2
fyy = = ⎜ ⎟= ( x + 2 xye xy ) = (4 x 2 y 2 + 2 x) e xy
∂y 2 ∂y ⎝ ∂y ⎠ ∂y

∂2 f ∂ ⎛ ∂f ⎞ ∂ 2 xy 2 xy 2
fx y = = ⎜ ⎟ = (3x y + y e ) = 3x + (2 xy + 2 y )e
2 2 3
∂y ∂x ∂y ⎝ ∂x ⎠ ∂y

∂2 f ∂ ⎛ ∂f ⎞ ∂ 2 2
fyx = = ⎜ ⎟ = ( x3 + 2 xye xy ) = 3 x 2 + (2 xy 3 + 2 y ) e xy
∂x ∂y ∂x ⎝ ∂y ⎠ ∂x
It may be noted that in this example, fx y = fy x.

Example 5.2 If V (x, y, z) = (x 2 + y 2 + z 2) – 1/2, show that


∂ 2V ∂ 2V ∂ 2V
+ + =0
∂x 2 ∂y 2 ∂z 2

Solution Assume that (x, y, z) ≠ (0, 0, 0). Then


∂V
= – x(x 2 + y 2 + z 2)– 3/2
∂x
∂ 2V ∂ 2 x2 − y2 − z 2
= 2 2 2 – 3/2
[– x(x + y + z ) ] = 2
∂x 2 ∂x ( x + y 2 + z 2 )5/2
In a similar manner, we have
∂ 2V 2 y2 − x2 − z 2
=
∂y 2
( x 2 + y 2 + z 2 )5/2
∂ 2V 2 z 2 − x2 − y2
=
∂z 2
( x 2 + y 2 + z 2 )5/2
which on addition leads to
∂ 2V ∂ 2V ∂ 2V
+ + =0
∂x 2
∂y 2
∂z 2

Example 5.3 If f = x 2 tan – 1 ( y/x), find ∂2 f /∂x∂ y at the point (1, 1).
190 Mathematical Methods

Solution Here
∂f ∂ x3
= [ x 2 tan −1 ( y / x)] = 2
∂y ∂y x + y2
which when differentiated partially with respect to x yield

∂2 f ∂ ⎛ ∂f ⎞ ∂ ⎛ x3 ⎞ ( x 2 + y 2 ) (3x 2 ) − ( x3 )(2 x)
= ⎜ ⎟= ⎜ 2 ⎟=
∂x ∂y ∂x ⎝ ∂y ⎠ ∂x ⎜⎝ x + y 2 ⎟⎠ ( x 2 + y 2 )2
Therefore
⎡ ∂2 f ⎤
⎢ ⎥ =1
⎣ ∂x ∂y ⎦ (1,1)

5.4 Total Differential


If y = f (x) is a differentiable function, then the differential dx of the
independent variable x is an arbitrary real number, and the differential
dy of the dependent variable y is defined as the product of f ′(x) and dx,
i.e.,
d y = f ′(x) dx
The differential dy is in fact a function of two independent variables x
and dx. A change in either one or both will be responsible for a change in
the other. It is possible to extend this concept of differential for a function
of several variables and we have

Definition 5.4 If z = f (x, y) where x = g(t) and y = h(t), i.e., x and y both
are functions of a single variable t, then
dz ∂z dx ∂z dy
= + (4)
dt ∂x dt ∂y dt
is known as the total derivative of z with respect to t. This equation can
also be written as
∂z ∂z
dz = dx + dy (5)
∂x ∂y
and in this case, dz is called the total differential of z.
In a similar way, if z = f (x1, x2, x3, ..., xn) where x1, x2, x3, ..., xn are
all functions of t, then

dz ∂z dx1 ∂z dx2 ∂z dx3 ∂z dxn


= + + + ... + (6)
dt ∂x1 dt ∂x2 dt ∂x3 dt ∂xn dt
Partial Differentiation 191

Remarks
1. When f (x, y) is a function of x and y, and y is a function of x, then the
total derivative of f with respect to x is
df ∂f ∂f dy
= + (7)
dx ∂x ∂ y dx
Now, if we have an implicit relation between x and y of the form f (x, y)
= c, where c is a constant and y is a function of x, then Eq. (7) reduces to
∂f ∂f dy
0=+
∂ x ∂ y dx
which gives the following important relation between the ordinary and
partial derivatives of f
dy ∂f ∂f
= – / (8)
dx ∂x ∂y
Also, if f is a function of the n variables x1, x2, ..., xn and x2, x3, ..., xn
are all functions of x1, then
df ∂f ∂ f d x2 ∂ f d x3 ∂ f d xn
= + + + ... + (8a)
d x1 ∂ x1 ∂ x2 d x1 ∂ x3 d x1 ∂ xn d x1
2. For the sake of convenience, we have the following notations
∂f ∂f ∂2 f ∂2 f ∂2 f
= p, = q, 2 = r , = s, 2 = t (9)
∂x ∂y ∂x ∂ x∂ y ∂y
With these notations, Eq. (8) takes the form
dy p
=−
dx q
which on differentiation with respect to x leads to

d2y q dp
dx
− p dq
dx
= – (10)
dx 2 q2
But from Eq. (7), we have
dp ∂ p ∂p dy ⎛ p ⎞ qr − ps
= + = r + s⎜ − ⎟ =
dx ∂x ∂y dx ⎝ q⎠ q
and
dq ∂q ∂q dy ⎛ p ⎞ qs − pt
= + = s + t⎜− ⎟ =
dx ∂x ∂y dx ⎝ q⎠ q
192 Mathematical Methods

With these values, Eq. (10) becomes

d2y q 2 r − 2 pqs + p 2 t
= – (11)
dx 2 q3
which gives second order ordinary derivative in terms of partial
derivatives.

Example 5.4 If f = x 2 e y/x, find df.

Solution Here
∂f ⎛ y⎞ ∂f ⎛1⎞
= x 2 e y / x ⎜ − 2 ⎟ + 2 xe y / x , = x2e y / x ⎜ ⎟
∂x ⎝ x ⎠ ∂y ⎝ x⎠
Thus
∂f ∂f
df= dx + dy = (2xe y/x – ye y/x) dx + xe y/x dy
∂x ∂y

2
xy
Example 5.5 If f = e when x = t cos t, y = t sin t, find d f /dt at
t = π/2.

Solution From Eq. (6), we have


d f ∂ f dx ∂ f dy 2 2
= + = ( y 2 e xy ) (−t sin t + cos t ) + (2 xye xy ) (t cos t + sin t)
dt ∂x dt ∂y dt
so that at t = π/2 we have d f /dt = – π2/8.

Example 5.6 If f = x 2 – y 2 + sin yz where y = e x and z = log x, compute


d f /dx.

Solution From Eq. (8a), we have


df ∂ f ∂ f dy ∂ f dz y cos yz
= + + = 2x + (– 2y + z cos yz) e x +
dx ∂x ∂y dx ∂z dx x

Example 5.7 If x 3 + 3x 2 y + 6xy 2 + y 3 = 1, find dy/dx.

Solution Let f = x 3 + 3x 2 y + 6xy 2 + y 3, then


∂f ∂f
= 3x 2 + 6xy + 6y 2, = 3x 2 + 12xy + 3y 2
∂x ∂y
Partial Differentiation 193

Thus, from Eq. (8), we have


dy x 2 + 2 xy + 2 y 2
=− 2
dx x + 4 xy + y 2
Example 5.8 If f (x, y) = ax 2 + 2hxy + by 2, find d 2y/dx 2.

Solution Here
p = 2(ax + hy), q = 2(hx + by), r = 2a, s = 2h, t = 2b
Therefore, from Eq. (11), we have
d2 y (hx + by )2 a − 2(ax + hy ) (hx + by ) h + (ax + hy )2 b
=−
dx 2 (hx + by )3

5.5 Change of Variables


If
z = f (x, y) (12)
where x = g(t1, t2) and y = h(t1, t2) then often it is required to change
expressions involving z, x, y, ∂z/∂x, ∂z/∂y, ∂ 2z/∂x 2, etc. into expressions
involving z = t1, t2, ∂z/∂t1, ∂z/∂t2, etc. This can be done as follows:
If t1 is considered as a constant, then from Eq. (4) (replacing the
ordinary derivatives by the partial derivatives as x and y are the functions
of the two variables t1 and t2 instead of only one variable t), we have

∂z ∂z ∂x ∂z ∂y
= + (13)
∂t1 ∂x ∂t1 ∂y ∂t1
Similarly, when t2 is taken as a constant, we have

∂z ∂z ∂x ∂z ∂y
= + (14)
∂t2 ∂x ∂t2 ∂y ∂t2
Equations (13) and (14) may be considered as a pair of simultaneous
linear equations and can be solved for ∂z/∂x and ∂z/∂y in terms of
∂z/∂t1, ∂z/∂t2 and the known quantities ∂x/∂t1, ∂y/∂t1, ∂x/∂t2 and ∂y/∂t2.
If we put these values of ∂z/∂x, ∂z/∂y and the values of x and y, as given
by Eq. (12) in any expression involving z, x, y, ∂z/∂x, ∂z/∂y we shall then
get the required transformation.
Now, if Eq. (12) is easily solvable for t1 and t2 in terms of x and y,
and
t1 = F (x, y), t2 = G(x, y) (15)
194 Mathematical Methods

then it is easy to use the formulas


∂z ∂z ∂t1 ∂z ∂t2
= + (16a)
∂x ∂t1 ∂x ∂t2 ∂x

∂z ∂z ∂t1 ∂z ∂t2
= + (16b)
∂y ∂t1 ∂y ∂t2 ∂y
Here the values of ∂t1/∂x, are to be substituted after finding them from
Eq. (15).
It may be noted that the higher order derivatives of z can be obtained
by a repeated application of Eqs. (13) and (14) (or, Eq. (16)). The above
formulas can be extended when more than two independent variables
are involved.
One of the familiar examples of the change of variables is the change
from the cartesian coordinates (x, y) to the polar coordinates (r, θ), where
x = r cos θ and y = r sin θ.

Example 5.9 Transform

∂2 z ∂2 z
+ =0
∂x 2 ∂y 2
into polar coordinates.

Solution It is known that a change from cartesian to polar coordinates is


given by

x = r cos θ, y = r sin θ, r = –1
x 2 + y 2 , θ = tan ( y/x) (17)
which on partial differentiation leads to

∂r x ∂r y
= = cos θ, = = sin θ
∂x x +y
2 2 ∂y x + y2
2

∂θ y sin θ ∂θ x cos θ
= − 2 =− , = 2 =
∂x x +y 2
r ∂y x + y 2
r
Therefore, from Eq. (16), we have
∂z ∂z ∂z ⎛ sin θ ⎞
= cos θ + ⎜− ⎟
∂x ∂r ∂θ ⎝ r ⎠
Partial Differentiation 195

which may be expressed as


∂z ⎛ ∂ sin θ ∂ ⎞
= ⎜ cos θ − ⎟z
∂x ⎝ ∂r r ∂θ ⎠
Thus, we have
∂2 z ∂ ⎛ ∂z ⎞ ⎛ ∂ sin θ ∂ ⎞ ⎛ ∂z sin θ ∂z ⎞
= ⎜ ⎟ = ⎜ cos θ − ⎟ ⎜ cos θ − ⎟
∂x 2
∂x ⎝ ∂x ⎠ ⎝ ∂r r ∂θ ⎠ ⎝ ∂r r ∂θ ⎠
which after simplification reduces to
∂2 z ∂2 z 2sin θ cos θ ∂ 2 z sin 2 θ ∂ 2 z
= cos 2 θ − + 2
∂x 2 ∂r 2 r ∂r ∂θ r ∂θ2

sin 2 θ ∂z 2sin θ cos θ ∂z


+ + (18)
r ∂r r2 ∂θ
In a similar manner, we have
∂2 z ∂2 z 2sin θ cos θ ∂ 2 z cos 2 θ ∂ 2 z
= sin 2 θ + +
∂y 2 ∂r 2 r ∂r ∂θ r 2 ∂θ2

cos 2 θ ∂z 2sin θ cos θ ∂z


+ − (19)
r ∂r r2 ∂θ
Adding Eqs. (18) and (19), we get
∂2 z ∂2 z ∂2 z 1 ∂z 1 ∂ 2 z
+ = + +
∂x 2 ∂y 2 ∂r 2 r ∂r r 2 ∂θ2
Therefore the transformed equation is given by
∂2 z 1 ∂z 1 ∂ 2 z
+ + =0
∂r 2 r ∂r r 2 ∂θ2

5.6 Euler’s Theorem on Homogeneous Functions


If in the expression
a0 x n + a1 x n – 1 y + a2x n– 2 y 2 + ... + an – 1 xy n– 1 + an y n
each term is of degree n, then such a function is known as a homogeneous
function of degree n. This expression can also be written as
n
⎪⎧ ⎛ y ⎞ ⎪⎫
2
⎛ y⎞ ⎛ y⎞
x n ⎨a0 + a1 ⎜ ⎟ + a2 ⎜ ⎟ + ... + an ⎜ ⎟ ⎬
⎩⎪ ⎝ x⎠ ⎝ x⎠ ⎝ x ⎠ ⎭⎪
196 Mathematical Methods

and the function x n f ( y/x) is called a homogeneous function of degree n


whatever the function f may be.
In general, if the function f (x1, x2, x3, ..., xm) of m variables x1, x2, ...,
xm can be written as

⎛x x x ⎞
xrn F ⎜ 1 , 2 , ..., m ⎟
⎝ xr xr xr ⎠
then the function f (x1, x2, x3, ..., xm) is known as a homogeneous function
of x1, x2, ..., xm of degree n. A relation between a homogeneous function
and its partial derivatives is given by the following theorem.

Theorem 5.1 Euler’s Theorem. If f (x1, x2, x3, ..., xm) is a homogeneous
function of x1, x2, ..., xm of degree n, then
∂f ∂f ∂f ∂f
x1 + x2 + x3 + ... + xm = nf
∂x1 ∂x2 ∂x3 ∂xm

Example 5.10 Establish Euler’s theorem for f (x, y, z) = 3x 2yz + 5xy 2z +


4z 4.

Solution The given function f (x, y, z) = 3x 2y z + 5xy 2z + 4z 4 is a


homogeneous function of degree 4. Thus to verify the Euler’s theorem
for this function, we must show that
∂f ∂f ∂f
x +y +z = 4f (20)
∂x ∂y ∂z
Now, differentiate the given function partially with respect to x, y and z
and then multiply the resulting expressions by x, y and z, we get
∂f ∂f
x = 6 x 2 yz + 5 xy 2 z , y = 3x 2yz + 10xy 2z
∂x ∂y

∂f
z = 3x 2yz + 5xy 2z + 16z 4
∂z
so that on adding these, we get
∂f ∂f ∂f
x +y +z = 12x 2yz + 20xy 2z + 16z 4 = 4 f
∂x ∂y ∂z
which is Eq. (20) and thus the Euler’s theorem is verified.
Partial Differentiation 197

EXERCISES
1. Verify fx y = fyx when
(i) f = ax 2 + 2hxy + by 2 (ii) f = x sin y + y sin x
(iii) f = x log y (iv) f = log[(x 2 + y 2)/xy]
(v) f = log tan ( y/x) (vi) f = (ay – bx)/(by – ax)
∂3 f
2. If f = exyz, show that = (1 + 3xyz + x 2y 2z 2) e xyz.
∂x ∂y ∂z
∂2 f
3. If f = tan – 1 ( xy / 1 + x 2 + y 2 ), show that =
∂x ∂y
1
.
(1 + x + y 2 )3/ 2
2

∂2 z
4. If x xy yz z = c, show that at x = y = z, = – (x log ex) –1.
∂x ∂y
∂ 2u ∂ 2u
5. If u = f (x + ay) + g(x – ay), show that = a2 .
∂y 2 ∂x 2
∂z ∂z
6. If z = f ( y/x), show that x +y = 0.
∂x ∂y
7. Find dy/dx if
(i) ax 2 + 2hxy + by 2 = 1. [Ans. – (ax + hy)/(hx + by)]
x y
(ii) y + x = c. [Ans. – ( y log y + yx y–1)/(xy x– 1 + x y log x)]
x

8. Find df /dx if
(i) f = x log xy where x 3 + y 3 + 3xy = 1.
[Ans. 1 + log xy – x(x 2 + y)/y(x + y 2)]
(ii) f = sin (x 2 + y 2) where a 2 x 2 + b 2 y 2 = c 2.
[Ans. 2x[cos (x 2 + y 2)] (1 – a2/b2)]
9. If z = f (r), where r 2 = x 2 + y 2, prove that

∂2 z ∂2 z 1
+ = f ″(r) + f ′(r )
∂x 2
∂y 2 r
10. If u = f ( y – z, z – x, x – y), prove that
∂u ∂u ∂u
+ + =0
∂x ∂y ∂z
198 Mathematical Methods

11. If z = z(u, v), u = x 2 – 2xy – y 2, v = y, show that


∂z ∂z
(x + y) + (x – y) =0
∂x ∂y
is transformed into ∂z/∂v = 0.
12. If u = f (x, y) and x = r cos θ, y = r sin θ, then prove that
2 2 2 2
⎛ ∂u ⎞ ⎛ ∂u ⎞ ⎛ ∂u ⎞ 1 ⎛ ∂u ⎞
⎜ ⎟ +⎜ ⎟ =⎜ ⎟ + 2⎜ ⎟

⎝ ⎠x ∂
⎝ ⎠y ∂
⎝ ⎠r r ⎝ ∂θ ⎠
13. If z = f (x, y) and x = r cos θ, y = r sin θ, then prove that

∂z ∂z 1 ∂z sin θ
= cos θ −
∂x ∂r r ∂θ
14. Verify Euler’s theorem for
(i) f (x, y) = ax 2 + 2hxy + by 2
(ii) f (x, y, z) = axy + byz + czx
(iii) f = x 2(x 2 – y 2)3/(x 2 + y 2)3
15. If f is a homogeneous function of degree n, prove that
∂2 f ∂2 f
x +y = (n – 1) ∂ f
∂x 2 ∂x ∂y ∂x
∂2 f ∂2 f ∂f
x +y 2 = (n – 1)
∂x ∂y ∂y ∂y
∂f ∂f
16. From Euler’s theorem x +y = n f where f is a homogeneous
∂x ∂y
function of x and y of degree n, deduce that
∂2 f ∂2 f ∂2 f
x2 + 2 xy + y 2 2 = n(n – 1) f
∂x 2 ∂x ∂y ∂y
17. If F = sin–1[(x 2 + y 2)/(x + y)], show that
∂F ∂F
x +y = tan F
∂x ∂y
18. If F = tan –1 [(x 3 + y 3)/(x – y)], show that
∂F ∂F
x +y = sin 2F
∂x ∂y
Differential Equations 199

CHAPTER 6

Differential Equations

6.1 Introduction
In many physical problems the relation between the rates of change of
observable quantities is simpler than the relation between the quantities
themselves and due to this reason, differential equations arise naturally
as mathematical models for a number of problems of engineering, physical
sciences, biological and medical sciences as well as social sciences. In
this chapter, we shall be concerned with some basic concepts about
the differential equations and the methods for solving these equations
(For a detailed account of differential equations and their applications,
see [2, 3]).

Definition 6.1 An equation which contains independent and dependent


variables and the derivatives or differentials of one or more dependent
variables with respect to one or more independent variables is known as
a differential equation.

Some of the examples of the differential equations are as follows:


xy″ + y ′ + xy = 0 (1)
d2x
= – kx (2)
dt 2
d 2I dI
+5 + 10I = 50 sin 20t (3)
dt 2 dt
d4y
EI – ω (x) = 0 (4)
dx 4
W
y″ = 1 + y ′2 (5)
H
dv
v+m = v2 (6)
dm
200 Mathematical Methods

∂ 2V ∂ 2V ∂ 2V
+ + = 0 (7)
∂x 2 ∂y 2 ∂z 2

∂V ⎛ ∂ 2V ∂ 2V ⎞
= k⎜ 2 + 2 ⎟ (8)
∂t ⎜ ∂y ⎠⎟
⎝ ∂x

∂ 2V 2 ∂ 2V
= a (9)
∂t 2 ∂x 2
∂4φ ∂4φ ∂4φ
+2 + = F(x, y) (10)
∂x 4 ∂x 2 ∂y 2 ∂y 4
dy
= sin x + cos x (11)
dx
dy k
y = x + (12)
dx dy / dx
3/2
d2y ⎡ 2⎤
k = ⎢1 + ⎛⎜ dy ⎞⎟ ⎥ (13)
dx 2 ⎢⎣ ⎝ dx ⎠ ⎥⎦

dy ⎡ 2⎤
⎛ dy ⎞
y = x + k⎢ 1+ ⎜ ⎟ ⎥ (14)
dx ⎢ ⎝ dx ⎠ ⎥
⎣ ⎦
4
d3x d2x ⎛ dx ⎞
+ +⎜ ⎟ = et (15)
dt 3 dt 2
⎝ dt ⎠
2
∂ 3V ⎛ 2 ⎞
= K⎜∂ V ⎟ (16)
∂t 3 ⎜ 2⎟
⎝ ∂x ⎠
There are two types of differential equations:
(i) Ordinary differential equations: The differential equations which
involve derivatives with respect to a single independent variable.
Eqs. (1) to (6) and (11) to (15) are examples of ordinary differential
equations.
(ii) Partial differential equations: The differential equations that
involve two or more independent variables and partial derivatives
with respect to them. Eqs. (7) to (10) and (16) are examples of
partial differential equations.
Differential Equations 201

Definition 6.2 The order of the highest order derivative involved in a


differential equation is called the order of a differential equation.
Eqs. (6), (11), (12) and (14) are of the first order; Eqs. (1) to (3), (5),
(7) to (9) and (13) are of the second order and Eqs. (15) and (16) are of
third order; while Eqs. (4) and (10) are of order four.

Definition 6.3 The degree of a differential equation is the degree of the


highest order derivative present in the equation, after the equation has
been made free from the radicals and fractions as far as the derivatives
are concerned.
Eqs. (1) to (11), (15) and (16) except equation (5), are of degree one,
Eqs. (5) and (12) to (14) are of degree two.

Definition 6.4 A differential equation in which the dependent variables


and all its derivatives present occur in the first degree only and no products
of dependent variables and/or derivatives occur is known as a linear
differential equation. A differential equation which is not linear is called
nonlinear differential equation.
Equation (11) is a linear equation of order one, Eqs. (2) and (7) are
linear equations of order two; while Eqs. (12) to (16) are nonlinear
equations.

Definition 6.5 A solution of a differential equation is a relation between


the dependent and independent variables, not involving the derivatives
such that this relation and the derivatives obtained from it satisfies the
given differential equation. For example, y = ce2x is a solution of the
differential equation dy/dx – 2y = 0, because dy/dx = 2ce2x and y = ce2x
satisfy the given differential equation.

6.2 Formation of a Differential Equation


A differential equation can be formed by eliminating the arbitrary
constants from a relation in the variables and constants. Suppose we are
given an equation containing n arbitrary constants. Then, by
differentiating it successively n times we get n equations more containing
n arbitrary constants and derivatives. Now, by eliminating n arbitrary
constants from the above (n + 1) equations and obtaining an equation
which involves derivatives upto nth order, we get a differential equation
of order n.
We shall now illustrate this method with the help of the following
examples.
202 Mathematical Methods

Example 6.1 Form a differential equation whose solution is y = c1 cos x


+ sin x.

Solution Given that


y = c1 cos x + sin x (17)
which on differentiation leads to
dy
= – c1 sin x + cos x (18)
dx
From Eq. (17), we have
y − sin x
c1 =
cos x
which when substituted in Eq. (18) yields
dy
cos x + y sin x = 1
dx
which is the required differential equation.

Example 6.2 Form a differential equation whose solution is xy = c1 e x


+ c2e–x + x 2.
Solution Given that
xy = c1e x + c2e – x + x 2 (19)
Differentiate Eq. (19) twice with respect to x to get
dy
x + y = c1e x – c2e – x + 2x (20)
dx
d2y dy
x +2 = c1e x + c2e – x + 2 (21)
dx 2 dx
Now, eliminating c1 and c2 from Eqs. (19) to (21), we get
d2y dy
x +2= xy – x 2 + 2
dx 2 dx
which is the required differential equation.

Example 6.3 Find the differential equation for the curve y = ae 2x


+ be – 2x.

Solution Given that


y = ae 2 x + be – 2x (22)
Differential Equations 203

Differentiate it twice with respect to x to get


dy
= 2ae2x – 2be– 2x
dx
d2y
= 4(ae 2x + be–2x) (23)
dx 2
From Eqs. (22) and (23), we get
d2y
– 4y = 0
dx 2
as the required differential equation.

Remark
In Example 6.1, there is only one arbitrary constant; while in Examples
6.2 and 6.3, the number of arbitrary constants are two, and the resulting
differential equations in these examples are, respectively, of order one
and two. Thus we see that the number of arbitrary constants in a solution
of a differential equation depends upon the order of the differential
equation and is the same as its order. Hence, a differential of order n will
contain n arbitrary constants in its solution.

Definition 6.6 A solution which contains a number of constants equal to


the order of the differential equation is called the general solution or
complete solution of the differential equation. A solution obtained from
a general solution by giving particular values to the arbitrary constants
is called the particular solution. For example, y = ae 2 x + be – 2x is a general
solution and if we take a = 3 and b = 0, then y = 3e2x is a particular
solution.
A solution which can not be derived from the general solution but
still is a solution of the given differential equation is called the singular
solution.

6.3 Differential Equations of First Order and First Degree


An ordinary differential equation of the first order and first degree is of
the form
dy
+ f (x, y) = 0 (24)
dx
which is sometimes also written as
M dx + N dy = 0 (25)
204 Mathematical Methods

where M and N are functions of x and y, or constants. We can not solve


Eq. (25) in every case; however, the solution exists if these equations
belong to any one of the standard forms discussed here.
6.3.1 Equations in which Variables are Separable
If the differential equation (25) can be expressed in the form f1(x) dx +
f2( y) dy = 0, we say that the variables are separable and the solution is
obtained by integrating each term of this equation. Thus the solution is

∫ f1 ( x) dx + ∫ f 2 ( y) dy =c
Example 6.4 Solve the following differential equations
dy 12
(i) = x5 + x2 –
dx x
dy
(ii) (e x + e – x) = ex – e–x
dx
(iii) (x 2 – yx 2) dy + ( y 2 + x 2y 2) dx = 0
(iv) sec2 x tan ydx + sec2 y tan x dy = 0

1 − y2
(v) dy + dx = 0
1 − x2
(vi) 3e x tan ydx + (1 – e x) sec2 y dy = 0
Solution (i) The given differential equation can be written as
⎡ 5 12 ⎤
dy = ⎢ x + x − ⎥ dx
2
⎣ x⎦
which on integration gives the solution as
x6 x3
y= + – 12 log x + c
6 3
dy
(ii) The differential equation (e x + e – x) = e x – e – x can be written as
dx
⎛ e x − e− x ⎞
dy = ⎜⎜ x −x ⎟⎟ dx
⎝e + e ⎠
Integrating both sides, we get
y = log(e x + e –x) + c
as the required solution.
Differential Equations 205

(iii) Separating the variables, we get


⎛1 − y ⎞ ⎛1 + x2 ⎞
⎜⎜ 2 ⎟⎟ dy + ⎜⎜ 2 ⎟⎟ dx = 0
⎝ y ⎠ ⎝ x ⎠
Integrating each term, we get
1 1
x− = + log y + c
x y
as the required solution.
(iv) The given equation sec 2 x tan y dx + sec2 y tan x dy = 0 can be
written as
⎛ sec 2 x ⎞ ⎛ sec2 y ⎞
⎜⎜ ⎟⎟ dx = −⎜⎜ ⎟⎟ dy
⎝ tan x ⎠ ⎝ tan y ⎠
and thus the variables are separated. Now, integrating both sides, we get
log tan x = – log tan y + log c
which using the properties of logarithms (cf., Appendix C) may be written
as
tan x tan y = c
which is the required solution.
(v) Separating the variables, we get
dy dx
=−
1− y 2
1 − x2
which on integrating yields
sin – 1 x + sin – 1 y = c
as the required general solution.
(vi) The given differential equation can be written as

−3e x dx sec2 ydy


=
1 − ex tan y
Integrating both sides, we get
3 log (1 – e x) = log tan y + log c
which on using the properties of logarithms can be expressed as
(1 – e x)3 = c tan y
which is the required general solution of the given differential equation.
206 Mathematical Methods

6.3.2 Homogeneous Differential Equations


A differential equation M dx + N dy = 0 is said to be homogeneous if it
can be expressed as
dy f ( x, y )
= (26)
dx g ( x, y )
where f (x, y) and g(x, y) are homogeneous functions of the same degree
(say n). Taking x n common both from the numerator and denominator
of Eq. (26), we get

dy x n f ( y / x)
= n = F ( y/x) (27)
dx x g ( y / x)
Such types of equations can be solved by substitution y = vx. Equation
(27) thus becomes
dv dv dx
v+x = F (v) or =
dx F (v ) − v x
The variables have now been separated and the solution is
dv
∫ F (v ) − v = log x + c

After integrating the left hand side of this equation v should be replaced
by y/x to get the required solution.

Example 6.5 Solve the following differential equations


(i) x y ′ = x + y
(ii) (3xy + y 2)dx + (x 2 + xy) dy = 0
(iii) (x – y) y′ = x + 3y

Solution (i) The given differential equation may be written as


dy x+ y
= (28)
dx x
dy
which is a homogeneous differential equation. Put y = vx, then =v
dx
dv dx
+x and Eq. (28) reduces to dv = which on integration leads to
dx x
y
v= = log x + c as the required solution.
x
Differential Equations 207

(ii) The given equation (3xy + y 2) dx + (x 2 + xy)dy = 0 can be written as


dy 3xy + y 2
=− 2
dx x + xy
which is a homogeneous differential equation and thus put y = vx, and
dy dv
=v+x so that the given equation takes the form (after
dx dx
simplification)
dx 1 (1 + v) dv 1 ⎡ dv dv ⎤
=− =− ⎢ + ⎥
x 2 v(v + 2) 2 ⎣ 2v 2(v + 2) ⎦
Integrating both sides, we get
1 1
log x = – log v – log (v + 2) + log c = – log v1/4 – log (v + 2)1/4 + log c
4 4
or
log x + log v1/4 + log (v + 2)1/4 = log c ⇒ xv1/4(v + 2)1/4 = c
or
1/ 4
⎡y⎛ y ⎞⎤
x ⎢ ⎜ + 2 ⎟⎥ = c
x
⎣ ⎝ x ⎠⎦
which is the required solution.
(iii) The given equation can be expressed as
dy x + 3 y
=
dx x−y
dy
which is a homogeneous differential equation. Put y = vx, and =v
dx
dv
+x in this equation so that after simplification, we get
dx
dx 1− v − dv 2dv
= dv = +
x (1 + v) 2 1 + v (1 + v) 2
(using partial fractions). Integrating both sides, we get
2 ⎛x + y⎞ 2x
log x = – log (1 + v) – + c = – log ⎜ ⎟− +c
1+ v ⎝ x ⎠ x+ y
(using v = y/x). Thus, using the properties of logarithms, after
simplification, we get
208 Mathematical Methods

2x
log(x + y) + =c
x+ y
as the required solution.
6.3.3 Linear Differential Equations
A linear differential equation of first order is of the form
dy
+ Py = 0 (29)
dx
where P and Q are constants or functions of x alone.
The solution of Eq. (29) is given by
∫ Pdx dx + c
ye ∫
Pdx
= ∫Qe (30)

The quantity e ∫ Pdx is known as the integrating factor.


Example 6.6 Solve the following equations
(i) (x 2 – 1)y ′ + 2xy = 1
(ii) sin xdy + y cos x dx = x sin x dx
(iii) ydx – xdy + log xdx = 0
Solution (i) The given equation can be written as
dy ⎛ 2 x ⎞ 1
+⎜ 2 ⎟y = 2
dx ⎝ x − 1 ⎠ x −1
which is of the form of Eq. (29). Here
2x 2 xdx
P= and ∫ Pdx = ∫ x2 − 1 = log (x 2 – 1)
x −1 2

so that

e∫
Pdx 2
= elog( x −1) = x 2 − 1
Thus, from Eq. (30), the solution of the given differential equation is
1
y(x 2 – 1) = ∫ x2 − 1 (x 2 – 1) dx + c = x + c

(ii) The given equation can be written as


dy
+ (cot x) y = x
dx
which is a linear differential equation of first order. Here
Differential Equations 209

P = cot x ⇒ e ∫ = e∫
Pdx cot xdx
= elog sin x = sin x
Thus, from Eq. (30), after simplification, we get
( y – 1) sin x + x cos x = c
as the required solution.
(iii) The given equation can be written as
dy 1 1
− y = log x
dx x x
Here P = – 1/x, Q = (log x)/x and e ∫
Pdx
= e – log x = 1/x and thus from
Eq. (30), the solution is
y = cx – (1 + log x)
(using log x = t and integrating by parts).
6.3.4 Differential Equations Reducible to Linear Form
An equation of the form
dy
+ Py = Q y n (31)
dx
where P and Q are constants or functions of x alone, and n is constant
except 0 and 1, is called a Bernoulli’s equation.
The solution of Eq. (31) is obtained as follows:
Divide Eq. (31) by y n and put y – n + 1 = v so that (– n + 1) y – n dy/dx =
dv/dx, and Eq. (31) thus reduces to
dv
+ (1 – n) Pv = (1 – n)Q
dx
which is linear differential equation in v and can be solved by the method
given in Section 6.3.3.

Example 6.7 Solve the following differential equations

(i) (1 – x 2) dy + xydx = xy 2dx (ii) dy = (x 3y 3 – xy)dx

Solution (i) The given equation can be written as

dy x x
y–2
+ y −1 =
dx 1 − x 2
1 − x2
– 1 – 2
Put y = v so that – y (dy/dx) = dv/dx and the given equation
reduces to
210 Mathematical Methods

dv x x
− v=−
dx 1 − x 2 1 − x2
which is linear differential equation in v and the solution is (using
Eq. (30))

v(1 – x 2)1/2 = (1 – x 2)1/2 + c ⇒ (1 – y) 1 − x 2 = cy


where v is replaced by y – 1.
(ii) The given equation can be written as
dy
y −3 + xy – 2 = x 3
dx
Put y – 2 = v so that – 2y – 3 (dy/dx) = dv/dx and the given equation reduces
to
dv
– 2xv = – 2x 3
dx
which is linear differential equation in v and the solution is (using
Eq. (30))
2 2
x x
v = x 2 + 1 + ce ⇒ y – 2 = x 2 + 1 + ce
(using v = y – 2)
6.3.5 Exact Differential Equations
A differential equation is said to be exact if it can be derived from its
general soution directly by differentiation, without any subsequent
multiplication, elimination, etc.
The necessary and sufficient condition for the differential equation
M dx + N dy = 0 to be exact is that
∂M ∂N
=
∂y ∂x
To obtain the solution of an exact differential equation, we have the
following rule:
(i) Integrate M with respect to x, keeping y as constant.
(ii) Integrate with respect to y only those terms of N which do not
contain x.
(iii) Add the two expressions obtained in (i) and (ii) above and equate
the result to an arbitrary constant.
Differential Equations 211

Thus, the solution of an exact differential equation is

∫ M dx + ∫ N dy = c (32)
( y = const.) (terms without x )

Example 6.8 Solve


dy ax + hy + g
(i) + =0 (ii) (x + 2y – 3) dy = (2x – y + 1)dx
dx hx + by + f
∂M ∂N
Solution (i) Here, M = ax + hy + g and N = hx + by + f. Thus, = ,
∂y ∂x
which shows that the given differential equation is exact. Hence, the
required solution, from Eq. (32), is

∫ (ax + hy + g ) dx + ∫ (by + f ) dy = c1
or

ax 2 2
+ hxy + gx + by + f y = c1
2 2
⇒ ax + 2hxy + by 2 + 2gx + 2fy + c = 0
2

where c = –2c1.
∂M ∂N
(ii) Here, M = – 2x + y – 1 and N = x + 2y – 3. Thus, = , which
∂y ∂x
shows that the given differential equation is exact. Hence, the required
solution is

∫ M dx + ∫ N dy = c
( y = const.) (terms without x )

⇒ ∫ (− 2 x + y − 1) dx + ∫ (2 y − 3) dy = c
⇒ y 2 – x 2 + yx – x – 3y = c
⎡ x⎤
Example 6.9 Solve (1 + e x/y) dx + e x/y ⎢1 − ⎥ dy = 0.
⎣ y⎦
Solution Here
⎡ x⎤ ∂M ∂N
M = 1 + e x/y, N = e x/y ⎢1 − ⎥ and =
⎣ y ⎦ ∂y ∂x
Thus, the given differential equation is exact. Now
212 Mathematical Methods

∫ M dx = ∫ ⎡⎣1 + e x / y ⎤⎦ dx = x + ye x / y
( y = const.)

and

∫ N dy = 0
(terms without x )

Therefore, the required solution, from Eq. (32), is


x + ye x/y = c

6.4 Higher Order Linear Differential Equations


So far we have dealt with the linear differential equations of first order
and the methods for solving such equations. In this section, we shall be
concerned with the linear differential equations of order greater than
one. These equations have even greater importance. Motion of pendulums,
of elasting strings, of falling bodies, the flow of electric currents, the
study of chemical kinetics and many more such types of problems are
related to the solution of linear differential equations of order greater
than one (for a detailed account of such applications, see [3]).
Definition 6.7 A linear differential equation of order n is an equation of
the form
dn y d n −1 y dn−2y dy
an ( x) + an − 1 ( x ) n −1
+ an − 2 ( x) + ... + a1 ( x) + a0 ( x) y = Q( x)
dx n
dx dx n − 2 dx
(33)
where a0, a1, a2, ..., an and Q(x) are continuous real functions on a common
interval and an(x) ≠ 0. The right hand side of Eq. (33) is called the non-
homogeneous term. If Q(x) = 0, then Eq. (33) reduces to
dn y d n −1 y dy d n−2 y
an(x) n
+ an −1 ( x) n −1
+ an − 2 ( x)
+ a0 ( x) y = 0
n−2
+ ... + a1 ( x)
dx dx dx dx
(34)
Equation (34) is called a homogeneous linear differential equation
of order n.

6.5 Solution of Homogeneous Linear Differential


Equations of Order n with Constant Coefficients
In practice, equations of the form (34), where the coefficients are the
functions of x with no restriction placed on their nature, do not usually
have solutions expressible in terms of elementary functions (functions of
Differential Equations 213

the form x n, e x, log x, trigonometric functions and/or a combination of


them); and even when they do, it is very difficult to find them. However,
if each coefficient in Eq. (34) is constant, then the differential equation
(34) is called a linear differential equation with constant coefficients,
and the solution in terms of the elementary functions can be obtained.
Moreover, these differential equations are of great practical importance
as well as of theoretical interest.
The solution of the equation
n n −1 n−2
an d y + an −1 d y + an − 2 d y + ... + a1 dy + a0 y = 0 (35)
dx n dx n −1 dx n − 2 dx
where a0, a1, a2, ..., an are all constants and an ≠ 0, can be obtained as
follows:
Express Eq. (35) as
[an D n + an – 1 D n – 1 + an – 2 D n– 2 + ... + a1 D + a0] y = 0 (36)
where D ny = d ny/dx n, etc. Now, replace D n y = d n y/dx n etc. by m n etc. in
Eq. (36), we get
a n m n + an– 1mn – 1 + an – 2mn – 2 + ... + a1m + a0 = 0 (37)
Equation (37) is known as the auxiliary equation (A.E.) or the
characteristic equation of Eq. (35) or (36). This is a polynomial of degree
n and the solution of Eq. (35) depends upon the nature of the roots of the
A.E. (37). On solving Eq. (37), following cases may arise:
(i) All the roots are distinct and real.
(ii) All the roots are real but some are repeating.
(iii) All the roots are imaginary.
All these three possibilities shall now be discussed separately as

Case I: If the n roots m1, m2, ..., mn of Eq. (37) are distinct, then the
solution of Eq. (35) is
mx m x m x
yc = y = c1e 1 + c2 e 2 + ... + cn e n (38)
In Eq. (38), yc is known as complementary function.
Example 6.10 Solve
d3 y d2y dy
+6 + 11
+ 6y = 0
dx 3
dx 2 dx
Solution The A.E. for this differential equation is m 3 + 6m 2 + 11m + 6 =
0 which on solving leads to m = – 1, – 2, – 3. Therefore, from Eq. (38),
the required solution is
214 Mathematical Methods

y = c1e – x + c2e–2x + c3e–3x

Case II: If Eq. (37) has a root m = a, which repeats n times, then the
solution of Eq. (35) is
y = (c1 + c2 x + ... + cn x n–1) e ax (39)
If Eq. (37) has k roots each equal to m1 and the remaining (n – k) roots
are all distinct, then the solution of Eq. (35) is
y = (c1x k– 1 + c2 x k– 2 + ... + ck – 1x + ck) em1x + ck +1emk +1 x + ... + cn emn x
(40)
Example 6.11 Solve

d3y dy
−3 + 2y = 0
dx 3 dx
Solution The A.E. for this differential equation is m 3 – 3m + 2 = 0 whose
roots are m = 1, 1, – 2. Therefore, from Eq. (40), the required solution is
y = (c1x + c2) e x + c3e – 2x

Example 6.12 Solve

d2y dy
16 + 24 + 9y = 0
dx 2
dx
Solution The A.E. for the given differential equation is
16m 2 + 24m + 9 = 0 ⇒ (4m + 3)2 = 0 ⇒ m = –3/4, –3/4
Therefore, from Eq. (39), the required solution is
y = (c1 + c2 x) e(–3/4)x
Case III: If the constant coefficients in Eq. (37) are real, then any
imaginary root it may have must occur in conjugate pairs. Thus, if
α + iβ is one root, then α – iβ must be another root. If α ± iβ are two
imaginary roots of an auxiliary equation of a second order differential
equation, then the solution is
y = e α x (c1 cos βx + c2 sin βx) (41)
If α + iβ and α – iβ each occurs twice as a root, then the solution is
y = e αx [(c1 + c2 x) cos βx + (c3 + c4 x) sin βx) (42)

Example 6.13 Solve

d2y
+ 4y = 0
dx 2
Differential Equations 215

Solution Here, m = ± 2i are the roots of the A.E. m 2 + 4 = 0 and therefore,


from Eq. (41), the required solution is
y = c1 cos 2x + c2 sin 2x
Example 6.14 Solve
d4y d2y
4
+8
+ 16 y = 0
dx dx 2
Solution The A.E. m 4 + 8m 2 + 16 = 0 for the given differential equation
has a double root m = ± 2i and thus, from Eq. (42), the required solution
is
y = (c1 + c2 x) cos 2x + (c3 + c4 x) sin 2x

6.6 Solution of Nonhomogeneous Linear Differential


Equations of Order n with Constant Coefficients
The general solution of nonhomogeneous linear differential equations of
order n
dny d n −1 y d n−2 y (43)
+ P1 + P2 + ... + Pn y = Q ( x)
dx n dx n −1 dx n − 2
where P1, P2, ..., Pn are constants and Q(x) ≠ 0 is a function of x or
constant, is
y = yc + yp (44)
where yc and yp are known as the complementary function (C.F.) and
the particular integral (P.I.).
We have already discussed above the method of finding the
complementary function and it only remains to find the particular integral.
Here, we shall be concerned with the cases for which Q(x) consists of
such terms as b, x k, e ax, sin ax, cos ax and a finite number of combinations
of these, where a and b are constants and k is a positive integer.
Let D n = d n/dx n. Then, Eq. (43) can be expressed as
(D n + P1D n – 1 + ... + Pn)y = Q(x) (45)
which may be written as
f (D) y = Q(x) (46)
n
where f (D) = D + P1 D n – 1 + ... + Pn. Eq. (46) now leads to
1
y = yp = Q ( x) = [ f (D)] – 1 Q(x) (47)
f ( D)
which gives the particular integral and its exact form will depend upon
the nature of f (D) and Q (x).
216 Mathematical Methods

In case where f (D) can be factorised having the factors of the form
(D – α) (D – β) ... (D – γ), then
1 ⎡ 1 ⎤
Q ( x) =
⎢ ⎥ Q ( x)
f ( D) ⎣ ( D − α ) ( D − β)... ( D − γ ) ⎦
If f (D) has a factor of the form (D – α), then
1
Q ( x) = e αx ∫ Q ( x ) e − αx dx (48)
D−α
Example 6.15 Solve
d2y
+ a 2y = sec ax
dx 2
Solution The A.E. for the given differential equation is
m 2 + a 2 = 0 ⇒ m = ± ia
Thus, the complementary function yc is
yc = c1 cos ax + c2 sin ax (49)
while the particular integral yp is
1 1 1
yp = Q ( x) = 2 sec ax = sec ax
f ( D) D + a2 ( D + ia) ( D − ia)

1 ⎡ D − D + ia − ( − ia ) ⎤
= ⎢ ⎥ sec ax
2ia ⎣ ( D + ia ) ( D − ia ) ⎦

1 ⎡ ( D + ia ) − ( D − ia ) ⎤
= ⎢ ⎥ sec ax
2ia ⎣ ( D + ia ) ( D − ia ) ⎦
1 ⎡ 1 1 ⎤
= − sec ax
2ia ⎣ D − ia D + ia ⎥⎦

1 ⎡ 1 ⎤ 1 ⎡ 1 ⎤
= ⎢ ⎥ sec ax − sec ax (50)
2ia ⎣ D − ia ⎦ 2ia ⎢⎣ D + ia ⎥⎦
which is of the form (48). Now
1 ⎡ 1 ⎤ 1 iax
⎢ ⎥ sec ax = e ∫ sec ax (e −iax ) dx
2ia ⎣ D − ia ⎦ 2ia
1 iax 1
= e ∫ (cos ax – i sin ax) dx
2ia cos ax
Differential Equations 217

eiax
2ia ∫
= [1 − i tan ax ] dx

eiax ⎛ i ⎞
= ⎜ x + log {cos ax}⎟ (51)
2ia ⎝ a ⎠
In a similar manner, we have
1 ⎡ 1 ⎤ e−iax ⎛ i ⎞ (52)
⎢ ⎥ sec ax = ⎜ x − log {cos ax}⎟
2ia ⎣ D + ia ⎦ 2ia ⎝ a ⎠
where we have used Euler’s formula eiax = cos ax + i sin ax (cf., Chapter
1). Now, from Eqs. (51) and (52), the particular integral given by
Eq. (50) can be written, after simplification, as
x 1
yp = sin ax + 2 cos ax {log (cos ax)} (53)
a a
Therefore, from Eqs. (49) and (53), the complete solution of the given
differential equation is
x 1
y = yc + yp = c1 cos ax + c2 sin ax + sin ax + 2 cos ax {log (cos ax)}
a a

6.7 Special Methods for Finding the Paticular Integral


We shall now mention some methods for finding the particular integral.
Here, we shall deal with the cases for which Q(x) consists of such terms
as b, x k, e ax, sin ax, cos ax and a finite number of combinations of these,
where a and b are constants and k is a positive integer.
6.7.1 When Q(x) = e ax
In this case, the particular integral is
1 1 ax
P.I. = Q ( x) = e (54)
f ( D) f (a)
Example 6.16 Solve
d2y dy
+ 31 + 240y = 272e – x
dx 2 dx
Solution The given differential equation can be expressed as
(D 2 + 31D + 240) y = 272e – x
The A.E. m2 + 31m + 240 = 0 for this differential equation has the roots
m = – 15, – 16 and thus the complementary function is
218 Mathematical Methods

yc = c1e – 15x + c2e – 16x (55)


while the particular integral is
1 1
yp = Q( x) = 2 272e − x
f ( D) D + 31D + 240

1 136e − x
= 272e− x = (56)
(−1) + 31(−1) + 240
2 105
Therefore, form Eqs. (55) and (56), the required solution of the given
differential equation is
136e− x
y = yc + yp = c1 e – 15x + c2 e – 16 x +
105
Remark
If f (a) = 0 in Eq. (54), then
1 1 ax 1
P.I. = Q( x) = e V = e ax V (57)
f ( D) f (a) f ( D + a)
where Q(x) = e ax = e ax . 1 = e ax. V (V is constant).

Example 6.17 Solve

d3y d2 y dy
+3 +3 + y = e– x
dx 3
dx 2 dx
Solution For the given differential equation
(D 3 + 3D 2 + 3D + 1) y = e – x
3 2
the A.E. m + 3m + 3m + 1 = 0 has the roots m = – 1, – 1, – 1. Thus, the
complementary function is
yc = (c1 + c2x + c3x 2) e – x (58)
3 2 3
Here, f (D) = D + 3D + 3D + 1 = (D + 1) which gives f (a) = f (– 1) =
0, and thus from Eq. (57), the particular integral is
−x 1
yp = e (1)
f (D + a)

−x 1
= e
{( D + 1) − 1}3

1 −x x3
= e− x (1) = e – x D – 3 (1) = e (59)
D3 6
Differential Equations 219

Therefore, from Eqs. (58) and (59), the complete solution of the given
differential equation is
x3
y = yc + yp = (c1 + c2 x + c3 x 2 + ) e– x
6
6.7.2 When Q(x) = sin ax or cos ax
In this case, if f (– a 2) ≠ 0, the particular integral is
1 1 1
P.I. = yp = Q ( x) = sin ax = sin ax (60)
f ( D) f ( D2 ) f (− a 2 )
1 1 1
P.I. = y p = Q( x) = cos ax = cos ax (61)
f ( D) 2
f (D ) f (−a 2 )

1 1
It may be noted that if P.I. = yp = Q ( x) = sin ax, then put
f ( D) f (D2 )
– a2 for D 2, a 4 for D 4, – a6 for D6 and so on, in f (D) to calculate yp.
The above method, for calculating the particular integral, however
fails, if D 2 + a2 is a factor of f (D) and f (– a 2) = 0. In such case, we have
1 1 −x
P.I. = yp = Q( x) = 2 sin ax = cos ax (62)
f ( D) D +a 2 2a
1 1 x
P.I. = yp = Q ( x) = 2 cos ax = sin ax (63)
f ( D) D + a2 2a
Example 6.18 Solve
d2ydy
+ 9y = 40 sin 5x
−8
dx dx
2

Solution The A.E. for the given differential equation (D 2 – 8D + 9)y =


40 sin 5x is m 2 – 8m + 9 = 0 whose roots are m = 4 ± 7. Thus, the
complementary function is

yc = c1e(4 + 7 )x
+ c2 e (4 − 7)x
(64)
while, the particular integral is
1 1
yp = Q ( x) = 2 40 sin 5x
f ( D) D − 8D + 9
1 −1 1
= 40 sin 5 x = 40 sin 5 x
−5 − 8 D + 9
2
8 D+2
220 Mathematical Methods

− 40 D − 2 −5( D − 2) 5
= sin 5 x = sin5 x = (D – 2) sin 5x
8 D2 − 4 −5 − 4
2
29
5 5
= (D sin 5x – 2sin 5x) = (5 cos 5x – 2 sin 5x) (65)
29 29
Therefore, from Eqs. (64) and (65), the required solution is
5
y = yc + yp = c1e(4 + 7 )x
+ c2 e(4 − 7 )x
+ (5cos 5x – 2 sin 5x)
29
Example 6.19 Solve (D 2 + 4)y = cos 2x.
Solution The A.E. for the given differential equation is m2 + 4 = 0 which
has the roots m = ±2i, and thus the complementary function is
yc = c1 cos 2x + c2 sin 2x (66)
The particular integral, from Eq. (63), is
1 1 x x
yp = cos 2 x = cos 2 x = sin 2 x = sin 2 x (67)
D +a2 2
D +42 2(2) 4
Therefore, the complete solution of the given differential equation, from
Eqs. (66) and (67), is
x
y = yc + yp = c1 cos 2x + c2 sin 2x + sin 2x
4
Example 6.20 Solve (D 4 – 1) y = sin x.
Solution The A.E. m4 – 1 = 0 has the roots m = ± 1, ± i, and thus, the
complementary function is

yc = c1e x + c2e – x + c3 cos x + c4 sin x (68)


The particular integral, from Eq. (62), is
1 1
yp = sin x = sin x
D4 − 1 ( D 2 − 1) ( D 2 + 1)

1 ⎡ x ⎤ 1 1
= sin x = −
⎢− 2(1) ⎥ cos x = 4 x cos x (69)
(−1 − 1)( D + 1) 2
⎣ ⎦ 2
Therefore, the complete solution of the given differential equation, from
Eqs. (68) and (69), is
1
y = yc + yp = c1e x + c2e – x + c3 cos x + c4 sin x + x cos x
4
Differential Equations 221

6.7.3 When Q(x) = kx m, m is a positive integer, and k is a constant.


In this case, the particular integral is
1 1
P.I. = yp = Q ( x) = kx m
f ( D) f ( D)
Here, we write f (D) in the form of D – a ( f (D) may also has the form
D + a), then
1 1 1
P.I. = y p = kx m = k xm = kx m
f ( D) D−a − a (1 − D
a )
−1
1⎛ D⎞ m 1⎛ D D2 ⎞
= − ⎜1 − ⎟ kx = − ⎜
⎜1 + + 2 + ...⎟⎟ kx m (70)
a⎝ a⎠ a⎝ a a ⎠
−1
[expanding ⎛⎜1 − D ⎞⎟ in the powers of D].
⎝ a⎠
Remark
Here, if m = 0, then Q(x) = k and Eq. (45) leads to
f (D) y = (D n + P1 D n– 1 + ... + Pn) y = Q(x) = k
and in such case, the particular integral is

1 1 k
yp = Q ( x) = k= (70 a)
f ( D) f ( D) Pn
Example 6.21 Solve (D 2 + 4)y = sin2 x.
Solution The given differential equation can be written as
1
(D 2 + 4) y = sin2 x = (1 – cos 2x) (71)
2
The roots of the A.E. m 2 + 4 = 0 are m = ± 2i and the complementary
function thus has the form
yc = a1 cos 2x + a2 sin 2x

⎡ a1 a2 ⎤
= a12 + a22 ⎢ 2 cos2 x + sin 2 x⎥
⎢ a1 + a22 a12 + a22 ⎥
⎣ ⎦
= c1 (cos c2 cos 2x – sin c2 sin 2x) = c1 cos (2x + c2) (72)
where we have used the identity cos (A + B) = cos A cos B – sin A sin B
222 Mathematical Methods

and the substitutions cos c2 = a1/ a12 + a22 , sin c2 = −a2 / a12 + a22 , and

c1 = a12 + a22 . The particular integral, using Eqs. (63) and (70), is
given by
1 1 1 1 1 1 1
yp = Q( x) = 2 (1 − cos 2 x) = (1) − cos2 x
f ( D) D +42 2 D2 + 4 2 D2 + 4

−1
1⎛ D2 ⎞ 1 1 1
= ⎜⎜1 + ⎟ (1) − ( x sin 2 x) = − x sin 2 x (73)
8⎝ 4 ⎟⎠ 8 8 8
Therefore from Eqs. (72) and (73), the complete solution of the given
Eq. (71) is given by
1 1
y = yc + yp = c1 cos (2x + c2) + − x sin 2x
8 8

Example 6.22 Solve

d3 y d2y dy
+3 +2 = x2
dx 3
dx 2 dx
Solution For the given differential equation
(D 3 + 3D 2 + 2D) y = x 2
3 2
the A.E. m + 3m + 2m = 0 has the roots m = – 1, – 2, 0. Thus, the
complementary function is
yc = c1 + c2e – x + c3e – 2x (74)
The particular integral is
−1
1 1 1 ⎧⎪ ⎛ 3 D 2 ⎞⎫⎪ 2
yp = Q( x) = 3 x 2
= ⎨1 + ⎜ D + ⎟⎬ x
f ( D) D + 3D 2 + 2 D 2 D ⎪⎩ ⎜⎝ 2 2 ⎟⎠⎪⎭

1 ⎧⎪ 3 ⎫
2
D2 ⎛ 3 D2 ⎞ ⎪ 2
= ⎨1 − D − + ⎜ D + ⎟ − ...⎬x
2D ⎪ 2 2 ⎜⎝ 2 2 ⎟⎠
⎩ ⎭⎪
1 ⎧ 2 3 2 1 2 2 9 2 2 1 4 2 3 3 2 ⎫
= ⎨x − Dx − D x + D x + D x + D x + ...⎬
2D ⎩ 2 2 4 4 2 ⎭
1 −1 ⎧ 2 9 ⎫ 1 ⎧1 3 9 ⎫
= D ⎨ x − 3 x − 1 + ⎬ = ⎨ x 3 − x 2 − x + x⎬ (75)
2 ⎩ 2 ⎭ 2 ⎩3 2 2 ⎭
Differential Equations 223

Therefore, from Eqs. (74) and (75), the complete solution of the given
differential equation is

1 ⎧1 3 3 2 9 ⎫
y = c1 + c2e – x + c3e –2x + ⎨ x − x − x + x⎬
2 ⎩3 2 2 ⎭
Example 6.23 Solve

d3y d2y dy
+2 + = e 2x + x 2 + x
dx 3
dx 2 dx
Solution For the given differential equation, the A.E. m3 + 2m2 + m = 0
has the roots m = 0, – 1, – 1. Thus, the complementary function is
yc = c1 + (c2 + c3x) e – x (76)
The particular integral is
1 1
yp = Q( x) = 3 (e 2x + x 2 + x)
f ( D) D + 2D2 + D
1 1 1
= e2 x + x2 + x
D + 2D + D
3 2
D + 2D + D
3 2
D + 2D 2 + D
3

1 1 1 1 1
= e2 x + x2 + x
2 + 2(2 ) + 2
3 2
D (1 + 2 D + D )
2
D (1 + 2 D + D 2 )

1 2x 1 1
= e + {1 + (2 D + D 2 )}−1 x 2 + {1 + (2 D + D 2 )}−1 x
18 D D
Now, expanding the second and third terms of this equation using the
Binomial theorem, we get, after performing the indicated operations
1 2x
yp = e + ∫ x 2 dx − 4 ∫ xdx + 6 ∫ dx + ∫ xdx − 2 ∫ dx
18
1 2x 1
= e + x(2x 2 – 9x + 24) (77)
18 6
Therefore, from Eqs. (76) and (77), the given differential equation has
the solution

1 2x 1
y = c1 + (c2 + c3 x) e – x + e + x (2x 2 – 9x + 24)
18 6
224 Mathematical Methods

Example 6.24 Solve (D 2 – 2D – 3) y = 5.

Solution Here
yc = c1e – x + c2e 3x
Also, f (D) = D2– 2D – 3 with Q(x) = k = 5 and Pn = – 3 and thus from
Eq. (70a), we have
k k 5
yp = = =−
f ( D ) Pn 3
Thus, the solution of the given differential equation is
5
y = yc + yp = c1e – x + c2e 3 x –
3
6.7.4 When Q(x) = e axV, V is a function of x
In this case, the particular integral is
1 1 ax 1
P.I. = yp = Q( x) = e V = e ax V
f ( D) f ( D) f ( D + a)
Example 6.25 Solve
d2y dy
−4 + y = e 2x sin 2x
dx 2 dx
Solution For the given differential equation the A.E. m 2 – 4m + 1 = 0 has
the roots m = 2 ± 3. Thus, the complementary function is
(2 +
yc = c1e
3) x
+ c2 e (2− 3) x
(78)
The particular integral is
1 ax 1
yp = e V = 2 (e 2x sin 2x)
f ( D) D − 4D + 1

2x 1 1
= e sin 2 x = e 2 x sin 2 x
( D + 2) − 4( D + 2) + 1
2
D −3
2

2x 1 e2 x
= e sin 2 x = −sin 2 x (79)
−2 2 − 3 7
The complete solution of the given differential equation, from Eqs. (78)
and (79), is

(2 + e2 x
y = c1e
3) x
+ c2 e(2− 3) x
− sin 2 x
7
Differential Equations 225

Example 6.26 Solve


d3y dyd2y
−3 – y = xe x + e x
+3
dx 3
dx dx 2

Solution The given differential equation has A.E. as m3 – 3m2 + 3m –


1 = 0 whose roots are m = 1, 1, 1. Thus, the complementary function is
yc = (c1 + c2 x + c3 x2)e x (80)
The particular integral is
1 1
yp = Q( x) = 3 e x(x + 1)
f ( D) D − 3D + 3D − 1
2

x 1
= e ( x + 1)
( D + 1)3 − 3( D + 1) 2 + 3( D + 1) − 1
1
= e
x
( x + 1) = e x D −3 ( x + 1) = e x D −2 ∫ ( x + 1) dx
D3
⎛ x2 ⎞ ⎛ x2 ⎞
= e x D −2 ⎜⎜ + x ⎟⎟ = e x D −1 ∫ ⎜⎜ + x ⎟⎟ dx
⎝ 2 ⎠ ⎝ 2 ⎠
⎛ x3 x 2 ⎞ ⎛ x 4 x3 ⎞
= e x D −1 ⎜⎜ + ⎟⎟ = e x ⎜⎜ + ⎟⎟ (81)
⎝6 2⎠ ⎝ 24 6 ⎠
Therefore, from Eqs. (80) and (81), the solution of the given differential
equation is
⎛ x 4 x3 ⎞
y = (c1 + c 2 x + c3 x 2)e x + e x ⎜⎜ + ⎟⎟
⎝ 24 6 ⎠
Example 6.27 Solve

d2y dy
−4 + 4y = 8x 2 e 2x sin 2x
dx 2 dx
Solution The given differential equation has A.E. as m2 – 4m + 4 = 0
whose roots are m = 2, 2. Thus, the complementary function is
yc = (c1 + c2x) e 2x (82)
The particular integral is
1 1
yp = Q( x) = 2 8x 2e 2x sin 2x
f ( D) D − 4D + 4
226 Mathematical Methods

1
yp = 8e 2 x x 2 sin 2 x
( D + 2) 2 − 4( D + 2) + 4
1 1 2
= 8e2 x x 2 sin2 x = 8e2 x
D∫
2
x sin 2 x dx
D
1⎡ 2 ⎛ dx 2 ⎞ ⎤
= 8e 2 x ⎢ x ∫ sin2 x dx − ∫ ⎜⎜ ∫ sin 2 x dx ⎟⎟ dx⎥
D ⎢⎣ ⎝ dx ⎠ ⎥⎦
1⎡ 1 2 1 1 ⎤
= 8e2 x ⎢− x cos 2 x + x sin2 x + cos2 x⎥
D⎣ 2 2 4 ⎦
⎡ 1 1 1 ⎤
= 8e2 x ⎢− ∫ x 2 cos2 xdx + ∫ x sin 2 x dx + ∫ cos 2 xdx⎥
⎣ 2 2 4 ⎦
= e 2x (– 2x 2 sin 2x – 4x cos 2x + 3sin 2x) (83)
Therefore, the required solution, from Eqs. (82) and (83) is given by
y = (c1 + c2x) e 2x + e 2x (– 2x 2 sin 2x – 4x cos 2x + 3sin 2x)
6.7.5 When Q(x) = x m sin ax (or x m cos ax), m is a positive integer
In this case, the particular integral is
1 1
P.I. = yp = Q( x) = x m sin ax
f ( D) f ( D)
or,
1 1
P.I. = yp = Q( x) = x m cos ax
f ( D) f ( D)
Now, to get the exact value of the particular integral, use
eiax − e −iax eiax + e − iax
sin ax = and cos ax = (84)
2i 2
in the above equations so that the particular integral can take the form
1 1 ax
P.I. = y p = Q( x) = e V
f ( D) f ( D)
which can be solved by using the method given in Section 6.7.4.

Example 6.28 Solve


d2y dy
+ = x cos x
dx 2 dx
Differential Equations 227

Solution Here
yc = c1 + c2 e – x (85)
1 1 1 ⎡1 ⎤
yp = Q( x) = 2 x cos x = 2 x ⎢ (eix + e −ix )⎥
f ( D) D +D D +D ⎣ 2 ⎦
1 1 1 1
= eix x + e−ix x
2D +D
2
2D +D
2

1 ix 1 1 1
= e x + e−ix x
2 ( D + i)2 + ( D + i) 2 ( D − i) + ( D − i)
2

1 ix 1 1 1
= − e x − e −ix x
2 1 − D − 2 Di − D − i
2
2 1 − D + 2 Di − D + i
2

1 −1
= − eix ⎡⎣1 − ( D 2 + 2 Di + D + i )⎤⎦ x
2
1 −ix −1
– e ⎡⎣1 − ( D − 2 Di + D − i )⎤⎦ x
2
2
which on using the Binomial theorem and Eq. (84), after simplification,
reduces to
1 1
x(sin x − cos x) + cos x + sin x
yp = (86)
2 2
Thus from Eqs. (85) and (86), the required solution is
1 1
y = c1 + c2 e – x + x(sin x – cos x) + cos x + sin x
2 2
Example 6.29 Solve

d2y
+ y = x 2 sin 2x
dx 2
Solution Here
yc = c1 cos (x + c2) (87)
and
1 1 1 ⎡1 ⎤
yp = Q ( x) = 2 x 2 sin 2 x = 2 x 2 ⎢ (ei 2 x − e−i 2 x )⎥
f ( D) D +1 D + 1 ⎣ 2i ⎦

1 1 1 1
= ei 2 x x 2 − e −i 2 x x 2
2i D + 1
2
2i D + 1
2
228 Mathematical Methods

1 i2x 1 1 1
= e x 2 − e −i 2 x x2
2i ( D + 2i ) 2 + 1 2i ( D − 2i ) 2 + 1
1 i2x 1 1 1
= e x 2 − e −i 2 x 2 x2
2i D + 4iD − 3
2
2i D − 4i D − 3
−1
⎛ 1⎞ ⎛ 1 ⎞ ⎡ ⎛ D 2 4iD ⎞⎤ 2
= ⎜ − ⎟ ⎜ ⎟ e 2ix ⎢1 − ⎜ + ⎟⎥ x
⎝ 3 ⎠ ⎝ 2i ⎠ ⎜ 3 ⎟⎠⎥⎦
⎣⎢ ⎝ 3
−1
⎡ ⎛ 2 ⎞⎤
– ⎛⎜ − 1 ⎞⎟ ⎛⎜ 1 ⎞⎟ e−2ix ⎢1 − ⎜ D − 4iD ⎟⎥ x 2
⎝ 3 ⎠ ⎝ 2i ⎠ ⎜ 3 ⎟⎠⎥⎦
⎢⎣ ⎝ 3
which from Binomial theorem and Eq. (84), after simplification, leads
to
8 1
yp = − x cos2 x + (26 − 9 x 2 )sin 2 x (88)
9 27
The required solution from Eqs. (87) and (88) is
8 1
y = yc + yp = c1 cos (x + c2) – x cos 2x + (26 – 9x2) sin 2x
9 27

6.7.6 When Q(x) = xV, V is any Function of x


In this case, the particular integral is
1 1 1 f ′( D )
P.I. = yp = Q ( x) = xV = x V− V (89)
f ( D) f ( D) f ( D) [ f ( D)]2
Example 6.30 Solve

d2y dy
−2 + y = x sin x
dx 2 dx
Solution Here
yc = (c1 + c2x) ex (90)
1 1
yp = Q( x) = 2 x sin x
f ( D) D − 2D + 1
1 2D − 2
= x sin x − sin x
D − 2D + 1
2
( D − 2 D + 1) 2
2
Differential Equations 229

[using Eq. (89)]. Thus


1 2D − 2
yp = x sin x − sin x
−1 − 2 D + 1 ( −1 − 2 D + 1) 2

1 −1 1
= − xD sin x − D – 2 (2D – 2) sin x
2 4

1 1
= x cos x − D −2 (cos x – sin x)
2 2

1
= (xcos x + cos x – sin x) (91)
2
The required solution of the given differential equation, from Eqs. (90)
and (91), is therefore
1
y = (c1 + c2 x) e x + (x cos x + cos x – sin x)
2
Example 6.31 Solve

d2y dy
+2 + y = xcos x
dx 2 dx
Solution Here
yc = (c1 + c2 x) e – x (92)
and
1 1
yp = Q( x) = 2 x cos x
f ( D) D + 2D + 1
1 2D + 2
= x cos x − cos x
D + 2D + 1
2
( D + 2 D + 1) 2
2

which after using Eq. (89) and simplification leads to


x 1
yp = sin x − (sin x – cos x) (93)
2 2
Therefore, the required solution of the given differential equation, from
Eqs. (92) and (93), is
1
y = (c1 + c2 x) e – x + (x sin x + cos x – sin x)
2
230 Mathematical Methods

EXERCISES
1. Form the differential equation for the curve y = c(x – c)2.
⎡ dy ⎛ dy ⎞ ⎤
2
⎢ Ans. 8 y 2
= 4 xy − ⎜ ⎟ ⎥
⎣⎢ dx ⎝ dx ⎠ ⎦⎥
2. Form the differential equation for the curve (x – h)2 + ( y – k) 2
= r 2. [Ans. [1 + (dy/dx)2]3 = r 2(d 2y/dx 2)2]
3. Find the differential equation of all the circles which pass through
the origin and whose centre lies on y-axis.
[Ans. x 2 – y 2 – 2xy(dx/dy) = 0]
4. Solve the following differential equations:
(i) (1 + cos x) dy – (1 – cos x) dx = 0
[Ans. y = 2 tan (x/2) – x + c]
(ii) (x + 2) (dy/dx) = x 2 + 4x – 9
[Ans. y = (x 2/2) + 2x – 13 log (x + 2) + c]
(iii) (x – 1) y′ = 2x 3

[Ans. log y = (x 3/3) – (x 2/2) + x + log (x – 1) + c]


(iv) y′ = (1 + x) (1 + y 2) [Ans. tan – 1 y = x + (x 2/2) + c]
(v) x log xdy – ydx = 0 [Ans. y = log (log x) + c]
⎛ dy ⎞ ⎛ 2 dy ⎞
(vi) ⎜ y − x ⎟ = a ⎜ y + ⎟ [Ans. [(a + x) (1 – ay)]/y = c]
⎝ dx ⎠ ⎝ dx ⎠
5. Solve the following homogeneous differential equations:
(i) x 2 y ′ = x 2 + xy + y 2 [Ans. log x = tan – 1 ( y/x) + c]
(ii) 2xyy ′ = x 2 + 3y 2 [Ans. x 2 + y 2 = cx 3]
⎛ x⎞
(iii) (1 + e x/y) dx + e x/y ⎜1 − ⎟ d y = 0 [Ans. x + ye x/y = c]
⎝ y⎠
(iv) x 2dy + y(x + y) dx = 0 [Ans. x 2y = e c( y + 2x)]
6. Solve
(i) (x + 2y 3) dy = ydx [Ans. x = y ( y 2 + c)]
(ii) y ′ + 2y = 4x [Ans. y = 2x – 1 + ce–2x]
(iii) y′ + 2y = 6e x [Ans. y = 2e x + ce –2x ]
(iv) (1 – x2) dy + 2xydx = x (1 – x 2)1/2 dx
⎡Ans. y = 1 − x 2 + c(1 − x 2 )⎤
⎣⎢ ⎥⎦
Differential Equations 231

7. Solve
(i) xdy + ydx = y 2 log x dx [Ans. 1 = (1 + log x) y – cxy]
(ii) y(2xy + e x) dx – e x dy = 0 [Ans. e x y –1 = – x 2 + c]
(iii) cos xd y = ( y sin x – y) dx [Ans. 1/y = sin x + c cos x]
8. Solve
xdy − ydx
(i) xdx + ydy + = 0. [Ans. x 2 – 2 tan –1 (x/y) + y 2 = 2c]
x2 + y2
(ii) (sin x cos y + e 2x) dx + (cos x sin y + tan y) d y = 0

1 2x
[Ans. e – cos x cos y + log sec x = c]
2
9. Continuous compound interest. Find the amount A in the saving
dA
bank account after t years if: (i) = 0.08 A and A(0) = 1000 and
dt
dA
(ii) = 0.12 A and A(0) = 5250.
dt
[Ans. (i) 1000 e 0.08t (ii) A = 8000e 0.06t]
10. Price-demand. If the marginal price dp/dx at x units of demand
per week is proportional to the price p, and if at Rs. 100.00 there is
no weekly demand [p (0) = 100], and if at Rs. 77.88 there is a
weekly demand of 5 units [p(5) = 77.88], find the price-demand
equation. [Ans. p(x) = 100e – 0.05x]
11. Population growth. India had a population of 500 million people
in 1966 (t = 0) and growth rate of 3 percent per year (which is
assumed to be compounded continuously). If P is the population
in millions t years after 1966, and the same growth rate continues,
what will India’s population be in the year 2011?
[Ans. 1429 million people]
12. Archaeology. (i) A peice of animal bone was found at an
archaeological site. If 10 percent of the original amount of
radioactive carbon-14 was present, find the age of the bone. (ii) A
skull from an ancient tomb was discovered and found to have
5 percent of the original amount of carbon-14 present. Estimate
the age of the skull.
[Ans. (i) 18600 years approximately (ii) 24200 years approximately]
232 Mathematical Methods

13. Drug concentrations. A single injection of a drug is administered


to a patient. The amount X in the body then decreases at a rate
proportional to the amount present, and for this particular drug
the rate is 4 percent per hour. Thus

dX
= – 0.04X, X (0) = X0
dt
where t is the time in hours. If the initial injection is 3 milliliters
[X(0) = 3], find X = X(t) so that both the conditions are satisfied.
How many milliliters of the drug are still in the body after 10
hours? [Ans. X = 3e –0.04t, X(10) = 2.01 millilitres]
14. Effect of adverstisement. An automobile company is trying to
expose its new car to as many people as possible through an
advertisement of television. Let the rate of exposure to new people
is proportional to the number of those who have not seen this car
out of V possible viewers. If no one is aware of this new car at the
beginning of the campaign and after t days 40 percent of V are
aware of the new car, solve

dN
= k(V – N), n(0) = 0, N(10) = 0.4 V
dt
for N = N(t), the number of people who are aware of the new car
after t days of advertising. [Ans. N = V (1 – e – 0.05t)]
15. Spread of rumour. A group of 400 parents, relatives and friends
are waiting at New Delhi Airports for a student charter to return
after a year in South America. The weather is very bad and the
plane is late. A particular parent thought he had heard that the
radio of the plane has got out of order and passed this news to
some friends, who in turn passed it to others, and so on. Social
scientists have studied the propagation of rumour and found that a
rumour tends to spread at a rate which is directly proportional to
the number x, who heard it, and to the number P – x, who have not
heard it, where P is the total population. In our case, P = 400 and

dx
= 0.001x (400 – x), x (0) = 1
dt
where t is the time in minutes. Then
(i) Solve this equation for x (t).
Differential Equations 233

(ii) How many people heard the rumour after 5 min and 20 min?
(iii) Find lim x(t ).
t→∞

[Ans. (i) x(t) = 400/(1 + 399e – 0.4t), (ii) 7 and 353 (iii) 400]
d3y d2y dy
16. Solve − −6 = 0. [Ans. y = c 1 + c2e 2x + c 3e 3x]
dx 3
dx 2 dx
d2y dy
17. Solve − 2a + a 2y = 0. [Ans. y = (c1 x + c2)e ax]
dx 2 dx
d3y
18. Solve + y = 0.
dx3
[Ans. y = c1e – x + e (1/2) x [c2 cos( 3 /2) x + c3 sin( 3 /2) x]
2
2
⎛ 2 ⎞
19. Solve ⎜⎛ dy − y ⎟⎞ ⎜ d y + y ⎟ = 0.
⎝ dx ⎜
⎠ ⎝ dx 2 ⎟

[Ans. y = (c1 + c2x) e x + (c3 + c4x) cos x + (c5 + c6 x) sin x]
20. Solve
(i) (D 2 – 2D + 5) y = e – x.
1
[Ans. y = (c1 cos 2x + c2 sin 2x + ) e – x ]
8
(ii) (D 3 – D 2 – 4D + 4) y = e 3x
e3 x
[Ans. y = c1e x + c2e 2x + c3e – 2x + ]
10
21. Solve (D 2 + 6D + 9) y = 2e – 3x.
[Ans. y = (c1 + c2 x)e – 3x + x 2e – 3x]
22. Solve (D 3 + D2 – D – 1) y = cos 2x.
1
[Ans. y = c1e x + (c2 + c3 x) e – x – (2sin 2x + cos 2x)]
25
23. Solve (D 2 – 3D + 2) y = 3sin 2x.
3
[Ans. y = c1e x + c2e 2x + (3 cos 2x – sin 2x)]
20
24. Solve (D 2 – 3D + 2)y = 3 sin 2x.
3
[Ans. y = c1e x + c2e 2x – `(3sin 3x + 7 cos 3x)]
130
1 ⎛ 2 1⎞
25. Solve (D 2 – 4) y = x 2. [Ans. y = c1 e 2x + c2e – 2x – ⎜x + ⎟ ]
4 ⎝ 2⎠
234 Mathematical Methods

26. Solve (D 2 + 2D + 1)y = 2 x + x 2.


[Ans. y = (c1 + c2x)e – x + x 2 – 2x + 2]
27. Solve (D3 – 2D + 4)y = x 4 + 3x 2 – 5x + 2.
1 4
[Ans. y = c1e – 2x + e x (c2 cos x + c3 sin x) + (x + 2x 3 + 6x 2
4
7
– 5x – )]
2
28. Solve (D 4 + D 2 + 16) y = 16x 2 + 256.
( 7 /2)x
[Ans. y = e (c1 cos (3/2)x + c2 sin (3/2)x)
− ( 7 /2) x
+ e (c3 cos(– 3/2)x + c4 sin (– 3/2) x) + x 2 + (127/8)]
29. Solve (D 4 – 1)y = e x cos x.

ex
[Ans. y = c1 e x + c2e – x + (c3 cos x + c4 sin x) – cos x]
5
30. Solve (D 2 + 4D – 12)y = (x – 1)e 2x.
e2 x ⎛ x 2 9 ⎞
[Ans. y = c1e – 6x + c2e 2x + ⎜ − x⎟ ]
8 ⎝⎜ 2 8 ⎠⎟
31. Solve (D 4 + D 3 + D 2 – D – 2)y = x 2 + e x.
⎛ 7 ⎞
[Ans. y = c1e x + c2e – x + c3e – x/2 cos ⎜ x + c4 ⎟
⎜ 2 ⎟
⎝ ⎠
1 1 1 3
+ e x x − x2 + x − ]
8 2 2 4
32. Solve (D3 + 1) y = e – x sin x.

⎛ 3 ⎞ 1
[Ans. y = c1e – x + c2e x/2 cos ⎜ + c3 ⎟ − e – x (2cos x – 3sin x)]
⎜ 2 ⎟
⎝ ⎠ 13
33. Solve (D 4 – 1) y = x sin x.

x2 3
[Ans. y = c1e – x + c2e x + c3 cos x + c4 sin x + cos x − x sin x ]
8 8
34. Solve (D 2 – 2D + 1) y = xe x sin x.
[Ans. y = (c1 + c2x – x sin x – 2 cos x) e x]
Matrices and Determinants 235

CHAPTER 7

Matrices and Determinants

7.1 Introduction and Definition


Let there be 50 chairs lying outside a big room, and one is asked to
arrange them inside the room. There are several ways to arrange these
chairs in the room. One way is to put all these 50 chairs in a single row,
or one can keep these 50 chairs in two rows, each containing 25 chairs;
or one can put these 50 chairs in 5 rows, each containing 10 chairs (10
columns), or else one can arrange 50 chairs in 10 rows, each containing
5 chairs (5 columns). Thus by making different types of arrangements,
one can arrange these 50 chairs in rows and columns. Such type of
arrangement can be seen in a big Lecture theatre or a cinema hall. Also,
when dealing with some scientific data, it is often convenient to present
this data in a tabular form which is a rectangular arrangement of rows
and columns; and it is in this way one can extract the information from
the table by reading the entry corresponding to the intersection of a row
and a column. Any rectangular arrangement of quantities in rows and
columns is called a matrix.
Matrices were devised by Arthur Cayley (1821-1895) in 1858, while
the determinants were earlier invented by a Japanese mathematician Seki
Kowa in 1683, and independently, by a German mathematician Leibnitz
in 1693. To get the solution of a system of linear equations, the
determinants were rediscovered by a Swiss mathematician Cramer
(1704-1752), in 1750. The matrices and determinants have a number of
useful applications in different areas of mathematics, natural sciences
and social sciences. This Chapter deals with an elementary treatment of
matrices and determinants along with their applications.

Definition 7.1 An array of real numbers (or some other well defined
objects) which are arranged in rows and columns is called a matrix. The
entries of a matrix are known as its elements. Some of the examples of
matrices are
236 Mathematical Methods

⎡ 1 2 3 4⎤ ⎡ − 41 27⎤
⎢ 71 22 − 23 890⎥ , ⎢− 471 − 22⎥
⎢ ⎥ ⎢ ⎥
⎣⎢42 87 56 381⎥⎦ ⎢⎣ 4132 807⎥⎦
and
⎡− 41⎤
⎢− 41⎥
⎡10 − 20 − 43 − 4 54⎤ ⎢ ⎥
⎢61 822 − 2 −90 34⎥ , ⎢ 42⎥
⎣ ⎦ ⎢ ⎥
⎢ 76⎥
⎢⎣ 75⎥⎦
Definition 7.2 In general, a rectangular array of the form
⎡ a11 a12 " a1n ⎤
⎢a a22 " a2 n ⎥⎥
⎢ 21
⎢ . . . . ⎥
⎢ . . . . ⎥
⎢ ⎥
⎣am1 am 2 " amn ⎦
where a ij are scalars, is called a matrix. This matrix is also denoted by
(a ij), i = 1, 2, ..., m, j = 1, 2, ..., n or, only by (aij). The m-horizontal
n-tuples
[a11, a12, ..., a1n], [a21, a22, ..., a2n], ..., [am1, am2, ..., amn]
are known as the row of the matrix and the n vertical m-tuples
⎡a11 ⎤ ⎡a12 ⎤ ⎡a1n ⎤
⎢a21 ⎥ ⎢a22 ⎥ ⎢a2 n ⎥
⎢... ⎥ , ⎢... ⎥ , ...,
⎢... ⎥
⎢⎣am1 ⎥⎦ ⎢⎣am 2 ⎥⎦⎢⎣amn ⎥⎦
are the columns of the matrix. The element aij, which is known as
ij-entry or ij-component, appears in the ith row and jth column. A matrix
with m rows and n columns is called an m by n matrix, or m × n matrix
and the pair of numbers (m, n) is known as the size or shape or order of
the matrix.
It may be noted that a matrix is not a number, it is a rectangular
array of numbers.

Definition 7.3 A matrix with only one row is called a row vector or row
matrix and a one which has got only one column is the column vector or
column matrix.
Matrices and Determinants 237

Definition 7.4 If every element of a matrix is zero, then it is known as


null or zero matrix. A null matrix of the type m × n is denoted by Om× n or
simply by O. For example
⎡0 0 0 0⎤
⎢0 0 0 0⎥
⎢ ⎥
⎢⎣0 0 0 0⎥⎦
is a 3 × 4 null matrix.
Definition 7.5 If the number of rows and columns of a matrix are equal,
then the matrix is known as the square matrix.
⎡ a11 a12 " a1n ⎤
⎢a ⎥
⎢ 21 a22 " a2 n ⎥
The square matrix A = ⎢ . . . . ⎥
⎢ . . . . ⎥
⎢ ⎥
⎣an1 an 2 " ann ⎦
has two diagonals; one is starting from the upper left hand corner and
ending at the lower right hand corner while the other one is extending
from upper right hand corner to the lower left hand corner. Thus the
elements of these two diagonals are a11, a22, ..., ann and a1n, ..., an1. The
diagonals of the matrix A with entries a11, a22, ..., ann is known as the
principal diagonal.
The matrices
⎡ 11 12 33 44⎤
⎢−51 21 − 223 90⎥ ⎡ − 41 27 11⎤ 10 − 20
⎢ ⎥ , ⎢− 471 − 22 35⎥ , ⎡ ⎤
⎢ 4 7 5 81⎥ ⎢ ⎥ ⎣⎢61 822⎦⎥
⎢ ⎥ ⎢⎣ 4132 807 80⎥⎦
⎣⎢ 34 45 67 23⎦⎥
are the square matrices of orders 4, 3, and 2, respectively, and the principal
diagonals for each of these matrices, respectively, have entries as 11, 21,
5, 23; – 41, – 22, 80 and 10, 822.
Definition 7.6 If all the elements of a square matrix, except the diagonal
ones, are zero, then the matrix is a diagonal matrix.
The matrices
⎡11 0 0 0⎤
⎢ ⎥ ⎡− 41 0 0⎤
⎢ 0 21 0 0⎥ , ⎢ 0 − 22 0⎥ , ⎡10 0⎤
⎢ 0 0 5 0⎥ ⎢ ⎥ ⎢ 0 822⎥
⎢⎣ 0 ⎣ ⎦
⎢ ⎥ 0 80⎥⎦
⎣⎢ 0 0 0 7⎦⎥
238 Mathematical Methods

are the diagonal matrices

Definition 7.7 If all the elements of a diagonal matrix are same, then the
matrix is called a scalar matrix.
The matrices
⎡11 0 0 0⎤
⎢ 0 11 0 0⎥ ⎡− 41 0 0⎤
⎢ 0 ⎡10 0⎤
⎢ ⎥, − 41 0⎥⎥ , ⎢
⎢ 0 0 11 0⎥ ⎢
⎣0 10⎥⎦
⎢ ⎥ ⎢⎣ 0 0 − 41⎥⎦
⎣ 0 0 0 11⎦
are the scalar matrices.

Definition 7.8 If each element of a diagonal matrix is unity, then the


matrix is called an identity matrix. An identity matrix of order n is usually
denoted by In.
The identity matrices of order 4, 3 and 2 are given as

⎡1 0 0 0⎤
⎢0 ⎡1 0 0⎤
1 0 0⎥⎥ ⎢0 ⎡1 0⎤
⎢ , 1 0⎥⎥ , ⎢0
⎢0 0 1 0⎥ ⎢
⎣ 1⎥⎦
⎢ ⎥ ⎣⎢0 0 1⎦⎥
⎣0 0 0 1⎦

Definition 7.9 A matrix which is obtained by deleting the rows or columns


or both of a matrix is known as the sub matrix of the original matrix.
The matrix

⎡11 12 33 ⎤
⎢− 51 21 − 223⎥⎥

⎢4 7 5 ⎥
⎢ ⎥
⎣34 45 67 ⎦
is a submatrix of the matrix

⎡ 11 12 33 44⎤
⎢− 51 21 − 223 90⎥⎥

⎢ 4 7 5 81⎥
⎢ ⎥
⎣ 34 45 67 23⎦
Matrices and Determinants 239

7.2 Algebra of Matrices


This section deals with the addition, subtraction and multiplication of
matrices along with the related results and definitions and we have

Definition 7.10 Two matrices X and Y are equal if and only if X and Y
are of the same order and each element of X is equal to the corresponding
element of Y. That is, X = [xij]m × n and Y = [ yij]m × n are equal if and only
if xij = yij, for every i and j.
The matrices
⎡21 10⎤ ⎡42/2 20 − 10⎤
⎢ 6 ⎥ and ⎢ ⎥
⎣ 22⎦ ⎣ 36 22 ⎦
and equal, while the matrices
⎡21 10 ⎤ ⎡210 10⎤
⎢6 and
⎣ 22⎥⎦ ⎢ 6
⎣ 2 ⎥⎦
are not equal.

Definition 7.11 The sum of two matrices A and B is a matrix C of the


same order, whose entries are the sum of the corresponding entries of
the matrices A and B.
If
⎡3 1 2⎤ ⎡ 1 0 2⎤
A= ⎢ ⎥ and B = ⎢
⎣2 1 4⎦ ⎣− 1 3 0⎥⎦
then
⎡3 + 1 1 + 0 2 + 2⎤ ⎡4 1 4⎤
A+B = ⎢ ⎥=⎢ ⎥
⎣2 + (−1) 1 + 3 4 + 0⎦ ⎣1 4 4⎦
It may be noted that the sum of two matrices of different order is not
defined. Also, if
⎡3 1 2⎤ ⎡0 0 0⎤
⎢ 4⎥⎥ and O = ⎢0 0⎥⎥
A = ⎢2 1 ⎢ 0
⎢⎣4 5 6⎥⎦ ⎢⎣0 0 0⎥⎦
then
⎡3 + 0 1+ 0 2 + 0⎤ ⎡3 1 2⎤
A + O = ⎢2 + 0 1+ 0 4 + 0⎥⎥ = ⎢⎢2 1 4⎥⎥ = A

⎢⎣4 + 0 5+0 6 + 0⎥⎦ ⎢⎣4 5 6⎥⎦
240 Mathematical Methods

Thus, the zero matrix


⎡0 0⎤ 0
⎢0 0⎥⎥
O = ⎢0
⎢⎣0
0 0⎥⎦
may be regarded as the identity element for addition for the matrix
⎡3 1 2⎤
⎢2 1 4⎥⎥

⎢⎣4 5 6⎥⎦
Definition 7.12 The negative of a matrix A, denoted as –A, is the matrix
whose each element is the negative of each element of the matrix A.
If
⎡− 3 1 2⎤
⎢ −1 − 4⎥⎥
A= ⎢ 2
⎢⎣ 4 5 − 6⎥⎦
then the negative of the matrix A is
⎡ 3 −1 − 2⎤
⎢ 4⎥⎥
– A = ⎢− 2 1
⎢⎣− 4 −5 6⎦⎥

Definition 7.13 If A and B are matrices of the same order such that
A + B = 0, then B is known as the additive inverse of A, where O denotes
the zero matrix. Thus we have, B = – A.
If
⎡− 3 1 2⎤
⎢ −1 − 4⎥⎥
A= ⎢ 2
⎢⎣ 4 5 − 6⎥⎦
then
⎡ 3 −1 − 2⎤ ⎡0 0 0⎤
– A = ⎢− 2 1 4⎥ and A + (− A) = ⎢⎢0 0 0⎥⎥


⎢⎣− 4 −5 6⎥⎦ ⎢⎣0 0 0⎥⎦
Definition 7.14 If A and B are two matrices of the same order, then the
sum A + (– B) is called the difference or subtraction of A and B and is
Matrices and Determinants 241

denoted by A – B.
If
⎡21 10 ⎤ ⎡210 10⎤
A= ⎢ and B = ⎢
⎣6 22⎥⎦ ⎣ 6 2 ⎥⎦

⎡−189 0 ⎤
then A – B = A + (– B) = ⎢
⎣ 0 20⎥⎦
Properties of Addition of Matrices
If A, B and C are m × n matrices, then the sum of matrices has the
following properties:
(i) Closure Law. The sum of two matrices A and B is also a matrix
whose order is same as that of A and B.
(ii) Associative Law. (A + B) + C = A + (B + C ).
(iii) Commutative Law. A + B = B + A.
(iv) Identity for Addition. The matrix O is the additive identity, i.e.
A + O = O + A = A.
(v) Inverse for Addition. For every matrix A, the matrix – A is the
additive inverse, i.e., A + (– A) = (– A) + A = O.

Definition 7.15 Let A be a matrix of order m × p and B, matrix of order


p × n, then the product of A and B is the matrix AB of order m × n. Thus,
for the multiplication of two matrices A and B it is necessary that the
number of columns of A must be same as the number of rows of B. The
multiplication of two matrices is illustrated as follows:
If
⎡a11 a12 a13 ⎤ ⎡b11 b12 ⎤
A = ⎢a21 a22 a23 ⎥ and B = ⎢⎢b21
⎥ b22 ⎥⎥

⎢⎣a31 a32 a33 ⎥⎦ ⎢⎣b31 b32 ⎥⎦
⎡a11b11 + a12 b21 + a13b31 a11b12 + a12 b22 + a13b32 ⎤
then AB = ⎢a21b11 + a22 b21 + a23b31 a21b12 + a22 b22 + a23b32 ⎥⎥

⎢⎣a31b11 + a32 b21 + a33b31 a31b12 + a32 b22 + a33b32 ⎥⎦

Example 7.1 If

⎡ 1 2⎤ ⎡2 1⎤
A= ⎢ ⎥ and B = ⎢
⎣−1 3⎦ ⎣1 1⎥⎦
242 Mathematical Methods

then find AB and BA.


Solution Here
⎡ 1 2⎤ ⎡2 1⎤ ⎡4 3⎤
AB = ⎢ =
⎣−1 3⎥⎦ ⎢1
⎣ 1⎥⎦ ⎢⎣1 2⎥⎦
and
⎡2 1⎤ ⎡ 1 2⎤ ⎡ 1 7⎤
BA = ⎢ ⎥⎢ =
⎣ 1 1⎦ ⎣−1 3⎥⎦ ⎢⎣0 5⎥⎦
which shows that AB ≠ BA.

Example 7.2 If
⎡1 2⎤ ⎡0 3⎤
A= ⎢ ⎥ and B = ⎢
⎣0 3⎦ ⎣1 4⎥⎦
then find AB and BA.

Solution Here
⎡1 2⎤ ⎡0 3⎤ ⎡2 11⎤
AB = ⎢ =
⎣0 3⎥⎦ ⎢⎣ 1 4⎥⎦ ⎢⎣3 12⎥⎦
and
⎡0 3⎤ ⎡1 2⎤ ⎡0 9 ⎤
BA = ⎢ =
⎣1 4⎥⎦ ⎢0
⎣ 3⎥⎦ ⎢⎣1 14⎥⎦
which yields that AB ≠ BA.

Example 7.3 If
⎡ 1 0 − 4⎤ ⎡5 8 4⎤
A= ⎢ 0 −1 2⎥ and B = ⎢⎢2
⎥ 3 2⎥⎥

⎢⎣−1 2 1⎥⎦ ⎢⎣1 2 1⎥⎦
then find AB and BA.

Solution Here

⎡1 0 0⎤
⎢ 0⎥⎥ = BA.
AB = ⎢0 1
⎢⎣0 0 1⎥⎦
Matrices and Determinants 243

Remarks
(i) Whenever AB exists, BA is not always defined. For example, if A
is 6 × 2 matrix and B is a 2 × 3 matrix, then AB is defined but BA
is not.
(ii) If AB and BA are both defined, it is always not necessary that they
should be equal. For example, if A is a 2 × 3 matrix and B is a
3 × 2 matrix, then AB is defined and is a 2 × 2 matrix; while BA is
also defined and is a 3 × 3 matrix. Thus AB ≠ BA, as AB and BA
are of different orders.
(iii) If AB and BA are defined and of the same order, even then it is not
necessary that AB = BA. Thus, in general, the multiplication of
two matrices is not commutative.
In most of the parts of this chapter, we shall be concerned with the
square matrices, i.e. the matrices whose rows and columns are same in
number. Thus, if A is a square matrix, by A2, A3, ..., we mean that AA,
(AA) A, ... We also have

Theorem 7.1 If A is a n × n matrix and I is a n × n identity matrix, then


AI = IA = A
Moreover, if for a matrix B of order n × n we have
AB = BA = A, then B = I
This theorem illustrates that I is the identity element for the multiplication
of two matrices. We shall denote a n × n matrix as In.

Example 7.4 If

⎡ 5 2⎤
A= ⎢
⎣− 3 4⎥⎦
then show that AI2 = I2 A.
Solution Here
⎡ 5 2⎤ ⎡1 0⎤ ⎡ 5 2⎤
AI2 = ⎢ =
⎣− 3 4⎥⎦ ⎢⎣0 1⎥⎦ ⎢⎣− 3 4⎥⎦
and
⎡1 0⎤ ⎡ 5 2⎤ ⎡ 5 2⎤
I2A = ⎢ =
⎣0 1⎥⎦ ⎢− 3
⎣ 4⎥⎦ ⎢⎣− 3 4⎥⎦
which shows that AI2 = I2 A.
244 Mathematical Methods

Properties of Matrix Multiplication


Matrix multiplication shares the following properties:
(i) In general the multiplication of two matrices is not commutative.
(ii) The product of two matrices can be zero without either factor being
a zero (or null) matrix.
For example, if

⎡0 p⎤ ⎡r s⎤ ⎡0 0⎤
A= ⎢ and B = ⎢ then AB = ⎢
⎣0 q ⎥⎦ ⎣0 0⎥⎦ ⎣0 0⎥⎦
while A ≠ 0, B ≠ 0 because p, q, r, s are all not zero.
Example 7.5 If

⎡1 1 −1⎤ ⎡−1 −2 −1⎤ ⎡ −1 −1 1⎤


⎢ −3 4⎥ , B = ⎢⎢ 6
⎥ 6⎥ , C = ⎢⎢ 2
⎥ − 2⎥⎥
A = ⎢2 12 2
⎢⎣3 −2 3⎥⎦ ⎢⎣ 5 10 5⎥⎦ ⎢⎣− 3 −3 3⎥⎦

then show that AB and CA are null matrices, while BA ≠ 0, AC ≠ 0.

Solution Since A, B and C are square matrices, therefore the products


AB, BA, AC and CA are defined and we have

⎡0 0 0⎤ ⎡−8 7 −10⎤
⎢ 48 60⎥⎥
AB = ⎢⎢0 0 0⎥⎥ = a null matrix, and BA = ⎢ − 42
⎢⎣0 0 0⎥⎦ ⎢⎣ 40 −35 50⎦⎥
Also, we have
⎡0 0 0⎤ ⎡ 4 − 4⎤ 4
⎢ 0⎥⎥ = a null matrix, and AC = ⎢− 20
CA = ⎢0 0
⎢ − 20 20⎥⎥
⎢⎣0 0 0⎥⎦ ⎢⎣ −16
−16 16⎥⎦
(iii) Cancellation law for the multiplication of matrices is not true as
in the case of the multiplication of real numbers. That is, for real
numbers a, b and c, ab = ac implies that b = c is the cancellation
law. This law is not true for matrix multiplication, i.e. for the
matrices A, B and C, AB = AC does not imply that B = C. Thus AB
may be equal to AC with the condition that A ≠ 0 and B ≠ C. This
has been illustrated through the following Example 7.6.
Matrices and Determinants 245

Example 7.6 If

⎡ 1 2 0⎤ ⎡1 2 3⎤ ⎡1 2 3⎤
⎢ 0⎥⎥ , B = ⎢1 −1⎥ , C = ⎢⎢1
⎥ −1⎥⎥
A= ⎢ 1 1 ⎢ 1 1
⎢⎣−1 4 0⎥⎦ ⎢⎣2 2 2⎥⎦ ⎢⎣1 1 1⎥⎦

then show that the cancellation law for matrix multiplication does not
hold.
Solution Here A ≠ 0, B ≠ C and

⎡3 4 1⎤
⎢ 2⎥⎥ = AC
AB = ⎢2 3
⎢⎣3 2 − 7⎥⎦

which shows that AB = AC implies that B ≠ C.


(iv) Matrix multiplication is associative, i.e. for the matrices A, B and
C, A(BC) = (AB)C.
(v) Matrix multiplication is distributive with respect to matrix addition/
subtraction, i.e. for the matrices A, B and C,
A(B + C) = AB + AC and A(B – C) = AB – AC
(vi) If A is any square matrix and I is the identity matrix, then it is
possible to form polynomials in A. Thus for any polynomial
f (x) = a0 + a1 x + a2 x 2 + ... + a n x n
f (A) is defined by the matrix
f (A) = a0 I + a1 A + a 2 A 2 + ... + a n A n
If f (A) is the zero matrix, then A is the root of the polynomial f (x).

Example 7.7 If
⎡1 2⎤
A= ⎢ ⎥
⎣4 − 3⎦
then find (i) A 2 and A 3 (ii) f (A), where f (x) = 2x 3 – 4x + 5. Also, show
that A is the root of the polynomial g (x) = x 2 + 2x – 11.

Solution (i) Here

⎡ 9 − 4⎤ ⎡− 7 30⎤
A2 = ⎢ ⎥ and A3 = ⎢
⎣−8 17⎦ ⎣ 60 − 67⎥⎦
246 Mathematical Methods

(ii) Given that f (x) = 2x 3 – 4x + 5 then

⎡−13 52⎤
f (A) = 2 A 3 – 4A + 5I = ⎢
⎣104 −117⎥⎦
Moreover, replacing x by A and 11 by 11I in g (x), we get

⎡0 0⎤
g (A) = A 2 + 2A – 11I = ⎢
⎣0 0⎥⎦
Since g (A) is a zero matrix, therefore A is the root of the polynomial
g (x).

Definition 7.16 The matrix which is obtained by interchanging the rows


and columns of a given matrix A is called the transpose of A and is
denoted by A T or A ′.
For example, if

⎡1 2⎤ ⎡1 4⎤
A= ⎢ ⎥ then AT = ⎢
⎣4 − 3⎦ ⎣2 − 3⎥⎦
Remarks
(i) If A is any matrix, then
(A T ) T = A
(ii) If A and B are any two matrices, then
(A + B) T = A T + B T
(iii) If A is a m × p matrix and B is p × n matrix, then
(AB) T = B T A T
(iv) If A is a matrix and k is a scalar, then
(kA) T = k A T

Definition 7.17 A square matrix A = [aij] is symmetric if its (i, j)th element
is same as the ( j, i)th element. Thus A is symmetric if aij = aji for every
i and j.
For example,

⎡1 2 3⎤
⎡1 2⎤
A= ⎢ and B = ⎢⎢2 1 −1⎥⎥
⎣2 3⎥⎦
⎢⎣3 −1 1⎥⎦

are symmetric matrices. Also, A = A ′ and B = B′.


Matrices and Determinants 247

Remarks
(i) It may be noted that the matrix which is symmetric about the
principal diagonal is a symmetric matrix and the elements on one
side of the principal diagonal are the reflected images of the
elements on the other side of the principal diagonal.
(ii) Symmetric matrices are always square matrices.
(iii) The matrix A is symmetric if and only if it is equal to its transpose,
i.e. A = A ′.
(iv) Diagonal matrices are always symmetric.

Definition 7.18 A square matrix A is anti-symmetric or skew-symmetric


if the (i, j)th element of A is the negative of the ( j, i)th element of A.
Thus, a matrix A = [a ij] is skew-symmetric if aij = – aji, for every i and j.

Remarks
(i) If the matrix A is skew-symmetric, then aii = 0. Thus each element
on the principal diagonal of an anti symmetric matrix is zero. For
example, the matrix
⎡0 −1 −3⎤
⎢1 0 4⎥⎥

⎢⎣3 −4 0⎥⎦
is a skew-symmetric matrix.
(ii) The necessary and sufficient condition for a skew-symmetric matrix
A is that A ′ = – A.
(iii) A matrix which is both symmetric and anti symmetric is a square
null matrix.
For symmetric and skew-symmetric matrices, we have

Theorem 7.2 If A is any square matrix, then A + A′ is symmetric and


A – A′ is skew-symmetric.

Theorem 7.3 Every square matrix can uniquely be expressed as the sum
of a symmetric matrix and a skew-symmetric matrix.
Thus, if A is any square matrix, then
1 1
A= ( A + A′) + ( A − A′) = S + T
2 2
where S = 12 (A + A′) and T = 12 (A – A′) are the symmetric and skew-
symmetric parts of the matrix A.
248 Mathematical Methods

7.3 Determinants
Let A be any square matrix whose elements are real numbers. A real
number associated with this matrix A is called the determinant of A and
is denoted by ∆ A, or | A|, or det (A). Thus det (A) is a function the domain
of which is the set of all square matrices having elements as real numbers
and range is the set of all real numbers. If A is a square matrix of order n,
then its determinant is also of order n. The determinant of the matrix

⎡ a1 b1 ⎤
A= ⎢ is the number (a1 b2 – a2 b1)
⎣a2 b2 ⎥⎦
We shall denote the determinant of the matrix

⎡ a1 b1 ⎤
A= ⎢
⎣a2 b2 ⎥⎦
as

⎡a b1 ⎤ a1 b1
∆⎢ 1 = = a 1b 2 – a2b1
⎣a2 b2 ⎥⎦ a2 b2
It may be noted that a matrix is simply an arrangement of numbers
enclosed between square brackets, while its determinant is not an
arrangement of numbers enclosed between two vertical lines. It is a scalar
quantity having some definite value. Thus, if

⎡4 7⎤
A= ⎢
⎣6 2⎥⎦
then
4 7
det A = = (4) (2) – (7) (6) = 8 – 42 = – 34
6 2
Here, the order of the determinant is 2. Also, the determinant of a matrix
[a], having only one element, is the number a itself. This is a determinant
of order one.
If

⎡ a1 b1 c1 ⎤
⎢ c2 ⎥⎥
A = ⎢a2 b2
⎢⎣a3 b3 c3 ⎥⎦
Matrices and Determinants 249

is a 3 × 3 matrix, then
a1 b1 c1
det A = a2 b2 c2
a3 b3 c3
The order of this determinant is 3. The determinants of orders 4 and n,
thus, respectively, have the forms
a11 a12 a13 " a1n
a11 a12 a13 a14 a21 a22 a23 " a2 n
a21 a22 a23 a24 a31 a32 a33 " a3n
and
a31 a32 a33 a34 " " " "
a41 a42 a43 a44 " " " "
an1 an 2 an 3 " ann
The element aij in a determinant of order n occurs in the ith row and jth
column.
The value of the determinant of order 3 is given by
a1 b1 c1
b2 c2 a2 c2 a2 b2
a2 b2 c2 = a1 − b1 + c1
b3 c3 a3 c3 a3 b3
a3 b3 c3
= a1(b2c3 – b3c2) – b1(a2c3 – a3c2) + c1(a2b3 – a3b2)
Example 7.8 Find the value of
3 2 1
0 1 −8
0 −5 7
Solution We have
3 2 1
1 −8 0 −8 0 1
0 1 −8 = 3 −2 +1 = – 99
−5 7 0 7 0 −5
0 −5 7
Example 7.9 Find the value of
a h g
h b f
g f c
250 Mathematical Methods

Solution Here
a h g
b f h f h b
h b f =a −h +g
f c g c g f
g f c
= a(bc – f 2) – h (ch – f g) + g(h f – bg) = abc + 2f gh – a f 2 – bg 2 – ch2

Definition 7.19 The minor of an element in a determinant is a determinant


which is left after removing the row and the column which intersect the
element, and is of order one less than that of the given determinant.
For the determinant
a1 b1 c1
a2 b2 c2
a3 b3 c3
b2 c2 a2 c2
the minor of a1 is , the minor of b1 is , the minor of
b3 c3 a3 c3
a2 b2
c1 is , and so on.
a3 b3

Definition 7.20 If the minor of an element of a determinant has an


appropriate sign, we then have the cofactor of that element. Thus, the
cofactor of the element aij is defined as
cofactor of aij = (– 1)i+ j Mij
where Mij is the minor of aij and aij is the element occuring in the ith row
and jth column of the given determinant.

Example 7.10 Find the cofactors of the elements in the third row of the
determinant
−2 4 2
1 5 −3
5 −2 2
Solution The elements in the third row are 5, – 2, 2. We know that
cofactor of aij = (– 1)i + j Mij
Therefore
4 2
cofactor of 5 = (– 1)3+1 = (1) (– 12 – 10) = – 22
5 −3
Matrices and Determinants 251

−2 2
cofactor of – 2 = (– 1)3+2 = (– 1) (6 – 2) = – 4
1 −3
−2 4
cofactor of 2 = (– 1)3+3 = (1) (– 10 – 4) = – 14
1 5

7.3.1 Properties of the Determinants


The determinant of a matrix has a number of properties. These properties
are helpful in calculations and are mentioned as follows:
1. If each entry in any row, or column of a determinant is zero, then the
value of the determinant is zero. For example
−2 4 2
0 0 0 = – 2 (0 × 2 – 0 × – 2) – 4(0 × 2 – 0 × 5)
5 −2 2
+ 2(0 × – 2 – 0 × 5) = 0
2. If rows are changed into columns and columns into rows, then the
determinant remains unchanged. For example
3 2 1 3 0 0
0 1 −8 = − 99 and 2 1 −5 = – 99
0 −5 7 1 −8 7
which shows that
3 2 1 3 0 0
0 1 −8 = 2 1 −5
0 −5 7 1 −8 7
Thus we have
a1 b1 c1 a1 a2 a3
a2 b2 c2 = b1 b2 b3
a3 b3 c3 c1 c2 c3
3 (i). If any two rows (or columns) of a determinant are interchanged,
then the resulting determinant is the negative of the original determinant.
That is
a1 b1 c1 b1 a1 c1
a2 b2 c2 = − b2 a2 c2
a3 b3 c3 b3 a3 c3
252 Mathematical Methods

(ii). If any line of a determinant ∆ be passed over in parallel lines, the


resulting determinant ∆′ = (– 1) m ∆. For example, if

a1 b1 c1 b1 c1 a1
∆ = a2 b2 c2 and ∆′ = b2 c2 a2
a3 b3 c3 b3 c3 a3

then ∆′ = (– 1)2 ∆ = ∆.
4. If two rows (or two columns) in a determinant are identical, then the
determinant is zero. That is

a1 b1 c1
a1 b1 c1 = 0
a2 b2 c2

5 (i). If all the elements of any one row (or column) are multiplied by the
same number, then the determinant is multiplied by that number. That
is, if

a1 b1 c1 ka1 kb1 kc1


∆ = a2 b2 c2 and ∆′= a2 b2 c2
a3 b3 c3 a3 b3 c3

then ∆′ = k∆.
(ii). If two rows or columns are such that the elements of one are equi-
multiples of the elements of the other, then the determinant is equal to
zero.
6. If each entry in a row or column of a determinant is written as the sum
of two or more terms, then the determinant can be written as the sum of
two or more determinants. That is

a1 + x1 b1 c1 a1 b1 c1 x1 b1 c1
a2 + x2 b2 c2 = a2 b2 c2 + x2 b2 c2
a3 + x3 b3 c3 a3 b3 c3 x3 b3 c3

7. If each entry of one row (or column) of a determinant is multiplied by


a real number k and the resulting product is added to the corresponding
entry in another row (or column) in the determinant, then the resulting
determinant is equal to the original determinant. Thus, if
Matrices and Determinants 253

a1 b1 c1 a1 b1 c1
∆ = a2 b2 c2 ′
and ∆ = a2 b2 c2
a3 b3 c3 a3 + ka1 b3 + kb1 c3 + kc1

then ∆ = ∆′.
8. If to each element of a row or column of a determinant the equi-
multiples of the corresponding elements of one or more parallel lines
(either rows or columns) are added, then the determinant remains
unchanged. That is
a1 + pa2 + qa3 a2 a3 a1 a2 a3
b1 + pb2 + qb3 b2 b3 = b1 b2 b3
c1 + pc2 + qc3 c2 c3 c1 c2 c3
Notation By R2 → R2 + aR3 + bR5 we mean that we have added a times
third row and b times the fifth row to the 2nd row: and similarly, the
operation C4 → C4 + xC2 + yC3 will mean that we have added x times
second column and y times third column to the fourth column.
9. If the elements of a determinant which involves x are polynomials in
x, and if the determinant is equal to zero when a is substituted for x, then
x – a is the factor of the determinant. For example, if in the determinant
a b c
2 2
∆= a b c2
bc ca ab
a is substituted for b, then it takes the form
a a c
2 2
a a c2
∆=
ac ca a2
and therefore ∆ = 0. Thus, a – b is a factor of the determinant ∆; similarly
b – c and c – a are also factors of ∆.

Definition 7.21 A transformation of any of the following form is called


an elementry transformation:
(i) interchange of two rows or two columns.
(ii) multiplication of a row or column of a determinant by a constant
different from zero.
254 Mathematical Methods

(iii) addition to the entries of a line (row or column) of the determinant


the constant multiples of a parallel line (row or column).

Definition 7.22 Two determinants are said to be equivalent if one can be


obtained from the other by a finite number of elementary transformations.
Example 7.11 Show that

0 ab 2 ac 2
a 2b 0 bc 2 = 2a3b3c3
a2c b2 c 0

Solution We have

0 ab 2 ac 2 0 b2 c2
a 2b 0 bc 2 = abc a 2 0 c2
a2c b2c 0 a2 b2 0

0 1 1
= abc(a 2b 2c 2) 1 0 1 = 2a 3b 3c 3
1 1 0

7.3.2 Application of Determinants


The determinant of a matrix has a number of applications in social
sciences, business and economics. Here, we shall mention that how the
determinants can be used to find the area of a triangle and to find the
solution of a system of linear equations.

(i) Area of a Triangle


If (x1, y1), (x2, y2) and (x3, y3) are the vertices of a triangle, then its area
is given by the determinant

x1 y1 1
1
x2 y2 1
2
x3 y3 1

It may be noted that if the points A(x1, y1), B(x2, y2) and C(x3, y3) are in
straight line, then the area of the triangle ABC is zero, and conversely.
Matrices and Determinants 255

This is known as the condition of collinearity of three points.

Example 7.12 Find the value of λ so that the points (5, 5), (λ, 1) and
(10, 7) are collinear.

Solution We know that the points A(5, 5), B(λ, 1) and C(10, 7) lie on a
straight line only when the area of the triangle ABC is zero. Thus
5 5 1
1
λ 1 1 =0
2
10 7 1
which on solving leads to λ = – 5.

(ii) Solutions of Linear Equations


If there is a the system of n linear equations in n unknown given by
a11x1 + a12x2 + ... + a1nxn = b1
a21x1 + a22x2 + ... + a2nxn = b2
...
...
an1x1 + an2x2 + ... + annxn = bn
then the following theorem, known as Cramer’s rule, is used to find the
solutions of a system of linear equations.
Theorem 7.4 If
a11 a12 " a1n
a21 a22 " a2 n
D= . . . .
. . . .
an1 an 2 " ann
and Dj is the determinant obtained from D after replacing jth column by

b1
b2
. D1 D D
. then x1 = , x2 = 2 , ..., xn = n , where D ≠ 0.
D D D
bn

Thus, the solution of the simultaneous equations


256 Mathematical Methods

a11 x + a12 y = b1
a21 x + a22 y = b2
is given by
D1 D
x= ,y= 2
D D
a11 a12 b1 a12 a11 b1
where D = , D1 = and D2 = . In a
a21 a22 b2 a22 a21 b2
similar way, the solution of the simultaneous equations
a11 x + a12 y + a13 z = b1
a21 x + a22 y + a23z = b2
a31 x + a32 y + a33z = b3
is given by
D1 D D
x= , y = 2, z = 3
D D D
where
a11 a12 a13 b1 a12 a13
D = a21 a22 a23 , D1 = b2 a22 a23
a31 a32 a33 b3 a32 a33
a11 b1 a13 a11 a12 b1
D2 = a21 b2 a23 , D3 = a21 a22 b2
a31 b3 a33 a31 a32 b3
Remarks
(i) If D ≠ 0, then the solution of the above equation is unique and finite.
Simultaneous linear equations having finite and unique solutions are
called consistent.
(ii) If D = 0, then the system of equations have no real solutions and in
such cases the equations are said to be inconsistent. We also have
(a) If D = 0 and atleast one of the determinants D1, D2 and D3 is non-
zero, then the given system of equations is inconsistent.
(b) If d = D1 = D2 = D3 = 0, then the system of equations may or may not
be consistent as in this case, the values of x, y and z assume the
indeterminate form. If the indeterminate form has a unique finite solution,
then the equations are consistent. If it has a finite number of solutions,
then the equations are called dependent. If a system of equations is
Matrices and Determinants 257

dependent, then D = 0 and all of the determinants D1, D2, ..., Dn are
zero.

Example 7.13 Determine whether the system of equations


x – 3y + 2z = 4
2x + y – 3z = – 2
4x – 5y + z = 5
is consistent.

Solution We have
1 −3 2 4 −3 2
D= 2 1 − 3 = 0, D1 = − 2 1 −3 = – 7
4 −5 1 5 −5 1
Hence, D = 0 and atleast one of the determinants D1, D2, D3 is not zero,
and the system of equations is inconsistent.

Example 7.14 Solve the system of equations (using Cramer’s rule)


– 4x + 2y – 9z = 2
3x + 4y + z = 5
x – 3y + 2z = 8

Solution Here
−4 2 −9 2 2 −9
D = 3 4 1 = 63, D1 = 5 4 1 = 441
1 −3 2 8 −3 2

−4 2 −9 −4 2 2
D2 = 3 5 1 = −189, D3 = 3 4 5 = – 252
1 8 2 1 −3 8
Therefore
D1 D D
x= = 7, y = 2 = −3, z = 3 = – 4
D D D

7.4 Inverse of a Matrix


The theory of determinants is also used in finding the inverse of a square
matrix. But before discussing the inverse of a matrix, we have the
258 Mathematical Methods

following definitions and the related results.

Definition 7.23 A square matrix A is called singular if det A = 0, otherwise


it is a nonsingular.

Definition 7.24 If A and B are square matrices such that AB = BA = I,


then B is called the inverse of A and we write it as A – 1 = B; and similarly,
A is called the inverse of B and is written as B – 1 = A. Thus
A A– 1 = A–1 A = I
A non-zero square matrix A is said to be invertible if there is a square
matrix B such that AB = BA = I.
It may be noted that the inverse of a matrix exists only when the
matrix is a square matrix. Also, an invertible matrix has a unique inverse.

Definition 7.25 The minor of an element in a square matrix A is the


determinant which is left after removing the row and the column which
intersect the element, and is of order one less than that of the given
determinant.
For the matrix
⎡ a1 b1 c1 ⎤
⎢ c2 ⎥⎥
A = ⎢ a2 b2
⎢⎣ a3 b3 c3 ⎥⎦

b2 c2 a2 c2
the minor of a1 is , the minor of b1 is , the minor of c1
b3 c3 a3 c3
a2 b2
is , and so on.
a3 b3
Definition 7.26 If the minor of an element of a square matrix A has an
appropriate sign, we then have the cofactor of that element. Thus, the
cofactor Aij of the element aij is defined as
cofactor Aij of aij = (– 1)i + j Mij
where Mij is the minor of aij and aij is the element occuring in the ith row
and jth column of the given matrix.
Thus for the matrix
⎡ a11 a12 a13 ⎤
⎢ a23 ⎥⎥
A = ⎢ a21 a22
⎢⎣ a31 a32 a33 ⎥⎦
Matrices and Determinants 259

a22 a23
cofactor of a11 = A11 = (– 1)1+1 M11 =
a32 a33
a21 a23
cofactor of a12 = A12 = (– 1)1+2 M12 = –
a31 a33
a12 a13
cofactor of a31 = A31 = (– 1)3+1M31 =
a22 a23
a11 a13
cofactor of a32 = A32 = (–1)3+2 M32 = –
a21 a23
Definition 7.27 The transpose of the matrix of the cofactors of the
elements ai j of A is called the adjoint of A and is denoted by adj A. Thus
for the matrix
⎡ a11 a12 a13 ⎤
⎢ a23 ⎥⎥
A = ⎢ a21 a22
⎢⎣ a31 a32 a33 ⎥⎦
⎡ A11 A21 A31 ⎤
adj A = ⎢ A12 A22 A32 ⎥
⎢ ⎥
⎢⎣ A13 A23 A33 ⎥⎦
For the adjoints of square matrices, we have

Theorem 7.5 For any square matrix A,


A(adj A) = (adj A) A = | A | I
where | A | denotes the determinant of the matrix A.
It may be noted that adj I = I and adj O = O, where I and O denote,
respectively, the identity and null matrices.

Theorem 7.6 If A is a nonsingular square matrix of order n, then


| adj A | = | A | n – 1

Theorem 7.7 If A and B are two nonsingular square matrices of same


order, then
adj (AB) = (adj B) (adj A)

Theorem 7.8 If A is a square matrix of order n, then


1
A–1 = (adj A)
| A|
260 Mathematical Methods

Remarks
(i) If A is nonsingular, then adj A = | A | A–1.

1
(ii) For the square matrix A, (adj A –1) = A.
| A|
(iii) If A is a nonsingular matrix of order 3 and k is any non-zero real
1 –1
number, then (kA) – 1 = A .
k
⎡0 0 1⎤
(iv) The inverse of the matrix ⎢ 0 1 0 ⎥⎥ is itself.

⎢⎣ 1 0 0 ⎥⎦
Theorem 7.9 If A and B are two nonsingular matrices, then AB is also
nonsingular and
(i) (AB) – 1 = B –1 A – 1 (ii) (A T ) – 1 = (A –1)T (iii) [(A – 1)T ] – 1 = AT

Example 7.15 Find the inverse of the matrix


⎡ −2 6 4⎤
A = ⎢⎢ 1 −3 2 ⎥⎥
⎢⎣ 1 5 2 ⎥⎦
Solution Here
⎡ −2 6 4⎤
⎢ 1 −3 2 ⎥
|A| = ⎢ ⎥ = 64 ≠ 0
⎢⎣ 1 5 2 ⎥⎦
which shows that the matrix A is nonsingular and thus A – 1 exists. Now
A11 = – 16, A12 = 0, A13 = 8, A21 = 8, A22 = – 8, A23 = 16, A31 = 24, A32 =
8, A33 = 0 so that the matrix of the cofactors of A is
⎡ −16 0 8⎤
⎢ 8 −8 16 ⎥
⎢ ⎥
⎢⎣ 24 8 0 ⎥⎦
and thus the adjoint of A (which is the transpose of the above matrix) is
⎡ −16 8 24 ⎤
⎢ −8 8 ⎥⎥
adj A = ⎢ 0
⎢⎣ 8 16 0 ⎥⎦
Matrices and Determinants 261

Therefore, inverse of the matrix A is


⎡ − 16 8 24 ⎤
1 ⎢
8 ⎥⎥
1
A– 1 = (adj A) = 0 −8
| A| 64 ⎢
⎣⎢ 8 16 0 ⎦⎥

7.5 Solution of a Matrix Equation


Once the inverse of a matrix is known, we can solve the matrix equation
AX + B = C for the matrix X, where A, B and C are matrices of the same
order. For example, if A and B are matrices of the same order, then to
solve the matrix equation AX = B, we multiply both sides of this equation,
from the left, by A – 1 to get
A – 1 AX = A – 1B ⇒ IX = A – 1B ⇒ X = A – 1B
While on the other hand, the solution of the matrix equation AX + B = C
for X can be obtained through the following steps.
(i) Add (– B) on both sides of the equation AX + B = C to get
AX + B + (– B) = C – B ⇒ AX + 0 = C – B ⇒ AX = C – B
(ii) Now multiply both the sides of equation AX = C – B, from the left,
by A – 1 so that
A – 1 AX = A – 1 (C – B) ⇒ IX = A – 1(C – B) ⇒ X = A – 1(C – B)

Example 7.16 Solve the matrix equation AX = B for X, where


⎡3 1⎤ ⎡5 −1 ⎤
A = ⎢ ⎥ and B = ⎢
⎣4 1⎦ ⎣2 3 ⎥⎦
⎡ −1 1 ⎤
Solution The inverse of the matrix A is A−1 = ⎢ ⎥
⎣ 4 −3⎦
Thus
⎡ −1 1⎤ ⎡ 3 1⎤ ⎡ −1 1⎤ ⎡ 5 −1 ⎤
⎢ 4 X =⎢
⎣ − 3 ⎥⎦ ⎢⎣ 4 1 ⎥⎦ ⎣ 4 − 3 ⎥⎦ ⎢⎣ 2 3 ⎥⎦
which leads to
⎡1 0⎤ ⎡ −3 4⎤ ⎡ −3 4⎤
⎢0 ⎥ X =⎢ ⎥ ⇒ X =⎢
⎣ 1⎦ ⎣ 14 −13 ⎦ ⎣ 14 −13 ⎥⎦
Example 7.17 Solve the matrix equation AX + B = C for X, where

⎡ 2 1⎤ ⎡ −5 0⎤ ⎡3 −9 ⎤
A= ⎢ ,B=⎢ and C = ⎢
⎣ −3 2 ⎥⎦ ⎣ 2 4 ⎥⎦ ⎣7 1 ⎥⎦
262 Mathematical Methods

Solution Here we have to solve the equation


⎡ 2 1⎤ ⎡ −5 0 ⎤ ⎡ 3 − 9 ⎤
⎢ −3 2 ⎥ X + ⎢ 2 4 ⎥ = ⎢ 7 1 ⎥⎦
⎣ ⎦ ⎣ ⎦ ⎣
The solution of the equation AX + B = C, from the method given above,
is X = A – 1(C – B). But the inverse of A is
⎡ 2/7 −1/7 ⎤
A–1 = ⎢
⎣ 3/7 2/7 ⎥⎦
Thus
⎡ 2/7 −1/7 ⎤ ⎡8 − 9 ⎤ ⎡ 11/7 −15/7 ⎤
X= ⎢ =
⎣ 3/7 2/7 ⎥⎦ ⎢5
⎣ −3 ⎥⎦ ⎢⎣ 34/7 − 33/7 ⎥⎦

7.6 Solution of Linear Equations


The theory of matrices can also be used to find the solution of simultaneous
linear equations in two and three variables etc. Let
a1 x + b1 y = c1
a2 x + b2 y = c2
be two simultaneous equations in two variables x and y. These equations
when expressed in the matrix notation takes the form
⎡ a1 b1 ⎤ ⎡ x ⎤ ⎡ c1 ⎤
⎢a ⎥ ⎢ ⎥ = ⎢ ⎥ or AX = B
⎣ 2 b2 ⎦ ⎣ y ⎦ ⎣ c2 ⎦
where A is a 2 × 2 matrix and X and B are 2 × 1 column matrices.
Similarly, the system of equations in three variables x, y and z
a1 x + b1 y + c1z = d1
a2 x + b2 y + c2z = d2
a3 x + b3 y + c3z = d3
can be expressed in the matrix form as
⎡ a1 b1 c1 ⎤ ⎡ x ⎤ ⎡ d1 ⎤
⎢a ⎥⎢ ⎥ ⎢ ⎥
⎢ 2 b2 c2 ⎥ ⎢ y ⎥ = ⎢ d 2 ⎥ or AX = B
z d
⎣⎢ a3 b3 c3 ⎦⎥ ⎣ ⎦ ⎣ 3 ⎦
where A is a 3 × 3 matrix and X and B are 3 × 1 column matrices.

Remarks
Consider equation AX = B and recall that equations having one or more
solutions are called consistent equations, we then have
Matrices and Determinants 263

(i) If | A | ≠ 0, the system is consistent and has a unique solution.


(ii) If | A | = 0, the system AX = B has either no solution or an infinite
number of solutions.
(iii) (a) If (adj A) B ≠ 0, the system has no solution and is therefore
inconsistent.
(b) If (adj A) B = 0, the system is consistent and has infinitely many
solutions and in such cases, the equations are dependent.

Example 7.18 Solve the system of equations


4x – 3y = 11
3x + 7y = – 1
by matrix method.
Solution The given equations can be written in the matrix form as
⎡ 4 − 3 ⎤ ⎡ x ⎤ ⎡ 11 ⎤
⎢3 =
⎣ 7 ⎥⎦ ⎢⎣ y ⎥⎦ ⎢⎣ −1 ⎥⎦
which may be expressed as AX = B, where
⎡ 4 −3 ⎤ ⎡x⎤ ⎡ 11 ⎤
A= ⎢ ⎥ , X = ⎢ ⎥ and B = ⎢ ⎥
⎣3 7⎦ y
⎣ ⎦ ⎣ −1 ⎦
We also have, | A | = 37 ≠ 0, that is, A is nonsingular and thus the inverse
of A exists. Moreover, since | A | ≠ 0, the system AX = B has a unique
solution and we have X = A – 1 B, i.e.
⎡ x ⎤ ⎡ 7/37 3/37 ⎤ ⎡ 11 ⎤ ⎡ 74/37 ⎤ ⎡ 2 ⎤
⎢⎣ y ⎥⎦ = ⎢ −3/37 4/37 ⎥ ⎢⎣ −1 ⎥⎦ = ⎢ −37/37 ⎥ = ⎢ −1 ⎥
⎣ ⎦ ⎣ ⎦ ⎣ ⎦
Therefore x = 2 and y = – 1.

Example 7.19 Using the matrix method, solve the system of equations
5x + 3y + z = 16
2x + y + 3z = 19
x + 2y + 4z = 25

Solution The given system of equations can be expressed as AX = B,


where
⎡5 3 1⎤ ⎡x⎤ ⎡ 16 ⎤
⎢ 3 ⎥⎥ , X = ⎢ y ⎥ and B = ⎢ 19 ⎥
A = ⎢2 1
⎢z ⎥ ⎢ 25 ⎥
⎣⎢ 1 2 4 ⎦⎥ ⎣ ⎦ ⎣ ⎦
264 Mathematical Methods

Here | A | = – 22 ≠ 0 which shows that the given system of equations has


a unique solution. Moreover, since the matrix A is nonsingular, the inverse
of A exists and we have
⎡ 2/22 10/22 −8/22 ⎤
= ⎢ 5/22
A–1 −19/22 13/22 ⎥⎥

⎢⎣ −3/22 7/22 1/22 ⎥⎦
– 1
Thus X = A B leads to
⎡ x ⎤ ⎡ 2/ 22 10/ 22 −8/ 22 ⎤ ⎡ 16 ⎤ ⎡ 1 ⎤
⎢ y ⎥ = ⎢ 5/ 22 −19/ 22 13/ 22 ⎥ ⎢ 19 ⎥ = ⎢ 2 ⎥
⎢ ⎥ ⎢ ⎥⎢ ⎥ ⎢ ⎥
⎢⎣ z ⎥⎦ ⎢⎣ −3/ 22 7/ 22 1/22 ⎥⎦ ⎢⎣ 25 ⎥⎦ ⎢⎣ 5 ⎥⎦
which implies that x = 1, y = 2, z = 5.

7.7 Leontief Input-output Analysis


The matrices and their inverses can be used in studying a relatively new
branch of applied mathematics known as input-output analysis. This
study, mainly made by Wassily Leontief who was awarded Noble Prize
in economics (1973) for his poineering work on economic planning for
industrialized countries. He made a comprehensive study that how 500
sectors of American economy interacted with each other. The input-output
analysis can also be used in the theory of automatic control and
servomechanics (for further details, see [3]).
Input-output analysis deals with the establishment of equilibrium
conditions under which industries in an economy have appropriate output
to satisfy the demands of each other in addition to final (outside) demands.
The basic input-output problem can be stated as
Given the internal demands for each industry’s output, determine the
output levels for the various industries that will meet a given final
(outside) level of demand as well as the internal demands.
Here, we shall consider a two-industry model. These considerations can
be generalized to a multi-industry-model. Let there be two industries,
viz. electric company E and water company W. Outputs for E and W are
measured in rupees. We first form a matrix, known as technology matrix,
the entries of which represent how much each industry uses the outputs
of other industries as well as its own for its own output. In our case, the
industry E will use both electricity and water as input for its own output,
and the industry W will use both electricity and water as input for its
own output.
Matrices and Determinants 265

Let the worth of each rupee of E’s output requires Rs. a11 of its own
output and Rs. a21 of W’s output, and each rupee’s worth of W’s output
requires Rs. a12 of E’s output and Rs. a22 of its own output. These
requirements can conveniently be expressed in the following technology
matrix:
⎡ a11 a12 ⎤
M= ⎢ ⎥
⎣ a21 a22 ⎦
Here, the first column indicates that each rupee of E’s output requires
Rs. a11 of its own output as input and Rs. a21 of W ’s output as input. The
second column has a similar interpretation. The first row indicates that
Rs. a11 of electricity is needed to produce a rupee’s worth of electricity,
and Rs. a12 of electricity is needed to produce a rupee’s worth of water. A
similar interpretation can be assigned to the second row.
If d1 and d2 represent the final demand (the demand from the outside
sector), then the problem is to find what rupee outputs, Rs. x1 from the
company E and Rs. x2 from the company W, are required to meet these
final demands? In terms of the matrix notation, we have
⎡d ⎤
Final demand matrix D = ⎢ 1 ⎥
⎣ d2 ⎦
where d1 and d2 are measured in millions of rupees and
⎡x ⎤
Output matrix X = ⎢ 1 ⎥
⎣ x2 ⎦
We can thus state our problem as “If the technology matrix M and
the final demand matrix D are given, then find the output matrix X.”
The above considerations can now be stated as
Total output from E = (Input required by E from E) + (Input required by
W from E) + (Final demand from E)
and
Total output from W = (Input required by E from W) + (Input required by
W from W) + (Final demand from W)
Converting these equations into symbolic form, we have
x1 = a11 x1 + a12 x2 + d1 (1a)
x2 = a21 x1 + a22 x2 + d2 (1b)
Equation (1) can also be expressed as
⎡ x1 ⎤ ⎡ a11 a12 ⎤ ⎡ x1 ⎤ ⎡ d1 ⎤
⎢x ⎥ = ⎢a +
a22 ⎥⎦ ⎢⎣ x2 ⎥⎦ ⎢⎣ d 2 ⎥⎦
⎣ 2 ⎦ ⎣ 21
266 Mathematical Methods

or
X = MX + D (2)
which may be expressed as
X – MX = D ⇒ IX – MX = D ⇒ (I – M ) X = D
so that
X = (I – M ) – 1 D (3)
Thus, our problem reduces to find the solution of the matrix
equation (3).

Example 7.20 Solve Eq. (3) for the above two-industry model with the
following data:
a11 = 0.1, a12 = 0.4, a21 = 0.3, a22 = 0.2; d1 = 12 and d2 = 6

Solution Here, the technology matrix M and the final demand matrix D,
respectively, are

⎡ 0.1 0.4 ⎤ ⎡ 12 ⎤
M = ⎢ and D = ⎢ ⎥
⎣ 0.3 0.2 ⎥⎦ ⎣ 6⎦
Since

⎡1 0⎤
I= ⎢
⎣0 1 ⎥⎦
we have

⎡ 0.9 − 0.4 ⎤
I–M= ⎢
⎣ − 0.3 0.8 ⎥⎦
which leads to | I – M | = 0.6 ≠ 0. Thus, the inverse of I – M exists and
we have

1 1 ⎡ 0.8 0.4 ⎤ ⎡ 4/3 2/3 ⎤


(I – M ) – 1 = adj ( I − M ) = =
|I −M | 0.6 ⎢⎣ 0.3 0.9 ⎥⎦ ⎢⎣ 1/2 3/2 ⎥⎦
Therefore, from Eq. (3), we have

⎡ x1 ⎤ ⎡ 4/3 2/3 ⎤ ⎡ d1 ⎤ ⎡ 4/3 2/3 ⎤ ⎡ 12 ⎤ ⎡ 20 ⎤


⎢ x ⎥ = ⎢ 1/2 = = (4)
⎣ 2⎦ ⎣ 3/ 2 ⎥⎦ ⎢⎣ d 2 ⎥⎦ ⎢⎣ 1/ 2 3/2 ⎥⎦ ⎢⎣ 6 ⎥⎦ ⎢⎣ 15 ⎥⎦
Hence, the electric company E must have an output of Rs. 20 million
and the water company W have an output of Rs. 15 million so that each
company can meet both internal and final demands.
Matrices and Determinants 267

Remarks
1. It may be noted that Eq. (4) provides the solution of the original problem
for arbitrary final demands d1 and d2. In fact, Eq. (4) gives a solution not
only for the stated final demands but also to the original problem for
various other projected demands. Thus, if in the original problem the
projected final demands for 5 years from now were d1 = 18 and d2 = 12,
then from Eq. (4), we can determine the output of each company for this
projected demand, and we have
⎡ x1 ⎤ ⎡ 4/3 2/3 ⎤ ⎡ 18 ⎤ ⎡ 32 ⎤
⎢ x ⎥ = ⎢ 1/2 3/ 2 ⎥ ⎢ 12 ⎥ = ⎢ 27 ⎥
⎣ 2⎦ ⎣ ⎦⎣ ⎦ ⎣ ⎦
2. The advantage of expressing the input-output analysis in matrix form
(2) is that the matrix equation (2) and its solution (3) are the same for a
three-industry economy, a four-industry economy, or an economy with n
industries, where n is a natural number.

EXERCISES
1. Find the value of X such that

⎡ 1 3⎤ ⎡4 −1 ⎤ ⎡ ⎡3 −4 ⎤ ⎤
X+ ⎢ = ⎢ Ans. ⎢ ⎥
⎣ −1 2 ⎥⎦ ⎢⎣ 6 2 ⎥⎦ ⎣ ⎣7 0 ⎥⎦ ⎦
2. If

⎡2 2 2⎤ ⎡3 3 3⎤ ⎡4 4 4⎤
⎢ − 3 ⎥ , B = ⎢⎢ 3
⎥ 5 ⎥ and C = ⎢⎢ 5
⎥ −1 4 ⎥⎥
A = ⎢2 1 0
⎢⎣ 1 0 4 ⎥⎦ ⎢⎣ 6 9 −1 ⎥⎦ ⎢⎣ 7 8 −1 ⎥⎦
then find the value of (i) 3A – 6B + 9C (ii) 7A – 2(B – C).

⎡ ⎡ 24 24 24 ⎤ ⎡ 16 16 16 ⎤ ⎤
⎢ ⎢ 33 ⎥
⎢ Ans. (i) ⎢ −6 −18 ⎥⎥ (ii) ⎢⎢ 18 5 − 23 ⎥⎥ ⎥
⎢⎣ ⎢⎣ 12 −9 12 ⎥⎦ ⎢⎣ 9 −2 28 ⎥⎦ ⎥⎦

⎡1 −1 ⎤
⎡3 1 2⎤ ⎢2
3. If A = ⎢ and B = 1 ⎥⎥ , find AB.
⎣1 0 1 ⎥⎦ ⎢
⎢⎣ 3 1 ⎥⎦

⎡ ⎡ 11 0⎤⎤
⎢ Ans. ⎢ ⎥
⎣ ⎣4 0 ⎥⎦ ⎦
268 Mathematical Methods

⎡3 3 5⎤
⎢ 4 ⎥⎥ , show that AI3 = I3 A.
4. If A = ⎢ 2 3
⎢⎣ 5 2 3 ⎥⎦
5. For the matrices
⎡1 0 ⎤ ⎡2 0⎤ ⎡ −1 2 ⎤
A= ⎢ ⎥ , B=⎢ ⎥ and C = ⎢ ⎥
⎣1 1 ⎦ ⎣1 1⎦ ⎣ 3 1⎦
verify that (i) (AB)C = A(BC ) (ii) A(B + C ) = AB + AC
(iii) (B + C ) A = BA + CA.
6. If
⎡a
g⎤ h ⎡ x⎤
A = [x y z ], B = ⎢⎢ h
f ⎥⎥ , and C = ⎢⎢ y ⎥⎥
b
f ⎢⎣ g
c ⎥⎦ ⎢⎣ z ⎥⎦
then evaluate ABC. [Ans. [ax 2 + by 2 + cz 2 + 2hxy + 2gzx + 2f yz]]
⎡1 3⎤ ⎡ −1 4⎤
7. If A = ⎢ ⎥ and B = ⎢ , then verify that (A + B) 2 ≠ A 2
⎣2 6⎦ ⎣ 2 1 ⎥⎦
2
+ 2AB + B .
8. If
⎡2 3⎤ ⎡3 4⎤
A= ⎢ ⎥ and B = ⎢ , then verify that (AB)′ = B′A′.
⎣0 1⎦ ⎣2 1 ⎥⎦

⎡1 2 3⎤ ⎡ −1 1 0⎤
⎢ 0 ⎥ and B = ⎢⎢ 0
⎥ −1 1 ⎥⎥ , then verify that
9. If A = ⎢ 0 1
⎣⎢ 1 1 0 ⎦⎥ ⎣⎢ 2 3 4 ⎦⎥
(A + B) T = AT + BT.
⎡ cos x sin x ⎤
10. If A = ⎢ ⎥ , then verify that AA′ = I2 = A′A.
⎣ − sin x cos x ⎦
11. Find the symmetric and skew-symmetric parts of the matrix

⎡1 2 4⎤ ⎡ ⎡ 1 4 7/2 ⎤ ⎡ 0 −2 1/ 2 ⎤ ⎤
⎢ 1 ⎥⎥ ⎢ ⎢ ⎥
A= ⎢6 8 ⎢ Ans. ⎢ 4 8 3 ⎥⎥ , ⎢⎢ 2 0 − 2 ⎥⎥ ⎥
⎢⎣ 3 5 7 ⎥⎦ ⎢⎣ ⎢⎣ 7/ 2 3 7 ⎥⎦ ⎢⎣ −1/ 2 2 0 ⎥⎦ ⎥⎦
Matrices and Determinants 269

12. Find the values of the determinants


⎡1 2 3⎤ ⎡x+a x x ⎤
(i) ⎢ 4 −2 ⎥
3 ⎥ (ii) ⎢⎢ x x+a x ⎥⎥

⎣⎢ 0 5 −1 ⎦⎥ ⎢⎣ x x x + a ⎦⎥
[Ans. (i) 55 (ii) x 2(3x + a)]
13. Write the cofactors of each element of the first column of the
following determinants and evaluate the determinant in each case.
(i) ⎡ 1 a bc ⎤ (ii) ⎡ 0 2 6 ⎤
⎢ 1 bc a ⎥ , ⎢1 5 0⎥
⎢ ⎥ ⎢ ⎥
⎢⎣ 1 c ab ⎥⎦ ⎣⎢ 3 7 1 ⎦⎥
[Ans. (i) ab 2 – ac 2, – (a 2b – bc 2), a 2c – b2; ab(b – a) + bc(c – a)
+ ac(a – c) (ii) 5, 40, – 30; – 50]
14. Without expanding the determinant, show that
a b c c b a
b c a = b a c
c a b a c b
15. Evaluate
1 374 1893
1 372 1892 [Ans. 1]
1 371 1891
16. Show that
b+c c+a a+b a b c
q+r r+p p+q =2 p q r
y+z z+x x+ y x y z
17. Prove that
a + b + 2c a b
c b + c + 2a b = 2(a + b + c) 2
c a c + a + 2b
18. Show that

1+ a 1 1
⎛ 1 1 1⎞
1 1+ b 1 = abc ⎜ 1 + + + ⎟
⎝ a b c⎠
1 1 1+ c
270 Mathematical Methods

19. Prove that


1 + a2 − b2 2ab − 2b
2ab 1 − a2 + b2 2a = (1 + a 2 + b2)3
2b − 2a 1 − a2 − b2
20. Determine whether the system of equations
4x – 2y + 6z = 8
2x – y + 3z = 5
2x – y + 3z = 4
is consistent. [Ans. Inconsistent]
21. Determine whether the system of equations
2x + 3y – 2z = 4
x – 2y + 3z = – 2
5x – 5y + z = – 2
is consistent. [Ans. Dependent]
22. Solve the system of equations (using Cramer’s rule)
x+y+z = 4
2x – y + 2z = 5
x – 2y – 2z = – 3
5 4
[Ans. x =
, y = 1, z = ]
3 3
23. Solve the system of equations (using Cramer’s rule)
2x – y – z = – 6
x + 3y – z = 0
2x + y + z = – 2
[Ans. x = – 2, y = 1, z = 1]
24. If A is a singular square matrix order 3, then prove that
A (adj A) = 0.
25. For the matrices
⎡2 0⎤ ⎡0 1⎤
A= ⎢ and B = ⎢ verify that (AB) – 1 = B – 1 A – 1.
⎣3 1 ⎥⎦ ⎣2 4 ⎥⎦

⎡2 0 −1 ⎤
⎢ 0 ⎥⎥ prove that A – 1 = A 2 – 6A + 11I.
26. If A = ⎢ 5 1
⎣⎢ 0 1 3 ⎦⎥
Matrices and Determinants 271

27. Using the matrix method, solve the system of equations


4x – 3y = 5
3x – 5y = 1 [Ans. x = 2, y = 1]
28. Using the matrix method, solve the system of equations
2x + 8y + 5z = 5
x+y+z = –2
x + 2y – z = 2 [Ans. x = – 3, y = 2, z = – 1]
29. Using the matrix method, solve the system of equations
2x – y + 4z = 18
– 3x + z = – 2
–x+y = 0 [Ans. x = 2, y = 2, z = 4]
30. Using the matrix method, solve the system of equations
x+y = 0
y+z = 1
x+z = 3 [Ans. x = 1, y = – 1, z = 2]
31. Let an economy based on two industrial sectors, viz. agriculture A
and energy E. The technology matrix M and final demand matrix
D, in billions of rupees, be
⎡ 0.4 0.2 ⎤ ⎡8 ⎤
M= ⎢ ⎥ and D = ⎢ 5 ⎥
⎣ 0.2 0.1 ⎦ ⎣ ⎦
– 1
Find (i) (I – M ) and (ii) the output for each sector which is
required to satisfy the demand.
⎡ −1 ⎡ 1.8 0.4 ⎤ ⎡ 16.4 ⎤ ⎤
⎢ Ans. (i) ( I − M ) = ⎢ ⎥ (ii) X = ⎢ ⎥
⎣ ⎣ 0.4 1.2 ⎦ ⎣ 9.2 ⎥⎦ ⎦
32. Consider an economy based on three industries, viz. agriculture
A, building B and energy E. The technology matrix M and the
final demand matrix D are given by
⎡ 0.422 0.100 0.266 ⎤ ⎡4⎤
M = ⎢⎢ 0.089 0.350 0.134 ⎥ and D = ⎢⎢ 3 ⎥⎥

⎣⎢ 0.134 0.100 0.334 ⎦⎥ ⎣⎢ 2 ⎦⎥


Find the output for each sector that is needed to satisfy the final
demand.
⎡ ⎡ x1 ⎤ ⎡ 11.14 ⎤ ⎤
⎢ ⎢ ⎥ ⎢ ⎥⎥
⎢ Ans. X = ⎢ x2 ⎥ = ⎢ 7.45 ⎥ ⎥
⎢ ⎢⎣ x3 ⎥⎦ ⎢⎣ 6.36 ⎥⎦ ⎥⎦

272 Mathematical Methods

CHAPTER 8

Vector Analysis

8.1 Introduction
In many physical situations we come across with two types of quantities,
viz. one which are represented by some numbers; and the other that not
only represented by some number but also have some particular direction.
The former are known as scalars and the latter are called vectors. This
chapter deals with the study of vectors, their algebraic and geometric
properties as well as the calculus of vectors. We have

Definition 8.1 A quantity which has magnitude only but no direction is


known as scalar.
For each scalar quantity we can associate some measure and thus
can be represented by a real number which gives the magnitude of the
quantity according to some chosen scale or unit of measure. For example,
mass, length, density, area, volume, time, work, temperature, electric
charge, potential etc. are scalar quantities.

Definition 8.2 A quantity which has magnitude and direction both is


known as vector.
For example, force, velocity, acceleration, momentum, angular
velocity, torque etc. are vector quantities.
A vector can be represented by a directed line segment or an arrow
OP (Fig. 8.1). The arrow indicates the direction of the vector, while the
magnitude of the vector is represented by the length of the arrow. The
point O (the tail) of the directed line segment (or arrow) is known as the
initial point or the origin of the directed line segment, and the point P
(the head) is the terminal point or the end point. The vector (Fig. 8.1) is
→ →
denoted by a or P, or a, or OP and the magnitude of the vector is
→ →
denoted by | a |, or | P |, or | a |, or | OP |, or simply by a, or OP. In this
Vector Analysis 273

chapter, a vector quantity shall be denoted by a bold letter while its


magnitude (the scalar quantity) by italic.

Definition 8.3 A vector whose magnitude is zero and the direction is


indeterminate is known as a null or zero vector. For null vectors, the
initial and the terminal points are same and all null vectors are regarded
as equal to each other without any consideration of direction.
P

O
Fig. 8.1 Directed line segment

8.2 Algebra of Vectors


This section deals with the addition, subtraction and multiplication etc.
of vectors along with related results. We have

Definition 8.4 Two vectors a and b are said to be equal if and only if a
and b have the same magnitude and direction, without any restriction on
the position of the initial points. If a and b are equal vectors, we write
them as a = b (Fig. 8.2).

a b

Fig. 8.2 Equal vectors


It may be noted that a vector can be drawn without any reference to
any particular location. Thus, from the definition of a vector it implies
that any vector may be subjected to its parallel displacement without
changing its magnitude or direction. The vectors in Fig. 8.3 are all equal
to one another. Since we can write a = b = c = d, therefore any one of the
vectors may be considered to represent a whole class of equal vectors of
which the remaining ones are the members. Such type of vectors are
known as free vectors.
274 Mathematical Methods

Definition 8.5 The sum or resultant of the two vectors a and b of


Fig. 8.4a is a vector c which is formed by translating the initial point of
b on the terminal point of a and joining the initial point of a to the
terminal point of b (Fig. 8.4b). The sum c is written as c = a + b. This
definition is equivalent to the parallelogram law of addition of vectors
as indicated in Fig. 8.4c.

b c

Fig. 8.3 Free vectors

b
b c b b

a a a
(a) (b) (c)

Fig. 8.4 Sum of vectors


The sum of more than two vectors can also be obtained in a similar
way. For example, Fig. 8.5 shows how to obtain the sum e of the vectors
a, b, c and d.

b c

a
e=a+b+c+d

Fig. 8.5 Sum of vectors


Definition 8.6 The negative of a vector a is a vector whose magnitude is
Vector Analysis 275

same as that of the vector a but the direction is opposite to a. Such a


vector is denoted by – a (Fig. 8.6).

a
–a

Fig. 8.6 Negative of a vector


Definition 8.7 The difference of two vectors a and b, denoted as a – b, is
a vector c which when added to b gives a. Equivalently, a – b may be
defined as a + (– b). Fig. 8.7 depicts the sum and difference of the vectors
a and b.

C B
a–
b
b
b a+

a
A

–b
a–
b

D E

Fig. 8.7 Sum and difference of two vectors


It may be noted that if a = b, then a – b is defined as the null or zero
vector (see also, Definition 8.3).

Definition 8.8 The vectors having unit length are called unit vectors.
If A is any vector with length A > 0, then A/A is a unit vector having the
same direction as that of A and is denoted by a. Then A = Aa.

Definition 8.9 The multiplication of a vector a by a scalar m gives a


vector ma whose magnitude is | m | times the magnitude of a and the
direction is same or opposite to that of a according as m is positive or
negative.
It may be noted that when m = 0, then ma = 0 is the null vector.
276 Mathematical Methods

8.2.1 Laws of Vector Algebra


If a, b and c are vectors, and m and n are scalars, then
(i) a + b = b + a (Commutative Law for Addition)
(ii) a + (b + c) = (a + b) + c (Associative Law for Addition)
(iii) m(na) = (mn)a = n(ma) (Associative Law for Multiplication)
(iv) (m + n)a = ma + na (Distributive Law)
(v) m(a + b) = ma + mb (Distributive Law)

Definition 8.10 If a and b are two vectors and m and n are any two
scalars, then the expression ma + nb is called a linear function of a and
b. Similarly, ma + nb + pc is a linear function of a, b and c.
The linear function of more than three vectors can be defined in a
similar way. For linear functions, we have

Theorem 8.1 If a and b are any two nonparallel vectors in a plane, and
if c is any third vector in the plane of a and b, then c can be expressed
(Fig. 8.8) as a linear function of a and b, i.e., c = ma + nb, where m and
n are scalars.
The extension of this theorem to space is as given by

Theorem 8.2 If a, b and c are any three vectors which are not parallel to
a single plane, and if d is any other vector, then d can be expressed as a
linear function of a, b and c, i.e. d = ma + nb + pc, where m, n and p are
scalars.

Theorem 8.3 If a and b are two non-collinear vectors, then xa + yb = 0


implies that x = y = 0.
R

nb ma + nb
b
a ma
P Q

Fig. 8.8 Linear function of two vectors


Remarks
(i) When this theorem holds we say that the vectors a and b are linearly
independent.
Vector Analysis 277

(ii) This theorem can be extended for more than two vectors.

Theorem 8.4 If x1a + y1b = x2a + y2b, where a and b are non-collinear
vectors, then x1 = x2 and y1 = y2.
This theorem can be extended for more than two vectors.
8.2.2 Rectangular Cartesian Coordinates and Position Vector
In order to study the applications of vectors, it is convenient to introduce
a set of rectangular Cartesian coordinates. We consider a set of rectangular
Cartesian coordinates denoted by the symbols x, y and z. These coordinates
are said to have right-handed orientation or to be right-handed if when
the thumb of the right hand is made to point in the direction of the
positive z-axis, the fingers point in the direction of the 90° rotation so
that the positive x-axis coincides with the positive y-axis. Otherwise the
coordinates are left-handed. The axes of such a set of coordinates is
shown in Fig. 8.9.
Now, introduce a set of three unit vectors, one pointing in the
direction of each of three positive coordinate axes. These vectors are
denoted by ˆi, ˆj and kˆ . Consider a vector a. This vector has orthogonal

D
k a3
a

y a1 A
j
i a2
B C
x
Fig. 8.9 Rectangular Cartesian coordinates
projections in the directions of the positive coordinate axes, which are
denoted by a1, a2 and a3 (Fig. 8.9). These a1, a2 and a3 are called the
components of a, may be positive or negative. For example, a1 is positive
when the angle between a and the direction of the positive x-axis (angle
BAD in Fig. 8.9) is acute, and is negative when this angle is obtuse.
It may be noted from the Fig. 8.9 that a is the diagonal of a rectangular
parallelopiped the lengths of whose edges are | a1 |, | a2 | and | a3 |, and
thus the magnitude a of the vector a is

|a| = a= a12 + a22 + a32 (1)


278 Mathematical Methods

Also, from Fig. 8.9, we have


JJJG JJJG JJJG
a = AB + BC + CD (2)
JJJG
The vector AB is parallel to î but from the definition of a1 and the
JJJG JJJG
product of a scalar by a vector, we have AB = a1ˆi. Similarly, BC = a2 ˆj
JJJG
and CD = a kˆ . With these values Eq. (2) leads to
3

a = a1ˆi + a2 ˆj + a3kˆ (3)


which shows that the vector a can be expressed as a linear function
of the unit vectors ˆi , ˆj and kˆ , where the coefficients are the components
of a.
In particular, the position vector or radius vector r from the point
A(0, 0, 0) to the point D(x, y, z) (Fig. 8.9) can be expressed as

r = xˆi + yˆj + zkˆ (4)

having the magnitude as | r | = r = x2 + y2 + z 2 .


We now have
Theorem 8.5 If a and b are two vectors such that

a = a1ˆi + a2 ˆj + a3kˆ , b = b 1ˆi + b 2 ˆj + b 3kˆ


then

a + b = (a1 + b 1)ˆi + (a2 + b 2 )ˆj + ( a3 + b3 )kˆ


i.e., the components of the sum of two vectors are equal to the sums of
the components of two vectors.
This theorem can be extended to the case of the sum of a number of
vectors.
Example 8.1 If a = i + j – k, b = 2i – 3j + 4k and c = 3i – 2j + 5k, then
find the magnitude of a – 2b + c.
Solution We have
a – 2b + c = (i + j – k) – 2(2i – 3j + 4k) + (3i – 2j + 5k) = 5j – 4k
and thus the magnitude of a – 2b + c is given by

| a – 2b + c | = | 5j – 4k | = (5) 2 + (− 4)2 = 41
Vector Analysis 279

Example 8.2 Find the unit vector parallel to the resultant of vectors
a = 2i – 3j + 4k and b = 3i – 4j + 2k.

Solution The resultant c of the vectors a and b is the sum of a and b.


Thus
c = a + b = 5i – 7j + 6k, c = | c | = 110
The unit vector parallel to c is

c 5i − 7 j + 6k
=
|c| 110
Example 8.3 If a = 2i – j + k, b = i + 3j – 2k, c = – 2i + j – 3k
and d = 3i + 2j + 5k, then find the scalars p, q, r such that d = pa +
qb + rc.

Solution Here d = pa + qb + rc leads to


3i + 2j + 5k = p(2i – j + k) + q(i + 3j – 2k) + r(– 2i + j – 3k)
= (2p + q – 2r)i + (– p + 3q + r)j + ( p – 2q – 3r) k
Since i, j, k are non-coplanar unit vectors, we therefore have
2p + q – 2r = 3, – p + 3q + r = 2, p – 2q – 3r = 5
which on solving give rise to
p = – 2, q = 1, r = – 3 so that d = – 2a + b – 3c
This clearly shows that d is linearly dependent.

8.2.3 Products of Vectors


We shall now find the ways that how to multiply a vector by another
vector to get products of vectors. One can multiply the vectors in two
different manners, viz. one give rise to a scalar and the other yield a
vector quantity.

Definition 8.11 Let a and b be two vectors having magnitudes,


respectively, as a and b, and θ be the angle between them (Fig. 8.10a)
such that 0 ≤ θ ≤ π, then the product a⋅ b = ab cos θ is known as the
scalar product of the vectors a and b and read as a dot b. This product is
also known as the dot product.
280 Mathematical Methods

P
b

a F

O P′ A
(a) (b)
Fig. 8.10(a) Scalar product of two vectors and
(b) work done by a force F

Remarks
(i) The outcome of the dot product of two vectors is always a scalar
quantity, and thus it is not possible to obtain the scalar product of
three or more vectors.
(ii) The expression ab has no meaning.
(iii) Since cos θ = cos (– θ), it therefore makes no difference whether
we take θ or – θ in a ⋅ b = ab cos θ.
(iv) Geometrically, a ⋅ b is equal to the product of the directed magnitude
of the projection of a onto b and the magnitude of b.
(v) If a = OA represents the displacement of a mass point and F, the
force acting on this mass point (Fig. 8.10b), then physically a⋅ F
represents the work done by the force F.
(vi) A vector b can be expressed as the sum of a vector parallel to a
vector a and vector perpendicular to a and we have

⎛ a⋅b ⎞ ⎡ ⎛ a⋅b ⎞ ⎤
b= ⎜ ⎟ a + ⎢b − ⎜ ⎟a⎥ (5)
⎝ a⋅a ⎠ ⎣ ⎝ a⋅a ⎠ ⎦
Properties of scalar product
If a, b and c are vectors, m is a scalar and i, j and k are unit vectors, then
1. a ⋅b = b ⋅ a (Commutative Law for Dot Product)
2. a ⋅ (b + c) = a⋅ b + a⋅ c (Distributive Law for Dot Product)
3. m(a⋅ b) = (ma)⋅ b = a⋅ (mb) = (a⋅ b)m
4. i⋅i = j⋅ j = k⋅ k = 1, i⋅ j = j ⋅ k = k ⋅ i = 0
5. If a = a1i + a2j + a3k and b = b1i + b2j + b3k, then
a⋅ b = a1b1 + a2b2 + a3b3
Vector Analysis 281

a⋅ a = a2 = a12 + a22 + a32

b ⋅ b = b2 = b1 + b 2 + b 3
2 2 2

It is not allowed to write a in place of a 2 , since a is a vector and a2


is a number. In fact, we can write a 2 = | a |.
6. If a and b are perpendicular, then a⋅ b = 0. But if a⋅ b = 0 then it does
not necessarily mean that a is perpendicular to b. It only means that
either a = 0, b = 0; or a is perpendicular to b.
In the same way , if a⋅ b = a⋅ c, then it does not mean that b = c. Since
this relation can be expressed as a⋅[b – c] = 0, and thus we can say that
atleast one of the following is true: a = 0; b = c; a is perpendicular to the
vector b – c.

Definition 8.12 Let a and b be two vectors having magnitude, respectively,


as a and b, and θ be the smallest non-negative angle between them such
that 0 ≤ θ ≤ π, then the vector product of a and b is a third vector c
(Fig. 8.11) defined in terms of a and b such that the following conditions
are satisfied:
O
c
b

a
θ a F
A P
(a) (b)
Fig. 8.11 (a) Vector product of two vectors and (b) moment of the force
(i) c is perpendicular to both a and b,
(ii) the direction of c is right-handed, i.e. the direction is that indicated
by the thumb of the right hand when the figure point in the sense
of rotation θ from the direction of a to the direction of b,
(iii) c = ab sin θ.
These three conditions define c uniquely. The vector product of a and b
is denoted by a × b and read as a cross b. This product is also known as
the cross product. Thus c = a × b.
The cross product of the vectors a and b may also be defined as
a × b = ab sin θn
282 Mathematical Methods

where n is a unit vector showing the direction of a × b.

Remarks
(i) The cross product of two vectors is always a vector quantity.
(ii) The expression ab has no meaning.
(iii) Geometrically, | a × b | is equal to the area of the parallelogram
whose adjacent sides are a and b.
(iv) The cross products are used to study the effects of forces in
electromagnetism, flow of fluids and mechanics etc. Thus, for
example if A is the point of application of a force F then OA × F,
physically represents the moment of the force F about the point O
(Fig. 8.11b).
(v) If a = b or, if a is parallel to b, then sin θ = 0 and we define
a × b = 0.

Properties of vector product


If a, b and c are vectors, m is a scalar and i, j and k are unit vectors, then
1. a × b = – b × a (Commutative Law for cross product does not hold)
2. a × (b + c) = a × b + a × c (Distributive Law for Dot Product)
3. m(a × b) = (ma) × b = a × (mb) = (a × b)m
4. i × i = j × j = k × k = 0, i × j = k, j × k = i, k × i = j.
5. If a = a1i + a2j + a3k and b = b1i + b2j + b3k, then
i j k
a × b = a1 a2 a3 and a × a = 0
b1 b2 b3
6. If a × b = 0, then either a = 0; b = 0, or a and b are parallel. In the
same way, if a × b = a × c, then a × (b – c) = 0; and atleast one of the
conditions that a = 0; b = c; a is parallel to b – c must be true.

Example 8.4 Show that the vectors a = 2i – j + k, b = i – 3j – 5k and


c = 3i – 4j – 4k form the sides of a right-angled triangle.

Solution We have
a + b = (2i – j + k) + (i – 3j – 5k) = 3i – 4j – 4k = c
Therefore, the vectors a, b and c form the sides of a triangle. Further,
a ⋅ b = (2i – j + k ) ⋅ (i – 3j – 5k) = 0
Vector Analysis 283

which shows that a and b are perpendicular to each other. Hence, the
vectors a, b and c form the sides of a right-angled triangle.

Example 8.5 Find the angle between the vectors a = 2i + 2j – k and


b = 6i – 3j + 2k.

Solution Here a = 3, b = 7 and a⋅b = 4. Thus from a ⋅b = ab cos θ we have


a⋅b 4
cos θ = =
ab 21
which gives θ = 79° (approximately).

Example 8.6 Express b = 2i + j – 3k as the sum of a vector parallel to


the vector a = 3i – j and a vector perpendicular to a.

Solution Here a ⋅ b = 5, a ⋅ a = 10 and Eq. (5) reduces to

⎛3 1 ⎞ ⎛1 3 ⎞
b = ⎜ i − j ⎟ + ⎜ i + j − 3k ⎟ = c + d
⎝2 2 ⎠ ⎝2 2 ⎠
1
It may be noted that the vector c is parallel to the vector a as c = a and
2
the vector d is perpendicular to the vector a because d⋅ a = 0. Thus, the
vector b can be expressed as the sum of a vector parallel to a and a vector
perpendicular to a.

Example 8.7 Show that (a – b) × (a + b) = (2a) × b.

Solution We have
(a – b) × (a + b) = (a – b) × a + (a – b) × b
= – a × (a – b) + (a – b) × b
= –a×a+a×b+a×b–b×b
= 2(a × b) = (2a) × b

Example 8.8 Find the area of the parallelogram whose adjacent sides
are a = i + 2j + 3k and b = – 3i – 2j + k.

Solution We have

i j k
a×b= 1 2 3 = 8i – 10j + 4k
−3 −2 1
284 Mathematical Methods

Therefore, the area of the parallelogram whose adjacent sides are a and
b is | a × b | = 180 sq. units.

8.2.4 The Multiple Products of Vectors


The products of more than two vectors are called multiple products and
we shall discuss here the products of three and four vectors.

Definition 8.13 Let a, b and c be any three vectors, then the product
a ⋅ (b × c) is a scalar and is known as scalar triple product of three vectors
a, b and c.
Thus, if a = a1i + a2j + a3k, b = b1i + b2j + b3k and c = c1i + c2j +
c3k, then
a1 a2 a3
a ⋅ (b × c) = b 1 b2 b3
c1 c2 c3
Properties of scalar triple product
(a) i ⋅ ( j × k) = i ⋅ i = 1, and j⋅ ( j × i) = 0, etc.
(b) The scalar triple product is commutative, i.e., a⋅ (b × c) = (b × c) ⋅ a.
(c) If the vectors in the scalar triple products are subject to an odd number
of permutations, the value of the product is changed only in sign; and if
the number of permutations is even then the value of the product remains
unchanged.
This property of the scalar triple product leads to
a ⋅ (b × c) = b ⋅ (c × a) = c ⋅ (a × b)
= – c ⋅ (b × a) = – a⋅ (c × b) = – b ⋅ (a × c)
The above equations can also be expressed in the following form
[a b c] = [b c a] = [c a b] = – [b a c] = – [c b a] = –[a c b]
(d) If the vectors a, b and c form the three adjacent sides of a
parallelopiped, then the volume V of the parallelopiped is
V = | a ⋅ (b × c) |
(e) Three non-zero and non-collinear vectors a, b and c are coplanar if
and only if a ⋅ (b × c) = 0. Here, a ⋅ (b × c) = 0 will mean that either a and
b × c are perpendicular to each other; or atleast one of the vectors a, b
and c is a null vector.

Definition 8.14 If a, b and c are any three vectors, then the product
a × (b × c) is a vector and is known as vector triple product of three
vectors a, b and c.
Vector Analysis 285

Properties of vector triple product


(a) i × ( j × k) = i × i = 0, and j × ( j × i) = – i, etc.
(b) The vector triple product is not associative, i.e., a × (b × c) ≠
(a × b) × c. Thus the product a × b × c has no meaning.
(c) a × (b × c) = b(a ⋅ c) – c(a⋅ b).

Definition 8.15 If a, b, c and d are any four vectors, then the product
(a × b) ⋅ (c × d) is a scalar quantity and is known as scalar product of
four vectors a, b, c and d. We have
(a × b) ⋅ (c × d) = (a ⋅ c) (b ⋅ d) – (b ⋅ c) (a ⋅ d)
which may also be expressed as
a⋅c a⋅d
(a × b) ⋅ (c × d) =
b⋅c b⋅d
Definition 8.16 If a, b, c and d are any four vectors, then the product
(a × b) × (c × d) is a vector quantity and is known as vector product of
four vectors a, b, c and d. We have
(a × b) × (c × d) = c[(a × b) ⋅ d] – d[(a × b) ⋅ c]
Since a × b = – b × a, we, have
(a × b) × (c × d) = – (c × d) × (a × b)
Moreover, (a × b) × (c × d) may also be expressed as
(a × b) × (c × d) = b[(c × d) ⋅ a] – a[(c × d) ⋅ b]
It may be noted that the products a × b × c, ab, a ⋅ b ⋅ c, a × b ⋅ c,
(a × b) ⋅ c ⋅ d, a ⋅ b × c ⋅ d, etc. have no meanings.
Example 8.9 Prove that
a × (b × c) + b × (c × a) + c × (a × b) = 0

Solution. We have
a × (b × c) = b(a ⋅ c) – c(a ⋅ b)
b × (c × a) = c(b ⋅ a) – a(b ⋅ c)
c × (a × b) = a(c ⋅ b) – b(c ⋅ a)
Adding all these three equations we get the required result.

Example 8.10 If a, b and c are non-zero vectors, and (a × b) × c =


a × (b × c) then show that (a × c) × b = 0.
286 Mathematical Methods

Solution We know that


a × (b × c) + b × (c × a) + c × (a × b) = 0
But a × (b × c) = (a × b) × c. Therefore
a × (b × c) – (c × a) × b – (a × b) × c = 0
which leads to (a × c) × b = 0.

Example 8.11 Show that


(a × b) ⋅ [(b × c) × (c × a)] = [a ⋅ (b × c)]2

Solution We have
(a × b) ⋅ [(b × c) × (c × a] = (a × b) ⋅ [{c(b × c) ⋅ a} – a{(b × c) ⋅ c}]
= (a × b) ⋅ [c{a ⋅ (b ×c)} – a{b ⋅ (c × c)}]
= {(a × b) ⋅ c} {a ⋅ (b × c)} = [a ⋅ (b × c)]2

Example 8.12 Prove that


(a × b) × (a × c) = [a ⋅ (b × c)]a

Solution We know that


(a × b) × (c × d) = c[(a × b) ⋅ d] – d[(a × b) ⋅ c]
Therefore
(a × b) × (a × c) = a[c ⋅ (a × b)] – c[a ⋅ (a × b)]
= a[a ⋅ (b × c)] – c[b ⋅ (a × a )]
= a[a ⋅ (b × c)] = [a ⋅ (b × c)] a

8.3 Vector Calculus


Now, we shall be concerned with the differentiation and integration of
vectors. We first introduce the concept of a vector function and we have

Definition 8.17 If corresponding to each value of a scalar u there is


associated a vector a, then a is called a vector function of u and is denoted
by a(u). The concept of vector function can be extended easily - thus, if
to each point (x, y, z) one can associate a vector a, then a is a function of
(x, y, z), and is denoted by a(x, y, z). A vector function a(x, y, z) is often
known as a vector field while the function φ(x, y, z) is a scalar field,
where φ is a scalar.
In Chapter 3, we have introduced the concept of ordinary differentiation
of a real function. The derivative of a vector function can be defined in a
similar manner and we have
Vector Analysis 287

Definition 8.18 The derivative of a(u) is defined as


da a(u + ∆u ) − a(u )
= lim
du ∆u → 0 ∆u
provided that this limit exists.

Properties of Vector Differentiation


If a(u) and b(u) are two vectors which are functions of a scalar u and m
is a scalar function of u, then
d da db
(i) (a ± b) = ±
du du du
d da dm
(ii) ( ma ) = m + a
du du du
d da db
(iii) (a ⋅ b ) = ⋅b +a⋅
du du du

(iv) d (a × b) = da × b + a × db
du du du
(v) If a = a1i + a2j + a3k, where i, j and k are unit vectors in the
directions of the axes, then
di d j dk
= = =0
du du du
and
da da1 da da
= i + 2 j+ 3k
du du du du
da
(vi) If a(u) is a constant vector, then = 0.
du
(vii) If a(u) is a vector of constant magnitude, then its derivative is
da
perpendicular to it, i.e. a ⋅ = 0.
du
(viii) If a(u) is a vector of constant direction, then its derivative is
da
collinear with it, i.e. a × = 0.
du
288 Mathematical Methods

(ix) If the vector a(u) is a function of a scalar variable u and u is a


da da du
function of another scalar v, then = .
dv du dv
Example 8.13 If a = p cos u + q sin u, where p and q are constant
vectors, then show that

⎛ da d 2 a ⎞
a⋅ ⎜ × 2 ⎟=0
⎜ ⎟
⎝ du du ⎠
Solution We are given that a = p cos u + q sin u, which on differentiation
leads to
da
= – p sin u + q cos u
du
and
d 2a
= –a
du 2
so that, after simplification, we have
⎛ da d 2 a ⎞
a ⋅ ⎜⎜ × 2 ⎟⎟ = 0
⎝ du du ⎠
Example 8.14 Prove that

d ⎡ ⎛ da d 2 a ⎞⎤ ⎛ da ⎞ d 3 a
⎢ a ⋅ ⎜⎜ × 2 ⎟⎟ ⎥ = ⎜ a × ⎟⋅
du ⎢⎣ ⎝ du du ⎠ ⎥⎦ ⎝ du ⎠ du 3
Solution Since
d da db
(a ⋅ b) = ⋅b + a⋅
du du du
and
d da db
(a × b) = ×b +a×
du du du
Therefore

d ⎡ ⎛ da d 2a ⎞ ⎤ da ⎛ da d 2 a ⎞ ⎡ d ⎛ da d 2 a ⎞ ⎤
⎢ a ⋅ ⎜⎜ × 2 ⎟⎥ = ⋅⎜ × 2 ⎟+a⋅⎢ ⎜ × 2 ⎟⎥
du ⎣⎢ ⎝ du du ⎟⎠ ⎦⎥ du ⎜⎝ du du ⎟⎠ ⎜ ⎟
⎣⎢ du ⎝ du du ⎠ ⎦⎥
Vector Analysis 289

d 2 a ⎛ da da ⎞ ⎡ d ⎛ da ⎞ d 2 a da d ⎛ d 2a ⎞ ⎤
= ⋅ ⎜ × ⎟ + a ⋅ ⎢ ⎜ ⎟× 2 + × ⎜ ⎟⎥
du 2 ⎝ du du ⎠ ⎢⎣ du ⎝ du ⎠ du du du ⎜⎝ du 2 ⎟⎠ ⎥⎦

⎡ da d 3a d 2 a d 2a ⎤ ⎡ da d 3 a ⎤ ⎛ da ⎞ d 3a
= a ⋅ ⎢ × 3 + 2 × 2 ⎥ = a ⋅ ⎢ × 3 ⎥ = ⎜a × ⎟⋅
⎣ du du du du ⎦ ⎣ du du ⎦ ⎝ du ⎠ du 3

8.3.2 Integration of Vectors


The concept of integration of a real function was introduced in
Chapter 4. In an analogous manner, we can define the integration of a
vector function and we have

Definition 8.19 If a(u) = a1i + a2j + a3k is a vector function of u, where


i, j and k are unit vectors whose direction is independent of u, then the
indefinite integral of a with respect u (denoted by ∫ a(u) du) is defined as

∫ a(u )du = i ∫ a1 (u ) du + j∫ a2 (u ) du + k ∫ a3 (u ) du
Here, each of the integral on the right hand side gives rise to a constant
of integration.

Remarks
(i) From the definition, we have
d
du ∫
a(u ) du = i a1 (u ) + j a2 (u ) + k a3 (u ) = a

as expected, since integration and differentiation are inverse


operations.
(ii) If a(u) is a linear function of constant vectors, with coefficients
which are functions of u, then ∫ a(u ) du can be obtained by the
usual process of integration in which constant vectors are treated
as constants in ordinary integration, and the arbitrary constant
vectors are inserted where arbitrary constants would appear in
ordinary integration. Thus
1 2
∫ (au + b)du =
2
au + b u + c

and

∫ a cosudu = a sin u + c
290 Mathematical Methods

8.3.3 Partial Differentiation of Vectors


Partial derivatives of a real function of several variables were defined in
Chapter 5. For a vector function of several variables, we have

Definition 8.20 Let the vector a be a function of (x, y, z), i.e.


a = a(x, y, z). The partial derivative of a with respect to x (keeping y and
z as constants) is defined as
∂a a ( x + ∆x , y , z ) − a ( x , y , z )
ax = = lim
∂x ∆ x → 0 ∆x
Similarly, the partial derivatives of a with respect to y and z, respectively,
are
∂a a( x, y + ∆y, z ) − a( x, y, z )
ay = = lim
∂y ∆y → 0 ∆y
∂a a ( x, y , z + ∆ z ) − a ( x, y , z )
az = = lim
∂z ∆z → 0 ∆z
provided that the limits in the above equations exist.
If ax, ay, ... are themselves functions of x, y, z ... and if they are
differentiated again, we get the second order partial derivatives. Thus
∂ 2a ∂ ⎛ ∂a ⎞ ∂ 2 a ∂ ⎛ ∂a ⎞
= ⎜ ⎟, = ⎜ ⎟ , etc.
∂x 2 ∂x ⎝ ∂x ⎠ ∂x ∂y ∂x ⎝ ∂y ⎠

Properties of Partial Differentiation of Vectors

∂ ∂a ∂b
(i) (a ± b) = ±
∂x ∂x ∂x
∂ ∂a ∂m
(ii) ( ma ) = m + a
∂x ∂x ∂x
∂ ∂a ∂b
(iii) (a ⋅ b) = ⋅b + a⋅
∂x ∂x ∂x
∂ ∂a ∂b
(iv) (a × b ) = ×b +a×
∂x ∂x ∂x
(v) If a has continuous partial derivatives of second order, then

∂ 2a ∂ 2a
=
∂x∂y ∂y∂x
Vector Analysis 291

i.e., the order of differentiation does not matter.


(vi) The rules of differentials of vectors are same as that of the ordinary
differentiation, e.g., if a = a(x, y, z), then
∂a ∂a ∂a
da = dx + dy + dz
∂x ∂y ∂z
Moreover, if the variables x, y and z are themselves functions of
another variable t, then a is a vector function of t and we have
d a ∂a dx ∂a dy ∂a dz
= + +
dt ∂x dt ∂y dt ∂z dt
Example 8.15 If a = xyzi + xz 2j – y 3k and b = x 3i – xyzj + x 2zk, then
∂ 2a ∂ 2b
evaluate × at the point (1, 1, 0).
∂y 2 ∂x 2
Solution Given that a = xyzi + xz 2j – y 3k and b = x3i – xyzj + x2zk, then
∂a
= xzi – 3y 2 k
∂y
and
∂b
= 3x 2 i – yzj + 2xzk
∂x
so that
∂2a
= – 6yk
∂y 2
and
∂ 2b
= 6xi + 2zk
∂x 2
Therefore
i j k
∂ 2a ∂ 2b
× = 0 0 − 6 y = – j(36xy)
∂y 2 ∂x 2
6x 0 2z
Thus at the point (1, 1, 0), we have

∂ 2a ∂ 2b
× = – 36j
∂y 2 ∂x 2
292 Mathematical Methods

The remaining part of this section is devoted to the study of some


well known differential operators, namely, gradient, divergence and curl
(along with related results). These operators play a key role in the study
of hydrodynamics and electromagnetism apart from that of other branches
of applied mathematics and physical chemistry. We have
Definition 8.21 The points satisfying an equation of the form f (x, y, z) =
c, where c is a constant, constitute a family of surfaces in the 3-
dimensional space. The surfaces of the family are called level surfaces.
Definition 8.22 Let V be a region in a space and let there be a curve C in
it. Let X be a general point on C whose position vector is r
(Fig. 8.12a), and let s be the arc length of C measured from some fixed
point Q on C. The vector t = dr/ds is a unit vector tangent to C in the
direction of s increasing (Fig. 8.12a). This vector t is known as the unit
tangent vector.
Definition 8.23 Let s be the distance from any fixed point P(x, y, z) on
the level surface f (x, y, z) = c (Fig. 8.12b) in the direction of a unit
tangent vector t, then d f /ds is called the directional derivative of f in the
direction of t. We have

d f ∂ f dx ∂ f dy ∂ f dz
= + + (6)
ds ∂x ds ∂y ds ∂z ds

V X
t
X t V
C P
Q
z f (x, y, z ) = c
r
k

O O
j y
i
x

Fig. 8.12(a) Unit tangent vector (b) Directional derivative


Definition 8.24 The vector differential operator ∇ is defined as
∂ ∂ ∂
∇ = i + j +k (7)
∂x ∂y ∂z
Vector Analysis 293

and is known as del operator.

Definition 8.25 The gradient of a scalar field f, denoted by ∇ f, or


grad f, is defined as
∂f ∂f ∂f
grad f = ∇ f = i +j +k (8)
∂x ∂y ∂z
Remarks
(i) The gradient of a scalar is a vector field.
(ii) Since
df ∂ f dx ∂ f dy ∂ f dz
= + +
ds ∂x ds ∂y ds ∂z ds
∂f ∂f ∂f
grad f = i + j +k
∂x ∂y ∂z
and
dr dx dy dz
t = =i + j +k
ds ds ds ds
It can easily be verified from here that
df
= ∇f ⋅ t (9)
ds
where t is the unit vector in the direction in which the directional
derivative is taken. Thus, we can conclude that the directional
derivative of f in the direction of the unit tangent vector t is equal
to the projection of ∇ f in that direction.
(iii) The vector ∇ f points in the direction in which d f/ds has maximum
value; and this maximum value is | ∇ f |.
(iv) The vector field ∇ f is normal to the surface f (x, y, z) = constant.
(v) If f is a function of a single scalar field u, then
df
∇f = ∇u
du
Definition 8.26 For any vector field a, we define two more operators
a⋅ ∇ and a × ∇ as follows:
∂ ∂ ∂
a ⋅ ∇ = a1 + a2 + a3 (10)
∂x ∂y ∂z
294 Mathematical Methods

and

i j k
a×∇ = a1 a2 a3 (11)
∂ ∂ ∂
∂x ∂y ∂z

The operator a ⋅ ∇ is a scalar and can be applied to a scalar field or


a vector field. Thus, if f and b be any scalar and vector field, respectively,
then
∂f ∂f ∂f
(a ⋅ ∇) f = a1 + a2 + a3 (12)
∂x ∂y ∂z
∂b ∂b ∂b
(a ⋅ ∇)b = a1 + a2 + a3 (13)
∂x ∂y ∂z
It may also be noted that
(a ⋅ ∇) f = a ⋅ ∇ f (14)
On the other hand, the operator a × ∇ is a vector operator and can
be applied to a scalar field. Thus if f is a scalar field, then

i j k
(a × ∇) f = a1 a2 a3 (15)
∂f ∂f ∂f
∂x ∂y ∂z

Also
(a × ∇) f = a × ∇ f (16)
While writing the expressions like (a ⋅ ∇) f, (a ⋅ ∇) b and (a × ∇) f,
care must be taken about the order of appearance of the symbols, because
the operator ∇ and all operators constructed from it operate only on the
functions which are on their right. For example, (a ⋅ ∇) b ≠ b(a ⋅ ∇).
Here, the left hand side is a vector field, while the right hand side is a
vector operator.

Definition 8.27 If a is a vector field, then the divergence of a, denoted


by div a, or, ∇ ⋅ a is defined as
∂a1 ∂a2 ∂a3
∇⋅a = + + (17)
∂x ∂y ∂z
Vector Analysis 295

Remarks

(i) The divergence of a vector field is a scalar quantity.


(ii) ∇ ⋅ a ≠ a ⋅ ∇.
(iii) If ∇ ⋅ a = 0, then a is said to be a solenoidal vector.

Definition 8.28 If a is a vector field, then the curl or rotation of a, denoted


by curl a, or, rot a, or ∇ × a is defined as

i j k
∂ ∂ ∂
∇×a = ∂x ∂y ∂z (18)
a1 a2 a3
Remarks
(i) The curl of a vector field is a vector quantity.
(ii) If ∇ × a = 0, then a is said to be an irrotational vector.
Definition 8.29 The Laplacian operator ∇2 is defined by the relation
∂2 f ∂2 f ∂2 f
∇ 2 f = (∇ ⋅ ∇) f = + + (19)
∂x 2 ∂y 2 ∂z 2
Formulas involving ∇
If a, b are vector fields, f, g are scalar fields and r is the position vector
of a point, then
(i) ∇( f ± g) = ∇ f ± ∇g
(ii) ∇ ( f g) = f ∇ g + g∇ f

⎛ f ⎞ g (∆ f ) − f ( ∆g )
(iii) ∇ ⎜ ⎟ =
⎝g⎠ g2
(iv) ∇ ⋅ (a + b) = ∇ ⋅ a + ∇ ⋅ b
(v) ∇ × (a + b) = ∇ × a + ∇ × b
(vi) ∇ ⋅ ( f a) = f (∇ ⋅ a) + (∇ f ) ⋅ a
(vii) ∇ × ( f a) = f (∇ × a) + (∇ f ) × a
(viii) ∇ ⋅ (a × b) = b ⋅ (∇ × a) – a ⋅ (∇ × b)
(ix) ∇ × (a × b) = a(∇ ⋅ b) + (b ⋅ ∇) a – b(∇ ⋅ a) – (a ⋅ ∇)b
(x) ∇(a ⋅ b) = (a ⋅ ∇) b + (b ⋅ ∇) a + a × (∇ × b) + b × (∇ × a)
(xi) ∇ × (∇ f ) = 0 (or, curl grad f = 0)
296 Mathematical Methods

(xii) ∇ ⋅ (∇ × a) = 0 (or, div curl a = 0)


(xiii) ∇ × (∇ × a) = ∇(∇ ⋅ a) – (∇ ⋅ ∇)a
(xiv) ∇ ⋅ r = 3
(xv) ∇ × r = 0
(xvi) (a ⋅ ∇) r = a
These formulas can easily be proved by using the definitions.

Example 8.16 Prove that

⎛r⎞ 2
div ⎜ ⎟ =
⎝r⎠ r
Solution We have

⎛r⎞ ⎛r⎞ ⎛ ∂ ∂ ∂ ⎞ ⎛x y z ⎞
div ⎜ ⎟ = ∇ ⋅ ⎜ ⎟ = ⎜ i + j + k ⎟ ⋅ ⎜ i + j + k ⎟
⎝r⎠ ⎝ r ⎠ ⎝ ∂x ∂y ∂z ⎠ ⎝ r r r ⎠

∂ ⎛ x ⎞ ∂ ⎛ y ⎞ ∂ ⎛ z ⎞ ⎛ 1 x ∂r ⎞ ⎛ 1 y ∂r ⎞ ⎛ 1 z ∂r ⎞
= ⎜ ⎟+ ⎜ ⎟+ ⎜ ⎟=⎜ − ⎟+⎜ − ⎟+⎜ − ⎟
∂x ⎝ r ⎠ ∂y ⎝ r ⎠ ∂z ⎝ r ⎠ ⎝ r r 2 ∂x ⎠ ⎝ r r 2 ∂y ⎠ ⎝ r r 2 ∂z ⎠

3 1 2 2
= − 3 (x + y2 + z2 ) =
r r r
as ∂r/∂x = x/r, etc. and r 2 = x 2 + y 2 + z 2.

Example 8.17 If a = (x + y + 2) i + 2j – (x + y)k, then evaluate a ⋅ curl a.


Solution Here

i j k
∂ ∂ ∂
∇×a= ∂x ∂y ∂z
=–i+j–k
x+ y+2 2 −( x + y)
Thus a . curl a = 0.

8.4 Line, Surface and Volume Integrals and Integral


Theorems
This section deals with the concepts of line, surface and volume integrals.
These integrals are then used to establish the theorems of Gauss and
Stoke. We have
Vector Analysis 297

Definition 8.30 A curve is known as a regular arc if, in the rectangular


coordinate system, it can be expressed by the equation
x = x1(u) i + x2(u) j + x3(u) k
where x is the position vector of a general point on the curve, u is a
parameter such that α ≤ u ≤ β, and for this range of u the functions
x1(u), x2(u) and x3(u) are continuous with continuous first derivatives.

Definition 8.31 A curve that is composed of a finite number of regular


arcs joined end to end, and which does not intersect itself is known as a
regular curve. In this chapter, we shall be concerned only with the regular
arcs and shall refer them simply as curves.
8.4.1 Line Integral
Consider a curve C with A and B as terminal points (Fig. 8.13). Let
f (x1, x2, x3) be a single valued and continuous function on the curve C.
Divide C into n parts by n + 1 points q0, q1, q2, ..., qn (Fig. 8.13). Let ∆sp
denote the length of the line segment qp– 1 qp ( p = 1, 2, ..., n). Also
assume that there be a point xp on the arc qp–1 qp with coordinates as
( x p1 , x p2 , x p3 ). Then, the line integral of f over C is defined as
B
xp qp

qp –1

x2
q2
q1
x1
q0
A
Fig. 8.13 Partition of a curve
n

∫ fds = n → ∞lim
, ∆s → 0
∑ f ( x p , x p , x p ) ∆s p
1 2 3
(20)
C pp =1

Since f is single valued and continuous on C, therefore the limit on


the right side of this equation is independent of the manner in which the
curve C is divided into parts. Moreover, if f = 1 everywhere on the curve
C, then the above equation defines the arc length of the curve c.
Let X be a point on the curve C (Fig. 8.14) and s be the arc length of
C measured from the end A of C. The vector t = dr/ds is a unit vector
tangent to C in the direction of s increasing. Let a(x1, x2, x3) be a vector
298 Mathematical Methods

field defined over C. The orthogonal projection of a on the unit tangent


vector t is called the tangential component of a, and is denoted by at and
thus
at = a ⋅ t (21)

X B
t

z r
A

Fig. 8.14 Tangent vector to a curve


The line integral of a over C is defined as

∫ at ds = ∫ a ⋅ t ds = ∫ a ⋅ dr = ∫ (a1i + a2 j + a3k ) ⋅ (dxi + dyj + dzk)


C C C C

= ∫ (a1dx + a2 dy + a3 dz ) (22)
C
(using the definition of the unit tangent vector and the vector a).
We also have

∫ ads = i ∫ a1ds + j∫ a2 ds + k ∫ a3 ds (23)


C C C C

∫ a × t ds = ∫ a × dr
C C

= i ∫ (a2 dz − a3 dy ) + j∫ (a3 dx − a1dz ) + k ∫ (a1dy − a2 dx) (24)


C C C

8.4.2 Surface Integral


A regular surface element is defined to be a portion of a surface which,
for some orientation of the coordinate axes can be projected onto a region
S′ in the xy-plane enclosed by a regular closed curve, and which can be
represented by the equation z = g(x, y) where g(x, y) is continuous and
has continuous first derivatives in S ′. Throughout this chapter, we shall
be concerned only with the surfaces which are made up of a finite number
of regular surface elements.
Vector Analysis 299

Now, consider a surface S bounded by a closed curve C (Fig. 8.15).


Let f (x1, x2, x3) be a single valued and continuous function defined on S.
Now, divide the region S into n parts with area ∆Sp (Fig. 8.15), and let
its coordinates be ( x p1 , x p2 , x p3 ). Then, the surface integral of f over S
is defined as
n

∫ fdS = lim
n → ∞ , ∆S p →0
∑ f ( x p , x p , x p ) ∆S p
1 2 3
(25)
C p =1

The limit in Eq. (25) is independent of the way in which the surface
S is divided into parts, because f is single valued and continuous
throughout the region S. Moreover, if f = 1, then Eq. (25) gives rise to
the surface area of S.

Xp
S

∆Sp

Fig. 8.15 Surface bounded by a curve


To evaluate the surface integral given by Eq. (25), let S be a regular
surface element (Fig. 8.16) and X be a general point on it; also, let dS be
an element of area at X. Now, project S onto the region S ′ in the
xy-plane, X and dS projecting into X ′ and dS ′, respectively. Assume that
n be a unit vector normal at X to the surface S, making an acute angle
with z-axis. Then, dS ′ = n3 dS. Let the equation of the surface be
z – g(x, y) = 0 (26)
If we denote the left hand side of Eq. (25) by F, then from Remarks
(ii) and (iii) of Definition 8.25, we have
∂F ∂F ∂F ∂z ∂z
n | ∇F | = ∇ F = i+ j+ k = − i − j+k
∂x ∂y ∂z ∂x ∂y
Thus
−1/2
⎡ ∂z
2
⎛ ∂z ⎞ ⎤
2
n3 = ⎢1 + ⎛⎜ ⎞⎟ + ⎜ ⎟ ⎥ (27)
⎢⎣ ⎝ ∂x ⎠ ⎝ ∂y ⎠ ⎥⎦
We, therefore, have
300 Mathematical Methods

1/2
⎡ ⎛ ∂g ⎞ 2 ⎛ ∂g ⎞ 2 ⎤
(28)
∫ f dS = ∫ f { x, y , g ( x, y )} ⎢1 + ⎜ ⎟ + ⎜ ⎟ ⎥ dS ′
⎢⎣ ⎝ ∂x ⎠ ⎝ ∂y ⎠ ⎥⎦
S S′

To evaluate the integral on the right hand side of Eq. (28), we may replace
dS′ by dxdy, and then perform the double integration with respect to
x and y over the region S ′. Alternatively, one can make use of the polar
coordinates r and θ in the xy-plane and replacing dS′ by rdrd θ.

S dS
X
z

O y

x
S′ X′ dS′

Fig. 8.16 Projection of S in xy-plane


If S is not a regular surface element, we divide it into regular surface
elements. The surface integral of f over these regular surface elements
can be obtained as the sum of the surface integrals of f over these regular
surface elements.
The two sides of a surface can be distinguished - one as the positive
side and the other as the negative side. Let n be a unit vector normal to S
at a point X and lying on the positive side of S. Let a(x1, x2, x3) be a
vector field defined on S. The orthogonal projection of a on n is called
the normal component of a, and is denoted by an so that
an = a ⋅ n (29)
The surface integral of a over S is defined as

∫ an dS = ∫ a ⋅ ndS (30)
S S
Vector Analysis 301

Now, introduce an infinitesimal vector dS defined as dS = ndS, and thus


Eq. (30) takes the form

∫ an dS = ∫ a ⋅ dS (31)
S S
We also have

∫ a dS = i ∫ a1dS + j∫ a2 dS + k ∫ a3 dS (32)
S S S S

∫a × ndS = i ∫ (a2 n3 − a3 n2 ) dS
S S

+ j∫ (a3 n1 − a1n3 ) dS + k ∫ (a1n2 − a2 n1 )dS (33)


S S
Example 8.18 If f = x + z then evaluate the surface integral of f over the
region S consisting of the triangle cut from the plane 6x + 3y + 2z = 6 by
the three planes x = 0, y = 0 and x + y = 1.

z
3

x = 0 plane
y = 0 plane 1
y =
x+
S

2 y
1
z = 0 plane
x
Fig. 8.17 Region S of Example 8.18
Solution The region S is shown in Fig. 8.17. Let G = 6x + 3y + 2z – 6,
then ∂G/∂x = 6, ∂G/∂y = 3, ∂G/∂z = 2, ∇G = 6i + 3j + 2k and
| ∇ G | = 36 + 9 + 4 = 7 so that n = (6/7) i + (3/7) j + (2/7) k which
leads to n3 = 2/7. Thus dS = dxdy/n3 = (7/2) dxdy. But on S, we have
z = 3 – 3x – (3/2)y which gives x + z = 3 – 2x – (3/2)y = f.
Now, if S′ denotes the projection of S on the xy-plane, we then have
7 ⎛ 3 ⎞
∫ fdS = ∫ ⎜ 3 − 2 x − y ⎟ dxdy
2 S′ ⎝ 2 ⎠
S
302 Mathematical Methods

7
1 ⎡1− x ⎛ 3 ⎞ ⎤
2 ∫0
= ⎢ ∫ ⎜ 3 − 2 x − y ⎟ dy ⎥ dx
⎢⎣ 0 ⎝ 2 ⎠ ⎥⎦

1 1− x
7 ⎡ 3 2⎤
2 ∫0
= ⎢ 3 y − 2 xy − 2 y ⎥ dx
⎣ ⎦0

1 1
⎡ 5 x 2 − 14 x + 9 ⎤ dx = ⎡ x3 − 7 x 2 + 9 x ⎤ =
7 7 5 77
=
80∫ ⎣ ⎦ ⎢
8 ⎣3 ⎥
⎦ 0 24

8.4.3 Volume Integral


Consider a region V in space enclosed by a surface S (Fig. 8.18). Let
f (x1, x2, x3) be a single valued and continuous function defined on V.
Now, divide V into n parts with volumes δVp ( p = 1, 2, ..., n). Let Xp be
any point on the element of volume ∆Vp (Fig. 8.18), and let its coordinates
be ( x p , x p , x p ). Then, the volume integral or triple integral of f over
1 2 3
V is defined as
n

∫ fdV = n → ∞lim
, ∆V → 0
∑ f ( x p , x p , x p ) ∆V p 1 2 3
V p =1 p

The limit in the above equation is independent of the way in which the
surface V is divided into parts, because f is single valued and continuous.
We also have

∫ a dV = i ∫ a1dV + j∫ a2 dV + k ∫ a3 dV
V V V V

Example 8.19 If a = (2x 2 – 3z) i – 2xy j – 4xk then find the value of
(i) ∇ ⋅ adV (ii) ∫ ∇ × adV , where V is the closed region bounded by the
∫ V
V
planes 2x + 2y + z = 4, x = 0, y = 0, z = 0.
Solution (i) Here ∇⋅a = 2x and dV = dxdydz. The limits are
z = 0 to z = 4 – 2x – 2y, y = 0 to y = 2 – x, x = 0 to x = 2. Therefore
2 2− x 4 − 2 x − 2 y

∫ ∇ ⋅ a dV = ∫ ∫ ∫ 2 x dx dy dz = 8/3
V 0 0 0
Vector Analysis 303

XP

V ∆Vp

Fig. 8.18 Region V enclosed by a surface S


(ii) Here ∇ × a = j – 2yk and
2 2 − x 4− 2 x − 2 y
8
∫ ∇ × a dV = ∫ ∫ ∫ ( j − 2 yk ) dx dy dz = ( j − k )
3
V 0 0 0

8.4.4 Integral Theorems


We shall now mention few integral theorems. These theorems have a
number of important applications. In fact these theorems tell us that
how a line integral can be converted into a surface integral or a volume
integral can be converted into a surface integral.

Theorem 8.6 Green’s Theorem in Plane. Let S be a closed region in the


xy-plane bounded by a curve C, and a be a vector field which is continuous
and have the continuous first derivatives in the region S. Then

∫k ⋅ (∇ × a) dS = ∫ a ⋅ t ds (34)
S C

where the integration over C has been taken in the positive direction.
Equation (34) is usually expressed as follows:

⎛ ∂a1 ∂a2 ⎞
∫ ⎜⎝ ∂y −
∂x
⎟ dS = − ∫ (a1dx + a2 dy )

(35)
S C

It may be noted that this theorem is also true for the regions bounded by
two or more curves.

Theorem 8.7 Green’s Theorem in Space. Let V be a closed region bounded


by a surface S, and a be a vector field which is continuous and have the
continuous first derivatives in the region V. Then
304 Mathematical Methods

∫ ∇ ⋅ a dV = ∫ a ⋅ n dS (36)
V S

where n is the unit outer normal vector to S.


Since the left hand side of Eq. (36) involves the integration of the
divergence of the vector a, the above theorem is also known as the
divergence theorem. Equation (36) is usually expressed as follows:

⎛ ∂a1 ∂a2 ∂a3 ⎞


∫ ⎜⎝ ∂x +
∂y
+ ⎟ dV = ∫ (a1n1 + a2 n2 + a3 n3 ) dS
∂z ⎠
(37)
V S

Theorem 8.8 Symmetric Form of Green’s Theorem. If F and G are two


scalar fields with continuous second derivatives in a closed region V
bounded by a surface S, then

⎛ dG dF ⎞
∫ (F ∇ G − G∇ F ) dV = ∫ ⎜ F −G
2 2
⎟ dS (38)
V S⎝
dn dn ⎠
where dG/dn is the directional derivative of G in the direction of the
outer normal n to S.

Theorem 8.9 Stoke’s Theorem. Let S be a closed region on a surface, the


boundary of the surface is the curve C. Considering the positive side of
S, let n be the unit vector normal to S. Let t be the unit vector tangent to
C in the positive direction, and let a be a vector field having continuous
first derivatives in the closed region S. Then
(39)
∫n ⋅ (∇ × a) dS = ∫ a ⋅ t ds
S C

where the integration over C has been taken in the positive direction;
while the positive direction of C defined to be that in which an observer
on the positive side of S would travel so as to have the interior of S on his
left.
Equation (39) may also be exprssed as

⎡ ⎛ ∂a3 ∂a2 ⎞ ⎛ ∂a1 ∂a3 ⎞ ⎛ ∂a ∂a ⎞ ⎤


∫ ⎣⎢ n1 ⎜⎝
∂y
− ⎟ + n2 ⎜
∂z ⎠ ⎝ ∂z
− ⎟
∂x ⎠
+ n3 ⎜ 2 − 1 ⎟ ⎥ dS
⎝ ∂x ∂y ⎠ ⎦
S

= – ∫ (a1 dx + a2 dy + a3 dz ) (40)
C
Vector Analysis 305

EXERCISES
1. If ABCD is a quadrilateral where OB – OA = OC – OD, prove
that ABCD is a parallelogram.
2. Prove that the diagonals of a parallelogram bisect each other.
3. The vectors of magnitudes a, 2a, 3a meet at a point and their
directions are along the diagonals of the three adjacent sides of a
cube. Determine their resultant.

[Ans. (4a/ 2)i + (3a / 2) j + (5a / 2)k ]


4. Determine the magnitude of the position vector a = 4i + 3j + 12k.
Also, find a unit vector in the direction of a.
[Ans. 13, (4i + 3j + 12k)/13]
5. If a = 2i – 3j + 4k, b = i – j + 2k and c = 2i + 2j + k, then prove
that a ⋅ (b + c) = a ⋅ b + a ⋅ c.
6. Verify that the vector product is distributive with respect to addition
of vectors when a = 3i + j – k, b = i + 2j + k and c = i – j + 2k.
7. Find the area of the parallelogram whose adjacent sides are a = i –
j + k and b = 2j – 3k. [Ans. 14]
8. Find the area of the triangle having vertices at A(1, 3, 2),
⎡ 1 ⎤
B(2, – 1, 1) and C (– 1, 2, 3). ⎢ Ans. 2 107 ⎥
⎣ ⎦
9. If a = 3i – 4j + 2k, b = 2i + j – k and c = i – 2j + 2k, then obtain
the values of (a × b) × c and a × (b × c).
[Ans. 24i + 7j – 5k, 15i + 15j – 15k]
10. If a = i – 2j – 3k, b = 2i + j – k and c = i + 3j – 2k, then obtain the
value of
(i) | (a × b) × c | (ii) | a × (b × c) |
(iii) a ⋅ (b × c) (iv) (a × b) ⋅ c
(v) (a × b) × (b × c) (vi) (a × b) (b ⋅ c)
[Ans. (i) 5 26 (ii) 3 10 (iii) – 20 (iv) – 20
(v) – 4i – 20j + 20k (vi) 35i – 35j + 35k]
11. Prove that (a + b) ⋅ (b + c) × (c + a) = 2a ⋅ (b × c).
12. Prove that
306 Mathematical Methods

(a × b) ⋅ (c × d) + (b × c) ⋅ (a × d) + (c × a) ⋅ (b × d) = 0
13. Prove that
(a × [b × (c × d)] = (b ⋅ d) (a × c) – (b ⋅ c) (a × d)
14. Prove that
a[b ⋅ (c × d)] – b[c ⋅ (d × a)] + c[d ⋅ (a × b)] – d[a ⋅ (b × c)] = 0
15. Prove that
n ⋅ (b × c) n ⋅ (c × a ) n ⋅ (a × b)
n= a+ b+ c
a ⋅ (b × c ) b ⋅ (c × a ) c ⋅ (a × b)
where n is any vector.
16. Prove that

d 2b d 2a d⎛ d b da ⎞
a× − ×b = ⎜a × − ×b⎟
dt dt2 2
dt ⎝ dt dt ⎠
17. If da/dt = c × a and db/dt = c × b, then show that

d
(a × b) = c × (a × b)
dt
18. If a(u) = (u – u 2)i + 2u3j – 3k, then evaluate
2
(i) ∫ a(u ) du (ii) ∫ a(u ) du
1

⎡ ⎛ u 2 u3 ⎞ u4 5 15 ⎤
⎢ Ans. (i) ⎜⎜ − ⎟i + j − 3uk + C (ii) − i + j − 3k ⎥

⎢⎣ ⎝ 2 3 ⎠ 2 6 2 ⎥⎦
19. The acceleration of a particle at any time t ≥ 0 is a = dv/dt
= 12 cos 2t i – 8 sin 2t j + 16t k. If at t = 0, the velocity v and the
displacement r are zero then find r and v at any time t.
[Ans. v = 6 sin 2t i + (4cos 2t – 4) j + 8t 2k, r = (3 – 3 cos 2t)i
+ (2sin 2t – 4t) j + 8
3 t 3k]

d 2a da
20. Show that ∫ a × dt = a × + C.
dt 2 dt
π /2
21. Evaluate ∫ (3sin ui + 2cosuj) dt. [Ans. 3i + 2j]
0
Vector Analysis 307

22. If f = 3x 2 y – y 3z 2, find ∇ f at the point (1, – 2, – 1).


[Ans. – 12i – 9j – 16k]
23. If a = x yi + yzj + zxk, determine div a at the point (1, 2, 3).
[Ans. 6]
24. If the vector a = (µ x + 3y + 4z)i + (x – 2y + 3z)j + (3x + 2y – z)k
is solonoidal then find the value of µ. [Ans. 3]
25. If a = xz 3i – 2x 2yzj + 2yz 4k then obtain the value of curl a at the
point (1, – 1, 1). [Ans. 3j + 4k]
26. If the vector a = (2x + 3y + λz)i + (µx + 2y + 3z) j + (2x + νy + 3)k
is irrotational then find the value of λ, µ and ν. [Ans. 2, 3, 3]
27. If a = 2xyi + (x 2 – y 2)j, evaluate the line integral of a from the
point A(0, 0) to B(1, 1) along the curve y 2 = x. [Ans. 2/3]
28. If f = x 2 – y 2, evaluate the line integral of f along the line x + 2y
= 2 from the point A(0, 1) to the point B(2, 0). [Ans. 5]
29. If a = 2xyi + (x 2 – y 2) j, evaluate the line integral of a from the
point A(a, 0) to B(0, a) along the curve x 2 + y 2 = a 2. [Ans. – a 3/3]
30. If a = a 2xi + ayzj + xz2k, evaluate the line integral of a along the
curve r = (a cos u) i + (a sin u)j + a(1 – cos u)k, (0 ≤ u ≤ π/2),
where a is a constant. [Ans. – a4/4]
31. If f = x 2 + 2y + z – 1, then evaluate the surface integral of f over the
region S consisting of that part of the plane 2x + 2y + z = 2 lying in
the first octant. [Ans. 3/4]
32. If a = xi + y 2j, then evaluate the surface integral of a over the
region S consisting of the triangle cut from the plane 6x + 3y
+ 2z = 6 by three planes x = 0, y = 0, x + y = 1, the origin being on
the negative side of S. [Ans. 7/12]
33. If a = yi + zj, then evaluate the surface integral of a over the region
S consisting of that part of the plane 2x + 2y + z = 2 lying in the
first octant, the origin being on the negative side of S. [Ans. 1]

34. Verify Green’s theorem in plane for ∫ ( xy + y ) dx + x 2 dy where


2

C is the closed curve of the region bounded by y = x and y = x 2.


308 Mathematical Methods

35. Obtain the value of ∫ a ⋅ n dS where a = 4xzi – y 2j – yzk and S is


S
the surface of the cube bounded by x = 0, x = 1, y = 0, y = 1, z = 0,
z = 1. [Ans. 3/2]

36. Evaluate ∫ r ⋅ n dS , where S is a closed surface.


S

[Ans. 3V, V being the volume enclosed by the surface S]


37. Verify Stoke’s theorem for a = (2x – y) i – yz 2j – y 2zk, where S is
the upper half surface of the sphere x 2 + y 2 + z 2 = 1 and C is its
boundary.
Appendix A : Arithmetic, Geometric and Harmonic Progressions 309

Appendix A : Arithmetic,
Geometric and Harmonic
Progressions

A1 Sequence
A list of numbers arranged in a definite order (i.e., there is a rule by
means of which the numbers after the first number is formed) is called a
sequence. The numbers separated by commas are called terms of the
sequence. A sequence which has a last member is a finite sequence, for
example, 2, 8, 18, 32, 50 is a finite sequence. If the sequence has no last
member, then it is known as infinite sequence, thus, 2, 6, 18, 54, 162, ...
is an infinite sequence. It is possible to express a sequence either by
listing few of its terms till the rule for determining the other terms become
clear; or, by assigning a rule for the nth term of the sequence. Thus, for
example, 1, 3, 5, ... is a sequence whose nth term is 2n – 1, while the
sequence defined by n2, where n is a positive integer, has the terms as 1,
4, 9, 16, 25, ...
We can thus define a sequence to be function whose domain is the
set of natural numbers. A sequence is denoted by the letter ‘a’ and the
image a(n) of n ∈ ¥ under A by an. Since the domain of a sequence is the
set ¥ of natural numbers, therefore a sequence is represented by its range.
The images of 1, 2, 3, ..., n, ... of the terms of a sequence are generally
denoted by a1, a2, a3, ..., an, ... and are, respectively, known as first term,
second term, third term, ..., nth term, ... of the sequence. Thus, for the
sequence 1, 3, 5, ..., we use n as the element of the domain and the
general term is an = 2n – 1. If an is the nth term of a sequence a, then we
write a = {an}.
A sequence whose domain is the set ¥ of natural numbers and the
range is the subset of the set ¡ is called a real sequence.

Example A1 Find the first four terms of the sequence whose general
term is

(− 1)n −1 n +1 x 2 n −1
(i) 2 n– 1 (ii) (iii) (iv)
n +1 n! (2n + 1)!
310 Mathematical Methods

Solution (i) The first term is 21– 1 = 20 = 1, the second term is 22– 1 = 2,
the third term is 22 = 4 and the fourth term is 23 = 8.
(ii) The first term is 1/2, the second term is – 1/3, the third terms is 1/4
and the fourth term is –1/5.
(iii) Here the required terms are
1+1 2 +1 3 +1 4 +1 3 2 5
, , , or 2, , ,
1! 2! 3! 4! 2 3 24
(iv) The first, second, third and fourth terms, respectively, are

x x3 x5 x 7
, , ,
3! 5! 7! 9!
Example A2 Find the general term for each of the following sequences:
(i) 2, 5, 8, 11, 14, ...
(ii) 2.1, 2.01, 2.001, 2.0001, ...
(iii) 1/8, –1/27, 1/64, –1/125, ...
(iv) 2, 2, 6, 2 2, 10, ...
(v) 3/1, 4/2, 5/6, ...
(vi) x, – x 3/3!, x 5/5!, – x 7/7!, ...
Solution (i) The first term is 3 ⋅ 1 – 1, the second term is 3 ⋅ 2 – 1, the
third term is 3 ⋅ 3 – 1 and so on; the general term is 3n – 1.
(ii) The first term is 2 + (1/10), the second term is 2 + (1/10)2, the third
term is 2 + (1/10)3, the fourth term is 2 + (1/10)4 and so on. Thus the
general term is 2 + (1/10)n.
(iii) Here the successive denominators are the cubes of 2, 3, 4, 5, ... or of
1 + 1, 2 + 1, 3 + 1, 4 + 1, ... and the general term is thus (– 1)n–1/(n + 1)3.
(iv) The terms of the given sequence can be expressed as 2, 4, 6, 8,
10, ... Thus, the general term is 2n .
(v) The given sequence can also be written as
1+ 2 2 + 2 3+ 2 4 + 2
, , , ,...
1! 2! 3! 4!
Thus the general term is (n + 2)/n!.
(vi) Here the exponents of x are 2 ⋅ 1 – 1, 2 ⋅ 2 – 1, 2 ⋅ 3 – 1, .... The
general term is therefore (– 1)n – 1 x2n– 1/(2n – 1)!.
Appendix A : Arithmetic, Geometric and Harmonic Progressions 311

It is not always necessary that the terms of a sequence follow a definite


pattern or are defined through some formula for the general term. The
sequences which follow a definite pattern are known as progressions.
In what follows we shall be concerned with the arithmetic, geometric
and harmonic progressions.

A2 Arithmetic Progression
A sequence in which each term after the first term is obtained by adding
a fixed quantity to the preceding term is known as an arithmetic
progression. The fixed quantity is called the common difference.
Thus, if an and an+ 1 are the nth and (n + 1)th terms of a sequence
and an + 1 – an = constant (= d ) for all n ∈ ¥, then the sequence is an
arithmetic progression (A. P.). Here, the constant difference is the common
difference.
The sequence 3, 6, 9, 12, 15, ... is an arithmetic progression because
6 – 3 = 9 – 6 = 12 – 9 = 15 – 12 = ... = 3 which is the common difference.
While, the sequence 5, 10, 14, 20, 25 is not an A.P. since 10 – 5 ≠ 14 –
10.
If a is the first term, d is the common difference and n denotes the
number of terms of an arithmetic progression, then
a, a + d, a + 2d, a + 3d, ..., a + (n – 1)d (1)
are the successive terms of the arithmetic progression. The nth or the
last term l is given by
l = a + (n – 1)d (2)
In order to find whether a sequence is in A.P. or not, we first find an
and then replace n by n + 1 in an to get an + 1. Now, calculate an – an+ 1. If
this difference is independent of n, then the given sequence is in A.P.;
otherwise not.

Example A3 Determine whether the sequences defined by the general


terms an = 4n + 5 and an = 2n 2 + 1 are in A.P. or not.

Solution Given that an = 4n + 5. Replace n by n + 1 in an, we get an+ 1 =


4(n + 1) + 5 = 4n + 9. Now, an + 1 – an = 4 which is independent of n.
Thus the sequence defined by an = 4n + 5 is in A.P. On other hand, for
the sequence defined by an = 2n 2 + 1, we have an+ 1 = 2n 2 + 4n + 3. Here
an+ 1 – an = 4n + 2 which depends upon n, and thus the sequence defined
by an = 2n2 + 1 does not form an arithmetic progression.
312 Mathematical Methods

Consider a sequence {an} whose nth term is a linear expression in


n, i.e., an = An + B, where A and B are constants, then an + 1 = A(n + 1) +
B and this leads to an+ 1 – an = A. This difference is independent of n, and
the sequence {an} thus forms an arithmetic progression with common
difference A. Therefore, we have

Result A1 A sequence {an} is in A.P. if its nth term is a linear expression


in n and in such case the common difference is equal to the coefficient
of n.
Thus, if an = 3n – 2 (a linear expression in n) is the nth term of a
sequence, then according to the above result, this sequence is in A.P.
Here the common difference (the coefficient of n) is 3.

Example A4 Show that the sequence 9, 12, 15, 18, ... forms an arithmetic
progression. Find the 16th and the general terms of this sequence.
Solution Here 12 – 9 = 15 – 12 = 18 – 15 = 3, and the given sequence is
in A.P. with common difference being 3. Also, from Eq. (2), we have
16th term = a16 = a + (16 – 1)d = 9 + 15 × 3 = 54
and
General term = nth term = an = a + (n – 1)d = 3n + 6

Example A5 For the sequence 72, 70, 68, 66, ..., find the term which is
40.
Solution Here 70 – 72 = 68 – 70 = 66 – 68 = – 2 and the given sequence
is in A.P. The common difference is – 2. Let an be the 40th term, then
from Eq. (2), we have 40 = 72 + (n – 1) ( – 2) which on solving leads to
n = 17. Thus the 17th term of the given sequence is 40.

Example A6 Find the number of terms in the sequence 3, 6, 9, 12, ...,


111.
Solution Here, the first term is a = 3 and the common difference is d = 3.
If there are n terms in the sequence, then nth term is an = 111. Now, from
Eq. (2), we have 111 = 3 + (n – 1)3 which on solving leads to
n = 37. Thus there are 37 terms in the given sequence.

Result A2 If a is the first term of an A.P. and d is the common difference,


then the sum Sn of n terms of the arithmetic progression is
n
Sn = [2a + (n – 1)d] (3)
2
Appendix A : Arithmetic, Geometric and Harmonic Progressions 313

or
n
Sn = (a + l ) (4)
2
where l = last term = a + (n – 1) d.

Remark. If the sum Sn of n terms of a sequence is given, then the nth


term an of the sequence is a n = Sn – Sn– 1.

Example A7 Determine the sum of 20 terms of the A.P. 1, 4, 7, 10, ...


Solution Here a = 1, d = 3 and n = 20. Using this data in Eq. (3), we get
S20 = 590.

Example A8 Find the sum of the series 5 + 13 + 21 + ... + 181.


Solution Here a = 5 and d = 8. If there are n terms in this series, then
an = 181 and from Eq. (2), we have n = 23. Thus, from Eq. (4), we have
n 23
Required sum = (a + l ) = (5 + 181) = 2139
2 2
Properties of Arithmetic Progression
We shall now state some of the properties (without proof ) of arithmetic
progression. These properties will be helpful in the subsequent discussions
on arithmetic, geometric and harmonic progressions, and we have

Result A3 If a constant is added or subtracted from each term of an


arithmetic progression, the resulting sequence is also in arithmetic
progression. In both the cases, the common difference is same.

Result A4 If d is the common difference of a given A.P., and if each term


of this given A.P. is multiplied or divided by a non-zero constant c, then
the resulting sequence is also in A.P. having cd or d/c as the common
difference.

Result A5 The sum of the terms equidistant from the beginning and end
of a finite A.P. is always same and equals to the sum of the first and last
term. That is ak + an – (k–1) = a1 + an, for all k = 1, 2, 3, ...,n – 1.

Result A6 Three numbers x, y, z are in A.P. if and only if 2y = x + z.

Result A7 The necessary and sufficient condition for a sequence to be in


A.P. is that sum of its first n terms can be expressed as An 2 + Bn, where
314 Mathematical Methods

A and B are constants independent of n. Here the common difference is


2 times the coefficient of n 2, i.e. 2A.

Result A8 The terms of an arithmetic progression form an A.P. if they


are chosen at the regular intervals.

Result A9 If an, an+1, an+2 are three consecutive terms of an arithmetic


progression, then 2an+1 = an + an+2.

Insertion of Arithmetic Means


1. If between two given quantities x and y, the n quantities X1, X2, ..., Xn
are inserted such that x, X1, X2, ..., Xn, y form an arithmetic progression.
Then, the quantities X1, X2, ..., Xn are called the arithmetic means between
x and y.
2. If X1, X2, ..., X n are n arithmetic means between x and y, then
x, X1, X2, ..., Xn, y is an arithmetic progression. Let d be the common
difference of this A.P. There are (n + 2) terms in this A.P. Then, y = last
term = (n + 2)th term = x + (n + 1)d which yields d = ( y – x)/(n + 1).
Thus, the required arithmetic means between x and y are
y−x
X1 = x + d = x +
n +1
2( y − x)
X2 = x + 2d = x +
n +1
M (5)
n( y − x )
Xn = x + nd = x +
n +1
3. When only one mean is to be inserted between two numbers, then it is
known as the average of two numbers. If X is an arithmetic mean (or
average) of x and y, then x, X, y are in A.P. Thus X – x = y – X which
leads to X = (x + y)/2.

Example A9 Insert three arithmetic means between 3 and 19.


Solution Let X1, X2, X3 be three arithmetic means between 3 and 19,
then 3, X1, X2, X3, 19 are in A.P. Then, d = ( y – x)/(n + 1) = (19 – 3)/
(3 +1) = 4. Thus, from Eq. (5), the required arithmetic means are
X1 = x + d = 3 + 4 = 7, X2 = x + 2d = 3 + 8 = 11, X3 = x + 3d = 3 + 12
= 15
Appendix A : Arithmetic, Geometric and Harmonic Progressions 315

Example A10 The digits of a positive integer, having three digits, form
an arithmetic progression. Their sum is 15. The number obtained by
reversing the digits is 594 less than the original number. Find the number.
Solution Let the digits at ones, tens and hundreds place be (a – d ), a and
(a + d), respectively. Then, the number is
(a + d ) × 100 + a × 10 + (a – d ) = 111a + 99d
The number obtained by reversing the digits is
(a – d ) × 100 + a × 10 + (a + d ) = 111a – 99d
But given that
(a – d ) + a + (a + d ) = 15 and 111a – 99d = 111a + 99d – 594
Solving these equations, we get a = 5 and d = 3. Therefore, the number
is 111a + 99d = 852.

Example A11 A man has to pay a loan of Rs. 3250 in monthly instalments.
In the first month he pays Rs. 20 and then increases the payment by Rs.
15 every month. How long will it take him to clear the loan?
Solution Let the loan be cleared in n months. The amounts that have to
be paid, according to the problem, are 20, 35, 50, 65, ... which is an A.P.
The first term is 20 and the common difference is 15. Also, given that
the sum of the amounts is 3250. Thus from Eq. (4), we have
n
[2 × 20 + (n – 1) × 15] = 3250
2
which on solving leads to n = 20, – 65/3. Therefore, the loan is cleared
in 20 months.

Example A12 A body is dropped and the distance s (in feet) through
which it falls freely in t seconds is given by s = 16t 2. (i) Show that the
distance through which the body falls during the first, second, third,
fourth, ... seconds form an arithmetic progression. (ii) How far the body
will fall in the 10th second?
Solution (i) From the given formula s = 16t 2, by taking t = 1, 2, 3, 4, ...,
the distances covered by the body during the first, second, third, fourth,
... seconds, respectively, are 16, 16(2)2 – 16 = 48, 16(3)2 – 16(2)3 = 80,
16(4)2 – 16(3)2 = 112, and so on. Here, 16, 48, 80, 112, ... are the terms
of an arithmetic progression with common difference as 32.
(ii) When n = 10, the distance covered in 10th second, from Eq. (2)
l = a + (n – 1)d, is 304 ft.
316 Mathematical Methods

A3 Geometric Progression
A sequence of non-zero numbers in which each term after the first is
obtained by multiplying the preceding term by a fixed number is called a
geometric progression. The fixed number is called the common ratio.
The geometric progression can also be defined as a sequence of numbers
in which the ratio of a term and the term preceding to it is always a
constant quantity. This constant ratio is the common ratio of the geometric
progression (G.P.). Thus, a sequence a1, a2, ..., an, ... is a geometric
progression if an+ 1/an = constant, for all n ∈ ¥.
The sequence 4, 12, 36, 108, ... is a geometric progression. Here,
the first term is 4 and the common ratio is 12/4 = 36/12 = 108/36 = ...
= 3, while the sequence 1, 1/4, 1/16, 1/48, ... is not a geometric progression
because (1/16)/(1/4) ≠ (1/48)/(1/16).

Example A13 Show that the sequence defined by an = 3(2n), for all
n ∈ ¥, is a G.P.
Solution Here an = 3(2n) and an+ 1 = 3(2 n + 1). Thus an+ 1/an = 2 and the
given sequence is in G.P.
If a1, a2, ..., an, ... is a geometric progression, then a1 + a2 + ... +
an + ... is known as a geometric series. It may be noted that a geometric
series is finite or infinite according as the corresponding G.P. consists of
finite or infinite number of terms.
If a is the first term of a G.P. and r is its common ratio, then the nth
term of the G.P. is a n = ar n– 1. Thus, geometric progression may be
expressed as
a, ar, ar 2, ar 3, ..., ar n – 1 or a, ar, ar 2, ar 3, ..., ar n– 1 , ...
according as G.P. is finite or infinite.
For a geometric progression, we have

Result A10 If a is the first term and r is the common ratio of a finite G.P.
consisting of m terms, then the nth term from the end is ar m – n.

Result A11 If l is the last term and r is the common ratio of a G.P., then
the nth term from the end is l(1/r)n– 1.

Example A14 Find the 9th and the general terms of the progression 1/4,
– 1/2, 1, –2, ...
Solution Here, the first term is a = 1/4 and the common ratio is r = – 2,
and thus the given progression is a G.P. Also, the 9th term is a9 = ar(9– 1)
= 64 and the general term is an = ar(n – 1) = (– 1)n – 1 2n – 3.
Appendix A : Arithmetic, Geometric and Harmonic Progressions 317

Example A15 Which term of the G.P. 2, 1, 1/2, 1/4, ... is 1/128?
Solution The first term of the given G.P. is a = 2 and the common ratio is
r = 1/2. If nth term is 1/128, then an = ar n– 1 leads to n = 9. Thus, 1/128
is the 9th term of the given G.P.

Example A16 The 4th, 7th and last term of a G.P. are 10, 80 and 2560
respectively. Find the first term and the number of terms in this G.P.
Solution If a is the first term and r the common ratio of the given G.P.,
then a4 = 10, a7 = 80 and ar 3 = 10, ar 6 = 80. Thus ar 6/ar 3 = 80/10 which
leads to r = 2. Put this value of r in ar 3 = 10, we get a = 10/8. Moreover,
if there are n terms in the given G.P., then a n = 2560 and thus
ar n – 1 = 2560 leads to n = 12.

Result A12 If a is the first term and r is the common ratio of a G.P., then
the sum of n terms of this G.P. is
⎛ rn − 1 ⎞ ⎛ 1 − rn ⎞
Sn = a ⎜⎜ ⎟⎟ or Sn = a ⎜⎜ ⎟⎟ , r ≠ 1 (6)
⎝ r −1 ⎠ ⎝ 1− r ⎠
Remarks
1. We also have
⎛ rn − 1 ⎞ ⎛ 1 − rn ⎞
Sn = a ⎜⎜ ⎟⎟ for r > 1 and Sn = a ⎜⎜ ⎟⎟, for r < 1 (7)
⎝ r −1 ⎠ ⎝ 1− r ⎠
The two formulas in Eq. (7) are identical, but do not hold for r = 1. In
case when r = 1, the sum of n terms of a G.P. is Sn = na.
2. If l is the last term of a G.P., then l = ar n– 1 and we have
a − lr lr − a
Sn = or Sn = , r≠1 (8)
1− r r −1
Example A17 Find the seventh term and the sum of the first seven terms
of the G.P., 12, 16, 64/3, ...
Solution Here a = 12, r = 4/3, n = 7. The seventh term is a7 = arn – 1 =
16383/243 and the sum of the first seven terms is
a − rl 56788
S= =
1− r 243
Example A18 Find the sum of the series 2 + 6 + 18 + ... + 4374.
Solution Here a = 2, r = 3 and l = 4374 and the given series is a geometric
series. The required sum is
318 Mathematical Methods

lr − a
Sn = = 6560
r −1
Result A13 If a is the first term and r is the common ratio such that
– 1 < r < 1 or (| r | < 1) of an infinite G.P., then the sum of this G.P. is
a
S = (9)
1− r
It may be noted that if r ≥ 1, then the sum of an infinite G.P. tends to
infinity.

Example A19 Find the sum to infinity of the G.P. – 5/4, 5/16, – 5/64, ...
Solution For the given G.P., the first term is a = – 5/4 and the common
ratio is r = – 1/4. Also, | r | < 1. Thus from Eq. (9), we have
a
S= =–1
1− r
Example A20 Find the sum of the infinite geometric series
1 1 1 1 1 1
+ 2 + 3 + 4 + 5 + 6 + ...∞
2 3 2 3 2 3
Solution The given geometric series can be expressed as
1 1 1 1 1 1
+ + + + + + ...∞
2 32 23 34 25 36
⎛1 1 1 ⎞ ⎛ 1 1 1 ⎞
= ⎜ + 3 + 5 + ... ∞ ⎟ + ⎜ 2 + 4 + 6 + ...∞ ⎟
⎝2 2 2 ⎠ ⎝3 3 3 ⎠
1 1
= (an infinite G.P. with a = ,r = 2)
2 2
1 1
+ (an infinite G.P. with a = 2
,r = )
3 32
⎡ (1/ 2) ⎤ ⎡ (1/32 ) ⎤ 19
= ⎢ 2 ⎥
+⎢ 2 ⎥
=
⎣ 1 − (1/ 2 ) ⎦ ⎣ 1 − (1/3 ) ⎦ 24
(using Eq. (5)).

Example A21 A tennis ball is dropped from a height of 81 ft. After


striking the ground each time, the ball rebounds 2/3rd of the distance
through which it has last fell, then find (i) the distance through which
Appendix A : Arithmetic, Geometric and Harmonic Progressions 319

did the ball fall when it struck the ground for the sixth time; and (ii) the
distance covered by the ball from the time when it was dropped until it
struck the ground for the sixth time.
Solution (i) The successive distance through which the ball falls form a
geometric progression for which the first term is a = 81 and the common
ratio is r = 2/3. When n = 6, l = ar n– 1 = 32/3 ft.
(ii) The sum of the distances for the first six falls and first five rebounds
is the required distance and we have
a(1 − r n ) 665
For the falls: a = 81, r = 2/3, n = 6 and S = =
1− r 3
a(1 − r n ) 422
For the rebounds: a = 54, r = 2/3, n = 5 and S = =
1− r 3
Therefore the total distance covered by the ball during the falls and
rebounds is (665/3) + (422/3) = 1087/3 ft.

Properties of Geometric Progression


We shall now mention some of the properties of the geometric progression
and geometric series. We have

Result A14 If all the terms of a geometric progression are multiplied by


the same non-zero constant, then the result is also a geometric progression
having the same common ratio as the previous one.
That is, if a1, a2, ..., an, ... form a G.P., and k is a non-zero constant, then
ka1, ka2, ..., kan, ... also form a G.P. The common ratio in both the cases
remains the same.

Result A15 The reciprocals of the terms of a given geometric progression


is also a geometric progression.

Result A16 If each term of a geometric progression is raised to the same


power, then the resulting expression also forms a geometric progression.
Thus, if a1, a2, ..., an, ... form a G.P., and k is a non-zero constant, then
a1k , a2k , ..., ank , ... also form a G.P. The common ratios in both of these
expressions remain the same.

Result A17 In a finite geometric progression, the product of the terms


equidistant from the beginning and the end is always same and equals to
the product of the first and the last term.
320 Mathematical Methods

Result A18 Three non-zero numbers x, y, z form a geometric progression


if and only if y 2 = xz.
From this result, it may be noted that if three non-zero numbers x, y, z
form a geometric progression, then y is known as the geometric mean of
x and z.

Result A19 If the terms of a geometric progression are chosen at regular


intervals, then the resulting sequence also forms a geometric progression.
The common ratio of these two geometric progressions remains the same.

Insertion of Geometric Means


1. If n numbers G1, G2, ..., Gn are inserted between two given numbers a
and b such that the sequence a, G1, G2, ..., Gn, b is a geometric progression,
then the numbers G1, G2, ..., Gn are known as n geometric means (G.M.)
between a and b.
If a single number G is inserted between two given numbers a and b
such that a, G, b form a geometric progression, then G is the geometric
mean of a and b and in this case G = ab .
It may be noted that, if two numbers are of opposite signs, then the
geometric mean between them does not exist.

2. If G1, G2, ..., Gn are n geometric means between two given numbers a
and b then a, G1, G2, ..., Gn, b is a geometric progression. Also, if r is the
common ratio of this geometric progression, then
b = (n + 2)th term = ar n + 1 ⇒ r n + 1 = b/a
Therefore
1/( n +1) 2/( n +1) n /( n +1)
⎛b⎞ ⎛b⎞ ⎛b⎞
G1 = ar = a ⎜ ⎟ , G2 = ar 2 = a ⎜ ⎟ ,..., Gn = ar n = a ⎜ ⎟
⎝a⎠ ⎝a⎠ ⎝a⎠
3. If n geometric means are inserted between two numbers, then the
product of n geometric means is the nth power of the single geometric
mean between the two numbers. Thus, if G1, G2, ..., Gn are n geometric
means between the numbers a and b, then G1G2 ... Gn = ( ab ) n = G n,
where G = ab is the single geometric mean between a and b.

4. The arithmetic and geometric means A and G, respectively between


two positive numbers a and b have the following properties:
(i) A > G.
(ii) The quadratic equation whose roots are a and b is x 2 – 2Ax + G 2 = 0.
Appendix A : Arithmetic, Geometric and Harmonic Progressions 321

5. If A and G are the arithmetic and geometric means between two positive
numbers, then the numbers are A ± A2 − G 2 .
Example A22 Insert four geometric means between 576 and 9.
Solution If G1, G2, G3, G4 are four geometric means between a = 576
and b = 9, then 576, G1, G2, G3, G4, 9 form a geometric progression
whose common ratio is r = 1/2 (using the formula r = (b/a)1/(n + 1)). Thus
G1 = ar = 288, G2 = ar 2 = 144, G3 = ar 3 = 72, G4 = ar 4 = 36.
Therefore 288, 144, 72, 36 are the desired geometric means between 576
and 9.

Example A23 Find two positive numbers whose arithmetic mean is 34


and geometric mean is 16.

Solution Let x and y be two numbers. But given that arithmetic mean is
34 and geometric mean is 16, that is
x+ y
= 34 so that x + y = 68
2
and
xy = 16 so that xy = 256
Therefore
(x – y)2 = (x + y)2 – 4xy = 3600 so that x – y = 60
Now, solving the two equations
x + y = 68
x – y = 60
for x and y, we get x = 64 and y = 4.

A4 Harmonic Progression
If the reciprocals of a set of numbers form an arithmetic progression,
then this set of numbers is said to form a harmonic progression. Thus in
order to solve a problem of harmonic progression, first take the reciprocals
and then apply the appropriate formula of arithmetic progression.

Example A24 Show that 2, 2/3, 2/5, ... to 30 terms is a harmonic


progression. Find also the last term.

Solution It can be seen very easily that the reciprocals 1/2, 3/2, 5/2, ...
form an arithmetic progression whose 30th term is 59/2. Thus the 30th
term of harmonic progression is 2/59.
322 Mathematical Methods

Example A25 Find the harmonic mean between 2 and 4.


Solution Let H be the required harmonic mean. Since

1 1 + 1
3
= 2 4
=
H 2 8
is the arithmetic mean between 1/2 and 1/4, therefore H = 8/3 is the
harmonic mean between 2 and 4.

EXERCISES
1. Write the first four terms of the sequences whose general terms
are 4n – 3; n!; (– 1)n – 1 x 2n – 2/(n – 1)!
[Ans. 1, 5, 9, 13; 1, 2, 6, 24; 1, – x 2, x 4/2, – x 6/6]
2. Write the general term for each of the sequences: 2, 4, 6, 8, 10, 12,
...; 2, 6, 18, 54, 162, ...; 2, – 5, 8, – 11, 14, ...; x/2, x2/6, x 3/24,
x 4/120, ...; 3, 4, 5/2, 1, 7/24, ...
[Ans. 2n, 2(3)n– 1, (– 1)n– 1 (3n – 1), x n/(n + 1)!, (n + 2)/(n – 1)!]

⎛ a2 ⎞ ⎛ a3 ⎞
3. Show that the sequence log a, log ⎜ ⎟ , log ⎜ ⎟ , ... forms an
⎜ b ⎟ ⎜ b2 ⎟
⎝ ⎠ ⎝ ⎠
arithmetic progression.
4. Show that the sequence log a, log (ab), log (ab 2), log(ab3), ...
forms an arithmetic progression. Find the nth term.
[Ans. log (abn–1)]
5. Which term of the sequence 4, 9, 14, 19, ... is 124.
[Ans. 25th term]
6. Is 184 a term of the sequence 3, 7, 11, ...? [Ans. No]
7. Find the sum of all the natural numbers between 250 and 1000
which are exactly divisible by 3. [Ans. 156375]
8. Insert six arithmetic means between 7 and 77.
[Ans. 17, 27, 37, 47, 57, 67]
9. What is the arithmetic mean of 8 and – 56. [Ans. – 24]
10. Show that the following sequences form a geometric progression.
Find next three terms also:
(i) 4, 8, 16, 32, 64 (ii) 12, – 4, 4/3, – 4/9
[Ans. (i) 128,256, 512 (ii) 4/27, – 4/81, 4/243]
Appendix A : Arithmetic, Geometric and Harmonic Progressions 323

11. Find the 4th term from the end of the G.P. 3, 6, 12, 24, ..., 3072.
[Ans. 384]
12. Which term of the G.P. 5, 10, 20, 40, ... is 5120? [Ans. 11th terms]
13. The first term of a G.P. is 1. The sum of the third and fifth terms is
90. Find the common ratio. [Ans. ± 3]
14. If the fourth and ninth terms of a G.P. are 54 and 13122, respectively.
Find the G.P. [Ans. 2, 6, 18, 54, ...]
15. The fourth and eighth terms of a G.P. are 1 and 1/256, respectively.
Find the tenth term. [Ans. 1/4096]
16. Find the sum of 10 terms of the G.P. 1, 1/2, 1/4, 1/8, ...
[Ans. 1023/512]
17. Find the sum of the G.P. 8, – 4, 2, ..., 1/128. [Ans. 683/128]
18. Insert five geometric means between 8 and 1/8.
[Ans. – 4, 2, – 1, 1/2, – 1/4]
19. Insert four geometric means between 81/2 and – 16/3.
[Ans. – 27, 18, – 12, 8]
20. Find the geometric mean of 1/3 and 243. [Ans. 9]
21. Find two positive numbers whose difference is 12 and whose
arithmetic mean exceeds the geometric mean by 2. [Ans. 16, 4]
22. The sum of three numbers in arithmetic progression is 24. If the
first number is decreased by 1 and the second by 2, then the three
numbers are in geometric progression. Find the arithmetic
progression. [Ans. 4, 8, 12 or 13, 8, 3]
23. Find the 16th term of a harmonic progression 3/4, 3/11, 1/6, ...
[Ans. 3/109]
24. Find the 20th term in the harmonic progression whose first term
is 1 and whose 13th term is 1/19. [Ans. 2/59]
25. Show that the harmonic mean between x and y is 2xy/(x + y).
26. Show that the geometric mean between two positive numbers is
also the geometric mean between the arithmetic and harmonic
means of the numbers.
324 Mathematical Methods

Appendix B : Permutation,
Combination and Binomial
Theorem

B1 Permutation
Any arrangement of a set of objects in a specific order is known as a
permutation of the set taken all at a time. For example, xyz, yzx, zxy are
permutations of the set of letters x, y, z taken all at a time.
It may be noted that in a permutation, the order of the arrangement
is important. If the order of the arrangement is changed, we get a different
permutation.

Example B1 From the letters a, e, i, o, u obtain the possible permutations


when three letters are taken at a time and starting with the letter a.
Solution The required permutations are
aei, aie, aeo, aoe, aeu, aue, aio, aoi, aiu, aui, aou, auo

Notations
1. We shall use the symbol nPr or P(n, r) to denote the number of all
permutations of n distinct objects, taken r at a time, where n and r are
positive integers such that 1 ≤ r ≤ n.
2. The symbol n! (read as factorial n) denotes the product of the positive
integers from 1 to n inclusive. That is
n! = 1 ⋅ 2 ⋅ 3⋅ ... ⋅ (n – 1) ⋅ n (1)
Thus
1! = 1, 3! = 1⋅ 2 ⋅ 3, 5! = 1 ⋅ 2 ⋅ 3 ⋅ 4 ⋅ 5, etc.
From Eq. (1) we have
n! = 1 ⋅ 2 ⋅ 3 ⋅ ... ⋅ (n – 1) ⋅ n = [(n – 1)!] n = n(n – 1)! (2)
Thus
9! = 9(8!), 7! = 7(6!), 5! = 5(4!), 3! = 3(2!), etc.
It may be noted that the factorial notation is not defined for proper
fractions or negative integers.
We also have
Appendix B : Permutation, Combination and Binomial Theorem 325

Result B1 The number of permutations of n different objects taken r at a


time, when r < n, is given by

nP
n(n − 1) ... (n − r + 1) (n − r )...2 ⋅ 1
r = n(n –1) ... (n – r + 1) =
(n − r ) ...2 ⋅ 1
n!
= (3)
(n − r )!
and the number of permutations of n different objects taken n at a time is
given by
nP = n(n – 1) ... 2 ⋅ 1 = n! (4)
n
Remark
Take r = n in Eq. (3), we get
n! n!
nP
n
= =
(n − n)! 0!
But from Eq. (4), nP n = n! and the above equation becomes n!
= n!/0! and thus
n!
0! = =1 (5)
n!
Example B2 Evaluate (i) 20!/18! (ii) 10!/6!4!
Solution From Eq. (2), we have
20! 20(19!) (20) (19) (18!)
= = = (20 (19) = 380
18! 18! 18!
and
10! (10) (9) (8) (7)6! (10) (9) (8) (7)
= = = 210
6!4! 6!4! (4) (3) (2) (1)
Example B3 Find n, if (n + 2)! = 2550 × n!.
Solution We have
(n + 2)! = 2550 × n! ⇒ (n + 2) (n + 1) × n! = 2550 × n!
⇒ (n + 2) (n + 1) = 2550 ⇒ n 2 + 3n – 2548 = 0 which on solving yields
n = 49 as n ≥ 0.

Example B4 Evaluate 5P3.


Solution We have
5! 5!
5P
3 = = = 60d
(5 − 3)! 2!
326 Mathematical Methods

Example B5 If 2 5P3 = nP4, find n.


Solution From the definition, we have
n! 5!
nP
4 = and 2 5P3 = 2
(n − 4)! (5 − 3)!
Thus nP4 = 2 5P3 leads to
n(n − 1) (n − 2) (n − 3) (n − 4)! 2(5)!
= ⇒ n(n – 1) (n – 2) (n – 3) = 5!
(n − 4)! 2!
which may be expressed as
n(n – 1) (n – 2) (n – 3) = 5(5 – 1) (5 – 2) (5 – 3)
so that on comparing the two sides, we get n = 5.

Example B6 If P(n – 1, 3) : P(n, 4) = 1 : 9, find n.


Solution We have
P(n − 1,3) 1
P(n – 1, 3) : P(n, 4) = 1 : 9 ⇒ =
P (n,4) 9
which from the definition of P(n, r) can be expressed as
( n −1)!
( n −1−3)! 1 (n − 1)! 1 (n − 1)! 1
n!
= ⇒ = ⇒ =
( n − 4)!
9 n! 9 n(n − 1)! 9
so that n = 9.
Example B7 Prove that
(i) P (n, n) = 2P (n, n – 2) (ii) P (n, n) = P (n, n – 1)
(iii) P(n, r) = P(n – 1, r) + rP(n – 1, r – 1) (iv) P(n, r) = nP(n – 1, r – 1)

Solution
(i) From the definition, we have
n! ⎛ n! ⎞
2 P (n, n – 2) = 2 = 2⎜ ⎟ = n! = P(n, n)
[n − (n − 2)]! ⎝ 2! ⎠
(ii) We have
n!
P (n, n – 1) = = n! = P(n, n)
[n − (n − 1)]!
(iii) From the definition, we have
(n − 1)! (n − 1)!
P (n – 1, r) + r P(n – 1, r – 1) = +r
(n − 1 − r )! [(n − 1) − (r − 1)]!
Appendix B : Permutation, Combination and Binomial Theorem 327

(n − 1)! r (n − 1)! (n − 1)! ⎡ r ⎤


= + = 1+
(n − r − 1)! (n − r ) (n − r − 1)! (n − r − 1)! ⎢⎣ n − r ⎥⎦
(n − 1)! n n!
= = = P(n, r)
(n − r − 1)! n − r (n − r )!
(iv) We have
(n − 1)! n!
nP(n – 1, r – 1) = n = = P (n, r)
[(n − 1) − (r − 1)]! (n − r )!

Example B8 If Pk denotes kPk , then prove that


1 + 1 ⋅ P1 + 2 ⋅ P2 + 3 ⋅ P3 + ... + n ⋅ Pn = (n + 1)!
Solution Since Pk = kPk = k!, we have
1 + 1⋅ P1 + 2⋅ P2 + 3 ⋅ P3 + ... n ⋅ Pn = 1 + 1 + 2⋅ 2! + 3 ⋅ 3! + 4 ⋅ 4! + ...
+ n ⋅ n!
n n n n
= 1 + ∑ r ⋅ r! = 1 + ∑[(r + 1) − 1)r! = 1 + ∑[(r + 1)r! − r!] = 1 + ∑[(r + 1)! − r!]
r =1 r =1 r =1 r =1
= 1 + [(2! – 1!) + (3! – 2!) + (4! – 3!) + ... + {(n + 1)! – n!}] = 1 + [(n + 1)! – 1!]
= (n + 1)!

Example B9 Seven persons are participating in a quiz competition. In


how many ways can the first prize be won?
Solution The total number of ways in which the first three prizes can be
won is the number of arrangements of seven different objects taken three
at a time. Thus, the required number of arrangements is 7P3 and we have
7! 7!
7P
3 = = = 210
(7 − 3)! 4!
Example B10 Find the number of different signals that can be made by
5 flags from 8 flags of different colours.
Solution The total number of signals is the number of arrangments of 8
flags by considering 5 flags at a time, which is 8P5 and thus, from the
definition, we have 8P5 = 6720.

Example B11 Three men have four shirts, five trousers and six caps. In
how many ways can they wear them?
Solution The total number of ways in which three men can wear four
shirts is the number of arrangements of four different shirts taken three
at a time. Thus, three men wear four shirts in 4P3 different ways. Similarly,
328 Mathematical Methods
5P and 6P3 are the number of ways in which three men can wear five
3
trousers and six caps, respectively. Therefore, the required number of
ways = 4P3 × 5P3 × 6P3 = 172800.

Example B12 How many numbers lying between 100 and 1000 can be
formed with the digits 1, 2, 3, 4, 5 if the repetition is not allowed?

Solution Since every number lying between 100 and 1000 is a three
digit number, therefore, we have to obtain the number of permutations
of five digits 1, 2, 3, 4, 5 taken three at a time. Hence, the required
number of permutation is 5P3 = 60.

Example B13 How many words can be formed using all the letters of
the word EQUATION, using each letter exactly once (the meanings of
the words thus formed are not taken into consideration).

Solution The word EQUATION has eight letters in it. Thus, the total
number of words is equal to the number of arrangements of these letters
taken all at a time, and the number of such arrangements is 8P8 = 8!.

B2 Permutation Under Certain Conditions


Here we shall consider the permutations of those situations in which
either the repetitions of objects are allowed, or distinction between some
objects are ignored, or a particular object occurs in every arrangements,
etc. We can also have the situation where the permutations of a given
number of objects is considered such that the objects under consideration
are not all different. The following results will be helpful to deal with
such situations.

Result B2 When a particular object is to be included always in each


arrangement, then the number of all permutations of n different objects
taken r at a time is r (n – 1 Pr – 1).

Result B3 When a particular object is never included in each arrangement,


then the number of all permutations of n different objects taken r at a
time is n – 1Pr.

Result B4 When two specified objects always occur together, then the
number of all permutations of n different objects taken r at a time is
2!(r – 1) (n – 2Pr – 2).

Example B14 Find the number of ways in which the letters of the word
Appendix B : Permutation, Combination and Binomial Theorem 329

PENCIL be arranged such that (i) N is always next to E (ii) N and E are
always together.

Solution (i) Consider the letters EN as one letter, then we have five
letters which can be arranged in 5P5 = 120 ways. Thus, the number of
ways N is always next to E is 120.
(ii) Take E and N together and consider it as one letter, then we have five
letters which can be arranged in 5P5 = 5! ways. But E and N can be put
together in 2! ways (i.e., EN, NE). Therefore, the total number of ways is
5! × 2! = 240.

Example B15 How many words can be formed from the letters of the
word TRIANGLE and how many of these will begin with T and end
with E?

Solution The word TRIANGLE consists of eight letters. The total number
of words that can be formed with these eight letters is the number of
arrangements of eight letters taken all at a time, which is equal to
8P = 8! = 40320. Moreover, if we fix the letter T in the beginning and
8
the letter E at the end, then the six letters are left which can then be
arranged in 6P6 ways. Thus, the total number of words which begin with
the letter T and end with the letter E is 6!.

Result B5 If there are l objects out of which m are alike of one kind and
n are alike of second kind such that m + n = l, then the number of mutually
distinguishable permutations of l objects taken all at a time is l!/m!n!.

Result B6 If there are l objects out of which m1 are alike of one kind, m2
are alike of second kind, m3 are alike of third kind and so on such that
m1 + m2 + m3+ ... + mr = l, then the number of permutations of l objects
is l!/m1! m2! ... mr!.

Result B7 If there are l objects out of which m1 are alike of one kind, m2
are alike of second kind and the remaining are all distinct, then the
number of permutations of l objects is l!/m1!m2!.

Result B8 If there are r objects which have to be arranged and the


repetition is allowed. Also, if m1, m2, m3, ..., mr are the integers such that
the first object occurs exactly m1 times, the second occurs exactly m2
times and so on, then the total number of permutations of these r objects
is
330 Mathematical Methods

(m1 + m2 + ... + mr )!
m1 !m2 !... mr !
Result B9 The number of permutations of n different objects taken r at a
time, when each may be repeated any number of times in each
arrangement, is nr.

Example B16 Find the number of words that can be formed out of the
letters of the word MISSISSIPPI.

Solution The given word contains eleven letters and in this word there
are four S, four I and two P. Thus the total number of words is the number
of arrangements of eleven words, of which similar four are of one kind,
similar four are of second kind and similar two are of third kind. Hence
11!
total number of words = = 34650
4!4!2!
Example B17 Find the number of arrangements that can be made from
the letters of the word MATHEMATICS.

Solution The word MATHEMATICS contains eleven letters, out of which


there are two M, two A, two T and all other are distinct. Thus
11!
total number of arrangements = = 4989600
(2!) (2!) (2!)
Example B18 If all the letters of the word AGAIN are arranged as in a
dictionary, then find 50th word.

Solution We know that the word, at each stage, in a dictionary are


arranged in alphabetical order, so starting with the letter A and arranging
the four letters GAIN, we get 4! = 24 words. Thus there are 24 words
which start with A. These are the first 24 words. Then, starting with the
letter G and arranging the other four letters A, A, I, N in different ways,
we get 4!/2! = 12 words, which shows that there are 12 words which
start with G. Now, start with the letter I, then the remaining four letters
A, G, A, N can be arranged in 4!/2! = 12 ways. This means that there are
12 words which start with the letter I.
So far thus we have constructed fortyeight words. The 49th word is
NAAGI and hence the 50th word is NAAIG.

Example B19 Find the number of three digits that can be formed from
the digits 1, 2, 3, 4, 5 when the repetition of digits is allowed.
Appendix B : Permutation, Combination and Binomial Theorem 331

Solution The unit’s place can be filled in five ways. As the repetition of
the digits are allowed, therefore, ten’s place and hundred’s place can be
filled in five ways. Thus, the number of three digit numbers is 5 × 5 × 5
= 125.

B3 Combination
A selection of objects which is made by taking all or a number of objects,
irrespective of their arrangements, is known as a combination. For
example, the different combinations of the three letters xyz taken two at
a time are xy, xz, yz.
It may be noted that for the combination, the word selection is used,
while for the permutation, the word arrangement is used.

Notation
The number of all combinations of n objects, taken r at a time is denoted
by C(n, r), or nCr. Thus, nCr denotes the number of ways of selecting r
objects from n objects. Moreover, nCr is defined only when n and r are
non-negative integers such that 0 ≤ r ≤ n.

Comparison between a permutation and a combination


(i) A combination consists only of a selection, while a permutation consists
of not only a selection but also involves an arrangement in a definite
order.
(ii) The ordering of the objects is immaterial in a combination, while in
a permutation the ordering is essential.
(iii) While finding the permutations of n different objects taken r at a
time, we first select r objects from n objects and then arrange them.
Thus, usually the number of permutations exceeds the number of
combinations.
(iv) Each combination corresponds to many permutations. Thus, for
example the permutations xyz, xzy, yzx, yxz, zyx and zxy correspond to
the same combination xyz.
We also have

Result B10 The number of all combinations of n distinct objects taken r


at a time is
n! n(n − 1) (n − 2)...(n − r + 1)
nC
r = = (6)
(n − r )!r! 1 ⋅ 2 ⋅ 3... r
332 Mathematical Methods

Remarks
1. Put r = n in Eq. (6), we get
n! n!
nC
n = = =1 (7)
(n − n)!n! ( n!) (0!)
2. Put r = 0 in Eq. (6), we get
n! n!
nC = = (8)
0 (n − 0)!0! n!
Thus
nC nC
n= 0 =1
3. From Eq. (6), we have

n! 1 ⎡ n! ⎤ n Pr
nC = = ⎢ = (9)
r
(n − r )! r! r! ⎣ (n − r )! ⎥⎦ r!
Result B11 For 0 ≤ r ≤ n,
nC
= nC n – r
r
This result can also be stated as : If x and y are non-negative integers
such that x + y = n, then nCx = nCy and conversely.

Example B20 Find the value of n, if nC7 = nC4.


Solution Since nC7 = nC4, then from the above result, we have n = 7 + 4
= 11.

Result B12 If r and n are non-negative integers such that r ≤ n, then


nC + nC n+ 1C
r r– 1 = r

Result B13 If r and n are non-negative integers such that 1 ≤ r ≤ n, then


n n– 1
nC
r = Cr– 1
r
Result B14 If r and n are non-negative integers such that 1 ≤ r ≤ n, then

n (n– 1 Cr – 1) = (n – r + 1) nCr – 1

Result B15 If nCx = nCy, then either x = y or x + y = n.


From this result, it may be noted that if nCx = nCy and x ≠ y, then
x + y = n.

Example B21 If nCr = 120 and nPr = 720, find the value of r.
Appendix B : Permutation, Combination and Binomial Theorem 333

Solution From Eq. (9), we have


720
120 = which leads to r! = 6 so that r! = 3! and r = 3
r!

Example B22 Show that

2nC
2n [1 ⋅ 3 ⋅ 5...(2n − 1)]
n= n!
Solution From the definition, we have
2n! (2n!) (2n) (2n − 1) (2n − 2)...4 ⋅ 3 ⋅ 2 ⋅ 1
2nC
n = = =
(2n − n)!n! n!n! n!n!

[1 ⋅ 3 ⋅ 5...(2n − 1)] (2 ⋅ 4 ⋅ 6...2n) [1 ⋅ 3 ⋅ 5...(2n − 1)] 2n (1 ⋅ 2 ⋅ 3... n)


= =
n!n! n! n!

[1 ⋅ 3 ⋅ 5 ... (2n − 1)] 2n n! 2n [1 ⋅ 3 ⋅ 5 ... (2n − 1)]


= =
n!n! n!
Example B23 Find rC2 when nCr – 1 = 36, nCr = 84 and nCr + 1 = 126.

Solution Since

n
Cr r +1
n
=
Cr +1 n−r

Therefore

r + 1 84
=
n − r 126

which on solving leads to

2n – 5r = 3 (10)

n
Cr
Now put r = r – 1 in , we get
n
Cr +1
n
Cr −1 r 36
n
= =
Cr n − (r − 1) 84
334 Mathematical Methods

which on solving yields


3n – 10r = – 3 (11)
Solving Eqs. (10) and (11), we get r = 3. Thus

rC
3!
2 = 3C 2 = =3
(3 − 2)!2!
Example B24 Determine the values of n and r, if nPr = nPr + 1 and
nC = nC .
r r–1
Solution Since nPr = nPr + 1 and nCr = nCr – 1 , we have
n! n! n! n!
= and =
(n − r )! (n − r − 1)! (n − r )!r! (n − r + 1)!(r − 1)!
which, after simplifications, leads to
n – r = 1 and n – 2r = – 1
Solving these two equations, we get n = 3 and r = 2.

Example B25 A question paper has two parts, part A and part B. Each
part contains ten questions. If the student has to choose eight questions
from part A and five questions from part B, then find the number of ways
in which the student can choose the questions.

Solution There are ten questions in part A out of which the student can
choose eight questions in 10C8 ways, and similarly the student can choose
five questions from part B in 10C5 ways. Thus, the total number of ways
in which the student can select questions from part A and part B is
10C × 10C = 11340.
8 5

Example B26 Twenty three applications are being received against the
five posts of lecturers in an institution. Out of these five posts, two are
reserved for the candidates belonging to backward classes. If there are
seven candidates belonging to backward class (BC) among the twenty
three applicants, find the number of ways in which the selection can be
made.

Solution In this example, there are seven BC applicants and sixteen


other applicants, and we have to select two out of seven BC applicants
and three out of sixteen other applicants. This can be done in 7C2 × 16C3
ways. Thus, the total number of ways in which the selection can be made
is 7C2 × 16C3 = 11760.

Example B27 Find the number of triangles that can be formed by joining
the vertices of a hexagon.
Appendix B : Permutation, Combination and Binomial Theorem 335

Solution Since there are six vertices of a hexagon, one triangle can be
formed by selecting a group of three vertices from the given six vertices.
This can be done in 6C3 ways. Thus

6!
total number of triangles = 6C3= = 20
3!3!
Example B28 Find the number of diagonals in a n-sided polygon.
Solution It is known that a n-sided polygon has n vertices. By joining
any two vertices of the polygon, we either get a side of the polygon, or a
diagonal of the polygon. Thus, the number of line segments obtained by
joining the vertices of a n sided polygon taken two at a time is equal to
n ( n −1)
the number of ways of selecting two out of n, which is nC2 = 2 . Out
of these lines, n lines are the sides of the polygon. Therefore, number of
n(n − 1) n(n − 3)
the diagonals of the polygon = −n= .
2 2
Example B29 There are ten points P, Q, R, ... in a plane such that no
three points lie on a straight line. Find the number of (i) lines which are
determined by these points (ii) lines which are passing through the point
P(iii) triangles determined by the points (iv) triangles having P as a
vertex (v) triangles having PQ as a side.
Solution (i) Since a line is determined by any two points, therefore there
are 10C2 = 45 lines.
(ii) To find a line through P, one more point is to be selected. Thus there
are nine lines through the point P.
(iii) Since a triangle is formed by joining any three points, therefore
there are 10C3 = 120 triangles.
(iv) Here two more points are needed to form a triangle. These two points
may be selected from the nine points in 9C2 = 36 ways, which is the
required number of triangles having P as the vertex.
(v) Here only one additional point is needed to form a triangle. Thus
there are eight triangles having PQ as a side.
Example B30 There are 21 consonants and 5 vowels in the English
alphabet. Determine (i) in how many ways can four consonants and two
vowels be selected? (ii) the number of words formed which consist of
336 Mathematical Methods

four consonants and two vowels (iii) the number of words in (ii) which
begin with R (iv) the number of words in (iii) which contain E.
Solution (i) The four consonants can be selected in 21C4 ways and two
vowels can be selected in 5C2 ways. Thus, the required number of selection
is 21C4 × 5C2 = 59850.
(ii) From each of the selection in (i) above, 6! words may be formed by
permuting the letters. Therefore, (59850) (6!) = 43092000 words can be
formed.
(iii) Since the position of the consonant R is fixed, we must select three
other consonants in 20C3 ways; and two vowels in 5C2 ways. Now, arrange
each selection of five letters in all possible ways. Thus, the required
number of words are 20C3 × 5C2 × 5! = 1368000.
(iv) Here the position of the consonant R is fixed while the position of
the vowel E is not fixed. Thus, three other consonants and one other
vowel must be selected in 20C3 and 4 ways, respectively. Now, arrange
each set of five letters in all possible ways. Therefore, the required number
of words are 20C3 × 4 × 5! = 547200.

B4 Binomial Theorem
An algebraic expression which contains two terms is known as a binomial
⎛ 1 ⎞ ⎛ 16 6
expression. For example, (x + y), (3a – 5b), ⎜ a + ⎟ , ⎜ +
⎞ etc.,
⎟,
⎝ b ⎠ ⎝ x x2 ⎠
are binomial expressions.

Consider few powers of the binomial (x + a) as follows:


(x + a)0 = 1
(x + a)1 = x + a
(x + a)2 = x 2 + 2ax + a2
(x + a)3 = x 3 + 3x 2a + 3xa 2 + a 3
(x + a)4 = x 4 + 4x 3a + 6x 2a 2 + 4xa 3 + a 4
(x + a)5 = x 5 + 5x 4a + 10x 3a 2 + 10x 2a 3 + 5xa4 + a 5
Now, observe the pattern of coefficients in the powers of (x + a)
from 0 to 5 in the above display. If we remove all and retain the
coefficients, we get the following pattern
0th row 1
1st row 1 1
2nd row 1 2 1
Appendix B : Permutation, Combination and Binomial Theorem 337

3rd row 1 3 3 1
4th row 1 4 6 4 1
5th row 1 5 10 10 5 1
In the the above pattern, every row starts with 1 and ends also with
1; and that each of the interior number is the sum of the two numbers
directly above it. Such type of pattern is known as Pascal triangle. This
triangle has a number of interesting properties - one of which is that it
provides coefficients for the expression (x + a)n. Note, for example, that
the 5th row gives the coefficients for (x + a)5. Also, the fifth row can be
expressed as
5C 5C 5C 5C 5C 5C
0 1 2 3 4 5
n
where Cr, (n = 5 and r = 0, 1, 2, 3, 4, 5) is defined by Eq. (6). It may be
noted that nCr occuring in the above expression is also sometimes known
as the binomial coefficient. Thus, (x + a)n, (n = 0, 1, 2, 3, 4, 5), in terms
of binomial coefficient, can also be written as
(x + a)0 = 1C1
(x + a)1 = 1C0 xa0 + 1C1 x0a
(x + a)2 = 2C0 x2a0 + 2C1 ax + 2C2 x0a2
(x + a)3 = 3C0 x3a0 + 3C1x2a + 3C2 xa 2 + 3C3 x0a3
(x + a)4 = 4C0 x4a0 + 4C1 x3a + 4C2 x2a2 + 4C3 xa3 + 4C4 x0a4
(x + a)5 = 5C0 x5a0 + 5C1 x4a + 5C2 x3a2 + 5C3x2a3 + 5C4 xa4 + 5C5 x0a5
The above expressions suggest a general formula for expanding
(x + a)n which is known as the binomial theorem.

Result B16 Binomial theorem for positive integral index. If x and a are
real numbers, then for all natural numbers n
(x + a)n = nC0 x na0 + nC1 x n– 1a1 + nC2 x n – 2 a2 + ... + nCr x n – r ar
+ ... + nCn – 1 x1a n – 1 + nCn x 0a n (12)
Equation (12) can also be written as
n
(x + a)n = ∑ n Cr x n − r a r (13)
r =0

Remarks
1. From Eq. (13) it may be noted that r can have values from 0 to n,
thus the total number of terms in the expansion is (n + 1).
2. In each term, the sum of the indices of x and a is n.
3. Put a = – a in Eq. (13), we get
338 Mathematical Methods

n
(x – a)n = ∑ (−1)r n Cr x n −r a r (14)
r =0

which clearly shows that the terms in the expansion (x – a)n are
alternatively positive and negative and the last term is positive or negative
according as n is even or odd.
4. Put x = 1 and a = x in Eqs. (12) and (13), we get
(1 + x)n = nC0 + nC1 x + nC2 x 2 + ... + nCr x r + ... + nCn xn (15)
and
n
(1 + x) n = ∑ n Cr x r (16)
r =0
Equation (15)/(16) provides the expansion of (1 + x)n in ascending powers
of x.
5. Put a = 1 in Eqs. (12) and (13), we have
(1 + x)n = nC0 x n + nC1x n –`1 + nC2 x n – 2
+ ... + nCr x n – r + ... + nCn –1 x + nCn (17)
and
n
(1 + x)n = ∑ n Cr x n − r (18)
r =0
which is the expansion of (1 + x)n in descending powers of x.
6. Put x = 1 and a = – x in Eqs. (12) and (13), we get
(1 – x)n = nC0 – nC1x + nC2 x2 + ... + (– 1)r nCr xr + ... + (–1)n nCn xn
(19)
and
n
(1 – x)n = ∑ (−1)r n Cr x r (20)
r =0

7. The coefficient of (r + 1)th term in the expansion of (1 + x)n is nCr.


8. The coefficient of xr in the expansion of (1 + x)n is nCr.
From Eq. (12) it may be noted that
the first term = nC0 x n a0
the second term = nC1 x n – 1 a1
the third term = nC2 x n – 2a2
the fourth term = nC3 x n – 3a3
M
the (r + 1)th term = nCr x n – r ar
Appendix B : Permutation, Combination and Binomial Theorem 339

If we denote the (r + 1)th term by Tr +1, then


Tr +1 = nCr x n – r a r (21)
This is known as the general term of the binomial expansion of (x + a)n.
Since there are (n + 1) terms in the binomial expansion of (x + a)n,
therefore

(i) ( n2 + 1)th term is the middle term provided that n is even.

(ii) ( n2+1 )th and ( n 2+3 )th terms are the two middle terms when n is
odd.

Example B31 Expand [(x + y)5 + (x – y)5] using binomial theorem and
hence find the value of [( 2 + 1)5 + ( 2 − 1)5 ].
Solution From Eqs. (13) and (14), we have
(x + y)5 + (x – y)5 = 2[5C0x5 + 5C2x3y2 + 5C4x1y4] = 2(x5 + 10x3y2 + 5xy4)
Now, put x = 2 and y = 1 in the above equation, we get after
simplification
( 2 + 1)5 + ( 2 − 1)5 = 58 2

Example B32 What is the general term in the expansion of (x 2 – y)6?


Solution The given binomial can be expressed as (x 2 – y) 6 =
[x 2 + (– y)6]. Thus, from Eq. (21), the general term is given by
Tr + 1 = 6Cr (x2)6 – r (– y)r = (– 1)r 6Cr x12–2r y r

( )
12
Example B33 Determine the tenth term in the expansion of 2 x 2 + 1
x
.
Solution Since the (r + 1)th term in the expansion of (x + a)n is
Tr + 1 = nCr x n – r ar

( )
12
therefore in the expansion of 2 x 2 + 1
x
, we have (here, n = 12, r = 9,
x = 2x2 and a = 1/x)
9
⎛1⎞ 3⎛ 1 ⎞ 1760
T10 = T9+1 = 12C9 (2x2)12–9 ⎜ ⎟ = C9 2 ⎜ 3 ⎟ = 3
12
⎝x⎠ ⎝x ⎠ x
Example B34 Prove that the middle term in the expansion of (1 + x)2n is
1 ⋅ 3 ⋅ 5...(2n − 1) n n
2 x
n!
340 Mathematical Methods

Solution The index of the given expression (1 + x)2n is 2n which is even.


Thus the (n + 1)th term, i.e. the ( 22n + 1)th term is the middle term and
we have
(2n)!
the middle term = Tn+1 = 2nCn(1)2n – n xn = 2nCn x n = xn
(2n − n)! n!
1 ⋅ 2 ⋅ 3 ⋅ 4...(2n − 3) (2n − 2) (2n − 1)(2n) n
= x
n!n!
[1 ⋅ 3 ⋅ 5...(2n − 3) (2n − 1)][2 ⋅ 4 ⋅ 6...(2n − 2) (2n)] n
= x
n!n!

[1 ⋅ 3 ⋅ 5...(2n − 3) (2n − 1)][1 ⋅ 2 ⋅ 3 ... (n − 1) (n)]2n n


= x
n!n!
[1 ⋅ 3 ⋅ 5...(2n − 3) (2n − 1)] n!2n x n 1 ⋅ 3 ⋅ 5 ... (2n − 1) n n
= = 2 x
n!n! n!
Example B35 Determine the coefficient of x10 in the expansion of

(2 x )
11
2
− 3
x
, when x ≠ 0.

( )
11
Solution Let (r + 1)th term contain x10 in the expansion of 2 x 2 − 3
,
x

then
r
⎛ −3 ⎞
Tr + 1 = 11Cr (2x2)11–r ⎜ r 11
⎟ = (– 1) Cr 2
11–r 3r x22–3r (22)
⎝ x ⎠
This term will contain x10 only when 22 – 3r = 10 which yields r = 4.
Thus, (4 + 1)th, i.e. 5th term contains x10. Now, put r = 4 in Eq. (22), we
get
T5 = (– 1)4 11C4 211–4 34 x10 = 11C4 27 34 x10
Therefore
the coefficient of x10 = 11C4 27 34

Example B36 Compute (99)5 using binomial theorem.


Solution We have
(99)5 = (100 – 1)5
= 5C0 (100)5 – 5C1(100)4 + 5C2(100)3 – 5C3(100)2 + 5C4(100)1 – 5C5(100)0
= (1010 + 107 + 5(10)2) – (5(10)8 + 105 + 1)) = 9509900499
Appendix B : Permutation, Combination and Binomial Theorem 341

Result B17 Binomial theorem for any index. If n is a rational number


and x is a real number such that | x | < 1, then
n(n − 1) 2 n(n − 1) (n − 2) ... (n − r + 1) r
(1 + x)n = 1 + nx + x + ... + x + ... ∞
2! r!
(23)
Remarks
1. If n is a natural number, then the condition | x | < 1 is not required,
while this condition is necessary if n is a rational number.
2. If n is a negative integer or a fraction, then there are infinite number
of terms in the expansion of (1 + x)n.
3. If the index of the binomial is either a negative integer or a fraction,
then (x + a)n has the following expansion
n n
⎡ ⎛ x ⎞⎤ ⎛ x⎞ ⎡ x n(n − 1) ⎛ x ⎞
2 ⎤
(x + a)n = ⎢ a ⎜ 1 + ⎟ ⎥ = a n ⎜ 1 + ⎟ = a n ⎢1 + n + ⎜ ⎟ + ...⎥
⎣ ⎝ a ⎠⎦ ⎝ a⎠ ⎣⎢ a 2! ⎝ a ⎠ ⎥⎦
n( n − 1) n – 2 2
= a n + na n –`1 x + a x + ... (24)
2!
The expansion (24) is valid when | x/a | < 1 or | x | < | a |.
4. If n is a positive integer, then the expansion (24) coincides with
(1 + x)n = nC0 + nC1x + nC2 x2 + ... + nCn (25)
because
nC n( n − 1) n n( n − 1) ( n − 2)
0 = 1, nC1 = n, nC2 = , C3 = , ... (26)
2! 3!
5. The general term in Eq. (23) is
n( n − 1) ( n − 2) ...[ n − ( r − 1)] r
Tr + 1 = x (27)
r!
6. When x > a, i.e. a/x < 1, then
n n
⎡ ⎛ a ⎞⎤ n⎛ a⎞ n
⎡ a n(n − 1) ⎛ a ⎞
2 ⎤
(x + a)n x
= ⎢ ⎜ 1 + ⎟⎥ = x ⎜ 1 + ⎟ = x ⎢ 1 + n + ⎜ ⎟ + ...⎥
⎣ ⎝ x ⎠⎦ ⎝ x⎠ ⎣⎢ x 2! ⎝ x ⎠ ⎥⎦
(28)
while, if x < a, then we have
n
⎡ x ⎤ x
n ⎡ x n( n − 1) ⎛ x ⎞
2 ⎤
(x + a)n = ⎢ a ⎛⎜ 1 + ⎞⎟ ⎥ = a n ⎛⎜ 1 + ⎞⎟ = a n ⎢1 + n + ⎜ ⎟ + ...⎥
⎣ ⎝ a ⎠⎦ ⎝ a⎠ ⎢⎣ a 2! ⎝ a ⎠ ⎥⎦
342 Mathematical Methods

which is same as Eq. (24).


7. Put n = – n in Eq. (23), we get
n(n + 1) 2 n(n + 1) (n + 2)...(n + r − 1) r
(1 + x)–n = 1 – nx + x + ... + (−1)r x + ...
2! r!
(29)
The general term of Eq. (29) is
n( n + 1) ( n + 2) ... (n + r − 1) r
Tr + 1 = (– 1)r x (30)
r!
8. Put x = – x in Eq. (29), we get
n(n + 1) 2 n(n + 1) (n + 2) ... (n + r − 1) r
(1 – x)– n = 1 + nx + x + ... + x + ...
2! r!
(31)
The general term of Eq. (31) is
n( n + 1) ( n + 2) ... (n + r − 1) r
Tr + 1 = x (32)
r!
9. Put x = – x in Eq. (23), we get
n(n − 1) 2 n(n − 1) (n − 2) ... (n − r + 1) r
(1 – x) n = 1 – nx + x + ... + (−1)r x + ...
2! r!
(33)
The general term in Eq. (33) is
n ( n − 1) ( n − 2) ...[ n − ( r − 1)] r
Tr + 1 = (– 1)r x (34)
r!
10. Put n = 1, 2, 3 in Eq. (29), we get
(1 + x)–1 = 1 – x + x 2 – x 3 + x 4 – ... + (– 1)r xr + ...
(1 + x)–2 = 1 – 2x + 3x2 – 4x3 + 5x4 – ... + (– 1)r (r + 1) xr + ...
(r + 1) (r + 2) r
(1 + x)–3 = 1 – 3x + 6x 2 – 10x 3 + .. + (–1)r x + ...
2
If we replace x by – x in the above expressions, we get
(1 – x)–1 = 1 + x + x2 + x3 + ... + xr + ...
(1 – x)–2 = 1 + 2x + 3x2 + 4x3 + ... + (r + 1) x r + ...
(r + 1) (r + 2) r
(1 – x)–3 = 1 + 3x + 6x2 + 10x 3 + ... + x + ...
2
Appendix B : Permutation, Combination and Binomial Theorem 343

Example B37 Expand (3 + 2x)– 4 upto four terms in (i) ascending powers
of x and (ii) descending powers of x. Also, find the values of x for which
the expansion is true in each case.
Solution (i) we have
−4 −4
(3 + 2x)– 4 = ⎢⎡ 3⎜⎛ 1 + 2 x ⎟⎞ ⎥⎤ ⎛
= 3− 4 ⎜ 1 +
2x ⎞

⎣ ⎝ 3 ⎠⎦ ⎝ 3 ⎠
⎡ ⎛ 2 x ⎞ (− 4) (− 4 − 1) ⎛ 2 x ⎞
2
(− 4) (− 4 − 1) (− 4 − 2) ⎛ 2 x ⎞
3 ⎤
= 3− 4 ⎢1 + (− 4) ⎜ ⎟+ ⎜ ⎟ + ⎜ ⎟ + ...⎥
⎢⎣ ⎝ 3 ⎠ 2! ⎝ 3 ⎠ 3! ⎝ 3 ⎠ ⎥⎦

⎛ 8 40 2 160 3 ⎞
= 3− 4 ⎜ 1 − x + x − x + ... ⎟
⎝ 3 9 27 ⎠
This expression is true only when | 2x/3 | < 1, which means that | x | <
3/2 or – 3
2
< x < 32 .
(ii) We have
−4 −4
⎡ ⎛ 3 ⎞⎤ ⎛ 3 ⎞
(3 + 2x)– 4 = ⎢ 2 x ⎜ 1 + ⎟ = (2 x)− 4 ⎜ 1 + ⎟
⎣ ⎝ 2 x ⎠ ⎥⎦ ⎝ 2x ⎠

1 −4
⎡ ⎛ ⎞ 3 ( − 4) ( − 4 − 1) ⎛ 3 ⎞
2
( − 4) ( − 4 − 1) (− 4 − 2) ⎛ 3 ⎞
3 ⎤
= 16 x ⎢1 + (− 4) ⎜ 2 x ⎟ + ⎜ ⎟ + ⎜ ⎟ + ...⎥
⎢⎣ ⎝ ⎠ 2! ⎝ 2x ⎠ 3! ⎝ 2x ⎠ ⎥⎦

1 −4 ⎛ 45 − 2 135 − 3 ⎞
x ⎜ 1 − 6 x −1 +
= x − x + ... ⎟
16 ⎝ 2 2 ⎠
This expression is true only when | 3/2x | < 1, which means that
3 < | 2x | or | x | > 32 .

Example B38 Find the general term in the expansion of (1 – x)– 4.


Solution If Tr + 1 denotes the general term in the expansion of (1 – x)– 4,
then
(− 4) ( − 4 − 1) ( − 4 − 2) ... ( − 4 − ( r − 1))
Tr + 1 = (– x)r
r!
(−1) r ⋅ 4 ⋅ 5 ⋅ 6 ... (r + 3) (−1)2 r ⋅ 4 ⋅ 5 ⋅ 6 ... (r + 3) r
= (−1)r x r = x
r! r!
4 ⋅ 5 ⋅ 6 ... r (r + 1) (r + 2) (r + 3) r (r + 1) (r + 2) (r + 3) r
= x = x
1 ⋅ 2 ⋅ 3... (r − 1) r 1⋅ 2 ⋅ 3
344 Mathematical Methods

Example B39 Determine the coefficient of x 6 in the expansion of


(1 – 2x)–5/2.
Solution Let (r + 1)th term in the expansion of (1 – 2x)–5/2 contain x6.
Then

Tr + 1 =
( )(
−5
2
−5
2
−1 )( −5
2 ) (
− 2 ...
−5
2
− (r − 1) ) (– 2x) r
r!
( 52 )( 72 )( 92 ) ... ( 2r2+3 )
= (– 1)r (– 1)r 2r x r
r!
5 ⋅ 7 ⋅ 9 ... (2r + 3) 5 ⋅ 7 ⋅ 9 ... (2r + 3) r
= (– 1)2r r
2r x r = x
2 r! r!
Now, if this term is going to contain x 6, then r must be equal to 6.
Therefore
5 ⋅ 7 ⋅ 9 ⋅ 11 ⋅ 13 ⋅ 15 15015
coefficient of x 6 = =
6! 16
Example B40 Find the value of 26 upto four places of decimal.
Solution We have

2 1/2 2 1/2 1 2 −1/ 2 ( 12 )( −21 ) (52 )−3/ 2 (1)2 +


26 = (5 + 1) = (5 ) + (5 ) (1) +
2 1⋅ 2

+
( )( )( ) (52 )−5/2 (1)3 + ( )( )( )( −25 ) (52)–7/2 (1)4 + ...
1
2
−1
2
−3
2
1
2
−1
2
−3
2
1⋅ 2 ⋅ 3 1⋅ 2 ⋅ 3 ⋅ 4
⎛ 1⎞⎛1⎞ ⎛ 1 ⎞⎛ 1 ⎞ ⎛ 1 ⎞⎛ 1 ⎞ ⎛ 1 ⎞⎛ 1 ⎞
= 5 + ⎜ ⎟ ⎜ ⎟ − ⎜ 3 ⎟ ⎜ 3 ⎟ + ⎜ 4 ⎟ ⎜ 5 ⎟ − ⎜ 7 ⎟ ⎜ 6 ⎟ + ...
⎝ 2⎠⎝5⎠ ⎝ 2 ⎠⎝5 ⎠ ⎝ 2 ⎠⎝5 ⎠ ⎝ 2 ⎠⎝5 ⎠
= 5.00000 + 0.10000 – 0.00100 + 0.00002 – ... = 5.09902 (approx.)
Therefore 26 = 5.0990, correct to four places of decimals.
It may be noted that if we express 2 1/2
26 as (1 + 5 ) , then using the
binomial theorem, we have
2 1/2
26 = (1 + 5 ) = 1 + 12.5 – 78.125 + 976.5625 – ...
which obviously is not the valid expansion.
Example B41 Find the cube root of 127 correct to four places of decimals.
Solution Since
1/3
⎛ 2 ⎞
(127)1/3 = (125 + 2)1/3 = (125)1/3 ⎜1 + ⎟
⎝ 125 ⎠
Appendix B : Permutation, Combination and Binomial Theorem 345

therefore, from binomial theorem, we have

⎡ 2 ⎛1⎞⎛ 2 ⎞
2
⎛ 5 ⎞⎛ 2 ⎞
3 ⎤
(127)1/3 = ⎢
5 1 + − ⎜ ⎟⎜ ⎟ + ⎜ ⎟⎜ ⎟ + ...⎥
⎢⎣ 375 ⎝ 9 ⎠ ⎝ 125 ⎠ ⎝ 81 ⎠ ⎝ 125 ⎠ ⎥⎦
= 5(1 + 0.005333 – 0.000028 + 0.000000253 – ...)
= 5(1.005333 – 0.000028) = 5.026525 = 5.0265

Example B42 Evaluate (0.98)–3 upto two decimal places.


Solution We have
( −3) ( − 3 − 1)
(0.98)–3 = (1 – 0.02)–3 = [1 + (– 3) (– 0.02) + (– 0.02)2
2!

( −3) ( −3 − 1) ( −3 − 2)
+ (– 0.02)3 + ...] = 1 + 0.0600 + 0.0024 + ...
3!
= 1.0624 = 1.06 upto two places of decimals
Remarks
While finding the approximate values upto n places of decimals, the
procedure is that first write the value of each term upto (n + 2) places of
decimals and then neglect those terms which contain (n + 1) or more
consecutive zeros just after the decimals.

EXERCISES
1 1 x
1. Find x, when + = . [Ans. 112]
9! 10! 11!

n! n!
2. If and are in the ratio 2 : 1, find the value of
2!(n − 2)! 4!(n − 4)!
n. [Ans. 5]
3. Evaluate P(15, 3) and P(5, 5). [Ans. 2730, 120]
4. If P(n, 4) = 20P(n, 2), find n. [Ans. 7]
5. If P(5, n) = 2P(6, n – 1), find n. [Ans. 3]
6. If P(10, n) = 5040, find n. [Ans. 4]
7. 9 9 10
If P5 + 5 P4 = Pn, find n. [Ans. 5]
8. 56 54
If Pn+6 : Pn+3 = 30800 : 1, find n. [Ans. 41]
9. In how many ways can six persons stand in a queue? [Ans. 720]
346 Mathematical Methods

10. How many different signals can be made from any number of flags
from five flags of different colours? [Ans. 325]
11. In how many ways seven paintings can be hung from five painting
nails on a wall? [Ans. 2520]
12. How many four letter words can be formed out of the letters of the
word LOGARITHMS, if the repetition of letters is not allowed
(the meanings of the words thus formed are not taken into
consideration)? [Ans. 5040]
13. How many words can be formed with the letters of the word
ORDINATE such that the vowels occupy the odd place?
[Ans. 576]
14. How many words can be formed from the letters of the word
DAUGHTER such that (i) the vowels always occur together (ii)
the vowels never occur together? [Ans. (i) 4320 (ii) 36000]
15. Find the number of permutations of the words APPLE and
BANANA. [Ans. 60, 60]
16. In how many ways five rings of different types can be worn in four
fingers? [Ans. 45]

∑ j =1 52− j C3 .
5
17. Find the value of 47C4 + [Ans. 52C4]

18. Evaluate (i) 10C8 (ii) 100C98 (iii) 52C52. [Ans. 45, 4950, 1]
19. If 15 = 8, find the value of nC12.
nC nC [Ans. 253]
20. Find the value of n, when 2nC3 : nC3 = 11 : 1. [Ans. 6]
21. Find the number of sides of a polygon which has 44 diagonals.
[Ans. 11]
22. Using binomial theorem, expand the following:
(i) (x2 + 2a)5 [Ans. x10 + 10x8a + 40x6a2 + 80x4a3 + 80x2a4
+ 32a5]
(ii) (2x – 3y) [Ans. 16x – 96x y + 216x y – 216xy + 81y4]
4 4 3 2 2 3

(iii) (1 + x + x2)3 [Ans. x6 + 3x5 + 6x4 + 7x3 + 6x2 + 3x + 1]


9
⎛ 4x 5 ⎞
23. Determine the sixth term in the expansion of ⎜ − ⎟ .
⎝ 5 2x ⎠
[Ans. –5040/x]
Appendix B : Permutation, Combination and Binomial Theorem 347

7
⎛ 3 x3 ⎞
24. Find the fourth term from the end in the expansion of ⎜ − ⎟ .
⎜ 2 6 ⎠⎟
⎝x
[Ans. 35x6/48]
7
⎛ x3 ⎞
25. Determine the middle term in the expansion of ⎜ 3x − ⎟ .
⎜ 6 ⎠⎟

[Ans. 35x15/48]
26. Determine the coefficients of x32 and x–17 in the expansion of
⎛ 4 1 ⎞
⎜x − 3 ⎟. [Ans. 1365, – 1365]
⎝ x ⎠
27. From the binomial theorem, evaluate (i) (102)6 (ii) (10.1)5.
[Ans. (i) 1126162419264 (ii) 105101.00501]
2
28. Expand (4 – 3x ) –1/3 to four terms and find the values of x for
which the expansion is valid.

[Ans. 4–1/3 (1 + (x2/4) + (x4/8) + (7x6/96) +...), (−2/ 3) < x < (2/ 3)]
29. Find the general term in the expansion of (4 – 5x2)–1/2.
⎡ 1 ⋅ 3 ⋅ 5...(2r − 1) 2 r ⎤
⎢ Ans. x ⎥
⎣ r!2r +1 ⎦
2
30. Find the general term in the expansion of (2 – 3x ) . –2/3

⎡ −2/3 2 ⋅ 5 ⋅ 8 ... (3r − 1) 2 r ⎤


⎢ Ans. (2 ) x ⎥
⎣ r!2r ⎦
10
31. Determine the coefficient of x in the expansion of
(1 + 3x2) (1 – x2)–3. [Ans. 66]
32. Determine the coefficient of x 4 in the expansion of
(1 – x)2 (1 + x)–2. [Ans. 16]
33. Show that the coefficient of x in the expansion of (1 – 4x)–1/2 is
n

(2n)!/(n!)2.
34. Show that the coefficient of x n in the expansion of (1 – 2x)–1/2 is
(2n)!/2n (n!)2.
35. Determine the coefficient of x7 in the expansion of (x – 2x2)–3.
[Ans. 67584]
36. Evaluate 23 correct to four places of decimals. [Ans. 4.7958]
348 Mathematical Methods

Appendix C : Some Useful


Formulas

C1 Algebra
(i) Laws of Exponents
am
a ma n = a m + n = am – n
an
(a m) n = a m n (ab) n = a nb n
n
⎛a⎞ an
⎜ ⎟ = n, b≠0 a0 = 1
⎝b⎠ b
(ii) Fractional Exponents
a1/n = n a
where n is called the index and a the radicand.

a m/n = n
a m = ( n a )m
where m is an integer, n is a positive integer and a is a real number. If n
is even then a ≥ 0.
(iii) Operations with Radical Expressions
n
n
a a
anb = n
ab n = n , b≠0
b b

a = mn a
m n cn cm = n m
a a
(iv) Properties of Logarithms
If M and N are positive real numbers, a > 0 and a ≠ 1, then
⎛M ⎞
loga (MN) = loga M + loga N log a ⎜ ⎟ = log M – log N
⎝N ⎠ a a

1
loga M n = n loga M log a = – log a M
M
Appendix C : Some Useful Formulas 349

1
log a n M = loga M loga 1 = 0
n
loga (a x) = x a (log a x ) = x

C2 Geometry
The following notations have been used in these formulas:
A = area, V = volume, b = base, h = altitude, r = radius (including base
radius), C = circumference, B = base area.
1
Triangle : A = bh.
2
Parallelogram : A = bh.
1
Trapezoid : A = (a + b) h (a and b being the lengths of parallel sides).
2
Circle : C = 2πr, A = πr2.
Ellipse : A = π ab, a and b are semi-major and semi-minor axes.
Parallelopiped : V = Bh.
1
Tetrahedron : V = Bh.
3
Right circular cylinder : Curved surface A = 2πrh, V = πr2h.
1 2
Right circular cone : Curved surface A = πr r + h , V = πr h.
2 2
3
1
General cone, or pyramid : V = Bh.
3
4 3
Sphere : A = 4πr2, V = πr .
3
4
Ellipsoid : V = πabc, a, b and c are the semi axes.
3

C3 Trigonometry
(i) Basic Trigonometric Identities
1 1 1 1
sin θ = , cos θ = , tan θ = , cot θ =
cos θ sec θ cot θ tan θ

1 1 sin θ cos θ
sec θ = , csc θ = , tan θ = , cot θ =
cos θ sin θ cos θ sin θ
350 Mathematical Methods

It may be noted that all the trigonometric ratios are positive in the first
quadrant. In the second quadrant, only sin θ and csc θ are positive. In
the third quadrant, only tan θ and cot θ are positive, while in the fourth
quadrant, only cos θ and sec θ are positive. We also have
sin2 θ + cos2 θ = 1, 1 + tan2 θ = sec2 θ, cot2 θ + 1 = csc2 θ
These identities are also known as Pythagorean identities.
sin (– θ) = – sin θ, cos(– θ) = cos θ, tan(– θ) = – tan θ
cot (– θ) = – cot θ, sec (– θ) = sec θ, csc (– θ) = – csc θ
sin (90° – θ) = cos θ, cos(90° – θ) = sin θ, tan(90° – θ) = cot θ
cot(90° – θ) = tan θ, sec(90° – θ) = csc θ, csc(90° – θ) = sec θ
sin(90° + θ) = cos θ, cos(90° + θ) = – sin θ, tan(90° + θ) = – cot θ
cot(90° + θ) = – tan θ, sec(90° + θ) = – csc θ, csc(90° + θ) = sec θ
sin (180° – θ) = sin θ, cos (180° – θ) = – cos θ, tan (180° – θ) = – tan θ
cot (180° – θ) = – cot θ, sec(180° – θ) = – sec θ, csc (180° – θ) = csc θ
sin (180° + θ) = – sin θ, cos(180° + θ) = – cos θ, tan(180° + θ) = tanθ
cot (180° + θ) = cot θ, sec (180° + θ) = – sec θ, csc(180° + θ) = – csc θ
(ii) Trigonometric Functions for a given Value of θ

Angle θ sin θ cos θ tan θ


0° 0 1 0
30° 1/ 2 3 /2 1/ 3
45° 1/ 2 1/ 2 1
60° 3/2 1/2 3
90° 1 0 ±∞
180° 0 −1 0
270° −1 0 ±∞
(iii) Sum and Difference Formulas
sin (A ± B) = sin A cos B ± cos A sin B
cos(A ± B) = cos A cos B + sin A sin B
tan A ± tan B
tan (A ± B) =
1 B tan A tan B
(iv) Double-angle Formulas
2tan A
sin 2A = 2sin A cos A =
1 + tan 2 A
1 − tan 2 A
cos 2A = cos2 A – sin2A = 2cos2 A – 1 = 1 – 2sin2 A =
1 + tan 2 A
Appendix C : Some Useful Formulas 351

2tan A
tan 2A =
1 − tan 2 A
(v) Half-angle Formulas
A 1 − cos A
sin = ±
2 2
A 1 + cos A
cos = ±
2 2
A 1 − cos A
tan =
2 sin A
(vi) Alternate Half-angle Formulas
1
sin2 A = (1 – cos 2A)
2
1
cos2 A = (1 + cos 2A)
2
1 − cos 2 A
tan2 A =
1 + cos 2 A
(vii) Formula for Converting the Sum or Difference into Product
⎛ A+ B⎞ ⎛ A−B⎞
sin A + sin B = 2sin ⎜ ⎟ cos ⎜ ⎟
⎝ 2 ⎠ ⎝ 2 ⎠
⎛ A+ B⎞ ⎛ A− B⎞
sin A – sin B = 2cos⎜ ⎟ sin ⎜ ⎟
⎝ 2 ⎠ ⎝ 2 ⎠
⎛ A+ B⎞ ⎛ A− B⎞
cos A + cos B = 2cos ⎜ ⎟ cos⎜ ⎟
⎝ 2 ⎠ ⎝ 2 ⎠
⎛ A+ B⎞ ⎛ A−B⎞
cos A – cos B = −2sin ⎜ ⎟ sin ⎜ ⎟
⎝ 2 ⎠ ⎝ 2 ⎠
(viii) Formulas for Converting the Product into Sum or Difference
2sin A cos B = sin(A + B) + sin(A – B)
2cos A sin B = sin(A + B) – sin(A – B)
2cos A cos B = cos(A + B) + cos(A – B)
2sin A sin B = cos(A – B) – cos (A + B)
sin(A + B) sin (A – B) = sin2A – sin2B
cos(A + B) cos(A –B) = cos2A – sin2B
352 Mathematical Methods

(ix) Laws of sines and cosines


If A, B and C are the angles of a triangle ABC and a, b and c are the sides
opposite to the angles A, B and C, respectively, then
a b c
= =
sin A sin B sin C
is the law of sines. The laws of cosines are
b2 + c2 − a2
cos A =
2bc
c2 + a 2 − b2
cos B =
2ca
a2 + b2 − c2
cos C =
2ab
(x) Projection formulas
a = b cos C + c cos B; b = c cos A + a cos C; c = a cos B + b cos A

C4 Some well Known Expansions


x x 2 x3
ex = 1+ + + + ... ∞
1! 2! 3!
x x 2 x3
e–x = 1 − + − + ... ∞
1! 2! 3!
ax (ax )2 (ax)3
eax = 1 + + + + ...∞
1! 2! 3!

1 1 1 1
e = 1+ + + + ... ∞ =
1! 2! 3!
∑ n!
n=0
2
x x3
a x = 1 + x(loge a) + (log e a)2 + (loge a)3 + ... ∞
2! 3!
where a > 0 and x is a real number.
x 2 x3 x 4
loge(1 + x) = x − + − + ... ∞
2 3 4
where | x | < 1.
x 2 x3 x 4
loge(1 – x) = − x − − − − ... ∞
2 3 4
Appendix C : Some Useful Formulas 353

⎛1 + x ⎞ ⎛ x3 x5 ⎞
log e ⎜ ⎟ = 2 ⎜ x + + + ... ∞ ⎟
⎝1− x ⎠ ⎜ 3 5 ⎟
⎝ ⎠
1 1 1 1
log 2 = 1 − + − + − ...
2 3 4 5
1 1 1
log 3 = 1 + + + + ...
12 80 448

x 3 x5
sin x = x − + − ... ∞
3! 5!

x2 x4
cos x = 1 − + − ... ∞
2! 4!
1 3 2 5 17 7
tan x = x + x + x + x + ... ∞
3 15 315

x3 x5 e x − e− x
sinh x = x + + + ... ∞ =
3! 5! 2

x2 x4 e x + e− x
cosh x = 1 + + + ... ∞ =
2! 4! 2

e x − e− x
tanh x =
e x + e− x
ex = cosh x + sinh x
e– x = cosh x – sinh x
sin nx = nC1 cosn – 1 x sin x – nC3 cosn–3 x sin3 x + ... ∞
cos nx = cosn x – nC2 cos n – 2 x sin2 x + nC4 cosn – 4 x sin4 x + ... ∞

π2 1 1 1 1
= 2
+ 2
+ 2
+ + ...
6 1 2 3 42

π4 1 1 1 1
= 4 + 4 + 4 + 4 + ...
90 1 2 3 4

π2 1 1 1 1
= 2
+ 2
+ 2
+ + ...
8 1 3 5 72
354 Mathematical Methods

π4 1 1 1 1
= 4 + 4 + 4 + 4 + ...
96 1 3 5 7
π2 1 1 1 1
= 2 − 2 + 2 − 2 + ...
12 1 2 3 4
π3 1 1 1 1
= 3 − 3 + 3 − 3 + ...
32 1 3 5 7
7 π4 1 1 1 1
= 4 − 4 + 4 − 4 + ...
720 1 2 3 4
References 355

References

1. Ahmad, K. : Text Book of Calculus. Real World Education


Publishers, New Delhi (2014).
2. Ahmad, K. : Text Book of Differential Equations. Real World
Education Publishers, New Delhi (2014).
3. Ahsan, Zafar : Differential Equations and Their Applications. 2nd
Edition. Prentice-Hall of India, New Delhi (2004).
4. Barnett, R.A., Ziegler, M.R. and Burke, C.J. : Applied Mathematics.
Deller Publishing Co. California (1986).
5. Thomas Jr., G.B. and Finney, R.L. : Calculus and Analytic
Geometry. 9th Edition. Addision-Wesley Publishing Co. Inc./First
ISE Reprint (1998).
Index 357

Index

Absolute maximum value 108 algebra of 13


Absolute minimum value 108 amplitude of a 17
Algorithm 2 conjugate of a 14
Al-jabr wal-muqabala 2 geometrical form of a 16
Al-Khawarizmi, Abu Abdallah imaginary part of a 13
Mohammad bin Musa 1 modulus of a 15
Archaeology 231 polar form of a 18
Area function 168 real part of a 13
Argand diagram 17 vector form of a 18
Argand plane 17 Complex plane 17
Argument 17 Compound interest 36, 186, 231
Arithmetic mean 314 Condition of collinearity 255
Automatic ventilation system 36 Consistent equations 256
Auxiliary equation 213 Cost equation 114
Bernoulli's equation 209 Cramer's rule 255
Binomial coefficient 337 Critical point 103
Binomial theorem 337 Cross product 281
Brain weight 33 Cube roots of unity 16
Brightness of a star 38 Curl of a vector 295
Carton industry 35 Daily protein intake 34
Cauchy's mean value theorem 120 Decibel 39
Chain rule 82 Del operator 293
Characteristic equation 213 Demand equation 114
Charle's law 29 Demand matrix 264
Closed interval 29 De Moivre's theorem 19
Cofactor 250, 258 Depereciation of equipment 40
Column vector 236 Derivative of a function 72, 287
Combination 331 geometric meaning of 74
Comlementary function 213 higher order 92
Complex number(s) 12 physical meaning of 74
358 Mathematical Methods

Determinant(s) 248 First fundamental theorem of integral


equivalent 254 calculus 168
Diastolic pressure 23 First mean value theorem 119
Differential coefficient 72 First principle 73, 78
Differential equation(s) 199 Function 23
complete solution of a 203 absolute value 31
degree of a 201 constant 29, 60
exact 210 continuous 59
general solution of a 203 decreasing 102, 103
higher order 212 differentiable 73
homogeneous 206 domain of a 27, 43
linear 201 even 42
non-linear 201 exponential 32, 104
non-homogeneous linear 215 graph of a 25
order of a 201 greatest integer 32
particular solution of 203 homogeneous 195
singular solution of a 203 identity 30, 60
solution of a 201 implicit 87
Differentiation 72 increasing 102, 103
of implicit function 87 inverse trigonometric 42
logarithmic 88 linear 29, 276
parametric 90 logarithmic 32, 104
by substitution 83 multiple-valued 187
Directional derivative 292 odd 42
Discriminant 7 periodic 42
Divergence 294 polynomial 29, 60
Divergence theorems 304 power 30
Dot product 279 quadratic 29
Double-declining balance formula 41 range of a 27, 43
Drug concentration 232 rational 30, 61
Earth quake 39 reciprocal 30
Effect of advertisement 232 signum 31
Elementary transformation 253 single-valued 187
Euler's formula 19 square root 30
Euler's theorem 195 trigonometric 41
Exponential decay 33 vector 286
Exponential growth 33 Gradient 293
Extreme value 106 Geometric mean 320
Factorial 324 Geometric series 316
First derivative test 106 Green's theorem
Index 359

in plane 303 column 236


in space 303 diagonal 237
symmetric form of 304 identity 238
Harmonic mean 322 invertible 258
Height of a male/female 34 inverse of a 257
Imaginary axis 17 non-singular 258
Inconsistent equations 256 null 237
Infinite limit 54 order of a 237
Initial point 272 row 236
Input-output analysis 264 scalar 238
Integral 130 singular 258
definite 164 skew/anti-symmetric 247
indefinite 129, 289 square 237
Integrating factor 208 transpose of a 246
Integration 129 zero 237
by parts 155 Matrix equation 261
by partial fraction 149 Maxima/minima of a function 105
by substitution 136 Minor of an element 250, 258
by successive reduction 160 Muhyi al-Din al-Maghribi 46
Intelligence quotient (IQ) 186 Muscle contraction 125
Intermediate value property 61 Natural numbers 12
Inverse of a function 44 Noise pollution 38
Irrational number 12 Normal component 300
Irrotational vector 295 Number line 12
Lagrange's mean value theorem 119 Open interval 29
Laplacian operator 295 Optical reader 26
Leontief input-output model 264 Output matrix 265
Level surface 292 Parallelogram law 274
Limit of a function 46 Partial derivative 187, 290
Line integral 297 Partial fraction 150
Linear polynomial 4 Particular integral 215
Linearly independent 276 Pascal triangle 337
Local maximum/minimum 105 Permutation 324
Loudness of sound 38 pH value 37
Marginal analysis 113 Photosynthesis 35
Marginal cost 113 Point of inflexion 106, 110
Marginal profit 114 Polynomial 4
Marginal revenue 114 linear 4
Matrix 235 Pollution in lakes 37
adjoint of a 259 Population growth 231
360 Mathematical Methods

Price demand 231 Sequence 309


Principal argument 17 finite 309
Principal diagonal 237 infinite 309
Profit equation 115 real 309
Progression 311 Simple interest 186
arithmetic 311, 313 Solenoidal vector 295
geometric 316, 319 Spread of rumour 232
harmonic 321 Stoke's theorem 304
Pythagorean identities 350 Submatrix 238
Quadratic equation 4 Surface integral 298
Rational numbers 12 Systolic pressure 23
Real axis 17 Tangential component 298
Real line 12 Technology matrix 264
Real numbers 12 Terminal point 272
Rectangular hyperbola 30 Total derivative 190
Reduction formula 160, 163 Total differential 190
Regular arc 297 Track and field events 34
Regular curve 297 Triple integral 302
Regular surface element 295 Two-industry model 261
Removable discontinuity 63 Unit tangent vector 292
Revenue equation 114 Upward parabola 30
Richter number 40 Vector(s) 272
Richter scale 40 component of a 277
Rolle's theorem 116 equal 273
Rotation 295 free 273
Row vector 236 null 273, 275
Sandwich theorem 49 position 278
Scalar 272 unit 275
Scalar field 286 zero 273, 275
Scalar product 279 Vector differentiation 287
Scalar triple product 284 Vector field 286
Second derivative test 110 Vector function 286
Second fundamental theorem of Vector integration 289
integral calculus 169 Vector product 281
Second mean value theorem 120 Vector triple product 284
Semi closed interval 29 Vertical line test 27
Semi open interval 29 Volume integral 302
Sensitivity to drugs/medicine 35, 113

View publication stats

You might also like